Download as pdf or txt
Download as pdf or txt
You are on page 1of 722

QUIZ 1

1. Reporting obligations: What is a binding precedent?


A. A decision of another court that is of persuasive authority
B. With respect to Australia, it is a decision of the UK Supreme Court
C. A decision of a court that binds judges in a lower court in the same court hierarchy
D. A decision of a different court on the same subject matter

2. Which are some of the main features of the Electronic Transactions Act 1999 (Cth):
A. Validity of electronic transactions
B. Recognition of writing by electronic
means
C. Recognition of retaining information in electronic form
D. All of the above

3. Peter verbally offers Tara his car for $5000. Tara refuses saying it’s not worth that, but : she
will pay $4000. What is the status of Peter’s first offer?
A. It can still be accepted by Tara
B. Peter should have made it in writing so Tara had proof he made it
C. Peter needs to make it again
D. It has lapsed because of Tara’s counteroffer

4. What was one of the “practical benefits” received by Roffey in Williams v Roffey Bros &:
Nicholls (Contractors) Ltd [1990] 1 All ER 512?
A. Williams did not sue Roffey for breach of contract
B. Roffey had no reason to doubt Williams would not complete his side of the bargain
C. Roffey received additional payment from Williams
D. Roffey did not need to find another subcontractor

5. Changing the Constitution: Section 128 of the Commonwealth Constitution provides


that the Constitution can be: changed by referendum that requires a “yes” vote:
A. In at least 2 States
B. In at least 3 States
C. In the ACT and NT (the Territories)
D. By the majority of voters and in a majority of States

6. Substantive and procedural law: Substantive law:


A. Refers to actual rights under the law
B. Is subsidiary to procedural law
C. Refers to the formal steps to enforcement of rights and duties under the law
D. Includes the rules of evidence

7. In which of the following situations is the offer most likely not to have lapsed?
A. Where Wrench had an option to purchase land but, unknown to Wrench, the seller
died: prior to Wrench’s acceptance
B. Where Carter, a prospective purchaser of land, makes a counter-offer that is rejected and:
then states that he will accept the earlier offer
C. Where no time was stated for acceptance by Jane, but Ben does not accept within a :
reasonable time
D. Where Jim has become overtaken by insanity prior to accepting

8. Reception of English law in Australia: Which Latin phrase explained the rationale for
applying English laws to the new Colony of : New South Wales?
A. actus reus
B. terra nullius
C. ratio decidendi
D. prima facie

A. Andy promises Ellie $100 on her 21st birthday. If this promise was contained in a simple :
contract, the legal position is that such a promise is:
B. Never binding as it must have consideration supplied by Ellie Binding only if Ellie and Andy had
contractual capacity
C. Sometimes binding but only after Ellie turned 21
D. Always binding because promises should not be broken

9. In Elizabeth City Center Pty Ltd v Corralyn Pty Ltd (1995) 63 SASR 235, the option to :
exercise renewal of the lease:
A. Was effective because of the operation of the postal rule
B. Effective communication of the renewal requirement had not been made
C. Negated the postal rule because the notification was not sent by certified mail as : required
D. b and c

10. Interpretation – extrinsic materials: In interpretation, extrinsic materials:


A. Are required to be referred to by a court
B. Are only to be referred to where the meaning of the provision is ambiguous
C. Are allowed to be referred to by a court
D. Are to be referred to regardless of the length of delay this will cause proceedings

11. With respect to revocation, which of the following statements is not correct?
A. Revocation need not be in words
B. The offeror must personally communicate the revocation to the offeree
C. The offeree may accept the offer until such time as they become aware of the
revocation
D. Where an offer has been made to the world at large, revocation does not need to be
seen : by everyone in order to be effective

12. Which of the following circumstances are likely to affect the consent of one or both :
parties to a contract:
A. Mistake
B. Duress and undue influence
C. Lack of writing
D. a & b

13. Simon and Stella, both of full legal capacity, agree to go on a date. Stella is to pay for the :
dinner, but she is running late and does not meet Simon at all. Simon is embarrassed and
: angry and calls Stella the next day to threaten to sue her for his taxi fares and dinner :
expenses. Will Simon succeed in his claims?
A. Yes, because there is agreement
B. Yes, because there was consideration in that Simon incurred taxi and dinner expenses
C. No, because Simon and Stella did not intend the agreement to create legally enforceable
: obligations
D. No, because there was no genuine consent of either party

14. Royal Assent: Who or what gives Royal Assent to an Act of the Commonwealth Parliament?
A. The Queen
B. The Governor-General on the Queen’s behalf
C. The Prime Minister
D. Cabinet

15. In Crown Melbourne Ltd v Cosmopolitan Hotel (Vic) Pty Ltd [2016] HCA 26 why did the :
High Court disagree with the decision of the Victorian Court of Appeal in finding there was
: no case for promissory estoppel:
A. Even though the elements of estoppel had been proven no remedy was provided as the
: initial claim was brought too late and equity does not assist claims lodged with unreasonable :
delay
B. The plaintiff could not establish that the statement that it would be “looked after at :
renewal time” was capable of conveying to a reasonable person that it was a genuine offer :
of a further lease
C. There was a need for certainty as to what the parties had agreed on at the end of the :
negotiations to found a claim for estoppel
D. B&C

16. Which of the following is not a way in which an offer can be terminated?
A. Revocation after acceptance has been mailed where the postal rule applies
B. Lapse
C. Lapse where no time was stipulated
D. Revocation before acceptance

17. Farah agreed to take care of an elderly woman Marge and in return was provided with a :
house to live in that was promised by Marge to be Farah’s after she died. Farah cared for
the : woman for 23 years, but upon Marge’s death Farah discovered that their oral
agreement : was never put into writing. The woman’s son moved into the house and
Farah made a claim : to the house. The court:
A. Is likely to reject Farah’s claim to the house as she provided no consideration of any legal:
value
B. Is likely to reject Farah’s claim to the house as the contract was not set down in writing as :
required
C. Is likely to order specific performance of the oral agreement because simple contracts do :
not need to be in writing
D. Is likely to apply the doctrine of part performance

18. Which of the following is most likely to be considered an offer?


A. A catalogue of books with discounted prices shown
B. A radio advertisement for drinks at “the coolest little pub in Victoria”
C. “I’ll pay you $3,000 if you complete a total rewrite of chapters 12 and 15 of this :
textbook.”
D. “Would you allow me to pay off the $2,000 over six weeks?”

19. Australian Constitution : Which Act established the federal legal and political system and
converted the separate : colonies into states?
A. Commonwealth of Australia Constitution Act 1900
B. The Australia Act 1986
C. Statute of Westminster Adoption Act 1942 (Cth)
D. statute of Westminster 1931 (IMP)

20. Contracts voidable by a minor do not include:


A. Those not binding unless ratified by the minor during their
minority
B. Those binding unless repudiated by the minor during their minority
C. Those binding unless repudiated by the minor within a reasonable time after attaining : their
majority
D. Those not binding unless ratified by the minor within a reasonable time of attaining : majority

21. Where an agreement has been made during the course of trade or commerce between
businesses, the situation will commonly indicate that the parties:
A. intended to create moral binding relations based on trust
B. intended to create binding legal relations
C. intended that there was no contract unless clearly specified in the agreement
D. intended to avoid court proceedings by inserting exclusion clauses in the agreement

22. Which of the following is not a class of persons regarded by the law as wholly or partly:
incapable of entering into legally binding contracts?
A. Intoxicated persons
B. Minors
C. Women
D. Mentally incapacitated persons

23. Separation of powers: Which of the following is not one of the three branches of the Commonwealth
government: in Australia:
A. The Governor
B. The executive
C. The judiciary
D. The legislature

24. Criminal offences: Which of the following is incorrect with respect to criminal offences?
A. Indictable offences are generally the more serious offences.
B. The prosecution must prove its case beyond reasonable doubt.
C. Summary offences are determined by a magistrate without a jury.
D. A committal hearing is held before most summary offence matters.

25. Federal system: Which of the following statements is correct?


A. Australia is a federal system, with one Constitution and a number of non-
law-making: States and Territories.
B. Australia is a unitary system, where the States and Territories are constrained in their :
law-making powers by the Commonwealth.
C. Australia is a federal system, with two legal systems for each citizen.
D. Australia is a federal system with three branches of government: the judiciary, the :
executive and the Crown.

26. Native Title: Native Title was first recognised in which of the following cases?
A. Mabo v State of Queensland (No 2) (1992) 175 CLR 1
B. Wik Peoples v State of Queensland (1996) 187 CLR 1
C. Brandy v Human Rights and Equal Opportunity Commission (1995) 183 CLR 245
D. Commonwealth v Jones (1901) 174 CLR 25

27. Separation of powers: Which of the following statements about separation of powers in Australia is
correct?
A. The judiciary is the body that makes statute law.
B. The legislature is the branch that declares what the law is and interprets the law.
C. The executive is the body that administers the law.
D. The legislature is the body that resolves disputes concerning the application of law and :
polices the law.

28. The postal acceptance rule:


A. Can be negated where the mailed offer was misdirected and the misdirection was the :
fault of the offeror
B. Can be impliedly excluded by offeror specifying actual receipt of acceptance
C. Can create a situation where a revocation is validly made days after the offer has
actually : been accepted
D. May apply to electronic communications, such as email, if the offeree chooses

29. Jack was subpoenaed to give evidence on Rods behalf. Jack claimed Rod promised him :
$2000 to give evidence. Would Jack recover that money?

A. No - Jack’s public duty is to give evidence in response to that subpoena.


B. No this is illegal
C. Yes, if Rod put that in writing
D. Always binding because promises should not be broken

30. Law Reports: Where are decisions of the High Court of Australia found?: only online
A. In the High Court reports
B. In the Commonwealth Law Reports
C. In the Common Law Reports

31. The Age of Majority Act 1977 (Vic) reduced the age for contractual capacity of a minor
A. 16
B. 18
C. 21
D. 25

32. Delegated legislation: An example of delegated legislation is the:


A. Partnership Act 1958 (Vic)
B. Local Government Act 1993 (Qld), s 25
C. Corporations Bill 1988 (Cth)
D. Corporations Regulations 2001 (Cth)

33. The Executive Power : The Executive power is administered by:


A. The Senate and the House of Representatives
B. The Queen, the Prime Minister and the Cabinet
C. The Governor, the Senate and the House of Representatives
D. The Judiciary, the Senate and the House of Representatives

34. Delegated legislation: Which of the following is not correct?: Delegated legislation:

A. Is subordinate legislation
B. Is made under the authority of an Act of Parliament
C. Often contains more detailed rules than those of the associated Act
D. Can only be made by Government Ministers

35. Common law: Which one of the following statements about common law is correct?

A. Common law is judge made law


B. Common law overrides statute law
C. Common law is made by Parliaments
D. Common law and equity are the same types of laws and provide the same
remedies

36. In Ashton v Pratt [2015] NSWCA 12 what was the main reason for the court deciding that : there was
no intention to create a legally binding contract?
A. The agreement was not reduced to writing
B. The verbal language of the agreement greatly lacked detail from either party and
did not : indicate definite obligations
C. Pratt lacked the necessary contractual capacity due to his age and ill health
D. The plaintiff had been a worker in the escort business and not morally entitled to any:
monies under the verbal agreement

37. Public and private law: Which of the following is not classified as private law?

A. The law of contract


B. Criminal law
C. The law of property
D. Corporations law

38. Criminal proceedings: An indictable offense is:


A. A more serious civil wrong
B. A less serious civil wrong
C. A more serious criminal offence
D. A less serious criminal offence

39. A qualified acceptance may also be known as a/an:


A. Counteroffer
B. Invitation to treat
C. Condition subsequent
D. Conditional agreement

40. Local or Magistrates Courts:


A. Are the lowest courts in the state hierarchy
B. Are the lowest courts in the federal hierarchy
C. Are an intermediate court
D. Are above Country Courts in the hierarchy
41. The legislature: Exclusive powers are those that:

A. Can be exercised by either the Commonwealth or the States


B. Can be exercised only by the Commonwealth
C. Can be exercised only by the States
D. Where there is any inconsistency, the Commonwealth law will prevail

42. Which of the following statements about contracts is true?


A. A contract must be in writing to be enforceable
B. It is a common law requirement that certain contracts are in writing
C. A simple contract is another term for an oral contract
D. It is a statutory requirement throughout Australia that some contracts must be in
writing: and others must be evidenced in writing

43. Sources of law: Sources of law in Australia do not include:


A. Consolidating statutes
B. Judge-made law
C. Repealed statutes
D. Equity

44. Which of the following statements about acceptance is false?


A. Acceptance can be by telephone or email or conduct
B. Acceptance can be in any manner chosen by the offeree
C. Acceptance must be unconditional
D. Acceptance can only be made by the person/s to whom the offer was made

45. The main legal issue in Felthouse v Bindley (1862) 11CB (NS) 869 was:
A. the contract was formed due to offer and acceptance being proven
B. An offer to sell a horse was simply an invitation to treat
C. generally silence /lack of action does not constitute acceptance of an offer
D. An offer can be revoked at any time prior to acceptance
46. Ratio decidendi*: The ratio decidendi of a case:
A. Is the full judgment of all judges
B. Is a statement of principle that is important, yet not crucial to the decision
C. Is the reason given for deciding the case
D. Is contained within the last published judgment

47. Hierarchy of courts: Which of the following is higher than the Federal Court of Australia in
the federal court : system?
A. Supreme Court
B. Magistrates Court
C. High Court
D. Federal Magistrates Court

48. Which of the following statements about consideration is correct?


A. A gratuitous promise is enforceable if contained in a simple contract
B. Good consideration may consist of performing an existing legal obligation
C. Good consideration can be present, future and even “past”
D. Consideration need not be adequate

49. Civil and criminal law: Which of the following is not correct with respect to civil law?
A. The typical purpose of a civil action is to obtain damages
B. Under civil law, one person may sue another who has committed a wrongful act.
C. Interrogatories and discovery are procedures that are available in civil proceedings.
D. The document filed by the defendant is called a writ.

50. If one party has threatened another party to enter into a contract, the element that is :
missing is:
A. Acceptance
B. Intention to create legal relations
C. Real/genuine consent
D. Legality

51. Court hierarchy: The highest court in Australia is:


A. The Privy Council
B. The Federal Court of Australia
C. The High Court of Australia
D. The Supreme Court of Australia

51. With respect to the contractual capacity of corporations, s 124 of the Corporations Act :
2001 (Cth) gives them:
A. Only the capacity to make contracts approved by shareholders in general meeting
B. The capacity to make contracts but only in the state or territory where the corporation :
conducts its business
C. All the legal capacity of a natural person (together with some additional powers that can : only
be exercised by a corporation)
D. Different capacity to contract depending on whether the corporation is public or : proprietary

52. The High Court’s decision in Waltons Stores (Interstate) Ltd v Maher (1988) 164
CLR 387:
A. Extended the doctrine of promissory estoppel such that it applies even where
the: promisor does not create or encourage the false assumption of the
promisee
B. Extended the doctrine of equitable estoppels such that it applies even where
there is no : pre-existing contractual relationship between the parties
C. Restricted the doctrine of equitable estoppel such that, where the contract is of a :
commercial nature, the doctrine will not apply
D. Restricted the doctrine of promissory estoppel such that where an innocent third party :
has been affected, no remedy can be ordered
53. Which of the following statements about bilateral and unilateral contracts is not correct?

A. More than one party is required for both bilateral and unilateral contracts
B. A unilateral contract does not require consideration but simply a promise to
perform an : act is sufficient
C. Both parties are obligated to perform their promises in a bilateral contract
D. Only one of the parties is obligated to perform an action in a unilateral contract

54. Why are illusory terms in a contract problematic?

A. they are vague or ambiguous so they fail to create a legal obligation


B. They exist in one party’s imaginary belief only
C. Courts have difficulty interpreting them
D. Interpreting them in any way at all could result in an unfair decision
55. Business law: Which of the following no longer regulates business or commercial law?
A. Trade Practices Act 1974 (Cth)
B. Competition and Consumer act 2010 (cth)
C. Corporations Act 2001 (cth)
D. The law of contract
56. Sources of law – statute: Which of the following is incorrect?: Statutes may:
A. Codify the law
B. Bring new laws into existence
C. Repeal laws
D. Not overrule existing common law
57. The rule of law: Which of the following concepts is closely related to the rule of law?
A. Representative government
B. Responsible government
C. Due process
D. Separation of powers
58. Interpretation: Which of the following is not correct?: With respect to interpretation,
the main approach used by courts is:
A. The literal approach
B. The purposive approach
C. The requirement that the natural and ordinary meaning to be given to words
D. The courts are free to exercise judicial discretion
59. In which of the following types of agreements is there a presumption that parties
intend : to be legally bound?
A. Partnership between husband and wife
B. Living arrangement between mother and daughter
C. Agreement between father and daughter
D. Dinner arrangement between friends
60. Doctrine of precedent: With respect to a case being decided under the appellate jurisdiction
of the County Court of : Victoria, a decision of the appellate division of the District Court of
New South Wales on the : same Commonwealth legislation is:
A. Strictly binding
B. Persuasive
C. Available for consideration if required
D. Not of any consequence
61. In Elizabeth City Centre Pty Ltd v Corralyn Pty Ltd (1995) 63 SASR 235, the option to :
exercise renewal of the lease:
A. Was effective because of the operation of the postal rule
B. Effective communication of the renewal requirement had not been made
C. Negated the postal rule because the notification was not sent by certified mail as :
required
D. B&C
62. Sally puts up a sign stating that she has lost her dogs and will pay a reward of $100 for : their
return to her within three days. Which of the following statements is correct?
A. Jim will not get the reward because he returned the dog but did not verbally :
communicate acceptance of the offer to Sally
B. Nadia will get the reward as she returned the dog to the RSPCA
C. Chang will not get the reward because he returned the dog knowing it was Sally’s, but :
was unaware of her offer of a reward
D. Isabel will get the reward because she was only one day late in returning the dogs (and to :
not give her the reward would be unreasonable)
63. Which of the following is not one of Brennan J’s six criteria for estoppel from Waltons:
Stores (Interstate) Ltd v Maher (1988) 164 CLR 387?
A. The promisor induced an assumption
B. The promisee acted in reliance on that assumption
C. The promisor knew or was reckless as to whether the promisee intended to act in that
D. The promisee will suffer a material loss if the assumption is not fulfilled
64. Constitution: Which of the following statements about constitutions in Australia is NOT
correct?
A. The Commonwealth Constitution sets out the relationship of the Commonwealth :
government with the States
B. A written constitution is a feature of a federal legal system such as Australia.
C. The Commonwealth Constitution sets out the powers of the Commonwealth government.
D. Australia has one constitution only (States have no constitutions).
65. Section 51 powers: Which of the following is not an area that the Commonwealth Parliament
has power to : legislate on under s 51 of the Commonwealth Constitution?
A. Education
B. Bankruptcy and insolvency
C. Foreign corporations
D. Banking and insurance
66. A contract dividing the proceeds of a cocaine importing venture that has no technical :
defects and that is not overly harsh/unfair to either party is:
A. Valid
B. Void
C. Voidable
D. Unenforceable
67. Sources of law: The two main types of law in Australia are
A. Common law and judge-made law
B. Statute law and judge-made law
C. Statute law and Acts of Parliament
D. Common law and equity
68. Constitution: Section 51 Commonwealth Constitution grants what type of law making power
to the : Commonwealth Parliament?
A. Concurrent powers
B. exclusive powers
C. residual
D. inherent
69. Simone advertises a car for $4,000. John responds to the advertisement and asks : whether
Simone will take $3,000. Simone says ‘no’, but that she will hold the offer open for : a week.
The next day she sells it to Samantha. : Which of the following statements is correct?
A. Simone has to hold the offer open as she made a promise
B. Simone does not have to hold the offer open as no consideration was given
C. John’s question is an invitation to treat
D. Simone does not have to hold the offer open as John made a counter-offer
70. Which of the following statements about the contractual obligations of government policy
proposals is incorrect?
A. The courts tend not to assign contractual obligations to government policy promises
B. Where the government enters into an ordinary commercial agreement to buy services, :
the law of contract regulates this agreement
C. Where the government makes a policy promise and a citizen relies on it the court will :
always regard that policy commitment as a binding contractual obligation
D. In Australian Woollen Mills Pty Ltd v Commonwealth of Australia (1954) 92 CLR 424, the :
government’s wool subsidy was found to be an administrative scheme and not contractual
: obligations
71. Section 15AA of the Acts Interpretation Act: Section 15AA of the Acts Interpretation Act
1901 (Cth)
A. Is an inflexible section
B. Provides that express mention of one matter suggests that other matters are excluded
C. Provides that where there is a conflict between a specific and a general provision, the :
specific provision should be applied
D. Provides that the purpose of the statute or its object should be the preferred method of :
determining the statute’s meaning

QUIZ 2
1. Which of the following is not true? A signature on a contract containing an
exemption: clause:
A. Will not guarantee the effectiveness of the clause where the doctrine of non est factum :
applies
B. Incorporates the exemption clause into the contract
C. Guarantees that an exemption clause will be effective
D. Will be effective even if one party did not read the contract

2. Which of the following is correct?


A. There cannot be any legal remedy where a contract is unenforceable
B. There can be no remedy where a contract is not terminated
C. There can be no remedy where a contract remains on foot after a breach
D. Damages must not be too remote from a breach

3. Which of the following is not correct? Restitution:


A. May be awarded where there is less than substantial performance
B. Is said to be based on unjust enrichment
C. Is subject to a number of defences, such as estoppel
D. Was not awarded in the case of Pavey & Matthews Pty Ltd v Paul (1987) 162 CLR 221 as : the
contract was not in writing as required by statute and therefore not enforceable

5. Fraser crashes his car with his friend Angus as a passenger. Angus is injured. Angus was :
not wearing a seatbelt. Which of the following is not correct?
A. Liability would likely be apportioned between Fraser and Angus
B. Damages cannot be apportioned for breach of contractual duty of care where
there is : contributory negligence
C. Damages awarded to Angus, if any, would be reduced by his contributory
negligence
D. Angus’ contributory negligence relates only to his own safety, not to that of others

6. Which of the following would not constitute the unfair practice of harassment or :
coercion under the Australian Consumer Law? Where there is a debt in connection
with the : possible of goods and the creditor:
A. Consciously calculates to intimidate the debtor with the content of her demands
B. Tells the debtor, among repeated demands for payment give the debtor a discount for
: early payment
C. Makes demands constantly in order to exhaust the debtor
D. Chooses particularly threatening demands in order to demoralise the debtor

7. When considering whether a contract is a standard form contract, the court does not :
have to consider:
A. The bargaining power of the parties as unequal bargaining power does not constitute a :
special disadvantage
B. Whether both parties had the opportunity to negotiate the terms of the contract
C. Whether one party’s vicarious liability for its agents is limited
D. Whether the terms of the contract take into account the specifics of the particular :
transaction

8. Which of the following is not an example of an unfair contract term?


A. Stan is not allowed to terminate the contract, but Flo is
B. The specifics of Rose’s situation were not taken into consideration by Melton Apartment :
Construction Ltd
C. Interpretation of the contract is only on the terms of Big Guns Insurance Co
D. The burden of proof for all matters rests on Ms Smith

9. Which of the following is not true? Economic duress:


A. Is a threat to the innocent party’s economic interests
B. Permits the threatened party to avoid the contract
C. Needs to be distinguished from a legitimate level of commercial pressure – an example : of an
actionable level of pressure is a threat to break a contract without any legal : justification in
order to extract money from an innocent party
D. Was found in North Ocean Shipping Co Ltd v Hyundai Construction Co Ltd [1979] 1 QB : 705
and the threatened party was able to recover the payments made under duress

10.In Barton v Armstrong [1976] AC 104, the court held that:


A. The defendant’s threats against the plaintiff’s life were insufficient to constitute duress : as the
plaintiff had financial reasons for entering into the contract
B. The defendant’s threats against members of the plaintiff’s family were insufficient to :
constitute duress as they were not “immediate” family
C. The threats to the plaintiff’s life contributed to the plaintiff’s decision to sign the : contract
and therefore constituted duress
D. The threats to the plaintiff’s life were not sufficiently plausible to create a real : apprehension
in the plaintiff constituting duress

11. Which of the following statements is correct with respect to mistake?


A. Mistakes of fact include bilateral mistake, common mistake and mutual mistake
B. Any mistake by either party will always void a contract
C. Rectification is a remedy for certain types of mistakes in written contracts
D. If a party mistakenly signs a document unaware of the nature of what they are signing : they
will be bound

12.Why are late payment charges on credit cards not penalties?


A. Banks penalties on cards are legitimate to cover their costs
B. They are considered penalties but in Paciocco v ANZ Banking Group Ltd the court held : they
were not
C. The Reserve Bank states they are not
D. They are considered just a consequence of the use of credit
13.Where a loan contract allows for termination by the loan provider in the event of default : by
the borrower, this is:
A. An implied power to terminate
B. An express power to terminate
C. A condition precedent
D. A release

14.Which of the following is not a way that a person seeking to rely on an exemption clause :
can show that the clause has become part of the contract?
A. By showing it is in writing and the other party has signed the contract
B. By showing it is in writing in a document a reasonable person would expect to contain :
contractual terms and was brought to the other party’s notice
C. By showing that the parties had previously contracted on terms that included the : exemption
clause and that they intended to contract on the same basis
D. By showing it was brought to the notice of the other party at the time of, or subsequent : to,
entering into the contract

15. Non est factum means:


A. It is not [my] deed
B. It is not true
C. Mistake of fact
D. It is unknown

16. Tom agrees to fix Richie’s car for $1,000. Richie pays a deposit of $200 and Tom :
guarantees the job will be completed within two weeks. Before Tom commences the
repairs, : the car is stolen from Richie’s locked garage and destroyed by fire. Richie
demands the : return of his $200 deposit and Tom refuses to give it to him. Which of
the following is not : correct?
A. The contract is automatically terminated
B. Tom is no longer obligated to fix Richie’s car
C. Under the Australian Consumer Law and Fair Trading Act 2012 (Vic), Tom is entitled
to:retain the $200 deposit
D. Under the Australian Consumer Law and Fair Trading Act 2012 (Vic), Tom is entitled to :
retain the portion of the $200 deposit he spent on parts and other relevant expenses, if
any

17. Which of the following is not correct? An action in restitution:


A. Is often brought where one party has been unjustly enriched at the expense of the other
B. Cannot be brought where there is no contract between the parties
C. Can be brought where the contract between the parties is unenforceable
D. Can be brought where there has been only part performance

18. Chris and Tama buy a noodle bar from Udon4U Pty Ltd. In the course of negotiations,
: Udon4U Pty Ltd’s agent, Nadia, misrepresents the turnover of the noodle bar,
stating an : amount that is double the actually turnover. Nadia herself received this
information on : turnover from Geoff, a director of Udon4U Pty Ltd.: In making the
misrepresentation, what sections has Nadia likely contravened?
A. As she merely passed on the information with no intention to procure a contravention
: she probably will not be personally liable
B. Section 2(1)
C. Section 18
D. Section 52
19. Which of the following is not one of the three elements of an unfair term under s
24(1) : of the Australian Consumer Law? Where:
A. It would cause a significant imbalance in the parties’ rights and
obligations
B. The term is not transparent
C. It is not reasonably necessary to protect the interests of the disadvantaged party
D. It would cause detriment to a party if it were applied

20. Which unfair practice involves participation in a trading scheme where persons
at the : top receive most of the benefits?
A. referral selling
B. pyramid selling
C. unconscionable conduct
D. misleading and deceptive conduct

21. Janey takes her fur stole to the dry cleaner. When she returns to collect it, there is a :
black mark in the shape of an iron in the middle of her stole and it is ruined. The dry
cleaner : tells Janey that he is not liable for the damage to her fur stole as there was
an exemption : clause in their agreement. Which of the following is not correct?
A. If the exemption clause was merely contained within a notice on the wall, the dry :
cleaner will be liable for the damage
B. If the exemption clause was on the dry cleaning docket Janey was given and she was
told : “don’t worry about this, it’s just the docket you have to give us when you come to
collect : your stole”, the dry cleaner will not be able to rely on the exemption clause
C. In order for the exemption clause to be effective where it was not in a signed
document: it had to have been brought to Janey’s attention
D. If the exemption clause was on the dry cleaning docket Janey was given rather than a :
formal written contract, the onus will be on the dry cleaner to prove that Janey was
aware it : contained conditions that would modify the agreement

22. Which of the following statements is correct?


A. An exemption clause is a term that completely excludes one party’s liability
B. An exemption clause is a term that excludes or limits the liability of one or more parties
C. An exemption clause limits liability to a certain monetary amount
D. An exclusion clause limits one party’s liability whereas an exemption clause limits all:
parties’ liability

23. Which of the following is not usually recoverable by way of damages?


A. Compensation for mere inconvenience or disappointment
B. Loss that is difficult to estimate
C. Nominal loss
D. Expenses incurred in reliance on the other party’s promise to perform

24. A party repudiates a contract when


A. They decide to terminate it
B. They are no longer able or are unwilling to perform their
obligations
C. The other party decides to terminate it
D. They take too long to perform an obligation that has a time limit under the contract, such
: as payment of rent

25. Chung is looking to purchase a carwash business from Terry. Terry tells Chung that
she : serves 100-150 cars per day and that she has just concluded an agreement to
service the : fleet of cars of a local business. In reality, Terry has only served
around 15 cars per day since : her customer service received a bad review in the
local newspaper and a competing carwash : business opened one street away.
Terry intends for Chung to buy her business based on : what she has stated. This is
an example of:
A. unilateral mistake
B. negligent misrepresentation
C. fraudulent misrepresentation
D. mutual mistake

26. Undue influence differs from duress in that:


A. With undue influence, the contract is not voidable but void
B. With undue influence, no unlawful act is required
C. Duress arises only within a closed list of special fiduciary relationships
D. With undue influence, actual physical violence is required

27. Which of the following constitutes referral selling?


A. Jaz is approached by her neighbor to join the sales team of Thames Beauty Products. :
Thames sells its products door-to-door through agents and is a close-knit family type of
: company. Agents that introduce other agents to “the family” are rewarded with a
finders’: fee. It is only $500 to join the Thames family, and Jaz only has to spend $400 of
products in: her first month
B. Jaz starts work at a chemist and notices that the chemist, as retailers often do, puts a :
sale price on a sticker on most of the products in the shop. However, the chemist does
not: put the sale price sticker on top of the original price sticker and when customers
bring the : products to the counter, the chemist tells them that unfortunately there has
been a mistake, : the true price is the higher price
C. Jaz approaches a salesperson on the forecourt to buy the car package she saw :
advertised on television and is told that there were only two cars for sale as a part of
that : deal and there are now none left. The salesperson tells Jaz that she would look
much better : in the convertible model and although there is no deal on that car, it is
only $50,000 more
D. Jaz purchases a painting through a new art gallery that has just opened in town. The :
price is high and Jaz is at first uncertain about the expense, but the art dealer promises
Jaz a : commission on sales to any of Jaz’s friends who she sends along to the gallery.
Jaz has a lot : of art collector friends so is sure that she will receive some commission
from the gallery in : the near future.

28. In Trident General Insurance Co Ltd v McNiece Bros Pty Ltd (1988) 165 CLR 107:
A. Blue Circle was a sub-contractor whose worker was injured, and privity precluded the :
worker from getting an indemnity under the contract
B. It was held that the subcontractor could get an indemnity even though the:
subcontractor had given no consideration under the contract
C. In Deane J’s view Trident, the insurance company, had an obligation to the
subcontractor because otherwise Trident would be unjustly enriched
D. The majority were of the view that the “settled and fundamental” doctrine of privity of :
contract should not be overturned by the court

29. Jacq and Jack robbed the Commercial Bank. Jacq thinks that Jack was too aggressive :
with the tellers and the robbery could have gone very wrong. Jacq is now refusing to
give : Jack his share of the takings. Which of the following is correct?
A. Jack provided consideration in the form of performing the robbery and is entitled to his :
share
B. Under the principle of freedom of contract, Jacq and Jack are entitled to agree to :
whatever they wish and the contract will be enforceable
C. The contract involved the commission of a crime and is therefore illegal and :
unenforceable
D. The contract will be unenforceable if the courts deem it to be against public policy

30. If one party has completed its obligations under the contract, and the other has not:
A. The contract can be terminated by mutual agreement
B. The contract cannot be terminated by simple agreement without consideration
C. The contract can be terminated if it is put down in writing; no further consideration is : require
D. The contract can be terminated where one party promises to abandon their rights under
: the contract and the other party promises to do the same

31. Ella contracted Anasaki to build a balcony for her house to certain specifications.
When : Anasaki had finished, Ella was shocked to see that the balcony was much
bigger than she : had specified. Ella refused to pay Anasaki. Which of the following is
incorrect?
A. Ella has received some benefit
B. The benefit Ella received was at Anasaki’s expense
C. It would be unjust for Ella to have to pay any money for a balcony that was not built to :
her specifications under the contract
D. Ella will probably not have to pay the contract price but must pay a reasonable amount

32. Which of the following is an element required for the doctrine of frustration to
operate?
A. Material loss to one party
B. No fault on the part of either party
C. Material loss to both parties
D. Hardship or inconvenience

33. Which of the following is not correct with respect to the prohibition of
unconscionable : conduct within the meaning of the unwritten law?
A. Unwritten law includes equity
B. There is no unconscionable conduct where both parties have made the same mistake in: good
faith
C. There must be a special disadvantage and an exploitation of that disadvantage
D. Unequal bargaining power of itself is enough to constitute a special disadvantage

34. In Jarvis v Swans Tours Ltd [1973] QB 233, where Jarvis was disappointed by, among :
other things, the little dry nut cakes on his holiday:
A. Damages were not awarded
B. Damages were awarded for anxiety and depression
C. Pleasure and enjoyment were not promises of the tour company with respect to the :
holiday in Switzerland
D. Damages were awarded partly for disappointment

35. What does the concept “quantum meruit” mean?


A. The contract is terminated because of a breach by both parties
B. The innocent party would be unjustly enriched if they were able to retain the benefit :
without compensating the party in breach for the “amount he deserves”
C. Voluntarily accepting a contractual benefit
D. Not my deed

36. Cathy and Mel entered into a contract where Cathy was to steam clean seven rooms
in : Mel’s house at $200 per room. After cleaning three rooms, Cathy abandoned the
job for a : more valuable contract. Which of the following is true?
A. As there has been substantial performance, Cathy will be able to enforce all the rights :
conferred by the contract
B. If the exact performance rule is applied, Cathy is entitled to payment for the work she
has : done
C. If the contract is divisible, Cathy is entitled to payment for the work she has done
D. Courts are inclined to treat contracts as indivisible and to require exact performance

37. Chang wants the court to imply an engineering custom into a term of a contract to
which : he is a party. The other party has no knowledge of this custom. It is not
contrary to any of : the express terms of the contract. The court:
A. Will not imply the term in these circumstances
B. May imply the term
C. Will imply the term even if the custom is not very well known
D. Will not imply the term unless it is implied by statute

38. Jong complains that the barbecue he bought is not fit for purpose because the metal :
sides melted the first time he cooked with it. The manufacturer maintains that Jong
should : not have used the barbecue for slow cooking and that it never represented
that the : barbecue would be fit for the purpose of slow cooking. What are the
circumstances in which : the guarantee as to fitness for purpose would apply?
A. Where Jong made known to the supplier that he was going to use the barbecue for slow
cooking meals
B. Where Jong bought the barbecue at auction
C. The guarantee always applies as it is not possible for a manufacturer or supplier to :
contract out of it
D. Where the unfitness for slow cooking of the barbecue was not drawn to Jong’s attention :
prior to his purchase by either the manufacturer or the supplier

39. It is important to distinguish between representations and terms because:


A. Damages cannot be awarded for misrepresentation
B. Damages can only be awarded for fraudulent or negligent misrepresentation
C. There is no remedy for misrepresentation
D. The remedy of rescission is not available for misrepresentation

40. Which of the following is not an enforcement measure under the Australian Consumer
: Law?
A. An order of the court requiring a supplier to publish an advertisement
B. A disclosure order issued by the ACCC
C. A two-year probation order of the court
D. A notice issued by the ACCC requiring a supplier to substantiate a claim about a
product : they are selling

41.Which of the following is not an exception to privity?


A. Insurance
B. Agency and trust
C. Employment
D. Property law

42. James is going through a difficult time and does not feel he can confide in anyone. :
Eventually he confides in the priest of his church and asks for support. The priest
forcefully : tells James to turn over all of his money to the church or he will be struck
down by : lightening. The priest does not believe that James will be struck down.
James makes the : payments. Which of the following is most likely legal grounds for
James to avoid the : transaction?
A. Mistake
B. Undue influence
C. Duress
D. Misrepresentation

43.In which High Court decision was it said: “[T]here are two relevant circumstances in
which : a breach of contract by one party may entitle the other to terminate. The first is
where the : obligation [is] essential … The second relevant circumstance is where there
has been a : sufficiently serious breach of a non-essential term …we rest our decision in
the appeal not : upon the ground of breach of an essential obligation, but upon
application of the doctrine : respecting intermediate terms.”
A. Gumland Property Holdings Pty Ltd v Duffy Bros Fruit Market (Campbelltown) Pty
Ltd : (2008) 234 CLR 237
B. Koompahtoo Local Aboriginal Land Council v Sanpine Pty Ltd (2007) 233 CLR 115
C. Ankar Pty Ltd v National Westminster Finance (Australia) Ltd (1987) 162 CLR 549
D. Shevill v Builders Licensing Board (1982) 149 CLR 620

44. Which of the following are sources of illegality?


A. Statute
B. Statute and common law
C. Statute, common law and equity
D. Statute, common law and parties’ subjective views on morality
45.In which situation is termination not available?
A. Where one party has repudiated the contract
B. Where one party has repudiated the contract and the other party has ignored the :
repudiation
C. Where one party still has obligations to perform under the contract
D. Where both parties still have obligations to perform under the contract

46. What is the main difference between the misleading or deceptive conduct provisions in :
the Trade Practices Act 1974 (Cth) and in the Australian Consumer Law?
A. The Australian Consumer Law no longer refers to “trade or commerce”
B. The Australian Consumer Law refers to a body corporate
C. The Australian Consumer Law refers to a corporation
D. The Australian Consumer Law refers to a person

47. With respect to consumer guarantees, which of the following is not correct?
A. It is possible to exclude the consumer guarantees in very limited circumstances
B. Remedies for non-compliance with the consumer guarantees depend on
whether there : was a “major failure” or not
C. With respect to fitness for purpose, a “disclosed purpose” relates to the supplier only
–: the manufacturer may make representations to the consumer, but the consumer
cannot : disclose a purpose to the manufacturer and receive a guarantee in return
D. A manufacturer does not need to provide repair facilities or spare parts after a
certain : period

48. Which of the following is not a type of contract illegal at common law on the grounds
of : public policy?
A. Contracts to oust the jurisdiction of the courts
B. Contracts to commit a tort
C. Contracts prejudicial to the public safety
D. Champertous contracts

49. The term “transparency” in unfair contracts means:


A. It is expressed in reasonably plain language that is readily available to the
affected party
B. It is easily interpretable by the court
C. It is easily interpretable by the industry
D. It is capable of only one meaning

50. In Howe v Teefy (1927) 27 SR (NSW) 301 where a leased racehorse was retaken
three : months into a three-year lease:
A. The court was unable to assess the damages because they were uncertain and there
was : no evidence on which they could be assessed
B. The fact that assessment was difficult did not prevent a court from granting
damages : where the plaintiff had been deprived of something of value
C. Although there was a value attributable to the plaintiff’s loss, it was too remote to be :
recoverable
D. The court was unable to assess the damages because the process was difficult and time
: consuming

51. A written contract allows Joseph, the neighbour of Sonny, to take “as many
strawberries : as he likes each strawberry season”. This contract:
A. Is void for uncertainty
B. Is valid but unenforceable because it is too uncertain
C. Is valid as the parties’ intention is clear
D. Is voidable for uncertainty
E.
52. Which of the following is not an equitable remedy?
A. Injunction
B. Specific performance
C. Restitution
D. Exemplary damages

53. Consumer guarantees as to title are contained within:

A. The “General Protections” chapter of the Australian Consumer Law


B. Section 38 of the Trade Practices Act 1974 (Cth)
C. Chapter 2 of the Australian Consumer Law
D. Sections 51-56 and ss 60-61 of the Australian Consumer Law

54. Which of the following is not one of the four ways a statute may render a contract illegal : set out in
Yango Pastoral Co v First Chicago Australia Ltd (1978) 139 CLR 410? The contract:
A. May require an action that statute forbids
B. May be performed in a prohibited manner
C. May have been made to effect an unlawful purpose
D. May set out an unreasonable restraint

55. Which of the following is not correct with respect to the Australian Consumer Law?

A. It is contained within Schedule 2 to the Competition and Consumer Act 2010 (Cth)
B. It applies to conduct engaged in outside of Australia
C. Owing to the Commonwealth’s limited lawmaking powers under s 51 of the
:Constitution, it applies only to corporations
D. The Australian Consumer Law is applied in each State and Territory
56. Which of the following is not one of the six elements required for a claim of fraudulent :
misrepresentation to succeed?

A. The representation must be fall


B. The representation must have been acted upon by the other party
C. The representation must be one of fact
D. The representation must be in writing (cannot be verbal only)

57. Ben planned the perfect surprise birthday party for his mother: He hired a hall, he hired : a
violinist and he organised catering. Two weeks prior to the event, the violinist called to : cancel.
Ben was so upset at the loss of what he thought would be the highlight of the : evening that he
called the whole event off. Ben lost both the deposit on the hall and the : deposit for the
catering. Which of the following is correct?

A. Ben can sue the violinist for all losses after Ben cancelled the hiring of the hall
B. Ben can recover his hire fees and catering fees as it was not his fault
C. The caterers and hall owners can sue the violinist for losses
D. Ben has a duty to mitigate his losses

58. Sandy is selling her horse, Flossy. Miranda is interested in buying Flossy. Sandy tells : Miranda that
Flossy: : • Is the best little racehorse in Australia: • Has all the registrations, licences and permits required
to race in the current season but : that Miranda should probably get this checked herself: These two
statements are:

A. Mere puff & term


B. Opinion & term
C. Opinion & representation
D. Mere puff & representation

59. Can a person use an official position they hold for their financial advantage?
A. Of course, this is a side benefit
B. Depends on the circumstances
C. The courts are silent on this specific point
D. No because if a contract is involved, this could lead to the promoting of corruption in :
public life

60. Smithy Builders have a contract with Big Bank Pty Ltd which contains the following : clause: “Where
Smithy Builders fails to complete the contract by 7 December, it will pay a : sum of $300,000 in full and final
satisfaction of its liability.” Smithy Builders fails to complete : by 7 December. If Smithy Builders wishes to
avoid paying the sum of $300,000, what must it : prove?

A. That the clause is an unliquidated damages clause


B. That the clause is a penalty clause (to punish the builders)
C. That the clause is a liquidated damages clause
D. That the clause is a genuine pre-estimate of the loss to Big Bank Pty Ltd

61.Who/what are exempt from the provisions of s18 of the Australian Consumer Law?
A. No-one is exempt
B. Corporations
C. Information providers such as media outlets
D. Advertisers

62. Which of the following is not a way to discharge a contract?


A. By performance
B. By operation of law
C. Through breach
D. By agreement
63. Which of the following statements about the parol evidence rule is true.?
A. It applies only to oral (parol) evidence
B. It does not apply to written contracts
C. It does not apply where it can be shown that the written contract was not
intended to : be a complete record of the agreement
D. It applies only to previous drafts of a written contract
64. In Curtis v Chemical Cleaning & Dyeing Co Ltd [1951] 1 KB 805:
A. The exemption clause in the sales agreement for the drycleaning was effective
B. The exemption clause in the sales agreement for the drycleaning was not
sufficiently : explained to the customer
C. The exemption clause in the sales agreement for the drycleaning was signed and :
therefore effective
65.Which of the following is not correct with respect to mitigation of loss?
A. There is a duty on the person claiming the damages to take all reasonable steps to : mitigate
their losses
B. Where loss could have been mitigated and was not, a person cannot claim that loss
C. The burden of proving that losses were mitigated falls upon the party claiming the :
damages ie the defendant
D. The question of whether a person took all reasonable steps to mitigate is one of fact :
dependent upon the particular circumstances
66.Which of the following is not correct with respect to termination by subsequent : agreement?
A. A subsequent simple contract can cancel a contract where one party has completed their
: obligations and the other has not
B. A subsequent agreement may cancel an original contract
C. A subsequent agreement must be valid to rescind an earlier contract
D. A subsequent simple contract can cancel a contract where there is still something to be :
done by each party
67.In the case of restraint of trade with respect to employment:

A. The court does not easily allow parties to contract out of their means of
employment
B. Whatever the parties determine will be accepted by the court
C. There are no restraints on employees unless set out in a contract with their
employer
D. These agreements are construed as strictly as restraints of trade between
purchaser and : vendor of a business
68. Krell v Henry [1903] 2 KB 740 illustrated what point?
A. That death can be regarded as frustration
B. That the parties were entitled to a discharge
C. That the parties must perform regardless of circumstance
D. That the parties are discharged if an event occurs which forms the basis of the
contract, : but which prevents performance at no fault of either party
69.Contingent conditions include:
A. Accord and satisfaction and consideration
B. Conditions subsequent and conditions precedent
C. Warranties and innominate terms
D. Releases and mutual terminations
70. Which of the following is NOT a requirement for an assignment of a debt or other
chose : in action under various State statutes?
A. The assignment is absolute and not by way of charge;
B. Fourteen days’ notice in writing is provided to debtor;
C. the assignment is in writing;
D. If not in writing then notice of the assignment is given by notice to the debtor.
71. The parol evidence rule performs much the same function as:
A. A collateral warranty
B. An oral collateral contract
C. Consideration
D. A merger clause
72. Which of the following is not one of the special relationships in which the
onus of : proving that undue influence was not employed shifts to the denying Party?
A. Parent and child where the child is living independently
B. Solicitor and client
C. Trustee and beneficiary
D. Doctor and patient
73. How may a court view substantial performance of a contract?:
A. The contract is complete, but the uncomplete portion may be allowed for as a
reduction : of the full contract price
B. it will not grant a discharge
C. It will sever the incomplete parts
D. It will treat the contract as breached
74. How is repudiation determined by the court?
A. Subjectively
B. By considering the actions of the repudiating party
C. By viewing the circumstances of the repudiation
D. Objectively
75.How is the Australian Consumer Law structured into the Competition and Consumer Act :
2010 (Cth)?
A. They are separate statutes
B. The Australian Consumer Law is an amendment to the Trade Practices Act 1974 (Cth)
C. The Australian Consumer Law is Sch 2 to the Competition and Consumer Act 2010 (Cth)
D. The Australian Consumer Law is App 3 to the Competition and Consumer Act 2010 (Cth)
76. In considering whether a contract is a standard form contract, which of the
following is : not something that the court must consider?
A. If one of the parties has most of the bargaining power
B. If one party prepared the contract prior to discussing it with the other party
C. If another party was given opportunity to negotiate the terms
D. Whether the price payable takes into account the specifics of the party and the :
transaction
77. In which one of the following cases was it held that for a loss in the contemplation of
the : parties to be recoverable it must be “a serious possibility”, “a real danger”, “liable
to result” : or “not unlikely” to occur?
A. Hadley v Baxendale (1854) 9 Exch 341; 156 ER 145
B. Burns v MAN Automotive (Aust) Pty Ltd (1986) 161 CLR 653
C. Koufos v Czarnikow Ltd [1969] 1 AC 350
D. Tabcorp Holdings Ltd v Bowen Investments Pty Ltd (2009) 236 CLR 272

78. Which of the following is a circumstance where specific performance would be


ordered?
A. Where the subject matter of the contract is personal property that is not rare or
unique
B. Where a famous actor is to work as a Master of Ceremonies at a function
C. Where the subject matter of the contract is an apartment in a complex of 20
apartments
D. Where a non-famous actor is to wear a gorilla suit every day to advertise a product
79. A consumer contract with respect to unfair contract terms does not involve:
A. The supply of goods or services
B. The sale of land
C. An individual who acquires goods or services or land
D. Wholly for personal, domestic or household use or resupply
80.Which of the following is not a valid assignment of a contract?
A. A bankrupt’s estate passing to the Official Trustee in Bankruptcy
B. A contract for personal services
C. A contract under which Sharma and his friend Melanie were to concrete a
driveway : devolving on Melanie after Sharma’s death
D. A contract under which Tom was to receive goods devolving on Tom’s
administrator : after letters of administration were granted
81. Sharma was selling his secondhand vacuum cleaner to Natasha and told Natasha that it :
could “do all the rooms in the house ten times over without the filter needing to be :
changed”. Natasha spent a few weeks shopping for other vacuum cleaners but eventually :
returned and purchased Sharma’s, telling Sharma it was the cheapest she had seen in two
: weeks and she liked the colour. When Natasha took it home she soon discovered that the
: vacuum cleaner could only do one room at a time and then the filter would need
changing. : Natasha cannot rely on Sharma’s statement for breach of contract because:
A. The statement was not put into writing
B. Natasha did not attach any importance to the statement when it was made
C. Natasha should have tested the vacuum cleaner before buying it
D. The statement was a merger clause
82.Which of the following is not correct?
A. The Australian Consumer Law contains provisions on unconscionable conduct
B. The Australian Consumer Law contains provisions on misrepresentation
C. The Australian Consumer Law contains provisions on misrepresentation in relation to : the sale
of land
D. The Australian Consumer does not have any provision for unconscionable conduct
83.The plaintiff in an action under s18 of the Australian Consumer Law is required to prove :
what?
A. the conduct is misleading or deceptive
B. the conduct is misleading or deceptive or likely to mislead or deceive
C. b, above, plus the intention of the defendant
D. The ACCC takes on the issues of proving the defendant’s intentions

84. In Shevill v Builders Licensing Board (1982) 149 CLR 620 at 625, Gibbs CJ listed all but the :
following various ways that a contract may be repudiated:
A. “[I]f one party discharges [themself] from performance by agreeing to a new agreement : that
supersedes the previous one”
B. “[I]f one party renounces his liabilities under it”
C. If one party “shows that [the party] intends to fulfil the contract only in a manner :
substantially inconsistent with [the party’s] obligations and not in any other way”
D. If one party “evinces an intention no longer to be bound by the contract”
85. A divisible contract:
A. Is one that provides expressly that performance is due after the other party has : performed
stages of the contract
B. Is one that provides impliedly that performance is due after the other party has : performed
stages of the contract
C. Is one that provides expressly or impliedly that performance is due after the other party : has
performed stages of the contract
D. Is one that provides that exact performance of the entire contract is required and duties :
under it are not severable
86. Which of the following is the general position of the courts when awarding damages?
A. Where it is possible to place an innocent part in the position he/she would have been in : if the
breach had not occurred
B. Where it is difficult to quantify the loss
C. Where the plaintiff is unable to establish that they have suffered any actual loss
D. Where the damages are unliquidated

87. Unliquidated damages are


A. Non-monetary sums
B. Prohibited by law as they are uncertain and difficult to quantify
C. Damages where the court is to determine the amount
D. Damages where an amount has been fixed in the contract
88.Which of the following sections of the Australian Consumer Law are relevant to :
misrepresentation?
A. Sections 23-25
B. Sections 18 and 29
C. Sections 20-22
D. Sections 52 and 53
89.Where a contract is made in writing, the express terms of that contract:
A. Are to be found in the writing
B. Are generally to be found in the writing
C. Are to be found in the writing unless it is an online agreement, in which case terms and :
conditions may be incorporated from the website of one or both parties
D. Cannot include anything said orally
90.Which of the following is not accurate? Duress involves:
A. Actual or threatened violence
B. Duress must be the only reason for entering into a contract
C. A party coerced into entering into a contract (against their will)
D. Coercion of a person or their immediate family or near relatives

91.Janey takes her fur stole to the dry cleaner. When she returns to collect it, there is a : black
mark in the shape of an iron in the middle of her stole and it is ruined. The dry cleaner : tells
Janey that he is not liable for the damage to her fur stole as there was an exemption : clause
in their agreement. Which of the following is not correct?
A. If the exemption clause was merely contained within a notice on the wall, the dry : cleaner
will be liable for the damage
B. If the exemption clause was on the dry cleaning docket Janey was given and she was told :
“don’t worry about this, it’s just the docket you have to give us when you come to collect :
your stole”, the dry cleaner will not be able to rely on the exemption clause
C. In order for the exemption clause to be effective where it was not in a signed document, : it
had to have been brought to Janey’s attention
D. If the exemption clause was on the dry cleaning docket Janey was given rather than a : formal
written contract, the onus will be on the dry cleaner to prove that Janey was aware it :
contained conditions that would modify the agreement
92.Chris maintains that a document that he and Kathy have signed is a binding contract. : Kathy
maintains that it is merely a receipt and does not include all the terms of their : agreement.
Which of the following statements is correct?
A. If both parties have signed the document then the court will look no further into the :
circumstances
B. Kathy should plead non est factum
C. Kathy will be able to establish the existence of an oral collateral contract even where it is :
inconsistent with the terms of the main binding contract
D. Parol evidence may be admissible
93.Which of the following statements about damages is not correct?
A. Damages may be recoverable for loss of profit
B. The onus of proving losses is on the plaintiff
C. Damages are a common law remedy
D. Damages are awarded to compensate a party for all losses caused by breach of contract

94. Which of the following is not one of the special relationships in which the onus of : proving
that undue influence was not employed shifts to the denying party?
A. Parent and child where the child is living independently
B. Solicitor and client
C. Trustee and beneficiary
D. Doctor and patient

95. A unilateral mistake does not include:


A. Mistake as to the nature of the document signed
B. Mistake as to the identity of the parties
C. Mistake as to the terms of the contract
D. Mistake as to capacity of the parties
96.Which of the following is not a type of termination?
A. Termination by bankruptcy
B. When the parties have fully and exactly performed their obligations to each other under : the
contract
C. When a deed displaces a simple contract
D. Where an event occurs that is not the fault of either party that causes a fundamental : change
to the nature of the contract and the parties obligations and although the contract : covers
that eventuality, it would cause hardship to one party to enforce it

97.Terms may not be implied into a contract by


A. The court
B. Representations
C. Trade usage
D. Custom
98.Which of the following is not a type of contract void at common law?
A. An agreement that offends the Australian Consumer Law (ACL).
B. A contract to oust the jurisdiction of the courts
C. A “marriage brokerage” agreement whereby money is paid in order to procure a : marriage
D. A price maintenance agreement that restrains trade.
99. In Codelfa Construction Pty Ltd v State Rail Authority of New South Wales (1982) 149 CLR :
337:
A. The High Court implied a term in the agreement granting a reasonable extension of time: b
Codelfa was able to prove that the term was necessary to make the contract work
B. Codelfa did not need to prove that the term was necessary to make the contract work : for the
term to be implied as it was sufficient that both parties needed to be rescued from a : difficult
position imposed upon them by the injunction
C. The High Court found that the contract was frustrated
100. In Hadley v Baxendale (1854) 9 Exch 341; 156 ER 145 the court recognized “reasonably :
foreseeable losses” as being:
A. All losses that a plaintiff can prove beyond reasonable doubt
B. Losses that arise naturally from a breach of contract
C. Losses that are actually contemplated by the parties
D. b & c
101. Which of the following is correct with respect to privity of contract?
A. It is unrelated to the doctrine of consideration
B. There are no exceptions to the doctrine of privity; the Trident v McNiece insurance : exception
is not a true exception
C. In circumstances where the doctrine applies, a third party beneficiary under a contract is : able
to acquire rights under it
D. Despite the doctrine, a new owner of land is able to enforce a covenant between the :
previous owner and another party
102. With respect to substantial performance and partial performance:
A. Partial performance is a lower level of performance than substantial performance, and : there
is no deduction of the contract price available where substantial performance has : occurred
B. Substantial performance is where the parties terminate the contract for less than full :
performance
C. Partial performance is where a party voluntarily accepts less than full performance by : the
other party
D. Partial performance is where the court terminates the contract for less than exact :
performance
103. Which of the following is not a situation where frustration would occur
A. Long term serious illness of a person contracted to perform a personal services contract
B. Where the government in exercising its powers has made completion of the contract :
impossible
C. Where a change in the law renders performance of the contract illegal
D. Where the performance of the contract is rendered illegal by the destruction of the : subject
matter of the contract by one of the parties
104. In determining whether a statement has become a term of the contract, the key test :
applied by the courts is:
A. Whether it is in writing
B. Contractual intention
C. The knowledge of the parties
D. The parties’ own beliefs

105. Which of the following are not usually compensated for with payment of damages?
A. Injured feelings/disappointment
B. Nominal loss
C. Loss that is difficult for the court to estimate
D. Loss of profit expected to be received
106. Which of the following is most correct? If an amount of $5,000 is specified in a written :
contract as the agreed genuine pre-estimate of damages in the event the contract is :
breached then this is commonly referred to as:
A. A penalty clause
B. Liquidated damages
C. Punitive damages
D. A mitigation clause
107. Which of the following is NOT a method of termination?
A. Agreement
B. Frustration
C. Breach
D. Rectification
108. Where a contract has not been completed within the specified time
A. If no time limit was specified in the contract, there is nothing the innocent party can do
B. The innocent party will be entitled to terminate the contract
C. The innocent party will be entitled to terminate the contract upon giving notice that time : is
of the essence
D. Only damages will be available no matter how crucial the time limit was to the innocent : party

109. Which of the following is not a requirement of a valid collateral contract?


A. The statement must be promissory
B. The parties must have intended the promise to be binding
C. The promise must be supported by consideration
D. The consideration must be of higher value than the main contract
110. Which of the following constitutes pyramid selling?
A. Jaz is approached by her neighbor to join the sales team of Thames Beauty Products.
: Thames sells its products door-to-door through agents and is a close-knit family
type of : company. Agents that introduce other agents to “the family” are rewarded
with a finders’: fee. It is only $500 to join the Thames family, and Jaz only has to
spend $400 of products in: her first month
B. Jaz starts work at a chemist and notices that the chemist, as retailers often do, puts a :
sale price on a sticker on most of the products in the shop. However, the chemist does
not: put the sale price sticker on top of the original price sticker and when customers
bring the : products to the counter, the chemist tells them that unfortunately there has
been a mistake, : the true price is the higher price
C. Jaz approaches a salesperson on the forecourt to buy the car package she saw :
advertised on television and is told that there were only two cars for sale as a part of
that : deal and there are now none left. The salesperson tells Jaz that she would look
much better : in the convertible model and although there is no deal on that car, it is
only $50,000 more
D. Jaz purchases a painting through a new art gallery that has just opened in town. The :
price is high and Jaz is at first uncertain about the expense, but the art dealer promises
Jaz a : commission on sales to any of Jaz’s friends who she sends along to the gallery. Jaz
has a lot : of art collector friends so is sure that she will receive some commission from
the gallery in : the near future.
111. James, a chef, tells Laura, who is looking to buy his antique lounge chair, that the chair is :
of Spanish origin. Three weeks later a written contract is drafted that makes no mention of :
the chair’s origin. The statement that the chair is of Spanish origin is most likely to be a/an::
a Mere puff
A. Representation
B. Opinion
C. Term
112. If the court held the sum the parties inserted into their contract ($100,000) to be a :
penalty, and the actual loss determined by a court was $30,000:
A. The plaintiff would be able to recover nothing as it was a penalty
B. The plaintiff would be able to recover $30,000 only
C. The plaintiff would be able to recover $100,000 as this was in the contract
D. The plaintiff would be able to recover$70,000 (the difference between the amount in : the
contract and the actual loss)
113. Which of the following is correct?
A. It is prohibited to offer free gifts with purchase
B. It is prohibited to offer goods at a special price, not have a sufficient amount of those : goods
available and then attempt to upsell to customers who are drawn to the shop for the : special
price deal
C. It is not prohibited for a supplier to take a customer’s money when the supplier believes : they
won’t be able to supply the service to the customer within a reasonable time
D. It is not prohibited for a supplier to take a customer’s money when the supplier believes : they
won’t be able to supply the service to the customer within a reasonable time
114. The definition of “consumer” is limited by what amount?
A. $40,000
B. Recently increased to $100,000
C. There is no limit
D. A negotiated amount
115. Coulls v Bagot’s Executor & Trustee Co Ltd (1967) 119 CLR 460; Beswick v Beswick [1968]
: AC 58 and Tweddle v Atkinson (1861) 1 B & S 393 all relate to the doctrine of:
A. Frustration
B. Consideration
C. Privity
D. Innominate terms
116. A party in breach of contract is liable to pay damages for:
A. Losses occurring in the usual course of things from the breach
B. All losses whether contemplated or not
C. Remote losses only
D. Losses that should have been mitigated
117. Matt sells his car to Tabitha, who purchases it on the condition that she can continue to:
keep it in his garage at no cost for the next six months. Tabitha pays in full. Two months:
later, Matt tells Tabitha she will have to remove her car from his garage as he has bought a:
new car and wants to keep it in the garage. Which of the following statements is correct?
A. If Tabitha agrees to remove her car, the contract has been terminated by partial: performance
B. The condition that Tabitha is allowed to keep her car in Matt’s garage is a condition: precedent
C. The condition that Tabitha is allowed to keep her car in Matt’s garage is a condition:
subsequent
D. If Tabitha promises to remove her car, Matt will not be able to enforce the promise: unless
the promise is supported by consideration or made under seal
118. A term that is vital to a contract is known as
A. A key term
B. A condition
C. A warranty
D. An innominate term
119. Which of the following statements is not correct?
A. A condition is a major term of the contract; a breach renders the contract substantially :
different from the agreed term
B. A breach of a warranty entitles the innocent party to be compensated with damages and :
termination of the contract
C. A breach of a condition entitles the innocent party to be compensated with damages
D. A warranty is a minor term of the contract; a breach renders the contract insubstantially :
different from the agreed term
120. When are nominal damages awarded?
A. The plaintiff has not mitigated his loss
B. The damages are so great the court has to estimate them
C. A legal right has been infringed but there is no actual loss
D. The plaintiff has calculated his losses incorrectly
121. Which of the following is a way that a contract may be terminated by operation of law?
A. Where a deed on a slightly different subject matter displaces a simple contract
B. Where a party to a delivery of goods contract becomes bankrupt
C. Where a deed where one of the parties is different displaces a simple contract
D. Where a simple contract merges into a deed and the security is of the same value in both
122. Which of the following is not a remedy for unconscionable conduct?
A. An injunction
B. Ordinary damages
C. A criminal prosecution
D. A pecuniary penalty
123. In which case was it that held that where an exemption clause is in a non-contractual :
document, the clause must be brought to the other party’s attention for it to be effective?
124. Which of the following statements is not correct? With respect to legality of object:
A. Illegality is not confined to criminal illegality
B. A contract may be illegal where it is made to effect a purpose that a statute renders : unlawful
C. A contract may be illegal where its object is impliedly prohibited by statute
D. Illegal objectives will not affect a contract provided both parties have agreed to it
125. Which of the following is not correct? With respect to s 18 of the Australian Consumer :
Law, silence:
A. May constitute misleading or deceptive conduct where a statement is literally true but is :
misleading without further qualification
B. May constitute misleading or deceptive conduct where there is a “reasonable : expectation”
that disclosure will be made
C. Is not relevant in considering the contravention
D. May constitute misleading or deceptive conduct where there is failure to disclose a : change in
circumstances
126. In which case did the court recognise that damages for disappointment and distress : could
be available subject to certain limitations?
A. Burns v MAN Automotive (Aust) Pty Ltd (1986) 161 CLR 653
B. Placer (Granny Smith) Pty Ltd v Thiess Contractors Pty Ltd (2003) 196 ALR 257
C. Howe v Teefy (1927) 27 SR (NSW) 301
D. Baltic Shipping Co v Dillon (1993) 176 CLR 344
127. In which case did the court find that passing off had been engaged in and the misleading :
and deceptive conduct provisions had been contravened?
A. Apand Pty Ltd v The Kettle Chip Co Pty Ltd (1994) 52 FCR 474
B. McWilliam’s Wines Pty Ltd v McDonald’s System of Australia Pty Ltd (1980) 33 ALR 394
C. Parkdale Custom Built Furniture Pty Ltd v Puxu Pty Ltd (1982) 149 CLR 191
D. eBay International AG v Creative Festival Entertainment Pty Ltd (2006) 170 FCR 450
128. A liquidated damages clause in a written contract:
A. States that the parties have agreed to leave the calculation of damages to the court
B. Sets out the price of the contract
C. Sets out the agreed amount payable to the innocent party for terminating the contract
D. Sets out the amount agreed payable to the innocent party if there is a breach of contract
129. In which of the following cases would a plea of frustration not succeed?
A. Where the event should have been foreseen
B. Where no specific provision was made for the event in the contract
C. Where performance is still possible but would be pointless
D. Where performance has become fundamentally different from what was contemplated : by
the parties
130. In Hong Kong Fir Shipping Co Ltd v Kawasaki Kisen Kaisha Ltd [1962] 2 QB 26, the court :
recognised which of the following terms which later became part of Australian law in :
Koompahtoo Local Aboriginal Land Council v Sanpine Pty Ltd (2007) 233 CLR 115?
A. Transitional term
B. Warranty
C. Innominate term
D. Intervening term
131. Which of the following is not correct? A condition:
A. If breached, entitles an innocent party to damages or specific performance under the :
common law
B. Is an essential term of the contract
C. Goes to the root of the subject matter of the contract
D. If breached, entitles an innocent party to rescind the contract
132. A party repudiates contract when
A. The party is able but unwilling to perform their obligations under it
B. The party decides to terminate the contract
C. The party breaches a representation
D. The party breaches a warranty
133. In Victoria Laundry (Windsor) Ltd v Newman Industries Ltd [1949] 2 KB 528:
A. All loss related to the delay in installation of the boiler was recoverable because the :
defendant should have known that delay would cause loss to a commercial operation
B. All loss related to the delay in installation of the boiler was recoverable because the : plaintiff
had made known to the defendant that the plaintiff was “most anxious” that the : boiler was
installed on time
C. Only the ordinary loss related to the delay in installation of the boiler was recoverable :
because the defendant was not made aware that additional lucrative contracts had been :
negotiated
D. No damages were recoverable as the defendant could not be expected to foresee that : late
delivery would cause any loss
134. Which of the following statements is incorrect?
A. Innominate terms are neither conditions nor warranties
B. Intermediate terms are the same as innominate terms
C. The remedies available for breach of an intermediate term depend on their classification : as
an intermediate term, not the effect of the particular breach
D. Intermediate terms are terms that are capable of being breached in minor and major : ways
135. Which of the following is incorrect? Repudiation:
A. Is where a party demonstrates an unwillingness to perform their obligations
B. Is the same as termination; the contract is at an end
C. Gives an innocent party the right to terminate the contract
D. Is where a party demonstrates an inability to perform their obligations
136. Which of the following is not an unfair practice under the Australian Consumer Law?
A. Where a free gift is offered with the purchase an electric toothbrush and the cost of the : “gift”
toothpaste is added to the price of the toothbrush
B. Where a television advertisement for a chocolate bar shows teddy bears working the :
manufacturing lines
C. Where a brochure advertising a beauty therapy treatment consisting of time in an : oxygen
tank lists the treatment as lasting two hours, when in fact the treatment lasts just : under one
hour
D. Where a shop assistant takes money from a customer knowing that the shirt the : customer
wants is sold out, but hoping that after a few days when the customer is told that : there are
no more shirts they will purchase something else from the shop

137. Marnie loans Bill $1000 so that Bill can pay a local politician to ensure his tender will be :
the winning tender. The contract between Marnie and Bill is:
A. A simple loan contract that is enforceable
B. A contract voidable at Marnie’s option
C. An illegal contract
D. A contract voidable at Bill’s option
138. In ACCC v Turi Foods Pty Ltd [2012] FCA 19:
A. The statement that the chickens were “free to roam” was misleading as there were too : many
chickens per square metre for the chickens to easily move
B. The advertising was found to be misleading as the chickens were not lounging on deck : chairs
C. The court had no power to issue a fine
D. The court ordered Turi to make a disclosure order
139. Which of the following is not an element requiring satisfaction before the court will : imply
a term into a contract?
A. It must be reasonable
B. It must be capable of clear expression
C. It must be equitable
D. It must complement an express term of the contract
140. Century Dragon Pty Ltd is contracted to export 1000 beach balls to Minerva Beach Surf :
Club by 1 January, with payment due by 1 April. Which of the following is correct?
A. The time of delivery is likely to be considered an essential term.
B. The time of payment is likely to be considered an essential term
C. The time of delivery and the time of payment are likely to be considered essential terms
D. Neither the time of delivery nor the time of payment are likely to be considered essential :
terms unless time is expressly stated to be of the essence
141. Which of the following is NOT correct?
A. The ACCC can seek both criminal and pecuniary penalties for breaches of the unfair : practices
provisions of the Australian Consumer Law?
B. The new maximum penalties may vary depending on turnover of a corporation
C. Injunctions can be sought as orders
D. A criminal penalty can be sought for a breach of s 18?

142. Where one party believes the contract refers to the Bonny Lass, a freight ship based out :
of Norway, and the other party believes the contract refers to the Bonny Lass, a freight ship :
based out of Hong Kong, this is an example of a:
A. Mistake of law
B. Unilateral mistake
C. Common mistake
D. Mutual mistake
143. Which of the following is not true with respect to misleading or deceptive conduct?
A. The plaintiff has to prove that conduct was misleading or deceptive
B. The plaintiff has to prove the causal link between the conduct and the loss
C. A person acting as a ”mere conduit” in passing on representations may also be liable
D. A corporation is not liable where it is clear that the corporation is not the source of the :
information
144. Which of the following matters is least likely to contribute to a finding of unconscionable :
conduct in connection with goods or services?
A. Where conditions imposed on a customer were not necessary for the protection of the :
supplier’s interests
B. Where the supplier makes a commercial decision to breach the contract knowing that : the
customer will seek a legal remedy for the breach
C. Where the supplier acted in bad faith
D. Where unfair tactics were used against the customer
145. Janine has sold her business to Simone. Simone insists on inserting a clause in the :
agreement that Janine cannot open a similar business within 10 kilometres of her former :
business, for the next two years. Which of the following is not true?
A. This clause is a restraint of trade
B. The court will always uphold restraint of trade provisions in contracts for sale of : business
C. Where a restraint of trade is unreasonable, the whole or parts of the contract will be void
D. It is impossible to say whether a court would deem this restraint of trade clause : reasonable
without further details
146. Which of the following is not a restraint of trade?
A. A price maintenance agreement
B. A contract restraining Laura from working as a welder within Ballarat for two years after : her
termination with Welders R Us
C. An agreement between John and Liz that John will not open a brew shop within 2km of : the
brew shop he just sold to Liz
D. An agreement by which Steve agrees to repay his daughter’s loan debt in return for the :
person who advanced the loan to his daughter agreeing not to report Steve’s daughter to :
the police for fraud
147. Parol evidence may not be admitted in which of the following circumstances?
A. To evidence an oral collateral contract
B. To evidence the identity of the parties to the contract
C. To contradict an unambiguous term
D. To explain a local custom
148. Which of the following is not correct? An injunction:
A. Is an order of the court
B. Restrains a person from doing something
C. Is sometimes granted to compel a party to do something they would not have been : ordered
to do by specific performance
D. Is an equitable remedy
149. When it was said in Re Hall & Barker [1878] 9 Ch D 538, “if a shoemaker agrees to make : a
pair of shoes, he cannot offer you one shoe and ask you to pay one half the price”, this is :
referring to:
A. The court’s tendency to regard contracts as divisible
B. The court’s tendency to regard contracts as severable
C. The court’s tendency to regard contracts as entire
D. The court’s tendency to require substantial performance
150. Which of the following guarantees applies to a sale by auction?
A. Guarantee as to undisturbed possession
B. Guarantee as to express warranties
C. Guarantee as to acceptable quality
D. Guarantee as to title
151. When a contract is made orally, the question of what the express terms are:
A. Is in most cases unascertainable
B. Depends on what a reasonable person would have said
C. Is a question of fact
D. Is not applicable; oral contracts only contain implied terms

152. Which of the following constitutes bait advertising?


A. Jaz is approached by her neighbor to join the sales team of Thames Beauty Products. :
Thames sells its products door-to-door through agents and is a close-knit family type
of : company. Agents that introduce other agents to “the family” are rewarded with a
finders’: fee. It is only $500 to join the Thames family, and Jaz only has to spend $400
of products in: her first month
B. Jaz starts work at a chemist and notices that the chemist, as retailers often do, puts a :
sale price on a sticker on most of the products in the shop. However, the chemist does
not: put the sale price sticker on top of the original price sticker and when customers
bring the : products to the counter, the chemist tells them that unfortunately there has
been a mistake, : the true price is the higher price
C. Jaz approaches a salesperson on the forecourt to buy the car package she saw :
advertised on television and is told that there were only two cars for sale as a part of
that : deal and there are now none left. The salesperson tells Jaz that she would look
much better : in the convertible model and although there is no deal on that car, it is
only $50,000 more
D. Jaz purchases a painting through a new art gallery that has just opened in town. The :
price is high and Jaz is at first uncertain about the expense, but the art dealer promises
Jaz a : commission on sales to any of Jaz’s friends who she sends along to the gallery. Jaz
has a lot : of art collector friends so is sure that she will receive some commission from
the gallery in : the near future.
153. Contingent conditions include:
A. Accord and satisfaction and consideration
B. Conditions subsequent and conditions precedent
C. Warranties and innominate terms
D. Releases and mutual terminations
Quiz 3
1. To claim damages a causal link must be established between what two things?
A. Between the defendant’s conduct and the actual monetary value of the
loss/damage
B. The conduct and initiating court proceedings
C. The fact that the plaintiff suffered loss or damage is sufficient. Nothing else is
required
D. The defendants breach and the plaintiff’s injury
2. The “neighbour principle” is often viewed as:
A. An objective test
B. A test of the reasonable person
C. A test of the foreseeability of harm
D. An identity test
3. In Tame v State of New South Wales (2002) 211 CLR 317 where Tame was given a false :
blood alcohol reading:
A. Whether the police officer was found to have owed Tame a duty of care was
entirely a : question of community standards
B. The psychotic depressive illness Tame developed was found to flow from the
breach of : duty of the police sergeant because it was entirely caused by his actions
that could have : been avoided had he exercised sufficient care
C. Part of the test of reasonable foreseeability is a question of fact
D. Because of the “eggshell skull” principle, pre-existing knowledge of Tame’s
susceptibility : to suffering nervous shock was not required to prove a breach
4. Which of the following is not correct?
A. If damages are too remote they will not be recoverable
B. The remoteness test will be satisfied where the damage suffered is of the same type
or : kind as foreseeable damage
C. In order for damage to not be too remote, it must be reasonably foreseeable
D. In The Wagon Mound No 1 and The Wagon Mound No 2, where there was an :
unfortunate combination of an oil spill, welding sparks and floating cotton waste, the
loss : was found to be not reasonably foreseeable in the circumstances
5. Which of the following is not a provision of the Wrongs Act 1958 (Vic) relating to:
professional liability?
A. If there are differing peer professional opinions across Australia, the court may
accept : just one of those opinions
B. The court must rely on peer professional opinion even where it considers that
opinion : irrational
C. If there are differing peer professional opinions across Australia, the court may
accept all : of those opinions
D. Peer professional opinion can be considered widely accepted even where it is not :
universally accepted
6. Why was the plaintiff successful in Overseas Tankship (UK) Ltd v Miller
Steamship Co Pty : Ltd (The Wagon Mound No 2) [1967] AC 617 (PC) when another
plaintiff failed in the earlier : related Wagon Mound case?

A. The plaintiffs in the second had better legal representation


B. The legal situation had changed between the two cases
C. The court was more disposed to think about the consequences of the damage suffered
by : so many as a result of the oil spill
D. In the first case, the plaintiffs failed to show that a reasonable man would have
foreseen : the risk of damage from the oil spill
7. Nicola drives the forklift at her place of work, Rooze’s Roofing. Nicola always leaves
the : forklift in a certain place where she has been told to leave it, with the forks up
off the : ground. One afternoon a customer who is collecting goods from the
workshop reverses his : car into the forks on the forklift. He is injured and his car is
damaged.: What is the principle that would make Nicola’s employer liable for her
actions?
A. Contributory negligence
B. Strict liability
C. Vicarious liability
D. Voluntary assumption of risk
8. Where a person gives advice, that advice is relied upon and the advice is incorrect,
the : person giving the advice may be liable in negligence. Which of the following is
incorrect?
A. The “special relationship” described in Mutual Life & Citizens’ Assurance Co Ltd v Evatt
: (1968) 122 CLR 556 has been accepted as the test for determining the existence of a
duty of : care with respect to negligent misstatements
B. The test in Mutual Life & Citizens’ Assurance Co Ltd v Evatt (1968) 122 CLR 556 was :
approved in San Sebastian Pty Ltd v Minister Administering Environmental Planning &:
Assessment Act 1979 (1986) 162 CLR 340
C. The duty of care only arises where the “special relationship” involves a person
seeking : advice from the advice-giver, not where the advice was given unrequested
and merely : accepted
D. The advice-giver does not need to be in the business of giving advice
9. Which of the following is not correct? The reform to the law of negligence that
took : place in Australia in the early 2000s:
A. Limits the scope of potential liability for negligence
B. Does not apply to claims in contract law
C. Was enacted in all Australian states and territories
D. Covers personal injury
10. In Australian Safeway Stores v Zaluzna (1987) 162 CLR 479:
A. The respondent was a lawful entrant upon the land of the respondent, establishing a :
relationship between them and there was therefore a duty of care owed by the
appellant to : avoid a foreseeable risk of injury
B. The respondent was limited by his particular status as entrant without specific
consent of : the appellant
C. The respondent contributed to the accident thus limiting damages
D. There was no liability.

QUIZ 4
1. The incorrect statement?
A. Partners are not bound when another partner misappropriates monies
B. If the giving of investment advice is within the scope of the firm’s business then all
partners : are jointly and severally liable for advice which is not in a client’s best
interests
C. the client should seek a personal indemnity from a partner
D. special skill is required to bind all partners
2. Which of the below statements is incorrect?
A. a written and signed notice of a partner’s intention to dissolve the partnership is
required, : where no fixed term has been agreed upon for the duration of the
partnership
B. a continuing guarantee given by or to a partnership is revoked as to future
transactions if there is : a change in the constitution of the partnership
C. where a partnership continues after a fixed term has expired, the rights and
duties of the partners : remain the same, but the partnership becomes a
partnership at will
D. partnership property must be used exclusively for the purposes of the
partnership and in the : manner set out in the partnership agreement
3. Which legislation controls the actual registration of the firm name?
A. Corporations Act 2001 (Cth)
B. Partnership Act
C. Statute of Frauds 1677
D. Business Names Registration Act 2011 (Cth)
4. Which of the following statements regarding partnerships is incorrect?
A. The second element of a partnership is carrying out a business in common
B. To be a partnership there must be a mutuality of rights and obligations
C. Each partner must take an active part in the direction and management of the firm
D. In Degiorgio v Dunn [2004] NSWSC 767 it was held that there was no
partnership because the : business was not run “in common”
5. Which of the following statements regarding dissolution of partnership is
incorrect?
A. In the event of dissolution, losses must be met first out of profits, followed by
capital, then by : partners in proportion to their share of profits
B. A firm’s assets as contributed by partners to make up capital losses need not be
applied to pay : the firm’s debts/liabilities to non-partners
C. After dissolution, each partner is entitled to advances and residue by the firm
D. After the dissolution, each partner’s authority to bind the firm continues, so far as
necessary for : winding up partnership affairs and completing unfinished transactions
6. Which is incorrect statement?
A. If a member of a firm of solicitors acting for a vendor in a sale absconds with the
deposit, : his/her partners are not liable to refund the money
B. In Lloyd v Grace, Smith & Co [1912] AC 716, the firm was held responsible for the
fraud committed : by a managing clerk of a firm, who misappropriated property while
acting within the scope of his : authority
C. In SJ Mackie Pty Ltd v Dalziell Medical Practice Pty Ltd [1989] 2 Qd it was held that the
transfer of a : share to a non-partner breaks the continuity of the firm, constituting a
new firm/partnership of the : remaining former partners and the new member
D. Partnership agreements can contain provisions to enable the transition from one firm
to another : to be effected without the disruption of a formal winding up.
7. Which of the following statements is not correct?
A. as an agent, a partner is able to bind the other partners and, as principal, be bound
by the actions : of the other partners
B. in New South Wales, Victoria, Queensland, South Australia, Western Australia and:
Tasmania provision for limited partnerships is made in the Partnership Act
C. incorporated limited partnerships have been introduced in all Australian States and
Territories
D. the Partnership Act provides that the rules of the common law and equity are to
continue in force : except insofar as they are inconsistent with the Act
8. Find one incorrect statement?
A. In the absence of special statutory provision, although each partner is liable with the
others for : the whole of the debts of the firm, their liability is only joint
B. A creditor can bring only one action against members of a partnership and any partner
can insist : that the action be stayed until all other partners are joined as parties.
C. A person admitted into an existing firm, liability may be incurred where it is
specially agreed upon
D. A creditor can enforce liability against an incoming partner whether or not
he/she is a party to : the contract
9. What constitutes an exception to the usual number of a partnership being 20 partners?
A. If one of the partners is married-his/her husband or wife is automatically a
partner
B. The exception relates only to limited partnerships
C. No more than 20 are allowed
D. The Corporations Regulations 2001 (Cth) provide greater numbers in certain
specified : professions
10. Which statement regarding an outsize partnership is incorrect?
A. it has more than 20 partners
B. its partnership agreement is invalid
C. it is liable to a criminal penalty ($500)
D. its agreement does not affect the enforceability of contracts or other
arrangements made
11. Which finding is correct?: In Popat v Schonchhatra (1997) 3 All ER 800, the Court
decided:
A. Popat was entitled to half the profits on the sale of the business and to a share of the
profits : that had accrued after the dissolution of the partnership, but before the final
settlement of : accounts.
B. Popat was not entitled to any profits on the sale of the business, as there was no
partnership : agreement and he was further not entitled to a share of the profits
accrued after the dissolution of : the partnership, but before the final settlement of
accounts.
C. Popat was entitled to half the profits on the sale of the business, but not to a share of
the profits : that had accrued after the dissolution of the partnership, but before the
final settlement of : accounts.
D. Popat was not entitled to half the profits on the sale of the business, but he was
entitled to a : share of the profits that had accrued after the dissolution of the
partnership but before the final : settlement of accounts.
12. Which of the following statements is incorrect?
A. a separate legal entity can commit civil wrongs and engage in criminal conduct
B. a public company has serious compliance obligations
C. a partnership must be in writing
D. a separate legal entity is entitled to own property, pay tax and enter into
contracts
13. Which of the below does not apply to the Partnership Act?
A. The rules in the Partnership Acts assist in determining whether a business is being
carried on in : common
B. The partnership under the Act must be registered with ASIC
C. Each rule states a negative: i.e. that a certain fact does not of itself create a
partnership, but is : only indicative of a partnership
D. Joint or part ownership or joint tenancy, or tenancy in common whether or not the :
owners/tenants share the profits, does not of itself create a partnership as to anything
so : held/owned
14. Which of the following statements regarding the written law is incorrect?
A. Under s 115 of the Corporations Act 2001 (Cth), the maximum number of
persons who may form a : partnership for the acquisition of gain is 20
B. Under the Corporations Regulations 2001 (Cth), reg 2A.1.01, partnerships of more
than 20 : partners may be formed for certain professions/callings
C. The Corporations Regulations 2001 (Cth), reg 2A.1.01 sets a maxima of partners for
medical and : legal practitioners, veterinary surgeons, patent and trademark
attorneys, sharebrokers and : stockbrokers and pharmaceutical chemists
D. The Corporations Regulations 2001 (Cth), reg 2A.1.01 does not set a maxima of
partners for : architects and accountants
15. Which court finding is incorrect?
A. In Mercantile Credit Co Ltd v Garrod [1962] 3 All ER 1103, damages were not
recovered even : though, from the plaintiff’s perspective, the sale of the car was
within the usual course of : business.
B. In Goldberg v Jenkins (1889) 15 VLR 36, the firm was not bound to the transaction
because : borrowing money on behalf of the firm at over 60% interest when comparable
rates were between : 6% and 10% was beyond ‘the usual
way’.
C. In Construction Engineering Pty Ltd v Hexyl Pty Ltd (1985) 155 CLR 541, Hexyl was not
liable : because the construction contract appeared to be between
Construction Engineering and Tembel
D. If judgment is obtained against one or more partners of a firm, no action may be taken
against the : other partners, even if satisfaction cannot be obtained from the partner(s)
sued
16. Cribb v Korn (1911) 12 CLR 205 established:
A. Joint ownership creates a partnership
B. A joint tenancy is just another name for a partnership
C. The sharing of joint returns does not in itself create a partnership
D. The statutory rules can be excluded by agreement
17. The incorrect statement is?
A. A partner has express actual and implied actual authority to engage in certain
activities with third: parties
B. A partner cannot pledge/sell partnership property, incur and pay debts on partnership :
accounts or hire employees
C. Contravention of an agreement to restrict a partner’s authority is not binding on the
firm if notice : of the agreement has been given
D. In Polkinghorne v Holland (1934) 51 CLR 143, the firm was held liable because Holland
provided : the advice in his role as solicitor; thus in the ordinary course of the firm’s
business
18. Regarding partnerships, which of the following statements is incorrect?
A. Partnerships are created with a view to profit, so partners must make a profit: b A partner does not
have to have a direct claim to a share of the profits
B. Associations and charities are not partnerships, as profits come from ancillary
business activities : and are reinvested, not distributed as dividends to their
members
C. The Partnership Acts do not govern members of a corporation incorporated under the
provisions : of the Corporations Act 2001 (Cth), a special Act of Parliament, or Royal
Charter
19. Which of the below statement is incorrect?
A. Partnership property is liable to be seized for the private (personal) debt of a partner
and : made liable on a judgment against the partnership
B. A creditor who has obtained judgment in respect of the separate debt of a partner may
obtain an : order charging that partner’s interest in the partnership property and profits
with the amount of the : debt and interest.
C. A creditor who has obtained judgment in respect of the separate debt of a partner may
obtain by : an order the appointment of a receiver of that partner’s share of profits and
of any other money : which may be coming to the partner in respect of the partnership.
D. The two sources of law relevant when considering how partners bind their partners
when dealing : with third parties are the common law (including equity) of agency and
the Partnership Act. The : common law complements the provisions of the
Partnership Act in relation to the authority that an : agent has.
20. Which statement is incorrect?
A. carrying on a business implies repetition (Smith v Anderson (1880) 15 Ch D 247)
B. partnership may be formed in order to undertake a single business transaction
C. in Khan v Miah [2000] 1 WLR 2123 it was held that work, such as finding, acquiring and
fitting out : a shop/restaurant, is undertaken with a view to profit
D. in Keith Spicer Ltd v Mansell [1970] 1 All ER 462 it was held that ordering goods and
opening a : joint bank account in contemplation of a business are insufficient for a
partnership
21. Find incorrect statement
A. A term often included in a partnership agreement that allows other partners to
purchase a : retiring/deceased partner’s interest at an agreed valuation avoids the
disruption of a formal winding
B. A partnership cannot be dissolved because the business is carried on at a loss
C. Partnerships can be dissolved because of mutual incompatibility, making it
impossible for partners : to carry on a business
D. On dissolution, partnership property can be applied towards the payment of
partnership : liabilities/debts and any surplus can be distributed among the
partners

22. Which below is incorrect? A firm is not bound by the acts of a partner if:
A. the act is not of the usual business of the kind carried on by the firm
B. the partner exceeds his/her actual authority in the particular matter
C. the person with whom the partner is dealing knows that he/she has exceeded
his/her authority
D. the person with whom the partner is dealing knows or believes him/her to be a partner

23. Legislation in Victoria: What Act regulates partnerships in Victoria?


A. Partnership Act 1892
B. Partnership Act 1958 (Vic)
C. Corporations Act 2001 (Cth), s 20
D. Competition & Consumer Act 2010 (Cth)

24. Which statement regarding persons of unsound mind and minors is incorrect?
A. a partner of unsound mind is capable of binding the firm and of being bound by
co-partners, : unless proven that he/she was of unsound mind when the partnership was
entered into and the : other partners knew this
B. if a partner who is a minor enters into a contract with a third party on behalf of the firm, the :
minor is liable as far as private assets are concerned
C. a creditor who has obtained judgment against the firm may not seize the minor’s
separate : property
D. a minor will become liable as an ordinary partner when he/she attains majority the
partnership is : not repudiated within a reasonable time

25. Which below statements is incorrect?

A. Partners share the profits in a partnership


B. In a partnership, partners have limited liability for the business debts
C. A partner is also an agent of the partnership
D. Agreement may be implied in a partnership
26. Which statement regarding dissolution of a partnership is incorrect?
A. Partners are not entitled to have partnership property applied towards the
payment of : partnership liabilities
B. After payment of the firm’s liabilities, partners do not have the right to have surplus assets :
applied in payment of what may be due to the partners respectively
C. On the termination of the partnership, any partner can apply to the court for a decree
to dissolve : the partnership and appoint a receiver to wind up the firm’s business/affairs
D. Should a sequestration order be made against a partner, a creditor of the firm cannot
receive a : dividend out of the bankrupt’s separate property until all separate
creditors of the bankrupt partner : have been paid in full.

27. Which court decision regarding fiduciary duties is incorrect?


A. In Chan v Zacharia (1984) 154 CLR 178, the court decided that that the fiduciary relationship :
continued until the partnership had finally been wound up.
B. In United Dominion Corp Ltd v Brian Pty Ltd (1985) 157 CLR 1, the Court found that a fiduciary :
relationship, with attendant fiduciary obligations ordinarily exist between prospective
partners who : have embarked upon the conduct of the partnership business or venture
before the precise terms of : any partnership agreement have been settled
C. In Popat v Schonchhatra (1997) 3 ALLER 800 in the absence of a statement to the contrary, :
partners are entitled to an equal share of profits
D. in Harvey v Harvey (1970) 120 CLR 529, the Court held that if a partner receives an additional :
annual fee to be “on call” for that client, the fee belongs to the partnership.

28. Which of the following statements regarding limited partners is incorrect?


A. A limited partner may assign his/her share in the partnership with the consent of the
general : partners
B. The consent of a limited partner is not required to admit a person as a partner
C. If a limited partner has suffered his/her share of the partnership property to be charged for a :
separate debt, the other partners are entitled to dissolve the partnership
D. A limited partner is not entitled to dissolve the partnership by notice.

29. Which legislation regulates partnerships in NSW?


A. Partnership Act 1892 (NSW)*
B. Competition & Consumer Act 2001 (Cth)
C. Partnership Act 1958 (Vic)
D. Partnership Act 1895
30. Find a statement that is incorrect
A. A retiring partner should give specific notice of their retirement to persons with whom
the firm : has had dealings to avoid being made liable for debts incurred after retirement.
B. Persons who represents themselves, or knowingly allow themselves to be represented,
as a : partner, are liable as a partner to anyone who has on the faith of such representation
given credit to : the firm
C. A wrongful act/omission includes breach of contract or of fiduciary duty, negligent :
misrepresentation and misleading or deceptive conduct
D. Innocent partners are liable for wrongful acts of other partners made in the
ordinary conduct of : business

31. Which statement regarding partnership is incorrect?


A. A partnerships (or firm) is a way of gathering resources or expertise for major
projects
B. Partnerships do not avoid taking on the formality and expense of an incorporated
company
C. Partners in a partnership complement each other with their skills and bring in new capital
and : broader funding options

32. How would a partner pledge the firm’s credit?


A. By deed
B. Only for a purpose directly connected to the firm’s business but must have express
authority : to do so
C. By agreement with other partners
D. By agreement with a majority of partners plus an appropriate credit application

33. Which statements is not correct?


A. if the agreement is that a person should be paid a fixed sum by the firm, he/she may be a
partner : and jointly liable to creditors of the firm
B. the rights and obligations of partners to each other arise from the partnership
agreement, the : statute and the equitable concept of the fiduciary
C. a partnership agreement must be in writing; it cannot be reached orally or by a
course of : conduct
D. the Partnership Act determines partners’ rights, duties and interests, if not
included in the : partnership agreement

34. Which of the statements below is incorrect?


A. A partner’s express authority must be in writing
B. A partner’s authority to buy/sell goods of a kind necessary for or usually employed in
the business : on account of the firm is implied
C. A partner cannot pledge the firm’s credit for purposes not connected with its
ordinary course of : business without express authority
D. A partner cannot pledge the firm’s assets for private debts without express
authority

35. Which of the following is NOT a ground for termination of a partnership?


A. Court order
B. Partner leaves the jurisdiction
C. Expiry of a fixed term
D. Bankruptcy of a partner

36. In Hadley v Baxendale (1854) 9 Exch 341; 156 ER 145 the court recognized “reasonably
: foreseeable losses” as being:
A. All losses that a plaintiff can prove beyond reasonable doubt
B. Losses that arise naturally from a breach of contract
C. Losses that are actually contemplated by the parties
D. b & c
37. Which statements is incorrect?
A. Like most partners, sole traders have unlimited personal liability of the business/firm
B. More complex business organisations are unsuitable for larger businesses
C. Upon incorporation, a corporation becomes a separate legal entity
D. Diverse ownership requires proper management and expertise in numerous areas

38. Which is incorrect statements regarding limited partnerships?

A. A limited partner must not take part in the management of the business and does
not have power : to bind the firm
B. If a limited partner partakes in the management of the business, he/she is liable as
a general : partner

C. A limited partner does not have the right to inspect the books of the firm

D. Any differences arising as to ordinary matters connected with the firm’s business
are to be : decided by a majority of the general partners
lOMoARcPSD|13289864

Quiz-BL-TỔNG-HỢP-GR - QUIZ

Introduction to Business Law (Western Sydney University)

StuDocu is not sponsored or endorsed by any college or university


Downloaded by huu nguyen (nguyenminhhuu75@gmail.com)
lOMoARcPSD|13289864

QUIZ 1:
Reporting obligations: What is a binding precedent?
A decision of another court that is of persuasive authority
With respect to Australia, it is a decision of the UK Supreme Court
A decision of a court that binds judges in a lower court in the same court hierarchy
A decision of a different court on the same subject matter

Which are some of the main features of the Electronic Transactions Act 1999
(Cth):
validity of electronic transactions
recognition of writing by electronic means
Recognition of retaining information in electronic form
All of the above

Peter verbally offers Tara his car for $5000. Tara refuses saying it’s not worth
that, but : she will pay $4000. What is the status of Peter’s first offer?
It can still be accepted by Tara
Peter should have made it in writing so Tara had proof he made it
Peter needs to make it again
It has lapsed because of Tara’s counteroffer

What was one of the “practical benefits” received by Roffey in Williams v Roffey
Bros &: Nicholls (Contractors) Ltd [1990] 1 All ER 512?
Williams did not sue Roffey for breach of contract
Roffey had no reason to doubt Williams would not complete his side of the bargain
Roffey received additional payment from Williams
Roffey did not need to find another subcontractor

Changing the Constitution : Section 128 of the Commonwealth Constitution


provides that the Constitution can be: changed by referendum that requires a
“yes” vote:
in at least 2 States
in at least 3 States
in the ACT and NT (the Territories)
by the majority of voters and in a majority of States

Substantive and procedural law: Substantive law:


Refers to actual rights under the law (bên t đáp án là câu này cơ)
Is subsidiary to procedural law
Refers to the formal steps to enforcement of rights and duties under the law

Downloaded by huu nguyen (nguyenminhhuu75@gmail.com)


lOMoARcPSD|13289864

Includes the rules of evidence

In which of the following situations is the offer most likely not to have lapsed?
Where Wrench had an option to purchase land but, unknown to Wrench, the seller
died : prior to Wrench’s acceptance (bên t là câu này)
Where Carter, a prospective purchaser of land, makes a counter-offer that is rejected
and: then states that he will accept the earlier offer
Where no time was stated for acceptance by Jane, but Ben does not accept within a :
reasonable time
Where Jim has become overtaken by insanity prior to accepting

Reception of English law in Australia: Which Latin phrase explained the rationale
for applying English laws to the new Colony of : New South Wales?
actus reus
terra nullius
ratio decidendi
prima facie

Andy promises Ellie $100 on her 21st birthday. If this promise was contained in a
simple : contract, the legal position is that such a promise is:
Never binding as it must have consideration supplied by Ellie
Binding only if Ellie and Andy had contractual capacity
Sometimes binding but only after Ellie turned 21
Always binding because promises should not be broken

In Elizabeth City Center Pty Ltd v Corralyn Pty Ltd (1995) 63 SASR 235, the option
to : exercise renewal of the lease:
Was effective because of the operation of the postal rule
Effective communication of the renewal requirement had not been made
Negated the postal rule because the notification was not sent by certified mail as :
required
b and c

Interpretation – extrinsic materials: In interpretation, extrinsic materials:


Are required to be referred to by a court
Are only to be referred to where the meaning of the provision is ambiguous
Are allowed to be referred to by a court
Are to be referred to regardless of the length of delay this will cause proceedings

With respect to revocation, which of the following statements is not correct?

Downloaded by huu nguyen (nguyenminhhuu75@gmail.com)


lOMoARcPSD|13289864

Revocation need not be in words


The offeror must personally communicate the revocation to the offeree
The offeree may accept the offer until such time as they become aware of the
revocation
Where an offer has been made to the world at large, revocation does not need to be
seen : by everyone in order to be effective

Which of the following circumstances are likely to affect the consent of one or
both : parties to a contract:
Mistake
Duress and undue influence
Lack of writing
a&b

Simon and Stella, both of full legal capacity, agree to go on a date. Stella is to pay
for the : dinner, but she is running late and does not meet Simon at all. Simon is
embarrassed and : angry and calls Stella the next day to threaten to sue her for
his taxi fares and dinner : expenses. Will Simon succeed in his claims?
Yes, because there is agreement
Yes, because there was consideration in that Simon incurred taxi and dinner expenses
No, because Simon and Stella did not intend the agreement to create legally
enforceable : obligations
No, because there was no genuine consent of either party

Royal Assent: Who or what gives Royal Assent to an Act of the Commonwealth
Parliament?
The Queen
The Governor-General on the Queen’s behalf
The Prime Minister
Cabinet

In Crown Melbourne Ltd v Cosmopolitan Hotel (Vic) Pty Ltd [2016] HCA 26 why
did the : High Court disagree with the decision of the Victorian Court of Appeal in
finding there was : no case for promissory estoppel:
Even though the elements of estoppel had been proven no remedy was provided as
the : initial claim was brought too late and equity does not assist claims lodged with
unreasonable : delay
The plaintiff could not establish that the statement that it would be “looked after at :
renewal time” was capable of conveying to a reasonable person that it was a genuine
offer : of a further lease

Downloaded by huu nguyen (nguyenminhhuu75@gmail.com)


lOMoARcPSD|13289864

There was a need for certainty as to what the parties had agreed on at the end of the :
negotiations to found a claim for estoppel
B&C

Which of the following is not a way in which an offer can be terminated?


Revocation after acceptance has been mailed where the postal rule applies

Farah agreed to take care of an elderly woman Marge and in return was provided
with a : house to live in that was promised by Marge to be Farah’s after she died.
Farah cared for the : woman for 23 years, but upon Marge’s death Farah
discovered that their oral agreement : was never put into writing. The woman’s
son moved into the house and Farah made a claim : to the house. The court:
Is likely to reject Farah’s claim to the house as she provided no consideration of any
legal: value
Is likely to reject Farah’s claim to the house as the contract was not set down in writing
as : required
Is likely to order specific performance of the oral agreement because simple contracts
do : not need to be in writing
Is likely to apply the doctrine of part performance

Which of the following is most likely to be considered an offer?


A catalogue of books with discounted prices shown
A radio advertisement for drinks at “the coolest little pub in Victoria”
“I’ll pay you $3,000 if you complete a total rewrite of chapters 12 and 15 of this :
textbook.”
“Would you allow me to pay off the $2,000 over six weeks?”

Australian Constitution : Which Act established the federal legal and political
system and converted the separate : colonies into states?
Commonwealth of Australia Constitution Act 1900
The Australia Act 1986
Statute of Westminster Adoption Act 1942 (Cth)
statute of Westminster 1931 (IMP)

Contracts voidable by a minor do not include:


Those not binding unless ratified by the minor during their minority
Those binding unless repudiated by the minor during their minority
Those binding unless repudiated by the minor within a reasonable time after attaining :
their majority

Downloaded by huu nguyen (nguyenminhhuu75@gmail.com)


lOMoARcPSD|13289864

Those not binding unless ratified by the minor within a reasonable time of attaining :
majority

Where an agreement has been made during the course of trade or commerce
between businesses, the situation will commonly indicate that the parties:
intended to create moral binding relations based on trust
intended to create binding legal relations
intended that there was no contract unless clearly specified in the agreement
intended to avoid court proceedings by inserting exclusion clauses in the agreement

Which of the following is not a class of persons regarded by the law as wholly or
partly: incapable of entering into legally binding contracts?
Intoxicated persons
Minors
Women
Mentally incapacitated persons

Separation of powers: Which of the following is not one of the three branches of the
Commonwealth government : in Australia:
A. The Governor
B. The executive
C. The judiciary
D. The legislature

Criminal offences: Which of the following is incorrect with respect to criminal offences?
A. Indictable offences are generally the more serious offences.
B. The prosecution must prove its case beyond reasonable doubt.
C. Summary offences are determined by a magistrate without a jury.
D. A committal hearing is held before most summary offence matters.

Federal system: Which of the following statements is correct?


A. Australia is a federal system, with one Constitution and a number of non-law-
making : States and Territories.
B. Australia is a unitary system, where the States and Territories are constrained in
their : law-making powers by the Commonwealth.

Downloaded by huu nguyen (nguyenminhhuu75@gmail.com)


lOMoARcPSD|13289864

C. Australia is a federal system, with two legal systems for each citizen.
D. Australia is a federal system with three branches of government: the judiciary,
the : executive and the Crown.

Native Title: Native Title was first recognised in which of the following cases?
A. Mabo v State of Queensland (No 2) (1992) 175 CLR 1
B. Wik Peoples v State of Queensland (1996) 187 CLR 1
C. Brandy v Human Rights and Equal Opportunity Commission (1995) 183 CLR 245
D. Commonwealth v Jones (1901) 174 CLR 25

Separation of powers: Which of the following statements about separation of powers in


Australia is correct?
A. The judiciary is the body that makes statute law.
B. The legislature is the branch that declares what the law is and interprets the law.
C. The executive is the body that administers the law.
D. The legislature is the body that resolves disputes concerning the application of
law and : polices the law.

The postal acceptance rule:


A. Can be negated where the mailed offer was misdirected and the misdirection
was the : fault of the offeror
B. Can be impliedly excluded by offeror specifying actual receipt of acceptance
C. Can create a situation where a revocation is validly made days after the offer has
actually : been accepted
D. May apply to electronic communications, such as email, if the offeree chooses

Jack was subpoenaed to give evidence on Rods behalf. Jack claimed Rod promised him :
$2000 to give evidence. Would Jack recover that money?
A. No - Jack’s public duty is to give evidence in response to that subpoena.
B. No this is illegal
C. Yes if Rod put that in writing
D. Always binding because promises should not be broken

Law Reports: Where are decisions of the High Court of Australia found?: only online

Downloaded by huu nguyen (nguyenminhhuu75@gmail.com)


lOMoARcPSD|13289864

A. In the High Court reports


B. In the Commonwealth Law Reports
C. In the Common Law Reports

The Age of Majority Act 1977 (Vic) reduced the age for contractual capacity of a minor
A. 16
B. 18
C. 21
D. 25

Delegated legislation: An example of delegated legislation is the:


A. Partnership Act 1958 (Vic)
B. Local Government Act 1993 (Qld), s 25
C. Corporations Bill 1988 (Cth)
D. Corporations Regulations 2001 (Cth)

The Executive Power : The Executive power is administered by:


A. The Senate and the House of Representatives
B. The Queen, the Prime Minister and the Cabinet
C. The Governor, the Senate and the House of Representatives
D. The Judiciary, the Senate and the House of Representatives

Delegated legislation: Which of the following is not correct?: Delegated


legislation:

A. Is subordinate legislation
B. Is made under the authority of an Act of Parliament
C. Often contains more detailed rules than those of the associated Act
D. Can only be made by Government Ministers

Common law: Which one of the following statements about common law is correct?

A. Common law is judge made law

Downloaded by huu nguyen (nguyenminhhuu75@gmail.com)


lOMoARcPSD|13289864

B. Common law overrides statute law


C. Common law is made by Parliaments
D. Common law and equity are the same types of laws and provide the same
remedies

In Ashton v Pratt [2015] NSWCA 12 what was the main reason for the court deciding
that : there was no intention to create a legally binding contract?

A. The agreement was not reduced to writing


B. The verbal language of the agreement greatly lacked detail from either party
and did not : indicate definite obligations
C. Pratt lacked the necessary contractual capacity due to his age and ill health
D. The plaintiff had been a worker in the escort business and not morally entitled to
any : monies under the verbal agreement

Separation of powers: Which of the following is not one of the three branches of the
Commonwealth government : in Australia:

1. The Governor
2. The executive
3. The judiciary
4. The legislature

Public and private law: Which of the following is not classified as private law?

A. The law of contract


B. Criminal law
C. The law of property
D. Corporations law

Criminal proceedings: An indictable offense is:

Downloaded by huu nguyen (nguyenminhhuu75@gmail.com)


lOMoARcPSD|13289864

A. A more serious civil wrong


B. A less serious civil wrong
C. A more serious criminal offence
D. A less serious criminal offence

Where an agreement has been made during the course of trade or commerce between
businesses, the situation will commonly indicate that the parties:
A. intended to create moral binding relations based on trust
B. intended to create binding legal relations
C. intended that there was no contract unless clearly specified in the agreement
D. intended to avoid court proceedings by inserting exclusion clauses in the agreement

A qualified acceptance may also be known as a/an:


A. Counteroffer
B. Invitation to treat
C. Condition subsequent
D. Conditional agreement

Court hierarchy: Local or Magistrates Courts:

A. Are the lowest courts in the state hierarchy


B. Are the lowest courts in the federal hierarchy
C. Are an intermediate court
D. Are above Country Courts in the hierarchy

The legislature: Exclusive powers are those that:

A. Can be exercised by either the Commonwealth or the States


B. Can be exercised only by the Commonwealth
C. Can be exercised only by the States
D. Where there is any inconsistency, the Commonwealth law will prevail

Contracts voidable by a minor do not include:

A. Those not binding unless ratified by the minor during their minority
B. Those binding unless repudiated by the minor during their minority
C. Those binding unless repudiated by the minor within a reasonable time after
attaining : their majority
You Answered

Downloaded by huu nguyen (nguyenminhhuu75@gmail.com)


lOMoARcPSD|13289864

D. Those not binding unless ratified by the minor within a reasonable time of
attaining : majority

Which of the following statements about contracts is true?

A. A contract must be in writing to be enforceable


B. It is a common law requirement that certain contracts are in writing
C. A simple contract is another term for an oral contract
D. It is a statutory requirement throughout Australia that some contracts must be in
writing : and others must be evidenced in writing

In which of the following situations is the offer most likely not to have lapsed?

A. Where Wrench had an option to purchase land but, unknown to Wrench, the
seller died : prior to Wrench’s acceptance
B. Where Carter, a prospective purchaser of land, makes a counter-offer that is
rejected and: then states that he will accept the earlier offer
C. Where no time was stated for acceptance by Jane, but Ben does not accept

within a : reasonable time

D. Where Jim has become overtaken by insanity prior to accepting

Interpretation – extrinsic materials: In interpretation, extrinsic materials:

A. Are required to be referred to by a court


B. Are only to be referred to where the meaning of the provision is ambiguous
C. Are allowed to be referred to by a court
D. Are to be referred to regardless of the length of delay this will cause proceedings

Sources of law: Sources of law in Australia do not include:

A. Consolidating statutes
B. Judge-made law
C. Repealed statutes
D. Equity

Downloaded by huu nguyen (nguyenminhhuu75@gmail.com)


lOMoARcPSD|13289864

Public and private law: Which of the following is not classified as private law?

A. The law of contract


B. Criminal law
C. The law of property
D. Corporations law

Which of the following statements about acceptance is false?

A. Acceptance can be by telephone or email or conduct


B. Acceptance can be in any manner chosen by the offeree
C. Acceptance must be unconditional
D. Acceptance can only be made by the person/s to whom the offer was made

The main legal issue in Felthouse v Bindley (1862) 11CB (NS) 869 was:

A. the contract was formed due to offer and acceptance being proven
B. An offer to sell a horse was simply an invitation to treat
C. generally silence /lack of action does not constitute acceptance of an offer
D. An offer can be revoked at any time prior to acceptance

Changing the Constitution : Section 128 of the Commonwealth Constitution provides


that the Constitution can be: changed by referendum that requires a “yes” vote:

A. in at least 2 States
B. in at least 3 States
C. in the ACT and NT (the Territories)
D. by the majority of voters and in a majority of States

Jack was subpoenaed to give evidence on Rods behalf. Jack claimed Rod promised him :
$2000 to give evidence. Would Jack recover that money?

A. No - Jack’s public duty is to give evidence in response to that subpoena.


B. No this is illegal
C. Yes if Rod put that in writing
D. Always binding because promises should not be broken

Downloaded by huu nguyen (nguyenminhhuu75@gmail.com)


lOMoARcPSD|13289864

Separation of powers: Which of the following statements about separation of powers in


Australia is correct?

A. The judiciary is the body that makes statute law.


B. The legislature is the branch that declares what the law is and interprets the law.
C. The executive is the body that administers the law.
D. The legislature is the body that resolves disputes concerning the application of
law and : polices the law.

QUIZ 2:

1. Which of the following is not true? A signature on a contract containing an


exemption : clause:
Will not guarantee the effectiveness of the clause where the doctrine of non est factum :
applies
Incorporates the exemption clause into the contract
Guarantees that an exemption clause will be effective
Will be effective even if one party did not read the contract

2. Which of the following would not constitute the unfair practice of


harassment or : coercion under the Australian Consumer Law? Where
there is a debt in connection with the : possible of goods and the creditor:
Consciously calculates to intimidate the debtor with the content of her demands
Tells the debtor, among repeated demands for payment give the debtor a discount for :
early payment
Makes demands constantly in order to exhaust the debtor
Chooses particularly threatening demands in order to demoralise the debtor

3. When considering whether a contract is a standard form contract, the court


does not : have to consider:
The bargaining power of the parties as unequal bargaining power does not constitute a :
special disadvantage
Whether both parties had the opportunity to negotiate the terms of the contract
Whether one party’s vicarious liability for its agents is limited
Whether the terms of the contract take into account the specifics of the particular :
transaction

Downloaded by huu nguyen (nguyenminhhuu75@gmail.com)


lOMoARcPSD|13289864

4. Non est factum means:


It is not [my] deed
It is not true
Mistake of fact
It is unknown

5. Tom agrees to fix Richie’s car for $1,000. Richie pays a deposit of $200 and
Tom : guarantees the job will be completed within two weeks. Before Tom
commences the repairs, : the car is stolen from Richie’s locked garage and
destroyed by fire. Richie demands the : return of his $200 deposit and Tom
refuses to give it to him. Which of the following is not : correct?
The contract is automatically terminated
Tom is no longer obligated to fix Richie’s car
Under the Australian Consumer Law and Fair Trading Act 2012 (Vic), Tom is entitled
to:retain the $200 deposit
Under the Australian Consumer Law and Fair Trading Act 2012 (Vic), Tom is entitled
to : retain the portion of the $200 deposit he spent on parts and other relevant
expenses, if any

6. Which of the following is not correct? An action in restitution:


Is often brought where one party has been unjustly enriched at the expense of the
other
Cannot be brought where there is no contract between the parties
Can be brought where the contract between the parties is unenforceable
Can be brought where there has been only part performance

7. Chris and Tama buy a noodle bar from Udon4U Pty Ltd. In the course of
negotiations, : Udon4U Pty Ltd’s agent, Nadia, misrepresents the turnover
of the noodle bar, stating an : amount that is double the actually turnover.
Nadia herself received this information on : turnover from Geoff, a director
of Udon4U Pty Ltd.: In making the misrepresentation, what sections has
Nadia likely contravened?
As she merely passed on the information with no intention to procure a contravention :
she probably will not be personally liable
Section 2(1)
Section 18
Section 52

Downloaded by huu nguyen (nguyenminhhuu75@gmail.com)


lOMoARcPSD|13289864

8. Which of the following is not one of the three elements of an unfair term
under s 24(1) : of the Australian Consumer Law? Where:
It would cause a significant imbalance in the parties’ rights and obligations
The term is not transparent
It is not reasonably necessary to protect the interests of the disadvantaged party
It would cause detriment to a party if it were applied

9. Which unfair practice involves participation in a trading scheme where


persons at the : top receive most of the benefits?
referral selling
pyramid selling
unconscionable conduct
misleading and deceptive conduct

10. Janey takes her fur stole to the dry cleaner. When she returns to collect it,
there is a : black mark in the shape of an iron in the middle of her stole and
it is ruined. The dry cleaner : tells Janey that he is not liable for the damage
to her fur stole as there was an exemption : clause in their agreement.
Which of the following is not correct?
If the exemption clause was merely contained within a notice on the wall, the dry :
cleaner will be liable for the damage
If the exemption clause was on the dry cleaning docket Janey was given and she was
told : “don’t worry about this, it’s just the docket you have to give us when you come to
collect : your stole”, the dry cleaner will not be able to rely on the exemption clause
In order for the exemption clause to be effective where it was not in a signed
document, : it had to have been brought to Janey’s attention
If the exemption clause was on the dry cleaning docket Janey was given rather than a :
formal written contract, the onus will be on the dry cleaner to prove that Janey was
aware it : contained conditions that would modify the agreement

11. Which of the following statements is correct?


An exemption clause is a term that completely excludes one party’s liability
An exemption clause is a term that excludes or limits the liability of one or more parties
An exemption clause limits liability to a certain monetary amount
An exclusion clause limits one party’s liability whereas an exemption clause limits all:
parties’ liability

12. Which of the following is not usually recoverable by way of damages?


Compensation for mere inconvenience or disappointment
Loss that is difficult to estimate

Downloaded by huu nguyen (nguyenminhhuu75@gmail.com)


lOMoARcPSD|13289864

Nominal loss
Expenses incurred in reliance on the other party’s promise to perform

13. A party repudiates a contract when


They decide to terminate it
They are no longer able or are unwilling to perform their obligations
The other party decides to terminate it
They take too long to perform an obligation that has a time limit under the contract,
such : as payment of rent

14. Chung is looking to purchase a carwash business from Terry. Terry tells
Chung that she : serves 100-150 cars per day and that she has just
concluded an agreement to service the : fleet of cars of a local business. In
reality, Terry has only served around 15 cars per day since : her customer
service received a bad review in the local newspaper and a competing
carwash : business opened one street away. Terry intends for Chung to
buy her business based on : what she has stated. This is an example of:
unilateral mistake
negligent misrepresentation
fraudulent misrepresentation
mutual mistake

15. Undue influence differs from duress in that:


With undue influence, the contract is not voidable but void
With undue influence, no unlawful act is required
Duress arises only within a closed list of special fiduciary relationships
With undue influence, actual physical violence is required

16. Which of the following constitutes referral selling?


Jaz is approached by her neighbor to join the sales team of Thames Beauty Products. :
Thames sells its products door-to-door through agents and is a close-knit family type
of : company. Agents that introduce other agents to “the family” are rewarded with a
finders’: fee. It is only $500 to join the Thames family, and Jaz only has to spend $400
on products in : her first month
Jaz starts work at a chemist and notices that the chemist, as retailers often do, puts a :
sale price on a sticker on most of the products in the shop. However, the chemist does
not : put the sale price sticker on top of the original price sticker and when customers
bring the : products to the counter, the chemist tells them that unfortunately there has
been a mistake, : the true price is the higher price

Downloaded by huu nguyen (nguyenminhhuu75@gmail.com)


lOMoARcPSD|13289864

Jaz approaches a salesperson on the forecourt to buy the car package she saw :
advertised on television and is told that there were only two cars for sale as a part of
that : deal and there are now none left. The salesperson tells Jaz that she would look
much better : in the convertible model and although there is no deal on that car, it is
only $50,000 more
Jaz purchases a painting through a new art gallery that has just opened in town. The :
price is high and Jaz is at first uncertain about the expense, but the art dealer promises
Jaz a : commission on sales to any of Jaz’s friends who she sends along to the gallery.
Jaz has a lot : of art collector friends so is sure that she will receive some commission
from the gallery in : the near future.

17. In Trident General Insurance Co Ltd v McNiece Bros Pty Ltd (1988) 165 CLR
107:
Blue Circle was a sub-contractor whose worker was injured, and privity precluded the :
worker from getting an indemnity under the contract
It was held that the subcontractor could get an indemnity even though the :
subcontractor had given no consideration under the contract
In Deane J’s view Trident, the insurance company, had an obligation to the
subcontractor because otherwise Trident would be unjustly enriched
The majority were of the view that the “settled and fundamental” doctrine of privity of :
contract should not be overturned by the court

18. Jacq and Jack robbed the Commercial Bank. Jacq thinks that Jack was too
aggressive : with the tellers and the robbery could have gone very wrong.
Jacq is now refusing to give : Jack his share of the takings. Which of the
following is correct?
Jack provided consideration in the form of performing the robbery and is entitled to his :
share
Under the principle of freedom of contract, Jacq and Jack are entitled to agree to :
whatever they wish and the contract will be enforceable
The contract involved the commission of a crime and is therefore illegal and :
unenforceable
The contract will be unenforceable if the courts deem it to be against public policy

19. If one party has completed its obligations under the contract, and the other
has not:
The contract can be terminated by mutual agreement
The contract cannot be terminated by simple agreement without consideration
The contract can be terminated if it is put down in writing; no further consideration is :
required

Downloaded by huu nguyen (nguyenminhhuu75@gmail.com)


lOMoARcPSD|13289864

The contract can be terminated where one party promises to abandon their rights
under : the contract and the other party promises to do the same

20. Ella contracted Anasaki to build a balcony for her house to certain
specifications. When : Anasaki had finished, Ella was shocked to see that
the balcony was much bigger than she : had specified. Ella refused to pay
Anasaki. Which of the following is incorrect?
Ella has received some benefit
The benefit Ella received was at Anasaki’s expense
It would be unjust for Ella to have to pay any money for a balcony that was not built to :
her specifications under the contract
Ella will probably not have to pay the contract price but must pay a reasonable amount

21. Which of the following is an element required for the doctrine of frustration
to operate?
Material loss to one party
No fault on the part of either party
Material loss to both parties
Hardship or inconvenience

22. Which of the following is not correct with respect to the prohibition of
unconscionable : conduct within the meaning of the unwritten law?
Unwritten law includes equity
There is no unconscionable conduct where both parties have made the same mistake
in : good faith
There must be a special disadvantage and an exploitation of that disadvantage
Unequal bargaining power of itself is enough to constitute a special disadvantage

23. In Jarvis v Swans Tours Ltd [1973] QB 233, where Jarvis was disappointed
by, among : other things, the little dry nut cakes on his holiday:
Damages were not awarded
Damages were awarded for anxiety and depression
Pleasure and enjoyment were not promises of the tour company with respect to the :
holiday in Switzerland
Damages were awarded partly for disappointment

24. Chung is looking to purchase a carwash business from Terry. Terry tells
Chung that she : serves 100-150 cars per day and that she has just
concluded an agreement to service the : fleet of cars of a local business. In

Downloaded by huu nguyen (nguyenminhhuu75@gmail.com)


lOMoARcPSD|13289864

reality, Terry has only served around 15 cars per day since : her customer
service received a bad review in the local newspaper and a competing
carwash : business opened one street away. Terry intends for Chung to
buy her business based on : what she has stated. This is an example of:
unliteral mistake
negligent misrepresentation
fraudulent misrepresentation
mutual mistake

25. What does the concept “quantum meruit” mean?


The contract is terminated because of a breach by both parties
The innocent party would be unjustly enriched if they were able to retain the benefit :
without compensating the party in breach for the “amount he deserves”
Voluntarily accepting a contractual benefit
Not my deed

26. Cathy and Mel entered into a contract where Cathy was to steam clean
seven rooms in : Mel’s house at $200 per room. After cleaning three rooms,
Cathy abandoned the job for a : more valuable contract. Which of the
following is true?
As there has been substantial performance, Cathy will be able to enforce all the rights :
conferred by the contract
If the exact performance rule is applied, Cathy is entitled to payment for the work she
has : done
If the contract is divisible, Cathy is entitled to payment for the work she has done
Courts are inclined to treat contracts as indivisible and to require exact performance

27. Chang wants the court to imply an engineering custom into a term of a
contract to which : he is a party. The other party has no knowledge of this
custom. It is not contrary to any of : the express terms of the contract. The
court:
Will not imply the term in these circumstances
May imply the term
Will imply the term even if the custom is not very well known
Will not imply the term unless it is implied by statute

28. Jong complains that the barbecue he bought is not fit for purpose because
the metal : sides melted the first time he cooked with it. The manufacturer
maintains that Jong should : not have used the barbecue for slow cooking
and that it never represented that the : barbecue would be fit for the

Downloaded by huu nguyen (nguyenminhhuu75@gmail.com)


lOMoARcPSD|13289864

purpose of slow cooking. What are the circumstances in which : the


guarantee as to fitness for purpose would apply?
Where Jong made known to the supplier that he was going to use the barbecue for slow
cooking meals
Where Jong bought the barbecue at auction
The guarantee always applies as it is not possible for a manufacturer or supplier to :
contract out of it
Where the unfitness for slow cooking of the barbecue was not drawn to Jong’s
attention : prior to his purchase by either the manufacturer or the supplier

29. Which of the following matters is least likely to contribute to a finding of


unconscionable : conduct in connection with goods or services?
Where conditions imposed on a customer were not necessary for the protection of the :
supplier’s interests
Where the supplier makes a commercial decision to breach the contract knowing that :
the customer will seek a legal remedy for the breach
Where the supplier acted in bad faith
Where unfair tactics were used against the customer

30. It is important to distinguish between representations and terms because:


A. Damages cannot be awarded for misrepresentation
B. Damages can only be awarded for fraudulent or negligent misrepresentation
C. There is no remedy for misrepresentation
D. The remedy of rescission is not available for misrepresentation

31. Which of the following is not an enforcement measure under the Australian
Consumer : Law?
A. An order of the court requiring a supplier to publish an advertisement
B. A disclosure order issued by the ACCC
C. A two-year probation order of the court
D. A notice issued by the ACCC requiring a supplier to substantiate a claim about
a product : they are selling

32. Which of the following is not an exception to privity?

A. Insurance
B. Agency and trust
C. Employment
D. Property law

Downloaded by huu nguyen (nguyenminhhuu75@gmail.com)


lOMoARcPSD|13289864

33. James is going through a difficult time and does not feel he can confide in
anyone. : Eventually he confides in the priest of his church and asks for
support. The priest forcefully : tells James to turn over all of his money to
the church or he will be struck down by : lightening. The priest does not
believe that James will be struck down. James makes the : payments.
Which of the following is most likely legal grounds for James to avoid the :
transaction?
A. Mistake
B. Undue influence
C. Duress
D. Misrepresentation

34 In which High Court decision was it said: “[T]here are two relevant
circumstances in which : a breach of contract by one party may entitle the other
to terminate. The first is where the : obligation [is] essential … The second
relevant circumstance is where there has been a : sufficiently serious breach of a
non-essential term …we rest our decision in the appeal not : upon the ground of
breach of an essential obligation, but upon application of the doctrine :
respecting intermediate terms.”
A. Gumland Property Holdings Pty Ltd v Duffy Bros Fruit Market (Campbelltown)
Pty Ltd : (2008) 234 CLR 237
B. Koompahtoo Local Aboriginal Land Council v Sanpine Pty Ltd (2007) 233 CLR
115
C. Ankar Pty Ltd v National Westminster Finance (Australia) Ltd (1987) 162 CLR
549
D. Shevill v Builders Licensing Board (1982) 149 CLR 620

35. Which of the following are sources of illegality?


A. Statute
B. Statute and common law
C. Statute, common law and equity
D. Statute, common law and parties’ subjective views on morality

36. What is the main difference between the misleading or deceptive conduct
provisions in : the Trade Practices Act 1974 (Cth) and in the Australian Consumer
Law?
A. The Australian Consumer Law no longer refers to “trade or commerce”
B. The Australian Consumer Law refers to a body corporate
C. The Australian Consumer Law refers to a corporation
D. The Australian Consumer Law refers to a person

Downloaded by huu nguyen (nguyenminhhuu75@gmail.com)


lOMoARcPSD|13289864

37. Non est factum means:


A. It is not [my] deed
B. It is not true
C. Mistake of fact
D. It is unknown

38. With respect to consumer guarantees, which of the following is not correct?
A. It is possible to exclude the consumer guarantees in very limited circumstances
B. Remedies for non-compliance with the consumer guarantees depend on
whether there : was a “major failure” or not
C. With respect to fitness for purpose, a “disclosed purpose” relates to the supplier
only –: the manufacturer may make representations to the consumer, but the
consumer cannot : disclose a purpose to the manufacturer and receive a
guarantee in return
D. A manufacturer does not need to provide repair facilities or spare parts after a
certain : period

39. Which of the following is not a type of contract illegal at common law on the
grounds of : public policy?
A. Contracts to oust the jurisdiction of the courts
B. Contracts to commit a tort
C. Contracts prejudicial to the public safety
D. Champertous contracts

40. The term “transparency” in unfair contracts means:


A. It is expressed in reasonably plain language that is readily available to the
affected party
B. It is easily interpretable by the court
C. It is easily interpretable by the industry
D. It is capable of only one meaning

41. Undue influence differs from duress in that:


A. With undue influence, the contract is not voidable but void
B. With undue influence, no unlawful act is required
C. Duress arises only within a closed list of special fiduciary relationships
D. With undue influence, actual physical violence is required

42. In Howe v Teefy (1927) 27 SR (NSW) 301 where a leased racehorse was
retaken three : months into a three-year lease:

Downloaded by huu nguyen (nguyenminhhuu75@gmail.com)


lOMoARcPSD|13289864

A. The court was unable to assess the damages because they were uncertain and
there was : no evidence on which they could be assessed
B. The fact that assessment was difficult did not prevent a court from granting
damages : where the plaintiff had been deprived of something of value
C. Although there was a value attributable to the plaintiff’s loss, it was too remote
to be : recoverable
D. The court was unable to assess the damages because the process was difficult
and time : consuming

43. A written contract allows Joseph, the neighbour of Sonny, to take “as many
strawberries : as he likes each strawberry season”. This contract:
A. Is void for uncertainty
B. Is valid but unenforceable because it is too uncertain
C. Is valid as the parties’ intention is clear
D. Is voidable for uncertainty
44. Which of the following is not an equitable remedy?
A. Injunction
B. Specific performance
C. Restitution
D. Exemplary damages
45. Which of the following matters is least likely to contribute to a finding of
unconscionable : conduct in connection with goods or services?

A. Where conditions imposed on a customer were not necessary for the protection
of the : supplier’s interests
B. Where the supplier makes a commercial decision to breach the contract knowing
that : the customer will seek a legal remedy for the breach
C. Where the supplier acted in bad faith
D. Where unfair tactics were used against the customer

46. Consumer guarantees as to title are contained within:

A. The “General Protections” chapter of the Australian Consumer Law


B. Section 38 of the Trade Practices Act 1974 (Cth)
C. Chapter 2 of the Australian Consumer Law
D. Sections 51-56 and ss 60-61 of the Australian Consumer Law

Downloaded by huu nguyen (nguyenminhhuu75@gmail.com)


lOMoARcPSD|13289864

47. Which of the following is not one of the four ways a statute may render a contract
illegal : set out in Yango Pastoral Co v First Chicago Australia Ltd (1978) 139 CLR 410?
The contract:

A. May require an action that statute forbids


B. May be performed in a prohibited manner
C. May have been made to effect an unlawful purpose
D. May set out an unreasonable restraint

48. Which of the following is not correct with respect to the Australian Consumer Law?

A. It is contained within Schedule 2 to the Competition and Consumer Act 2010


(Cth)
B. It applies to conduct engaged in outside of Australia
C. Owing to the Commonwealth’s limited lawmaking powers under s 51 of
the :Constitution, it applies only to corporations
D. The Australian Consumer Law is applied in each State and Territory

49. Which of the following is not one of the six elements required for a claim of
fraudulent : misrepresentation to succeed?

A. The representation must be fall


B. The representation must have been acted upon by the other party
C. The representation must be one of fact
D. The representation must be in writing (cannot be verbal only)

50. Which of the following is not a way that a person seeking to rely on an exemption
clause : can show that the clause has become part of the contract?

A. By showing it is in writing and the other party has signed the contract
B. By showing it is in writing in a document a reasonable person would expect to
contain : contractual terms and was brought to the other party’s notice
C. By showing that the parties had previously contracted on terms that included
the : exemption clause and that they intended to contract on the same basis

Downloaded by huu nguyen (nguyenminhhuu75@gmail.com)


lOMoARcPSD|13289864

Correct!

D. By showing it was brought to the notice of the other party at the time of, or
subsequent : to, entering into the contract

51. Ben planned the perfect surprise birthday party for his mother: He hired a hall, he
hired : a violinist and he organised catering. Two weeks prior to the event, the violinist
called to : cancel. Ben was so upset at the loss of what he thought would be the
highlight of the : evening that he called the whole event off. Ben lost both the deposit
on the hall and the : deposit for the catering. Which of the following is correct?

A. Ben can sue the violinist for all losses after Ben cancelled the hiring of the hall
B. Ben can recover his hire fees and catering fees as it was not his fault
C. The caterers and hall owners can sue the violinist for losses
D. Ben has a duty to mitigate his losses

52. Sandy is selling her horse, Flossy. Miranda is interested in buying Flossy. Sandy
tells : Miranda that Flossy: : • Is the best little racehorse in Australia: • Has all the
registrations, licences and permits required to race in the current season but : that
Miranda should probably get this checked herself: These two statements are:

A. Mere puff & term


B. Opinion & term
C. Opinion & representation
D. Mere puff & representation

53. What does the concept “quantum meruit” mean?

A. The contract is terminated because of a breach by both parties


B. The innocent party would be unjustly enriched if they were able to retain the
benefit : without compensating the party in breach for the “amount he deserves”
C. Voluntarily accepting a contractual benefit
D. Not my deed

Downloaded by huu nguyen (nguyenminhhuu75@gmail.com)


lOMoARcPSD|13289864

54. The term “transparency” in unfair contracts means:

A. It is expressed in reasonably plain language that is readily available to the


affected party
B. It is easily interpretable by the court
C. It is easily interpretable by the industry
D. It is capable of only one meaning

55. Can a person use an official position they hold for their financial advantage?

A. Of course, this is a side benefit


B. Depends on the circumstances
C. The courts are silent on this specific point
D. No because if a contract is involved, this could lead to the promoting of
corruption in : public life

56. Smithy Builders have a contract with Big Bank Pty Ltd which contains the
following : clause: “Where Smithy Builders fails to complete the contract by 7
December, it will pay a : sum of $300,000 in full and final satisfaction of its liability.”
Smithy Builders fails to complete : by 7 December. If Smithy Builders wishes to avoid
paying the sum of $300,000, what must it : prove?

A. That the clause is an unliquidated damages clause


B. That the clause is a penalty clause (to punish the builders)
C. That the clause is a liquidated damages clause
D. That the clause is a genuine pre-estimate of the loss to Big Bank Pty Ltd

57. Which of the following is not correct? An action in restitution

Downloaded by huu nguyen (nguyenminhhuu75@gmail.com)


lOMoARcPSD|13289864

A. Is often brought where one party has been unjustly enriched at the expense of the
other
B. Cannot be brought where there is no contract between the parties
C. Can be brought where the contract between the parties is unenforceable

D. Can be brought where there has been only part performance

58. Which of the following is correct?

A. It is prohibited to offer free gifts with purchase


B. It is prohibited to offer goods at a special price, not have a sufficient amount of
those : goods available and then attempt to upsell to customers who are drawn to
the shop for the : special price deal
C. It is not prohibited for a supplier to take a customer’s money when the supplier
believes : they won’t be able to supply the service to the customer within a
reasonable time
D. It is not prohibited for a bank to send an unsolicited credit card to its client; it is :
prohibited for a bank to send an unsolicited debit card to a person who is a
potential client : but not a yet a client

59. Which of the following is not true? A signature on a contract containing an


exemption : clause:
A. Will not guarantee the effectiveness of the clause where the doctrine of non est
factum : applies
B. Incorporates the exemption clause into the contract
C. Guarantees that an exemption clause will be effective
D. Will be effective even if one party did not read the contract

60. Which of the following is not an unfair practice under the Australian Consumer
Law?

A. Where a free gift is offered with the purchase an electric toothbrush and the cost
of the : “gift” toothpaste is added to the price of the toothbrush
B. Where a television advertisement for a chocolate bar shows teddy bears
working the : manufacturing lines

Downloaded by huu nguyen (nguyenminhhuu75@gmail.com)


lOMoARcPSD|13289864

C. Where a brochure advertising a beauty therapy treatment consisting of time in


an : oxygen tank lists the treatment as lasting two hours, when in fact the
treatment lasts just : under one hour
D. Where a shop assistant takes money from a customer knowing that the shirt the :
customer wants is sold out, but hoping that after a few days when the customer is
told that : there are no more shirts they will purchase something else from the
shop

A party repudiates contract when

A. The party is able but unwilling to perform their obligations under it


B. The party decides to terminate the contract
C. The party breaches a representation
D. The party breaches a warranty

Ella contracted Anasaki to build a balcony for her house to certain specifications.
When : Anasaki had finished, Ella was shocked to see that the balcony was much bigger
than she : had specified. Ella refused to pay Anasaki. Which of the following is
incorrect?

A. Ella has received some benefit


B. The benefit Ella received was at Anasaki’s expense
C. It would be unjust for Ella to have to pay any money for a balcony that was not
built to : her specifications under the contract
D. Ella will probably not have to pay the contract price but must pay a reasonable
amount

Janine has sold her business to Simone. Simone insists on inserting a clause in the :
agreement that Janine cannot open a similar business within 10 kilometres of her
former : business, for the next two years. Which of the following is not true?

You Answered

This clause is a restraint of trade


Correct Answer

The court will always uphold restraint of trade provisions in contracts for sale of :
business

Downloaded by huu nguyen (nguyenminhhuu75@gmail.com)


lOMoARcPSD|13289864

Where a restraint of trade is unreasonable, the whole or parts of the contract will be
void
It is impossible to say whether a court would deem this restraint of trade clause :
reasonable without further details

Where a loan contract allows for termination by the loan provider in the event of
default : by the borrower, this is:

You Answered

An implied power to terminate


Correct Answer

An express power to terminate


A condition precedent
A release

Which of the following is not a restraint of trade?

A price maintenance agreement


A contract restraining Laura from working as a welder within Ballarat for two years
after : her termination with Welders R Us
You Answered

An agreement between John and Liz that John will not open a brew shop within 2km
of : the brew shop he just sold to Liz
Correct Answer

An agreement by which Steve agrees to repay his daughter’s loan debt in return for
the : person who advanced the loan to his daughter agreeing not to report Steve’s
daughter to : the police for fraud

Downloaded by huu nguyen (nguyenminhhuu75@gmail.com)


lOMoARcPSD|13289864

Which of the following statements about the parol evidence rule is true.?

It applies only to oral (parol) evidence


It does not apply to written contracts
Correct!

It does not apply where it can be shown that the written contract was not intended to :
be a complete record of the agreement
It applies only to previous drafts of a written contract

QUIZ 3:
To claim damages a causal link must be established between what two things?
A. Between the defendant’s conduct and the actual monetary value of the
loss/damage
B. The conduct and initiating court proceedings
C. The fact that the plaintiff suffered loss or damage is sufficient. Nothing else is
required
D. The defendants breach and the plaintiff’s injury
The “neighbour principle” is often viewed as:
A. An objective test
B. A test of the reasonable person
C. A test of the foreseeability of harm
D. An identity test
In Tame v State of New South Wales (2002) 211 CLR 317 where Tame was given a
false : blood alcohol reading:
A. Whether the police officer was found to have owed Tame a duty of care was
entirely a : question of community standards
B. The psychotic depressive illness Tame developed was found to flow from the
breach of : duty of the police sergeant because it was entirely caused by his
actions that could have : been avoided had he exercised sufficient care
C. Part of the test of reasonable foreseeability is a question of fact
D. Because of the “eggshell skull” principle, pre-existing knowledge of Tame’s
susceptibility : to suffering nervous shock was not required to prove a breach
Which of the following is not correct?
A. If damages are too remote they will not be recoverable

Downloaded by huu nguyen (nguyenminhhuu75@gmail.com)


lOMoARcPSD|13289864

B. The remoteness test will be satisfied where the damage suffered is of the same
type or : kind as foreseeable damage
C. In order for damage to not be too remote, it must be reasonably foreseeable
D. In The Wagon Mound No 1 and The Wagon Mound No 2, where there was an :
unfortunate combination of an oil spill, welding sparks and floating cotton waste,
the loss : was found to be not reasonably foreseeable in the circumstances
Which of the following is not a provision of the Wrongs Act 1958 (Vic) relating to:
professional liability?
A. If there are differing peer professional opinions across Australia, the court may
accept : just one of those opinions
B. The court must rely on peer professional opinion even where it considers that
opinion : irrational
C. If there are differing peer professional opinions across Australia, the court may
accept all : of those opinions
D. Peer professional opinion can be considered widely accepted even where it is
not : universally accepted
Why was the plaintiff successful in Overseas Tankship (UK) Ltd v Miller
Steamship Co Pty : Ltd (The Wagon Mound No 2) [1967] AC 617 (PC) when
another plaintiff failed in the earlier : related Wagon Mound case?
A. The plaintiffs in the second had better legal representation
B. The legal situation had changed between the two cases
C. The court was more disposed to think about the consequences of the damage
suffered by : so many as a result of the oil spill
D. In the first case, the plaintiffs failed to show that a reasonable man would have
foreseen : the risk of damage from the oil spill
Nicola drives the forklift at her place of work, Rooze’s Roofing. Nicola always
leaves the : forklift in a certain place where she has been told to leave it, with the
forks up off the : ground. One afternoon a customer who is collecting goods from
the workshop reverses his : car into the forks on the forklift. He is injured and his
car is damaged.: What is the principle that would make Nicola’s employer liable
for her actions?
A. Contributory negligence
B. Strict liability
C. Vicarious liability
D. Voluntary assumption of risk
Where a person gives advice, that advice is relied upon and the advice is
incorrect, the : person giving the advice may be liable in negligence. Which of the
following is incorrect?

Downloaded by huu nguyen (nguyenminhhuu75@gmail.com)


lOMoARcPSD|13289864

A. The “special relationship” described in Mutual Life & Citizens’ Assurance Co Ltd
v Evatt : (1968) 122 CLR 556 has been accepted as the test for determining the
existence of a duty of : care with respect to negligent misstatements
B. The test in Mutual Life & Citizens’ Assurance Co Ltd v Evatt (1968) 122 CLR 556
was : approved in San Sebastian Pty Ltd v Minister Administering Environmental
Planning &: Assessment Act 1979 (1986) 162 CLR 340
C. The duty of care only arises where the “special relationship” involves a person
seeking : advice from the advice-giver, not where the advice was given
unrequested and merely : accepted
D. The advice-giver does not need to be in the business of giving advice
Which of the following is not correct? The reform to the law of negligence that
took : place in Australia in the early 2000s:
A. Limits the scope of potential liability for negligence
B. Does not apply to claims in contract law
C. Was enacted in all Australian states and territories
D. Covers personal injury
In Australian Safeway Stores v Zaluzna (1987) 162 CLR 479:
A. The respondent was a lawful entrant upon the land of the respondent,
establishing a : relationship between them and there was therefore a duty of care
owed by the appellant to : avoid a foreseeable risk of injury
B. The respondent was limited by his particular status as entrant without specific
consent of : the appellant
C. The respondent contributed to the accident thus limiting damages
D. There was no liability.
Where a person gives advice, that advice is relied upon and the advice is incorrect, the :
person giving the advice may be liable in negligence. Which of the following is
incorrect?
A. The “special relationship” described in Mutual Life & Citizens’ Assurance Co Ltd
v Evatt : (1968) 122 CLR 556 has been accepted as the test for determining the
existence of a duty of : care with respect to negligent misstatements
B. The test in Mutual Life & Citizens’ Assurance Co Ltd v Evatt (1968) 122 CLR 556
was : approved in San Sebastian Pty Ltd v Minister Administering Environmental
Planning &: Assessment Act 1979 (1986) 162 CLR 340
C. The duty of care only arises where the “special relationship” involves a
person seeking : advice from the advice-giver, not where the advice was
given unrequested and merely : accepted
D. The advice-giver does not need to be in the business of giving advice
Which of the following is not correct? The reform to the law of negligence that took :
place in Australia in the early 2000s:
A. Limits the scope of potential liability for negligence

Downloaded by huu nguyen (nguyenminhhuu75@gmail.com)


lOMoARcPSD|13289864

B. Does not apply to claims in contract law


C. Was enacted in all Australian states and territories
D. Covers personal injury
Nicola drives the forklift at her place of work, Rooze’s Roofing. Nicola always leaves
the : forklift in a certain place where she has been told to leave it, with the forks up off
the : ground. One afternoon a customer who is collecting goods from the workshop
reverses his : car into the forks on the forklift. He is injured and his car is damaged.:
What is the principle that would make Nicola’s employer liable for her actions?
A. Contributory negligence
B. Strict liability
C. Vicarious liability
D. Voluntary assumption of risk
In Australian Safeway Stores v Zaluzna (1987) 162 CLR 479:
A. The respondent was a lawful entrant upon the land of the respondent,
establishing a : relationship between them and there was therefore a duty of
care owed by the appellant to : avoid a foreseeable risk of injury
B. The respondent was limited by his particular status as entrant without specific
consent of : the appellant
C. The respondent contributed to the accident thus limiting damages
D. There was no liability.
Which of the following is not a provision of the Wrongs Act 1958 (Vic) relating to:
professional liability?
A. If there are differing peer professional opinions across Australia, the court may
accept : just one of those opinions
B. The court must rely on peer professional opinion even where it considers that
opinion : irrational
C. If there are differing peer professional opinions across Australia, the court may
accept all : of those opinions
D. Peer professional opinion can be considered widely accepted even where it is
not : universally accepted
To claim damages a causal link must be established between what two things?
A. Between the defendant’s conduct and the actual monetary value of the
loss/damage
B. The conduct and initiating court proceedings
C. The fact that the plaintiff suffered loss or damage is sufficient. Nothing else is
required
D. The defendants breach and the plaintiff’s injury
Which of the following is not correct?
A. If damages are too remote they will not be recoverable

Downloaded by huu nguyen (nguyenminhhuu75@gmail.com)


lOMoARcPSD|13289864

B. The remoteness test will be satisfied where the damage suffered is of the same
type or : kind as foreseeable damage
C. In order for damage to not be too remote, it must be reasonably foreseeable
D. In The Wagon Mound No 1 and The Wagon Mound No 2, where there was an :
unfortunate combination of an oil spill, welding sparks and floating cotton waste,
the loss : was found to be not reasonably foreseeable in the circumstances
The “neighbour principle” is often viewed as:
A. An objective test
B. A test of the reasonable person
C. A test of the foreseeability of harm
D. An identity test
Why was the plaintiff successful in Overseas Tankship (UK) Ltd v Miller Steamship Co
Pty : Ltd (The Wagon Mound No 2) [1967] AC 617 (PC) when another plaintiff failed in
the earlier : related Wagon Mound case?
A. The plaintiffs in the second had better legal representation
B. The legal situation had changed between the two cases
C. The court was more disposed to think about the consequences of the damage
suffered by : so many as a result of the oil spill
D. In the first case, the plaintiffs failed to show that a reasonable man would have
foreseen : the risk of damage from the oil spill
In Tame v State of New South Wales (2002) 211 CLR 317 where Tame was given a
false : blood alcohol reading:
A. Whether the police officer was found to have owed Tame a duty of care was
entirely a : question of community standards
B. The psychotic depressive illness Tame developed was found to flow from the
breach of : duty of the police sergeant because it was entirely caused by his
actions that could have : been avoided had he exercised sufficient care
C. Part of the test of reasonable foreseeability is a question of fact
D. Because of the “eggshell skull” principle, pre-existing knowledge of Tame’s
susceptibility : to suffering nervous shock was not required to prove a breach
In Australian Safeway Stores v Zaluzna (1987) 162 CLR 479:
A. The respondent was a lawful entrant upon the land of the respondent,
establishing a : relationship between them and there was therefore a duty of
care owed by the appellant to : avoid a foreseeable risk of injury
B. The respondent was limited by his particular status as entrant without specific
consent of : the appellant
C. The respondent contributed to the accident thus limiting damages
D. There was no liability.

Downloaded by huu nguyen (nguyenminhhuu75@gmail.com)


lOMoARcPSD|13289864

The “neighbour principle” is often viewed as:


A. An objective test
B. A test of the reasonable person
C. A test of the foreseeability of harm
D. An identity test
Which of the following is not correct?
A. if damages are too remote they will not be recoverable
B. The remoteness test will be satisfied where the damage suffered is of the same
type or : kind as foreseeable damage
C. In order for damage to not be too remote, it must be reasonably foreseeable
D. In The Wagon Mound No 1 and The Wagon Mound No 2, where there was an : unfortunate
combination of an oil spill, welding sparks and floating cotton waste, the loss : was found to
be not reasonably foreseeable in the circumstances

Where a person gives advice, that advice is relied upon and the advice is incorrect, the :
person giving the advice may be liable in negligence. Which of the following is
incorrect?
A. The “special relationship” described in Mutual Life & Citizens’ Assurance Co Ltd v
Evatt : (1968) 122 CLR 556 has been accepted as the test for determining the
existence of a duty of : care with respect to negligent misstatements
B. The test in Mutual Life & Citizens’ Assurance Co Ltd v Evatt (1968) 122 CLR 556
was : approved in San Sebastian Pty Ltd v Minister Administering Environmental
Planning &: Assessment Act 1979 (1986) 162 CLR 340
C. The duty of care only arises where the “special relationship” involves a person
seeking : advice from the advice-giver, not where the advice was given
unrequested and merely : accepted
D. The advice-giver does not need to be in the business of giving advice

To claim damages a causal link must be established between what two things?
A. Between the defendant’s conduct and the actual monetary value of the
loss/damage
B. The conduct and initiating court proceedings
C. The fact that the plaintiff suffered loss or damage is sufficient. Nothing else is
required
D. The defendants breach and the plaintiff’s injury

Nicola drives the forklift at her place of work, Rooze’s Roofing. Nicola always leaves
the : forklift in a certain place where she has been told to leave it, with the forks up off

Downloaded by huu nguyen (nguyenminhhuu75@gmail.com)


lOMoARcPSD|13289864

the : ground. One afternoon a customer who is collecting goods from the workshop
reverses his : car into the forks on the forklift. He is injured and his car is damaged.:
What is the principle that would make Nicola’s employer liable for her actions?
A. Contributory negligence
B. Strict liability
C. Vicarious liability
D. Voluntary assumption of risk

In Tame v State of New South Wales (2002) 211 CLR 317 where Tame was given a
false : blood alcohol reading:
A. Whether the police officer was found to have owed Tame a duty of care was
entirely a : question of community standards
B. The psychotic depressive illness Tame developed was found to flow from the
breach of : duty of the police sergeant because it was entirely caused by his
actions that could have : been avoided had he exercised sufficient care
C. Part of the test of reasonable foreseeability is a question of fact
D. Because of the “eggshell skull” principle, pre-existing knowledge of Tame’s
susceptibility : to suffering nervous shock was not required to prove a breach

Which of the following is not a provision of the Wrongs Act 1958 (Vic) relating to:
professional liability?
A. If there are differing peer professional opinions across Australia, the court may
accept : just one of those opinions
B. The court must rely on peer professional opinion even where it considers that
opinion : irrational
C. If there are differing peer professional opinions across Australia, the court may
accept all : of those opinions
D. Peer professional opinion can be considered widely accepted even where it is
not : universally accepted

Which of the following is not correct? The reform to the law of negligence that took :
place in Australia in the early 2000s:

A. Limits the scope of potential liability for negligence


B. Does not apply to claims in contract law
C. Was enacted in all Australian states and territories
D. Covers personal injury

Downloaded by huu nguyen (nguyenminhhuu75@gmail.com)


lOMoARcPSD|13289864

Why was the plaintiff successful in Overseas Tankship (UK) Ltd v Miller Steamship Co
Pty : Ltd (The Wagon Mound No 2) [1967] AC 617 (PC) when another plaintiff failed in
the earlier : related Wagon Mound case?

A. The plaintiffs in the second had better legal representation


B. The legal situation had changed between the two cases
C. The court was more disposed to think about the consequences of the damage
suffered by : so many as a result of the oil spill
D. In the first case, the plaintiffs failed to show that a reasonable man would have
foreseen : the risk of damage from the oil spill

The “neighbour principle” is often viewed as:

A. An objective test
B. A test of the reasonable person
C. A test of the foreseeability of harm
D. An identity test

Which of the following is not correct?


A. If damages are too remote they will not be recoverable
B. The remoteness test will be satisfied where the damage suffered is of the same
type or : kind as foreseeable damage
C. In order for damage to not be too remote, it must be reasonably foreseeable
D. In The Wagon Mound No 1 and The Wagon Mound No 2, where there was an :
unfortunate combination of an oil spill, welding sparks and floating cotton waste,
the loss : was found to be not reasonably foreseeable in the circumstances

Which of the following is not correct? The reform to the law of negligence that took :
place in Australia in the early 2000s:

A. Limits the scope of potential liability for negligence


B. Does not apply to claims in contract law
C. Was enacted in all Australian states and territories
D. Covers personal injury

In Australian Safeway Stores v Zaluzna (1987) 162 CLR 479:

Downloaded by huu nguyen (nguyenminhhuu75@gmail.com)


lOMoARcPSD|13289864

A. The respondent was a lawful entrant upon the land of the respondent,
establishing a : relationship between them and there was therefore a duty of care
owed by the appellant to : avoid a foreseeable risk of injury
B. The respondent was limited by his particular status as entrant without specific
consent of : the appellant
C. The respondent contributed to the accident thus limiting damages
D. There was no liability.

To claim damages a causal link must be established between what two things?

A. Between the defendant’s conduct and the actual monetary value of the
loss/damage
B. The conduct and initiating court proceedings
C. The fact that the plaintiff suffered loss or damage is sufficient. Nothing else is
required
D. The defendants breach and the plaintiff’s injury

In Tame v State of New South Wales (2002) 211 CLR 317 where Tame was given a
false : blood alcohol reading:
A. Whether the police officer was found to have owed Tame a duty of care was
entirely a : question of community standards
B. The psychotic depressive illness Tame developed was found to flow from the
breach of : duty of the police sergeant because it was entirely caused by his
actions that could have : been avoided had he exercised sufficient care
C. Part of the test of reasonable foreseeability is a question of fact
D. Because of the “eggshell skull” principle, pre-existing knowledge of Tame’s
susceptibility : to suffering nervous shock was not required to prove a breach

Where a person gives advice, that advice is relied upon and the advice is incorrect, the :
person giving the advice may be liable in negligence. Which of the following is
incorrect?

A. The “special relationship” described in Mutual Life & Citizens’ Assurance Co Ltd
v Evatt : (1968) 122 CLR 556 has been accepted as the test for determining the
existence of a duty of : care with respect to negligent misstatements

Downloaded by huu nguyen (nguyenminhhuu75@gmail.com)


lOMoARcPSD|13289864

B. The test in Mutual Life & Citizens’ Assurance Co Ltd v Evatt (1968) 122 CLR 556
was : approved in San Sebastian Pty Ltd v Minister Administering Environmental
Planning &: Assessment Act 1979 (1986) 162 CLR 340
C. The duty of care only arises where the “special relationship” involves a person
seeking : advice from the advice-giver, not where the advice was given
unrequested and merely : accepted
D. The advice-giver does not need to be in the business of giving advice

Nicola drives the forklift at her place of work, Rooze’s Roofing. Nicola always leaves
the : forklift in a certain place where she has been told to leave it, with the forks up off
the : ground. One afternoon a customer who is collecting goods from the workshop
reverses his : car into the forks on the forklift. He is injured and his car is damaged.:
What is the principle that would make Nicola’s employer liable for her actions?

A. Contributory negligence
B. Strict liability
C. Vicarious liability
D. Voluntary assumption of risk

Which of the following is not a provision of the Wrongs Act 1958 (Vic) relating to:
professional liability?
A. If there are differing peer professional opinions across Australia, the court may
accept : just one of those opinions
B. The court must rely on peer professional opinion even where it considers that
opinion : irrational
C. If there are differing peer professional opinions across Australia, the court may
accept all : of those opinions
D. Peer professional opinion can be considered widely accepted even where it is
not : universally accepted

QUIZ 4:
The incorrect statement?
A. Partners are not bound when another partner misappropriates monies

Downloaded by huu nguyen (nguyenminhhuu75@gmail.com)


lOMoARcPSD|13289864

B. If the giving of investment advice is within the scope of the firm’s business then
all partners : are jointly and severally liable for advice which is not in a client’s
best interests
C. the client should seek a personal indemnity from a partner
D. special skill is required to bind all partners
Which of the below statements is incorrect?
A. a written and signed notice of a partner’s intention to dissolve the partnership is
required, : where no fixed term has been agreed upon for the duration of the
partnership
B. a continuing guarantee given by or to a partnership is revoked as to future
transactions if there is : a change in the constitution of the partnership
C. where a partnership continues after a fixed term has expired, the rights and
duties of the partners : remain the same, but the partnership becomes a
partnership at will
D. partnership property must be used exclusively for the purposes of the
partnership and in the : manner set out in the partnership agreement
Which legislation controls the actual registration of the firm name?
A. Corporations Act 2001 (Cth)
B. Partnership Act
C. Statute of Frauds 1677
D. Business Names Registration Act 2011 (Cth)
Which of the following statements regarding partnerships is incorrect?
A. The second element of a partnership is carrying out a business in common
B. To be a partnership there must be a mutuality of rights and obligations
C. Each partner must take an active part in the direction and management of the
firm
D. In Degiorgio v Dunn [2004] NSWSC 767 it was held that there was no
partnership because the : business was not run “in common”
Which of the following statements regarding dissolution of partnership is
incorrect?
A. In the event of dissolution, losses must be met first out of profits, followed by
capital, then by : partners in proportion to their share of profits
B. A firm’s assets as contributed by partners to make up capital losses need not be
applied to pay : the firm’s debts/liabilities to non-partners
C. After dissolution, each partner is entitled to advances and residue by the firm
D. After the dissolution, each partner’s authority to bind the firm continues, so far
as necessary for : winding up partnership affairs and completing unfinished
transactions
Which is incorrect statement?

Downloaded by huu nguyen (nguyenminhhuu75@gmail.com)


lOMoARcPSD|13289864

A. If a member of a firm of solicitors acting for a vendor in a sale absconds with the
deposit, : his/her partners are not liable to refund the money
B. In Lloyd v Grace, Smith & Co [1912] AC 716, the firm was held responsible for
the fraud committed : by a managing clerk of a firm, who misappropriated
property while acting within the scope of his : authority
C. In SJ Mackie Pty Ltd v Dalziell Medical Practice Pty Ltd [1989] 2 Qd it was held
that the transfer of a : share to a non-partner breaks the continuity of the firm,
constituting a new firm/partnership of the : remaining former partners and the
new member
D. Partnership agreements can contain provisions to enable the transition from
one firm to another : to be effected without the disruption of a formal winding up.
Which of the following statements is not correct?
A. as an agent, a partner is able to bind the other partners and, as principal, be
bound by the actions : of the other partners
B. in New South Wales, Victoria, Queensland, South Australia, Western Australia
and: Tasmania provision for limited partnerships is made in the Partnership Act
C. incorporated limited partnerships have been introduced in all Australian States
and Territories
D. the Partnership Act provides that the rules of the common law and equity are to
continue in force : except insofar as they are inconsistent with the Act
Find one incorrect statement?
A. In the absence of special statutory provision, although each partner is liable with
the others for : the whole of the debts of the firm, their liability is only joint
B. A creditor can bring only one action against members of a partnership and any
partner can insist : that the action be stayed until all other partners are joined as
parties.
C. A person admitted into an existing firm, liability may be incurred where it is
specially agreed upon
D. A creditor can enforce liability against an incoming partner whether or not
he/she is a party to : the contract
What constitutes an exception to the usual number of a partnership being 20
partners?
A. If one of the partners is married-his/her husband or wife is automatically a
partner
B. The exception relates only to limited partnerships
C. No more than 20 are allowed
D. The Corporations Regulations 2001 (Cth) provide greater numbers in certain
specified : professions
Which statement regarding an outsize partnership is incorrect?
A. it has more than 20 partners

Downloaded by huu nguyen (nguyenminhhuu75@gmail.com)


lOMoARcPSD|13289864

B. its partnership agreement is invalid


C. it is liable to a criminal penalty ($500)
D. its agreement does not affect the enforceability of contracts or other
arrangements made
Which finding is correct?: In Popat v Schonchhatra (1997) 3 All ER 800, the Court
decided:
A. Popat was entitled to half the profits on the sale of the business and to a share
of the profits : that had accrued after the dissolution of the partnership, but before
the final settlement of : accounts.
B. Popat was not entitled to any profits on the sale of the business, as there was
no partnership : agreement and he was further not entitled to a share of the
profits accrued after the dissolution of : the partnership, but before the final
settlement of accounts.
C. Popat was entitled to half the profits on the sale of the business, but not to a
share of the profits : that had accrued after the dissolution of the partnership, but
before the final settlement of : accounts.
D. Popat was not entitled to half the profits on the sale of the business, but he was
entitled to a : share of the profits that had accrued after the dissolution of the
partnership but before the final : settlement of accounts.
Which of the following statements is incorrect?
A. a separate legal entity can commit civil wrongs and engage in criminal conduct
B. a public company has serious compliance obligations
C. a partnership must be in writing
D. a separate legal entity is entitled to own property, pay tax and enter into
contracts
Which of the below does not apply to the Partnership Act?
A. The rules in the Partnership Acts assist in determining whether a business is
being carried on in : common
B. The partnership under the Act must be registered with ASIC
C. Each rule states a negative: i.e. that a certain fact does not of itself create a
partnership, but is : only indicative of a partnership
D. Joint or part ownership or joint tenancy, or tenancy in common whether or not
the : owners/tenants share the profits, does not of itself create a partnership as to
anything so : held/owned
Which of the following statements regarding the written law is incorrect?
A. Under s 115 of the Corporations Act 2001 (Cth), the maximum number of
persons who may form a : partnership for the acquisition of gain is 20
B. Under the Corporations Regulations 2001 (Cth), reg 2A.1.01, partnerships of
more than 20 : partners may be formed for certain professions/callings

Downloaded by huu nguyen (nguyenminhhuu75@gmail.com)


lOMoARcPSD|13289864

C. The Corporations Regulations 2001 (Cth), reg 2A.1.01 sets a maxima of


partners for medical and : legal practitioners, veterinary surgeons, patent and
trademark attorneys, sharebrokers and : stockbrokers and pharmaceutical
chemists
D. The Corporations Regulations 2001 (Cth), reg 2A.1.01 does not set a maxima
of partners for : architects and accountants
Which court finding is incorrect?
A. In Mercantile Credit Co Ltd v Garrod [1962] 3 All ER 1103, damages were not
recovered even : though, from the plaintiff’s perspective, the sale of the car was
within the usual course of : business.
B. In Goldberg v Jenkins (1889) 15 VLR 36, the firm was not bound to the
transaction because : borrowing money on behalf of the firm at over 60% interest
when comparable rates were between : 6% and 10% was beyond ‘the usual
way’.
C. In Construction Engineering Pty Ltd v Hexyl Pty Ltd (1985) 155 CLR 541, Hexyl
was not liable : because the construction contract appeared to be between
Construction Engineering and Tembel
D. If judgment is obtained against one or more partners of a firm, no action may be
taken against the : other partners, even if satisfaction cannot be obtained from
the partner(s) sued
Cribb v Korn (1911) 12 CLR 205 established:
A. Joint ownership creates a partnership
B. A joint tenancy is just another name for a partnership
C. The sharing of joint returns does not in itself create a partnership
D. The statutory rules can be excluded by agreement
The incorrect statement is?
A. A partner has express actual and implied actual authority to engage in certain
activities with third: parties
B. A partner cannot pledge/sell partnership property, incur and pay debts on
partnership : accounts or hire employees
C. Contravention of an agreement to restrict a partner’s authority is not binding on
the firm if notice : of the agreement has been given
D. In Polkinghorne v Holland (1934) 51 CLR 143, the firm was held liable because
Holland provided : the advice in his role as solicitor; thus in the ordinary course of
the firm’s business
Find one incorrect statement?
A. In the absence of special statutory provision, although each partner is liable with
the others for : the whole of the debts of the firm, their liability is only joint

Downloaded by huu nguyen (nguyenminhhuu75@gmail.com)


lOMoARcPSD|13289864

B. A creditor can bring only one action against members of a partnership and any
partner can insist : that the action be stayed until all other partners are joined as
parties.
C. A person admitted into an existing firm, liability may be incurred where it is
specially agreed upon
D. A creditor can enforce liability against an incoming partner whether or not
he/she is a party to : the contract
Regarding partnerships, which of the following statements is incorrect?
A. Partnerships are created with a view to profit, so partners must make a profit: b
A partner does not have to have a direct claim to a share of the profits
B. Associations and charities are not partnerships, as profits come from ancillary
business activities : and are reinvested, not distributed as dividends to their
members
C. The Partnership Acts do not govern members of a corporation incorporated under
the provisions : of the Corporations Act 2001 (Cth), a special Act of Parliament, or
Royal Charter
Which of the below statements is incorrect?
A. a written and signed notice of a partner’s intention to dissolve the partnership is
required, : where no fixed term has been agreed upon for the duration of the
partnership
B. a continuing guarantee given by or to a partnership is revoked as to future
transactions if there is : a change in the constitution of the partnership
C. where a partnership continues after a fixed term has expired, the rights and duties
of the partners : remain the same, but the partnership becomes a partnership at
will
D. partnership property must be used exclusively for the purposes of the partnership
and in the : manner set out in the partnership agreement
Which of the following statements is incorrect?
A. a separate legal entity can commit civil wrongs and engage in criminal conduct
B. a public company has serious compliance obligations
C. a partnership must be in writing
D. a separate legal entity is entitled to own property, pay tax and enter into contracts
The incorrect statement?
A. Partners are not bound when another partner misappropriates monies
B. If the giving of investment advice is within the scope of the firm’s business then
all partners : are jointly and severally liable for advice which is not in a client’s
best interests
C. the client should seek a personal indemnity from a partner
D. special skill is required to bind all partners

Downloaded by huu nguyen (nguyenminhhuu75@gmail.com)


lOMoARcPSD|13289864

Which of the following statements is not correct?


A. as an agent, a partner is able to bind the other partners and, as principal, be bound
by the actions : of the other partners
B. in New South Wales, Victoria, Queensland, South Australia, Western Australia
and: Tasmania provision for limited partnerships is made in the Partnership Act
C. incorporated limited partnerships have been introduced in all Australian States
and Territories
D. the Partnership Act provides that the rules of the common law and equity are to
continue in force : except insofar as they are inconsistent with the Act
Which of the below statement is incorrect?
A. Partnership property is liable to be seized for the private (personal) debt of
a partner and : made liable on a judgment against the partnership
B. A creditor who has obtained judgment in respect of the separate debt of a partner
may obtain an : order charging that partner’s interest in the partnership property
and profits with the amount of the : debt and interest.
C. A creditor who has obtained judgment in respect of the separate debt of a partner
may obtain by : an order the appointment of a receiver of that partner’s share of
profits and of any other money : which may be coming to the partner in respect of
the partnership.
D. The two sources of law relevant when considering how partners bind their
partners when dealing : with third parties are the common law (including equity)
of agency and the Partnership Act. The : common law complements the
provisions of the Partnership Act in relation to the authority that an : agent has.
What constitutes an exception to the usual number of a partnership being 20 partners?
A. If one of the partners is married-his/her husband or wife is automatically a
partner
B. The exception relates only to limited partnerships
C. No more than 20 are allowed
D. The Corporations Regulations 2001 (Cth) provide greater numbers in certain
specified : professions
Which of the following statements regarding partnerships is incorrect?
A. The second element of a partnership is carrying out a business in common
B. To be a partnership there must be a mutuality of rights and obligations
C. Each partner must take an active part in the direction and management of
the firm
D. In Degiorgio v Dunn [2004] NSWSC 767 it was held that there was no
partnership because the : business was not run “in common”
Which is incorrect statement?
A. If a member of a firm of solicitors acting for a vendor in a sale absconds with the
deposit, : his/her partners are not liable to refund the money

Downloaded by huu nguyen (nguyenminhhuu75@gmail.com)


lOMoARcPSD|13289864

B. In Lloyd v Grace, Smith & Co [1912] AC 716, the firm was held responsible for the
fraud committed : by a managing clerk of a firm, who misappropriated property
while acting within the scope of his : authority
C. In SJ Mackie Pty Ltd v Dalziell Medical Practice Pty Ltd [1989] 2 Qd it was held
that the transfer of a : share to a non-partner breaks the continuity of the firm,
constituting a new firm/partnership of the : remaining former partners and the
new member
D. Partnership agreements can contain provisions to enable the transition from one
firm to another : to be effected without the disruption of a formal winding up.
Which statement is incorrect?
A. carrying on a business implies repetition (Smith v Anderson (1880) 15 Ch D 247)
B. partnership may be formed in order to undertake a single business transaction
C. in Khan v Miah [2000] 1 WLR 2123 it was held that work, such as finding,
acquiring and fitting out : a shop/restaurant, is undertaken with a view to profit
D. in Keith Spicer Ltd v Mansell [1970] 1 All ER 462 it was held that ordering goods
and opening a : joint bank account in contemplation of a business are insufficient
for a partnership
Which court finding is incorrect?
A. In Mercantile Credit Co Ltd v Garrod [1962] 3 All ER 1103, damages were not
recovered even : though, from the plaintiff’s perspective, the sale of the car was
within the usual course of : business.
B. In Goldberg v Jenkins (1889) 15 VLR 36, the firm was not bound to the
transaction because : borrowing money on behalf of the firm at over 60% interest
when comparable rates were between : 6% and 10% was beyond ‘the usual way’.
C. In Construction Engineering Pty Ltd v Hexyl Pty Ltd (1985) 155 CLR 541, Hexyl
was not liable : because the construction contract appeared to be between
Construction Engineering and Tembel
D. If judgment is obtained against one or more partners of a firm, no action may be
taken against the : other partners, even if satisfaction cannot be obtained from
the partner(s) sued
Find incorrect statement
A. A term often included in a partnership agreement that allows other partners to
purchase a : retiring/deceased partner’s interest at an agreed valuation avoids the
disruption of a formal winding
B. A partnership cannot be dissolved because the business is carried on at a loss
C. Partnerships can be dissolved because of mutual incompatibility, making it
impossible for partners : to carry on a business

Downloaded by huu nguyen (nguyenminhhuu75@gmail.com)


lOMoARcPSD|13289864

D. On dissolution, partnership property can be applied towards the payment of


partnership : liabilities/debts and any surplus can be distributed among the
partners

Which below is incorrect? A firm is not bound by the acts of a partner if:

A. the act is not of the usual business of the kind carried on by the firm
B. the partner exceeds his/her actual authority in the particular matter
C. the person with whom the partner is dealing knows that he/she has exceeded
his/her authority
D. the person with whom the partner is dealing knows or believes him/her to be a
partner

Find one incorrect statement?

A. In the absence of special statutory provision, although each partner is liable with
the others for : the whole of the debts of the firm, their liability is only joint
B. A creditor can bring only one action against members of a partnership and any
partner can insist : that the action be stayed until all other partners are joined as
parties.
C. A person admitted into an existing firm, liability may be incurred where it is
specially agreed upon
D. A creditor can enforce liability against an incoming partner whether or not
he/she is a party to : the contract

Legislation in Victoria: What Act regulates partnerships in Victoria?

A. Partnership Act 1892


B. Partnership Act 1958 (Vic)
C. Corporations Act 2001 (Cth), s 20
D. Competition & Consumer Act 2010 (Cth)

Which statement regarding an outsize partnership is incorrect?

Downloaded by huu nguyen (nguyenminhhuu75@gmail.com)


lOMoARcPSD|13289864

A. it has more than 20 partners


B. its partnership agreement is invalid
C. it is liable to a criminal penalty ($500)
D. its agreement does not affect the enforceability of contracts or other
arrangements made

Which of the following statements is incorrect?

A. a separate legal entity can commit civil wrongs and engage in criminal conduct
B. a public company has serious compliance obligations
C. a partnership must be in writing
D. a separate legal entity is entitled to own property, pay tax and enter into
contracts

Which court finding is incorrect?

A. In Mercantile Credit Co Ltd v Garrod [1962] 3 All ER 1103, damages were not
recovered even : though, from the plaintiff’s perspective, the sale of the car was
within the usual course of : business.
B. In Goldberg v Jenkins (1889) 15 VLR 36, the firm was not bound to the
transaction because : borrowing money on behalf of the firm at over 60% interest
when comparable rates were between : 6% and 10% was beyond ‘the usual way’.
C. In Construction Engineering Pty Ltd v Hexyl Pty Ltd (1985) 155 CLR 541, Hexyl
was not liable : because the construction contract appeared to be between
Construction Engineering and Tembel
D. If judgment is obtained against one or more partners of a firm, no action may be
taken against the : other partners, even if satisfaction cannot be obtained from
the partner(s) sued

Which statement regarding persons of unsound mind and minors is incorrect?

A. a partner of unsound mind is capable of binding the firm and of being bound by
co-partners, : unless proven that he/she was of unsound mind when the
partnership was entered into and the : other partners knew this

Downloaded by huu nguyen (nguyenminhhuu75@gmail.com)


lOMoARcPSD|13289864

B. if a partner who is a minor enters into a contract with a third party on behalf of
the firm, the : minor is liable as far as private assets are concerned
C. a creditor who has obtained judgment against the firm may not seize the minor’s
separate : property
D. a minor will become liable as an ordinary partner when he/she attains majority
the partnership is : not repudiated within a reasonable time

Which below statements is incorrect?

A. Partners share the profits in a partnership


B. In a partnership, partners have limited liability for the business debts
C. A partner is also an agent of the partnership
D. Agreement may be implied in a partnership

Which statement is incorrect?

A. carrying on a business implies repetition (Smith v Anderson (1880) 15 Ch D 247)


B. partnership may be formed in order to undertake a single business transaction
C. in Khan v Miah [2000] 1 WLR 2123 it was held that work, such as finding,
acquiring and fitting out : a shop/restaurant, is undertaken with a view to profit
D. in Keith Spicer Ltd v Mansell [1970] 1 All ER 462 it was held that ordering goods
and opening a : joint bank account in contemplation of a business are insufficient
for a partnership

Which statement regarding dissolution of a partnership is incorrect?

A. Partners are not entitled to have partnership property applied towards the
payment of : partnership liabilities
B. After payment of the firm’s liabilities, partners do not have the right to have
surplus assets : applied in payment of what may be due to the partners
respectively
C. On the termination of the partnership, any partner can apply to the court for a
decree to dissolve : the partnership and appoint a receiver to wind up the firm’s
business/affairs

Downloaded by huu nguyen (nguyenminhhuu75@gmail.com)


lOMoARcPSD|13289864

D. Should a sequestration order be made against a partner, a creditor of the firm


cannot receive a : dividend out of the bankrupt’s separate property until all
separate creditors of the bankrupt partner : have been paid in full.

Which of the below statement is incorrect?

A. Partnership property is liable to be seized for the private (personal) debt of a


partner and : made liable on a judgment against the partnership
B. A creditor who has obtained judgment in respect of the separate debt of a
partner may obtain an : order charging that partner’s interest in the partnership
property and profits with the amount of the : debt and interest.
C. A creditor who has obtained judgment in respect of the separate debt of a
partner may obtain by : an order the appointment of a receiver of that partner’s
share of profits and of any other money : which may be coming to the partner in
respect of the partnership.
D. The two sources of law relevant when considering how partners bind their
partners when dealing : with third parties are the common law (including equity)
of agency and the Partnership Act. The : common law complements the
provisions of the Partnership Act in relation to the authority that an : agent has.

Which court decision regarding fiduciary duties is incorrect?

A. In Chan v Zacharia (1984) 154 CLR 178, the court decided that that the fiduciary
relationship : continued until the partnership had finally been wound up.
B. In United Dominion Corp Ltd v Brian Pty Ltd (1985) 157 CLR 1, the Court found
that a fiduciary : relationship, with attendant fiduciary obligations ordinarily exist
between prospective partners who : have embarked upon the conduct of the
partnership business or venture before the precise terms of : any partnership
agreement have been settled
C. In Popat v Schonchhatra (1997) 3 ALLER 800 in the absence of a statement to
the contrary, : partners are entitled to an equal share of profits
D. in Harvey v Harvey (1970) 120 CLR 529, the Court held that if a partner
receives an additional : annual fee to be “on call” for that client, the fee belongs
to the partnership.

Downloaded by huu nguyen (nguyenminhhuu75@gmail.com)


lOMoARcPSD|13289864

Which of the following statements regarding limited partners is incorrect?

A. A limited partner may assign his/her share in the partnership with the consent of
the general : partners
B. The consent of a limited partner is not required to admit a person as a partner
C. If a limited partner has suffered his/her share of the partnership property to be
charged for a : separate debt, the other partners are entitled to dissolve the
partnership
D. A limited partner is not entitled to dissolve the partnership by notice.

Which legislation regulates partnerships in NSW?

A. Partnership Act 1892 (NSW)*


B. Competition & Consumer Act 2001 (Cth)
C. Partnership Act 1958 (Vic)
D. Partnership Act 1895

Find incorrect statement

A. A term often included in a partnership agreement that allows other partners to


purchase a : retiring/deceased partner’s interest at an agreed valuation avoids the
disruption of a formal winding
B. A partnership cannot be dissolved because the business is carried on at a loss
C. Partnerships can be dissolved because of mutual incompatibility, making it
impossible for partners : to carry on a business
D. On dissolution, partnership property can be applied towards the payment of
partnership : liabilities/debts and any surplus can be distributed among the
partners

Quiz 1:
1. Sources of law: Sources of law in Australia do not include:
A. Consolidating statutes
B. Judge-made law
C. Repealed statutes
D. Equity

Downloaded by huu nguyen (nguyenminhhuu75@gmail.com)


lOMoARcPSD|13289864

2. Criminal offences: Which of the following is incorrect with respect to criminal


offences?
A. Indictable offences are generally the more serious offences.
B. The prosecution must prove its case beyond reasonable doubt.
C. Summary offences are determined by a magistrate without a jury.
D. A committal hearing is held before most summary offence matters. orrect Answer
3. Common law: Which one of the following statements about common law is
A. Common law is judge made law Correct!
B. Common law overrides statute law
C. Common law is made by Parliaments
D. Common law and equity are the same types of laws and provide the same
remedies
4. The High Court’s decision in Waltons Stores (Interstate) Ltd v Maher (1988)
164 CLR 387:
A. Extended the doctrine of promissory estoppel such that it applies even where
the : promisor does not create or encourage the false assumption of the
promisee
B. Extended the doctrine of equitable estoppels such that it applies even where
there is no : pre-existing contractual relationship between the parties
C. Restricted the doctrine of equitable estoppel such that, where the contract is of
a : commercial nature, the doctrine will not apply
D. Restricted the doctrine of promissory estoppel such that where an innocent
third party : has been affected, no remedy can be ordered
5.Which are some of the main features of the Electronic Transactions Act 1999
(Cth)
A. validity of electronic transactions
B. recognition of writing by electronic means
C. Recognition of retaining information in electronic form
D. All of the above
6. In Ashton v Pratt [2015] NSWCA 12 what was the main reason for the court
deciding that : there was no intention to create a legally binding contract?

Downloaded by huu nguyen (nguyenminhhuu75@gmail.com)


lOMoARcPSD|13289864

A. The agreement was not reduced to writing


B. The verbal language of the agreement greatly lacked detail from either party and
did not : indicate definite obligations
C. Pratt lacked the necessary contractual capacity due to his age and ill health
7. Which of the following statements about bilateral and unilateral contracts is
not correct?
A. More than one party is required for both bilateral and unilateral contracts
B. A unilateral contract does not require consideration but simply a promise to
perform an : act is sufficient
C. Both parties are obligated to perform their promises in a bilateral contract
D. Only one of the parties is obligated to perform an action in a unilateral contract
8.Which of the following is not a class of persons regarded by the law as wholly
or partly : incapable of entering into legally binding contracts?
A. Intoxicated persons
B. Minors You Answered
C. Women
D. Mentally incapacitated persons
9. The legislature: Exclusive powers are those that:
A. Can be exercised by either the Commonwealth or the States
B. Can be exercised only by the Commonwealth
C. Can be exercised only by the States
D. Where there is any inconsistency, the Commonwealth law will prevail
10. Why are illusory terms in a contract problematic?
A. they are vague or ambiguous so they fail to create a legal obligation
B. They exist in one party’s imaginary belief only
C. Courts have difficulty interpreting them
D. Interpreting them in any way at all could result in an unfair decision
11. Delegated legislation: Which of the following is not correct?: Delegated Legislation:
A. Is subordinate legislation
B. Is made under the authority of an Act of Parliament
C. Often contains more detailed rules than those of the associated Act
D. Can only be made Government Ministers
12.Native Title: Native Title was first recognised in which of the following cases?
A. Mabo v State of Queensland (No 2) (1992) 175 CLR 1

Downloaded by huu nguyen (nguyenminhhuu75@gmail.com)


lOMoARcPSD|13289864

B. Wik Peoples v State of Queensland (1996) 187 CLR 1


C. Brandy v Human Rights and Equal Opportunity Commission (1995) 183 CLR 245
D. Commonwealth v Jones (1901) 174 CLR 25
13. Jack was subpoenaed to give evidence on Rods behalf. Jack claimed Rod promised
him : $2000 to give evidence. Would Jack recover that money?
A. No - Jack’s public duty is to give evidence in response to that subpoena. Correct!
B. No this is illegal
C. Yes if Rod put that in writing
D. Always binding because promises should not be broken
14. Business law: Which of the following no longer regulates business or commercial
law?
A. Trade Practices Act 1974 (Cth)
B. Competition and Consumer act 2010 (cth)
C. Corporations Act 2001 (cth)
D. The law of contract
15. The main legal issue in Felthouse v Bindley (1862) 11CB (NS) 869 was:
A. the contract was formed due to offer and acceptance being proven
B. An offer to sell a horse was simply an invitation to treat
C. generally silence /lack of action does not constitute acceptance of an offer
D. An offer can be revoked at any time prior to acceptance

Quiz 2:
1. Who/what are exempt from the provisions of s18 of the Australian Consumer
Law?
A. No-one is exempt
B. Corporations
C. Information providers such as media outlets
D. Advertisers
2. Which of the following is not a way to discharge a contract?
A. By performance
B. By operation of law
C. Through breach
D. By agreement
3. Which of the following statements about the parol evidence rule is true.?

Downloaded by huu nguyen (nguyenminhhuu75@gmail.com)


lOMoARcPSD|13289864

A. It applies only to oral (parol) evidence


B. It does not apply to written contracts
C. It does not apply where it can be shown that the written contract was not
intended to : be a complete record of the agreement
D. It applies only to previous drafts of a written contract
4, In Curtis v Chemical Cleaning & Dyeing Co Ltd [1951] 1 KB 805:
A. The exemption clause in the sales agreement for the drycleaning was effective
B. The exemption clause in the sales agreement for the drycleaning was not
sufficiently : explained to the customer
C. The exemption clause in the sales agreement for the drycleaning was signed and :
therefore effective
5. Which of the following is not correct with respect to mitigation of loss?
A. There is a duty on the person claiming the damages to take all reasonable steps
to : mitigate their losses
B. Where loss could have been mitigated and was not, a person cannot claim that
loss
C. The burden of proving that losses were mitigated falls upon the party claiming
the : damages ie the defendant
D. The question of whether a person took all reasonable steps to mitigate is one of
fact : dependent upon the particular circumstances
6. Which of the following is not correct with respect to termination by
subsequent : agreement?
A. A subsequent simple contract can cancel a contract where one party has
completed their : obligations and the other has not
B. A subsequent agreement may cancel an original contract
C. A subsequent agreement must be valid to rescind an earlier contract
D. A subsequent simple contract can cancel a contract where there is still something
to be : done by each party
7. In the case of restraint of trade with respect to employment:

A. The court does not easily allow parties to contract out of their means of
employment
B. Whatever the parties determine will be accepted by the court
C. There are no restraints on employees unless set out in a contract with their
employer

Downloaded by huu nguyen (nguyenminhhuu75@gmail.com)


lOMoARcPSD|13289864

D. These agreements are construed as strictly as restraints of trade between


purchaser and : vendor of a business
8. Krell v Henry [1903] 2 KB 740 illustrated what point?
A. That death can be regarded as frustration
B. That the parties were entitled to a discharge
C. That the parties must perform regardless of circumstance
D. That the parties are discharged if an event occurs which forms the basis of the
contract, : but which prevents performance at no fault of either party
9. Contingent conditions include:
A. Accord and satisfaction and consideration
B. Conditions subsequent and conditions precedent
C. Warranties and innominate terms
D. Releases and mutual terminations
10. Fraser crashes his car with his friend Angus as a passenger. Angus is
injured. Angus was : not wearing a seatbelt. Which of the following is not
correct?
A. Liability would likely be apportioned between Fraser and Angus
B. Damages cannot be apportioned for breach of contractual duty of care where
there is : contributory negligence
C. Damages awarded to Angus, if any, would be reduced by his contributory
negligence
D. Angus’ contributory negligence relates only to his own safety, not to that of
others
11.Which of the following is NOT a requirement for an assignment of a debt or other
chose : in action under various State statutes?
A. The assignment is absolute and not by way of charge;
B. Fourteen days’ notice in writing is provided to debtor;
C. the assignment is in writing;
D. If not in writing then notice of the assignment is given by notice to the debtor.
12. The parol evidence rule performs much the same function as:
A. A collateral warranty
B. An oral collateral contract
C. Consideration
D. A merger clause

Downloaded by huu nguyen (nguyenminhhuu75@gmail.com)


lOMoARcPSD|13289864

13. Which of the following is not one of the special relationships in which the
onus of : proving that undue influence was not employed shifts to the denying Party?
A. Paren and child where the child is living independently
B. Solicitor and client
C. Trustee and beneficiary
D. Doctor and patient
14. How may a court view substantial performance of a contract?:
A. The contract is complete, but the uncomplete portion may be allowed for as a
reduction : of the full contract price
B. it will not grant a discharge
C. It will sever the incomplete parts
D. It will treat the contract as breached
15. How is repudiation determined by the court?
A. Subjectively
B. By considering the actions of the repudiating party
C. By viewing the circumstances of the repudiation
D. Objectively

QUIZ 3
Question 1: To claim damages a causal link must be established between what two
things?
A. Between the defendant’s conduct and the actual monetary value of the
loss/damage
B. The conduct and initiating court proceedings
C. The fact that the plaintiff suffered loss or damage is sufficient. Nothing else is
required
D. The defendants breach and the plaintiff’s injury

Question 2:Which of the following is not correct?


A. If damages are too remote they will not be recoverable
B. The remoteness test will be satisfied where the damage suffered is of the same
type or : kind as foreseeable damage
C. In order for damage to not be too remote, it must be reasonably foreseeable
D. In The Wagon Mound No 1 and The Wagon Mound No 2, where there was an :
unfortunate combination of an oil spill, welding sparks and floating cotton waste, the loss
: was found to be not reasonably foreseeable in the circumstances

Downloaded by huu nguyen (nguyenminhhuu75@gmail.com)


lOMoARcPSD|13289864

Question 3: Which of the following is not a provision of the Wrongs Act 1958 (Vic)
relating to:
professional liability?
A. If there are differing peer professional opinions across Australia, the court may
accept : just one of those opinions
B. The court must rely on peer professional opinion even where it considers that
opinion : irrational
C. If there are differing peer professional opinions across Australia, the court may
accept all : of those opinions
D. Peer professional opinion can be considered widely accepted even where it is
not :
universally accepted

Question 4 : Why was the plaintiff successful in Overseas Tankship (UK) Ltd v Miller
Steamship Co Pty : Ltd (The Wagon Mound No 2) [1967] AC 617 (PC) when another
plaintiff failed in the earlier : related Wagon Mound case?
A. The plaintiffs in the second had better legal representation
B. The legal situation had changed between the two cases
C. The court was more disposed to think about the consequences of the damage
suffered by : so many as a result of the oil spill
D. In the first case, the plaintiffs failed to show that a reasonable man would have
foreseen : the risk of damage from the oil spill

Question 5 Where a person gives advice, that advice is relied upon and the advice is
incorrect, the : person giving the advice may be liable in negligence. Which of the
following is incorrect?
A. The “special relationship” described in Mutual Life & Citizens’ Assurance Co Ltd v
Evatt : (1968) 122 CLR 556 has been accepted as the test for determining the
existence of a duty of : care with respect to negligent misstatements
B. The test in Mutual Life & Citizens’ Assurance Co Ltd v Evatt (1968) 122 CLR 556
was : approved in San Sebastian Pty Ltd v Minister Administering Environmental
Planning &: Assessment Act 1979 (1986) 162 CLR 340
C. The duty of care only arises where the “special relationship” involves a person
seeking : advice from the advice-giver, not where the advice was given
unrequested and merely : accepted
D. The advice-giver does not need to be in the business of giving advice

Question 6: The “neighbour principle” is often viewed as:


A. An objective test

Downloaded by huu nguyen (nguyenminhhuu75@gmail.com)


lOMoARcPSD|13289864

B. A test of the reasonable person


C. A test of the foreseeability of harm
D. An identity test

Question 7: In Tame v State of New South Wales (2002) 211 CLR 317 where Tame was
given a false : blood alcohol reading:
A. Whether the police officer was found to have owed Tame a duty of care was
entirely a : question of community standards
B. The psychotic depressive illness Tame developed was found to flow from the
breach of : duty of the police sergeant because it was entirely caused by his
actions that could have : been avoided had he exercised sufficient care
C. Part of the test of reasonable foreseeability is a question of fact
D. Because of the “eggshell skull” principle, pre-existing knowledge of Tame’s
susceptibility: to suffering nervous shock was not required to prove a breach

Question 8:Nicola drives the forklift at her place of work, Rooze’s Roofing. Nicola always
leaves the : forklift in a certain place where she has been told to leave it, with the forks
up off the : ground. One afternoon a customer who is collecting goods from the
workshop reverses his : car into the forks on the forklift. He is injured and his car is
damaged.: What is the principle that would make Nicola’s employer liable for her
actions?
A. Contributory negligence
B. Strict liabilityCorrect!
C. Vicarious liability
D. Voluntary assumption of risk

Question 9: Which of the following is not correct? The reform to the law of negligence
that took : place in Australia in the early 2000s:
A. Limits the scope of potential liability for negligence
B. Does not apply to claims in contract law
C. Was enacted in all Australian states and territories
D. Covers personal injury

Question 10: In Australian Safeway Stores v Zaluzna (1987) 162 CLR 479:
A. The respondent was a lawful entrant upon the land of the respondent,
establishing a : relationship between them and there was therefore a duty of care
owed by the appellant to : avoid a foreseeable risk of injury
B. The respondent was limited by his particular status as entrant without specific
consent of : the appellant

Downloaded by huu nguyen (nguyenminhhuu75@gmail.com)


lOMoARcPSD|13289864

C. The respondent contributed to the accident thus limiting damages


D. There was no liability.

Downloaded by huu nguyen (nguyenminhhuu75@gmail.com)


lOMoARcPSD|13289864

Downloaded by huu nguyen (nguyenminhhuu75@gmail.com)


lOMoARcPSD|13289864

Downloaded by huu nguyen (nguyenminhhuu75@gmail.com)


lOMoARcPSD|13289864

Downloaded by huu nguyen (nguyenminhhuu75@gmail.com)


lOMoARcPSD|13289864

Downloaded by huu nguyen (nguyenminhhuu75@gmail.com)


lOMoARcPSD|13289864

Downloaded by huu nguyen (nguyenminhhuu75@gmail.com)


lOMoARcPSD|13289864

Downloaded by huu nguyen (nguyenminhhuu75@gmail.com)


lOMoARcPSD|13289864

Downloaded by huu nguyen (nguyenminhhuu75@gmail.com)


lOMoARcPSD|13289864

Downloaded by huu nguyen (nguyenminhhuu75@gmail.com)


lOMoARcPSD|13289864

Downloaded by huu nguyen (nguyenminhhuu75@gmail.com)


lOMoARcPSD|13289864

Downloaded by huu nguyen (nguyenminhhuu75@gmail.com)


lOMoARcPSD|13289864

Downloaded by huu nguyen (nguyenminhhuu75@gmail.com)


lOMoARcPSD|13289864

Downloaded by huu nguyen (nguyenminhhuu75@gmail.com)


lOMoARcPSD|13289864

Quiz 4:
Question 1:Which is incorrect statement?
A. If a member of a firm of solicitors acting for a vendor in a sale absconds with
the deposit, : his/her partners are not liable to refund the money
B. In Lloyd v Grace, Smith & Co [1912] AC 716, the firm was held responsible for
the fraud committed : by a managing clerk of a firm, who misappropriated
property while acting within the scope of his : authority
C. In SJ Mackie Pty Ltd v Dalziell Medical Practice Pty Ltd [1989] 2 Qd it was held
that the transfer of a : share to a non-partner breaks the continuity of the firm,
constituting a new firm/partnership of the : remaining former partners and the
new member
D. Partnership agreements can contain provisions to enable the transition from one
firm to another : to be effected without the disruption of a formal winding up.
Question 2: Find incorrect statement
A. A term often included in a partnership agreement that allows other partners to
purchase a : retiring/deceased partner’s interest at an agreed valuation avoids the
disruption of a formal winding
B. A partnership cannot be dissolved because the business is carried on at a loss
C. Partnerships can be dissolved because of mutual incompatibility, making it
impossible for partners : to carry on a business
D. On dissolution, partnership property can be applied towards the payment of
partnership : liabilities/debts and any surplus can be distributed among the
partners

Downloaded by huu nguyen (nguyenminhhuu75@gmail.com)


lOMoARcPSD|13289864

Q3: Find one incorrect statement?


A. In the absence of special statutory provision, although each partner is liable with
the others for : the whole of the debts of the firm, their liability is only joint
B. A creditor can bring only one action against members of a partnership and any
partner can insist : that the action be stayed until all other partners are joined as
parties.
C. A person admitted into an existing firm, liability may be incurred where it is
specially agreed upon
D. A creditor can enforce liability against an incoming partner whether or not he/she
is a party to : the contract
Q4: Find a statement that is incorrect
A. A retiring partner should give specific notice of their retirement to persons with
whom the firm : has had dealings to avoid being made liable for debts incurred
after retirement.
B. Persons who represents themselves, or knowingly allow themselves to be
represented, as a : partner, are liable as a partner to anyone who has on the faith
of such representation given credit to : the firm
C. A wrongful act/omission includes breach of contract or of fiduciary duty,
negligent : misrepresentation and misleading or deceptive conduct
D. Innocent partners are liable for wrongful acts of other partners made in the
ordinary conduct of : business
Q5: Which statement regarding partnership is incorrect?

A. A partnerships (or firm) is a way of gathering resources or expertise for major


projects
B. Partnerships do not avoid taking on the formality and expense of an incorporated
company
C. Partners in a partnership complement each other with their skills and bring in new
capital and : broader funding options
Q6: Which of the following statements regarding the written law is incorrect?
A. Under s 115 of the Corporations Act 2001 (Cth), the maximum number of persons
who may form a : partnership for the acquisition of gain is 20
B. Under the Corporations Regulations 2001 (Cth), reg 2A.1.01, partnerships of
more than 20 : partners may be formed for certain professions/callings
C. The Corporations Regulations 2001 (Cth), reg 2A.1.01 sets a maxima of partners
for medical and : legal practitioners, veterinary surgeons, patent and trademark
attorneys, sharebrokers and : stockbrokers and pharmaceutical chemists
D. The Corporations Regulations 2001 (Cth), reg 2A.1.01 does not set a maxima of
partners for : architects and accountants

Downloaded by huu nguyen (nguyenminhhuu75@gmail.com)


lOMoARcPSD|13289864

Q7: Which statement regarding persons of unsound mind and minors is


incorrect?
A. a partner of unsound mind is capable of binding the firm and of being bound by
co-partners, : unless proven that he/she was of unsound mind when the
partnership was entered into and the : other partners knew this
B. if a partner who is a minor enters into a contract with a third party on behalf of
the firm, the : minor is liable as far as private assets are concerned
C. a creditor who has obtained judgment against the firm may not seize the minor’s
separate : property
D. a minor will become liable as an ordinary partner when he/she attains majority
the partnership is : not repudiated within a reasonable time
Q8: Which of the following statements is not correct?
A. as an agent, a partner is able to bind the other partners and, as principal, be bound
by the actions : of the other partners
B. in New South Wales, Victoria, Queensland, South Australia, Western Australia
and: Tasmania provision for limited partnerships is made in the Partnership Act
C. incorporated limited partnerships have been introduced in all Australian States
and Territories
D. the Partnership Act provides that the rules of the common law and equity are to
continue in force : except insofar as they are inconsistent with the Act
Q9: Which finding is correct?: In Popat v Schonchhatra (1997) 3 All ER 800, the
Court decided:

A. Popat was entitled to half the profits on the sale of the business and to a share
of the profits : that had accrued after the dissolution of the partnership, but
before the final settlement of : accounts.
B. was not entitled to any profits on the sale of the business, as there was
no partnership : agreement and he was further not entitled to a share of the
profits accrued after the dissolution of : the partnership, but before the final
settlement of accounts.
C. Popat was entitled to half the profits on the sale of the business, but not to a
share of the profits : that had accrued after the dissolution of the partnership,
but before the final settlement of : accounts.
D. Popat was not entitled to half the profits on the sale of the business, but he
was entitled to a : share of the profits that had accrued after the dissolution of
the partnership but before the final : settlement of accounts.
Q10: What constitutes an exception to the usual number of a partnership being 20
partners?
A. If one of the partners is married-his/her husband or wife is automatically a

Downloaded by huu nguyen (nguyenminhhuu75@gmail.com)


lOMoARcPSD|13289864

partner
B. The exception relates only to limited partnerships
C. No more than 20 are allowed
D. The Corporations Regulations 2001 (Cth) provide greater numbers in certain
specified : professions
Q11: Which court decision regarding fiduciary duties is incorrect?
A. In Chan v Zacharia (1984) 154 CLR 178, the court decided that that the
fiduciary relationship : continued until the partnership had finally been wound
up.
B. In United Dominion Corp Ltd v Brian Pty Ltd (1985) 157 CLR 1, the Court
found that a fiduciary : relationship, with attendant fiduciary obligations
ordinarily exist between prospective partners who : have embarked upon the
conduct of the partnership business or venture before the precise terms of :
any partnership agreement have been settled
C. In Popat v Schonchhatra (1997) 3 ALLER 800 in the absence of a statement
to the contrary, : partners are entitled to an equal share of profits
D. in Harvey v Harvey (1970) 120 CLR 529, the Court held that if a partner
receives an additional : annual fee to be “on call” for that client, the fee
belongs to the partnership.
Q12: Which below is incorrect? A firm is not bound by the acts of a partner if:
A. the act is not of the usual business of the kind carried on by the firm
B. the partner exceeds his/her actual authority in the particular matter

C. the person with whom the partner is dealing knows that he/she has exceeded
his/her authority
D. the person with whom the partner is dealing knows or believes him/her to be
partner
Q13: How would a partner pledge the firm’s credit?
A. By deed
B. Only for a purpose directly connected to the firm’s business but must have
express authority : to do so
C. By agreement with other partners
D. By agreement with a majority of partners plus an appropriate credit application
Q14: Which statements is not correct?
A. if the agreement is that a person should be paid a fixed sum by the firm, he/she
may be a partner : and jointly liable to creditors of the firm
B. the rights and obligations of partners to each other arise from the partnership
agreement, the : statute and the equitable concept of the fiduciary

Downloaded by huu nguyen (nguyenminhhuu75@gmail.com)


lOMoARcPSD|13289864

C. a partnership agreement must be in writing; it cannot be reached orally or by a


course of : conduct
D. the Partnership Act determines partners’ rights, duties and interests, if not
included in the : partnership agreement
Q15:Which of the statements below is incorrect?
A. A partner’s express authority must be in writing
B. A partner’s authority to buy/sell goods of a kind necessary for or usually
employed in the business : on account of the firm is implied
C. A partner cannot pledge the firm’s credit for purposes not connected with its
ordinary course of : business without express authority
D. A partner cannot pledge the firm’s assets for private debts without express
authority

Q1: Find incorrect statement

A. A term often included in a partnership agreement that allows other partners to


purchase a : retiring/deceased partner’s interest at an agreed valuation avoids the
disruption of a formal winding
B. A partnership cannot be dissolved because the business is carried on at a loss
C. Partnerships can be dissolved because of mutual incompatibility, making it
impossible for partners : to carry on a business
D. On dissolution, partnership property can be applied towards the payment of
partnership : liabilities/debts and any surplus can be distributed among the
partners

Question 2: Which of the following statements regarding dissolution of partnership is


incorrect?
A. In the event of dissolution, losses must be met first out of profits, followed by
capital, then by : partners in proportion to their share of profits
B. A firm’s assets as contributed by partners to make up capital losses need not be
applied to pay : the firm’s debts/liabilities to non-partners
C. After dissolution, each partner is entitled to advances and residue by the firm
D. After the dissolution, each partner’s authority to bind the firm continues, so far as
necessary for : winding up partnership affairs and completing unfinished
transactions

Question 3: Which of the following statements regarding limited partners is incorrect?

Downloaded by huu nguyen (nguyenminhhuu75@gmail.com)


lOMoARcPSD|13289864

A. A limited partner may assign his/her share in the partnership with the consent of
the general : partners
B. The consent of a limited partner is not required to admit a person as a partner
C. If a limited partner has suffered his/her share of the partnership property to be
charged for a : separate debt, the other partners are entitled to dissolve the
partnership
D. A limited partner is not entitled to dissolve the partnership by notice.

Question 4: The incorrect statement is?


A. A partner has express actual and implied actual authority to engage in certain
activities with third: parties
B. A partner cannot pledge/sell partnership property, incur and pay debts on
partnership : accounts or hire employees
C. Contravention of an agreement to restrict a partner’s authority is not binding on
the firm if notice : of the agreement has been given
D. In Polkinghorne v Holland (1934) 51 CLR 143, the firm was held liable because
Holland provided : the advice in his role as solicitor; thus in the ordinary course of
the firm’s business

Question 5: Which finding is correct?: In Popat v Schonchhatra (1997) 3 All ER 800, the
Court decided:
A. Popat was entitled to half the profits on the sale of the business and to a share of
the profits : that had accrued after the dissolution of the partnership, but before
the final settlement of : accounts.
B. Popat was not entitled to any profits on the sale of the business, as there was no
partnership : agreement and he was further not entitled to a share of the profits
accrued after the dissolution of : the partnership, but before the final settlement
of accounts.
C. Popat was entitled to half the profits on the sale of the business, but not to a
share of the profits : that had accrued after the dissolution of the partnership, but
before the final settlement of : accounts.
D. Popat was not entitled to half the profits on the sale of the business, but he was
entitled to a : share of the profits that had accrued after the dissolution of the
partnership but before the final : settlement of accounts.

Question 6: Which of the below statements is incorrect?


A. a written and signed notice of a partner’s intention to dissolve the partnership is
required, : where no fixed term has been agreed upon for the duration of the
partnership

Downloaded by huu nguyen (nguyenminhhuu75@gmail.com)


lOMoARcPSD|13289864

B. a continuing guarantee given by or to a partnership is revoked as to future


transactions if there is : a change in the constitution of the partnership
C. where a partnership continues after a fixed term has expired, the rights and
duties of the partners : remain the same, but the partnership becomes a
partnership at will
D. partnership property must be used exclusively for the purposes of the
partnership and in the : manner set out in the partnership agreement

Question 7: Which of the following is NOT a ground for termination of a partnership?


A. Court order
B. Partner leaves the jurisdiction
C. Expiry of a fixed term
D. Bankruptcy of a partner

Question 8: Which below statements is incorrect?


A. Partners share the profits in a partnership
B. In a partnership, partners have limited liability for the business debts
C. A partner is also an agent of the partnership
D. Agreement may be implied in a partnership

Question 9: Find a statement that is incorrect


A. A retiring partner should give specific notice of their retirement to persons with
whom the firm : has had dealings to avoid being made liable for debts incurred
after retirement.
B. Persons who represents themselves, or knowingly allow themselves to be
represented, as a : partner, are liable as a partner to anyone who has on the faith
of such representation given credit to : the firm
C. A wrongful act/omission includes breach of contract or of fiduciary duty,
negligent : misrepresentation and misleading or deceptive conduct
D. Innocent partners are liable for wrongful acts of other partners made in the
ordinary conduct of : business

Question 10: Which of the below statement is incorrect?


A. Partnership property is liable to be seized for the private (personal) debt of a
partner and : made liable on a judgment against the partnership
B. A creditor who has obtained judgment in respect of the separate debt of a partner
may obtain an : order charging that partner’s interest in the partnership property
and profits with the amount of the : debt and interest.

Downloaded by huu nguyen (nguyenminhhuu75@gmail.com)


lOMoARcPSD|13289864

C. A creditor who has obtained judgment in respect of the separate debt of a partner
may obtain by : an order the appointment of a receiver of that partner’s share of
profits and of any other money : which may be coming to the partner in respect of
the partnership.
D. The two sources of law relevant when considering how partners bind their
partners when dealing : with third parties are the common law (including equity)
of agency and the Partnership Act. The : common law complements the
provisions of the Partnership Act in relation to the authority that an : agent has.

Question 11: Regarding partnerships, which of the following statements is incorrect?


A. Partnerships are created with a view to profit, so partners must make a profit: b
A partner does not have to have a direct claim to a share of the profits
B. Associations and charities are not partnerships, as profits come from ancillary
business activities : and are reinvested, not distributed as dividends to their
members
C. The Partnership Acts do not govern members of a corporation incorporated under
the provisions : of the Corporations Act 2001 (Cth), a special Act of Parliament, or
Royal Charter

Question 12: Which statements is not correct?


A. if the agreement is that a person should be paid a fixed sum by the firm, he/she
may be a partner : and jointly liable to creditors of the firm
B. the rights and obligations of partners to each other arise from the partnership
agreement, the : statute and the equitable concept of the fiduciary
C. a partnership agreement must be in writing; it cannot be reached orally or by a
course of : conduct
D. the Partnership Act determines partners’ rights, duties and interests, if not
included in the : partnership agreement

Question 13: Incorrect statement is?


A. The owners of a corporation have unlimited liability
B. A separate legal entity is separate from its owners and managers
C. A corporation can sue and be sued in its own name
D. Complex business structures are more expensive to establish

Question 14: Which is incorrect statement?


A. If a member of a firm of solicitors acting for a vendor in a sale absconds with the
deposit, : his/her partners are not liable to refund the money

Downloaded by huu nguyen (nguyenminhhuu75@gmail.com)


lOMoARcPSD|13289864

B. In Lloyd v Grace, Smith & Co [1912] AC 716, the firm was held responsible for the
fraud committed : by a managing clerk of a firm, who misappropriated property
while acting within the scope of his : authority
C. In SJ Mackie Pty Ltd v Dalziell Medical Practice Pty Ltd [1989] 2 Qd it was held
that the transfer of a : share to a non-partner breaks the continuity of the firm,
constituting a new firm/partnership of the : remaining former partners and the
new member
D. Partnership agreements can contain provisions to enable the transition from one
firm to another : to be effected without the disruption of a formal winding up.

Question 15: Which of the following statements regarding the written law is incorrect?
A. Under s 115 of the Corporations Act 2001 (Cth), the maximum number of persons
who may form a : partnership for the acquisition of gain is 20
B. Under the Corporations Regulations 2001 (Cth), reg 2A.1.01, partnerships of
more than 20 : partners may be formed for certain professions/callings
C. The Corporations Regulations 2001 (Cth), reg 2A.1.01 sets a maxima of partners
for medical and : legal practitioners, veterinary surgeons, patent and trademark
attorneys, sharebrokers and : stockbrokers and pharmaceutical chemists
D. The Corporations Regulations 2001 (Cth), reg 2A.1.01 does not set a maxima of
partners for : architects and accountants

Downloaded by huu nguyen (nguyenminhhuu75@gmail.com)


lOMoARcPSD|13289864

Downloaded by huu nguyen (nguyenminhhuu75@gmail.com)


lOMoARcPSD|13289864

Downloaded by huu nguyen (nguyenminhhuu75@gmail.com)


lOMoARcPSD|13289864

Downloaded by huu nguyen (nguyenminhhuu75@gmail.com)


lOMoARcPSD|13289864

Downloaded by huu nguyen (nguyenminhhuu75@gmail.com)


lOMoARcPSD|13289864

Downloaded by huu nguyen (nguyenminhhuu75@gmail.com)


lOMoARcPSD|13289864

Downloaded by huu nguyen (nguyenminhhuu75@gmail.com)


lOMoARcPSD|13289864

Downloaded by huu nguyen (nguyenminhhuu75@gmail.com)


lOMoARcPSD|13289864

Downloaded by huu nguyen (nguyenminhhuu75@gmail.com)


Quiz 1-4

1. Sources of law: sources of law in Australia do not include repealed statutes. Includes: 1.
Consolidating statutes, 2. judge-made law, and 3. equity.

2. Which are some of the main features of the Electronic Transactions Act 1999 (Cth): are
the validity of electronic transactions, recognition of writing by electronic means, and
recognition of retaining information in electronic form.

3. The doctrine of precedent: With respect to a case being decided under the appellate
jurisdiction of the Country Court of Victory, a decision of the appellate division of the
District Court of New South Wales on the same Commonwealth legislation is persuasive.

4. Extrinsic Materials: Which of the following is NOT an example of an extrinsic source of


material? The internet. An extrinsic source: 1. Law reform commission reports, 2.
Parliamentary Committee reports, and 3. Parliamentary Debates.

5. Simon and Stella, both of full legal capacity, agree to go on a date. Stella is to pay for the
dinner, but she is running late and does not meet Simon at all. Simon is embarrassed and
angry and calls Stella the next day to threaten to sue her for his taxi fares and dinner
expenses. Will Simon succeed in his claim? No, because Simon and Stella did not intend
the agreement to create legally enforceable obligations.

6. Which of the following statements about consideration is correct? Consideration need


not be adequate.

7. The rule of law: Which of the following concepts is closely related to the rule of law? The
due process. Not closely related: 1. Representative government, 2. a responsible
government, and 3. separation of powers.

8. In Crown Melbourne Ltd v Cosmopolitan Hotel (Vic) Pty Ltd [[2016]] HCA 26 why did the
High Court disagree with the decision of the Victorian Court of Appeal in finding there
was no case for promissory estoppel:
a) The plaintiff could not establish that the statement that it would be “looked after at
renewal time” was capable of conveying to a reasonable person that it was a genuine
offer of a further lease, and

b) There was a need for certainty as to what the parties had agreed on at the end of the
negotiations to find a claim for estoppel.

9. Delegated legislation: Which of the following is not correct? Delegated legislation can
only be made by Government Ministers. Correct: 1. Delegated legislation is subordinate
legislation, 2. is made under the authority of an Act of Parliament, and 3. often contains
more detailed rules than those of the associated Act.

10. Which of the following is not a way in which an offer can be terminated? Revocation
after acceptance has been mailed where the postal rule applies. True: Lapse,
revocation before acceptance, or lapse where no time was stipulated.

11. Section 51 powers: Which of the following is not an area that the Commonwealth
Parliament has the power to legislate on under s51 of the Commonwealth constitution?
Education. Has power over 1. bankruptcy and insolvency, 2. foreign corporations, and 3.
banking and insurance.

12. Interpretation-extrinsic materials: In interpretation, extrinsic materials are allowed to be


referred to by a court.

13. Ratio decidendi: The ratio decidendi of a case is the reason given for deciding the case.

14. Illusory terms in a contract are problematic since they are vague or ambiguous so they
fail to create a legal obligation.

15. Native title: Native title was first recognised in the case of Mabo v State of Queensland
(No 2)(1992) 175 CLR 1.
16. Changing the Constitution: Section 128 of the Commonwealth Constitution provides that
the Constitution can be changed by referendum that requires a “yes” vote by the
majority of voters and in a majority of States.

17. The main legal issue in Felthouse v Bindley (1862) 11CB (NS) 869 was generally silence/
lack of action does not constitute acceptance of an offer.

18. Which of the following is not a class of persons regarded by the law as wholly or partly
incapable of entering into legally binding contracts? Women.

19. Reporting obligations: What is a binding precedent? A decision of a court that brings
judges in a lower court in the same court hierarchy.

20. Separation of powers: Which of the following is not one of the three branches of the
Commonwealth government in Australia: The Governor. The three branches are 1. the
executive, 2. the judiciary, and 3. the legislative.

21. Criminal offences: Which of the following is incorrect with respect to criminal offences?
The committal hearing is held before most summary offense matters. Correct. Criminal
offenses are indictable offenses are generally the more serious offenses, summary
offenses are determined by a magistrate without a jury, and the prosecution must prove
its case beyond a reasonable doubt.

22. Australian Constitution: Which Act established the federal legal and political system and
converted the separate colonies into states? Commonwealth of Australia Constitution
Act 1900.

23. Public and private law: Which of the following is not classified as private law? Criminal
law (public law). Private law: 1. The law of contract, 2. the law of property, and 3. the
corporations law.

24. Farah agreed to take care of an elderly woman Marge and in return was provided with a
house to live in that was promised by Marge to be Farah’s after she died. Farah cared for
the woman for 23 years, but upon Marge’s death, Farah discovered that their oral
agreement was never put into writing. The woman’s son moved into the house and
Farah made a claim to the house. The court is likely to apply the doctrine of part
performance.

25. Andy promises Ellie $100 on her 21st birthday. If this promise was contained in a simple
contract, the legal position is that such a promise is never binding as it must have
consideration supplied by Ellie.

26. The High Court’s decision in Waltons Stores (Interstate) Ltd v Maher (1988) 164 CLR 387
is that they extended the doctrine of equitable estoppels such that it applies even
where there is no pre-existing contractual relationship between the parties.

27. Which of the following statements about acceptance is false? Acceptance can be in any
manner chosen by the offeree. True: 1. Acceptance can be by telephone or email or
conduct, 2. acceptance must be unconditional, and 3. acceptance can only be made by
the person(s) to whom the offer was made.

28. Section 15AA of the Acts Interpretation Act 1901 (Cth) provides that the purpose of the
statute or its object should be preferred method of determining the statute’s meaning.

29. Separation of powers: Which of the following statements about separation of powers in
Australia is correct? The executive is the body that administers the law.

30. Equitable remedies: Which of the following is incorrect? Equitable remedies must be
awarded in a separate proceeding to common law remedies such as damages. Correct:
Equitable remedies are discretionary, and include specific performance, and injunctions.

31. Civil and criminal law: Which of the following is not correct with respect to civil law? The
document filed by the defendant is called a writ. True: 1. The typical purpose of a civil
action is to obtain damages, 2. under civil law, one person may sue another who has
committed a wrongful act, and 3. interrogatories and discovery are procedures that are
available in civil proceedings.
32. Federal system: Which of the following statements is correct? Australia is a federal
system, with two legal systems for each citizen.

33. A contract dividing the proceeds of a cocaine importing venture that has no technical
defects and that is not overly harsh/unfair to either party is void.

34. If one party has threatened another party to enter into a contract, the element that is
missing is real/genuine consent.

35. Substantive and procedural law: Substantive law refers to actual rights under the law.

36. The Age of Majority Act 1977 (Vic) reduced the age for contractual capacity of a minor to
18.

37. Delegated legislation: An example of delegated legislation is the Corporations


Regulations 2001 (Cth).

38. In which of the following types of agreements is there a presumption that parties intend
to be legally bound? A partnership between husband and wife.

39. Court hierarchy: Local or Magistrates Courts are the lowest courts in the state hierarchy.

40. Which of the following statements about bilateral and unilateral contracts is not correct?
A unilateral contract does not require consideration but simply a promise to perform
an act if sufficient. True: 1. More than 1 party is required for both bilateral and unilateral
contracts, 2. both parties are obliged to perform their promises in a bilateral contract,
and 3. while only 1 of the parties is obligated to perform an action in a unilateral
contract.

41. Criminal proceedings: An indictable offense is a more serious criminal offense.

42. Law reports: Where are decisions of the High Court of Australia found only online? In
the Commonwealth Law Report.
43. Which of the following circumstances are likely to affect the consent of one or both
parties to a contract? Mistakes and duress and undue influence

44. Reception of English law in Australia: Which Latin phrase explained the rationale for
applying English laws to the new Colony of New South Wales? Terra Nullius.

45. With respect to revocation, which of the following statements is not correct? The offeror
must personally communicate the revocation to the offeree. True: 1. Revocation need
not be in words, 2. the offeree may accept the offer until such time as they become
aware of the revocation, and 3. where an offer has been made to the world at large, the
revocation does not need to be seen by everyone in order to be effective.

46. Sources of law: the two main types of law in Australia are statute law and
judgment-made law.

47. In Ashton v Pratt (2015) NSWCA 12, what was the main reason for the court deciding
that there was no intention to create a legally binding contract? The verbal language of
the agreement greatly lacked detail from either party and did not indicate definite
obligations.

48. Simone advertises a car for $4,000. John responds to the advertisement and asks
whether Simone will take $3,000. Simon says “no”, but that she will hold the offer open
for a week. The next day, she sells it to Samantha. Which of the following statements is
correct? Simone does not have to hold the offer open since no consideration was
given.

49. Which of the following is not one of Brennan J’s six criteria for estoppel from Waltons
Stores (Interstate) Ltd v Maher (1988) 164 CLR 387? The promissory knew or was
reckless as to whether the promisee intended to act in that. 3 of 6 criteria: The
promissory induced an assumption, the promisee acted in reliance on that assumption,
and the promisee will suffer a material loss if the assumption is not fulfilled.

50. Court hierarchy: The highest court in Australia is the High Court of Australia.
51. Interpretation: Which of the following is not correct? With respect to interpretation, the
main approach used by courts is the purposive approach. Correct: The main approach
used by courts is the literal approach, the requirement that the natural and ordinary
meaning be given to words, and the courts are free to exercise judicial discretion.

52. Which of the following no longer regulates business or commercial law? The Trade
Practices Act 1974 (Cth). Regulates: 1. Competition and Consumer Act 2010 (Cth), 2.
Corporations Act 2001 (Cth), and 3. the law of contract.

53. Jack was subpoenaed to give evidence on Rods behalf. Jack claimed Rod promised him
$2000 to give evidence. Would Jack recover that money? No, since Jack’s public duty is
to give evidence in response to that subpoena.

54. Which of the following statements about the contractual obligations of government
policy proposals is incorrect? Where the government makes a policy promise and a
citizen relies on it the court will always regard that policy commitment as a binding
contractual obligation. Correct: 1. The courts tend not to assign contractual obligations
to government policy promises, 2. Where the government enters into an ordinary
commercial agreement to buy services, the law of contract regulates this agreement,
and 3. In Australian Woollen Mills Pty Ltd v Commonwealth of Australia (1954) 92 CLR
424, the government’s wool subsidy was found to be an administrative scheme and not
contractual obligations.

55. Constitution: Section 51 Commonwealth Constitution grants what type of lawmaking


power to the Commonwealth Parliament? Concurrent powers.

56. Royal Assent: The Governor-General on the Queen’s behalf gives Royal Assent to an Act
of the Commonwealth Parliament.

57. Sally puts up a sign stating that she has lost her dogs and will pay a reward of $100 for
their return to her within 3 days. Which of the following statements is correct? Chang
will not get the reward because he returned the dog knowing it was Sally’s, but
unaware of her offer of a reward.
58. A qualified acceptance may also be known as a/an counteroffer.

59. Which of the following statements about contracts is true? It is a statutory requirement
throughout Australia that some contracts must be in writing and others must be
evidenced in writing

60. The legislature: Exclusive powers are those that can be exercised only by the
Commonwealth.

61. Sources of law – statute: Which of the following is incorrect? Statutes may not overrule
existing common law.

62. Civil law and criminal law: Where an act is both a crime and a tort, the State may
prosecute the offender for the crime, and the victim may bring a civil action.

63. Which of the following is most likely to be considered an offer? “I’ll pay you $3,000 if
you complete a total rewrite of chapters 12 and 15 of this textbook.”

64. Hierarchy of courts: Which of the following is higher than the Federal Court of Australia
in the federal court system? High Court. Lower than the Federal Court of Australia in the
federal court system: 1. Supreme Court, 2. Magistrates Court, and 3. Federal Magistrates
Court.

65. The Executive Power: The Executive power is administered by the Queen, the Prime
Minister, and the Cabinet.

66. Contracts voidable by a minor do not include those not binding unless ratified by the
minor during their minority. Include: 1. Those binding unless repudiated by the minor
during their minority, 2. Those binding unless repudiated by the minor within a
reasonable time after attaining their majority, and 3. Those not binding unless ratified by
the minor within a reasonable time of attaining majority.

67. The postal acceptance rule can be impliedly excluded by offeror specifying actual
receipt of acceptance.
68. With respect to the contractual capacity of corporations, s 124 of the Corporations Act
2001 (Cth) gives them all the legal capacity of a natural person (together with some
additional powers that can only be exercised by a corporation)

69. In which of the following situations is the offer most likely not to have lapsed? Where
Wrench had an option to purchase land but, unknown to Wrench, the seller died prior
to Wrench’s acceptance. Likely: 1. Where Carter, a prospective purchaser of land, makes
a counteroffer that is rejected and then states that he will accept the earlier offer, 2.
Where no time was stated for acceptance by Jane, but Ben does not accept within a
reasonable time, and 3. Where Jim has become overtaken by insanity prior to accepting.

70. Where an agreement has been made during the course of trade or commerce between
businesses, the situation will commonly indicate that the parties intended to create
binding legal relations.

71. Criminal offences: Which of the following is incorrect with respect to criminal offences?
A committal hearing is held before most summary offence matters. Correct: 1.
Indictable offences are generally the more serious offences, 2. The prosecution must
prove its case beyond reasonable doubt, and 3. Summary offences are determined by a
magistrate without a jury.

72. Peter verbally offers Tara his car for $5000. Tara refuses saying it’s not worth that, but
she will pay $4000. What is the status of Peter’s first offer? It has lapsed because of
Tara’s counteroffer.

73. Constitution: Which of the following statements about constitutions in Australia is NOT
correct? Australia has one constitution only (States have no constitutions). Correct: 1.
The Commonwealth Constitution sets out the relationship of the Commonwealth
government with the States, 2. A written constitution is a feature of a federal legal
system such as Australia, and 3. The Commonwealth Constitution sets out the powers of
the Commonwealth government.
74. What was one of the “practical benefits” received by Roffey in Williams v Roffey Bros &
Nicholls (Contractors) Ltd [1990] 1 All ER 512? Roffey did not need to find another
subcontractor.

75. In Elizabeth City Centre Pty Ltd v Corralyn Pty Ltd (1995) 63 SASR 235, the option to
exercise renewal of the lease Negated the postal rule because the notification was not
sent by certified mail as required.

76. Australian Constitution : Which Act established the federal legal and political system and
converted the separate colonies into states? Commonwealth of Australia Constitution
Act 1900.

77. Common law: Which one of the following statements about common law is correct?
Common law is judge made law.

78. Business law: which of the following no longer regulates business or commercial law?
Trade Practices Act 1974 (Cth).

79. Which of the following facts about a “contract under seal” is incorrect? It must always
be supported by consideration to be enforceable. Correct: 1. It must be in writing, 2. It
obtains its binding legal force from the form in which it was prepared, and 3. It can allow
a gratuitous promise to be enforced.

Quiz 5-13

1. A party in breach of contract is liable to pay damages for losses occurring in the usual
course of things from the breach.

2. Which of the following statements is correct? A contract may be illegal as performed


where only one party performs it in an illegal manner.

3. Which of the following is not accurate? Duress involves: Duress must be the only reason
for entering into a contract. Correct: 1. Actual or threatened violence, 2. A party
coerced into entering into a contract (against their will), and 3. Coercion of a person or
their immediate family or near relatives.

4. Which of the following is not one of the six elements required for a claim of fraudulent
misrepresentation to succeed? The representation must be in writing (cannot be verbal
only). 3 of the 6 elements are required for a claim of fraudulent misrepresentation to
succeed: 1. The representation must be false, 2. the representation must have been
acted upon by the other party, and 3. the representation must be one of fact.

5. Which of the following is not true? Economic duress was found in North Ocean
Shipping Co Ltd v Hyundai Construction Co Ltd [1979] 1 QB 705 and the threatened
party was able to recover the payments made under duress. True: 1. is a threat to the
innocent party’s economic interests, 2. permits the threatened party to avoid the
contract, and 3. needs to be distinguished from a legitimate level of commercial
pressure - an example of an actionable level of pressure is a threat to break a contract
without any legal justification in order to extract money from an innocent party.

6. Which of the following is not usually recoverable by way of damages? Compensation for
mere inconvenience or disappointment. Recoverable: Loss that is difficult to estimate,
nominal loss, and expenses incurred in reliance on the other party’s promise to perform.

7. No fault on the part of either party is an element required for the doctrine of
frustration to operate.

8. In Howe v Teefy (1927) 27 Sr (NSW) 301 where a leased racehorse was retaken three
months into a three-year lease, the fact that assessment was difficult did not prevent a
court from granting damages where the plaintiff had been deprived of something of
value.

9. The term “transparency” in unfair contracts means it is expressed in reasonably plain


language that is readily available to the affected party.
10. Where a contract restricts one party from exercising their trade, it may be valid
provided it was e reasonable as between the parties and the public.

11. Janine has sold her business to Simone. Simone insists on inserting a clause in the
agreement that Janine cannot open a similar business within 10 km of her former
business, for the next two years. Which of the following is not true? The court will
always uphold restraint of trade provisions in contracts for sale of business. True. 1.
The clause is a restraint of trade. 2. Where a restraint of trade is reasonable, the whole
parts of the contract will be void. 3. It is impossible to say whether a court would deem
this restraint of trade clause reasonable without further details.

12. In determining whether a statement has become a term of the contract, the key test
applied by the courts is contractual intention.

13. In which case did the court recognise that damages for disappointment and distress
could be available subject to certain limitations? Baltic Shipping Co v Dillon (1993) 176
CLR 344.

14. Which of the following is not an example of an unfair contract term? The specifics of
Rose’s situation were not taken into consideration by Melton Apartment Construction
Ltd.

15. Unliquidated damages are damages where the court is to determine the amount.

16. Which of the following is NOT a requirement for an assignment of a debt or other chose
in action under various State statutes? Fourteen days’ notice in writing is provided to
debtor. Requirement. 1. The assignment is absolute and not by way of charge, 2. The
assignment is in writing, and 3. If not in writing then notice of the assignment is given by
notice to the debtor.

17. Which of the following statements about the parol evidence rule is true? It does not
apply where it can be shown that the written contract was not intended to be a
complete record of the agreement.
18. In Jarvis v Swans Tours Ltd [1973] QB 233, where Jarvis was disappointed by, among
other things, the little dry nut cakes on his holiday damages were awarded partly for
disappointment.

19. With respect to substantial performance and partial performance, partial performance
is where a party voluntarily accepts less than full performance by the other party.

20. How is the Australian Consumer Law structured into the Competition and Consumer Act
2010 (Cth)? The Australian Consumer Law is Sch 2 to the Competition and Consumer
Act 2010 (Cth).

21. Century Dragon Pty Ltd is contracted to export 1000 beach balls to Minerva Beach Surf
Club by 1 January, with payment due by 1 April. Which of the following is correct? The
time of delivery is likely to be considered an essential term.

22. A term that is vital to a contract is known as a condition.

23. Which of the following is NOT a method of termination? Rectification.

24. Parol evidence may not be admitted in which of the following circumstances? To
contradict an unambiguous term.

25. The test of whether a statement is a term or a representation is not dependent on which
of the following? Whether the party making the statement believed in the truth of the
statement or not.

26. In Trident General Insurance Co Ltd v McNiece Bros Pty Ltd (1988) 165 CLR 107, it was
held that the subcontractor could get an indemnity even though the subcontractor had
given no consideration under the contract

27. Which of the following is not correct? An action in restitution cannot be brought where
there is no contract between the parties. Correct. 1. An action in restitution is often
brought where one party has been unjustly enriched at the expense of the other, 2. can
be brought where the contract between the parties is unenforceable, and 3. can be
brought where there has been only part performance.

28. In Codelfa Construction Pty Ltd v State Rail Authority of New South Wales (1982) 149
CLR 337, the High Court found that the contract was frustrated.

29. Compensatory damages are also known as actual/ordinary damages.

30. Which of the following is not a type of termination? Where an event occurs that is not
the fault of either party that causes a fundamental change to the nature of the
contract and the parties obligations and although the contract covers that eventuality,
it would cause hardship to one party to enforce it. A type of termination: 1.
Termination by bankruptcy, 2. When the parties have fully and exactly performed their
obligations to each other under the contract, and 3. when a deed displaces a simple
contract.

31. Which of the following is a circumstance where specific performance would be ordered?
Where the subject matter of the contract is an apartment in a complex of 20
apartments.

32. The remedy of restitution is based on the concept of unjust enrichment where the
defendant is unjustly enriched at the plaintiff’s expense.

33. Which of the following are not usually compensated for with payment of damages?
Injured feelings/ disappointment.

34. Which of the following is not correct with respect to the Australian Consumer Law?
Owing to the Commonwealth’s limited lawmaking powers under s51 of the
Constitution, it applies only to corporations. True: It is contained within Schedule 2 to
the Competition and Consumer Act 2010 (Cth), it applies to conduct engaged in outside
of Australia, and the Australian Consumer Law is applied in each State and Territory.

35. Which of the following is not one of the four ways a statute may render a contract illegal
set out in Yango Pastoral Co v First Chicago Australia Ltd (1978) 139 CLR 410? The
contract may set out an unreasonable restraint. True: The contract may require an
action that statute forbids, may be performed in a prohibited manner, and may have
been made to effect an unlawful purpose.

36. Jacq and Jack robbed the Commercial Bank. Jacq thinks that Jack was too aggressive
with the tellers and the robbery could have gone very wrong. Jacq is now refusing to
give Jack his share of the takings. Which of the following is correct? The contract
involved the commission of a crime and is therefore illegal and unenforceable.

37. Janey takes her fur stole to the dry cleaner. When she returns to collect it, there is a
black mark in the shape of an iron in the middle of her stole and it is ruined. The dry
cleaner tells Janey that he is not liable for the damage to her fur stole as there was an
exemption clause in their agreement. Which of the following is not correct? If the
exemption clause was merely contained within a notice on the wall, the dry cleaner
will be liable for the damage. True: 1. If the exemption clause was on the dry cleaning
docket Janey was given and she was told : “don’t worry about this, it’s just the docket
you have to give us when you come to collect your stole”, the dry cleaner will not be
able to rely on the exemption clause, 2. In order for the exemption clause to be effective
where it was not in a signed document, it had to have been brought to Janey’s attention,
and 3. If the exemption clause was on the dry cleaning docket Janey was given rather
than a formal written contract, the onus will be on the dry cleaner to prove that Janey
was aware it contained conditions that would modify the agreement

38. Criminal proceedings may be brought for unfair practices.

39. Chris maintains that a document that he and Kathy have signed is a binding contract.
Kathy maintains that it is merely a receipt and does not include all the terms of their
agreement. Which of the following statements is correct? Parol evidence may be
admissible.
40. In which of the following circumstances is the court likely to order specific performance
rather than damages? The contract involved the sale of an expensive waterfront
mansion on Sydney Harbour.

41. In the case of restraint of trade with respect to employment, the court does not easily
allow parties to contract out of their means of employment.

42. The general principle with respect to damages for breach of contract is damages are to
place the innocent party in the same position they would have been in if the contract
had been performed.

43. Which of the following is not correct with respect to the prohibition of unconscionable
conduct within the meaning of the unwritten law? Unequal bargaining power of itself is
enough to constitute a special disadvantage. True: 1. Unwritten law includes equity, 2.
There is no unconscionable conduct where both parties have made the same mistake in
good faith, and 3. There must be a special disadvantage and exploitation of that
disadvantage.

44. Which of the following is correct? Damages must not be too remote from a breach.

45. James, a chef, tells Laura, who is looking to buy his antique lounge chair, that the chair is
of Spanish origin. Three weeks later a written contract is drafted that makes no mention
of the chair’s origin. The statement that the chair is of Spanish origin is most likely to be
a/an mere puff representation.

46. Which of the following is not true? A signature on a contract containing an exemption
clause guarantees that an exemption clause will be effective. True: 1. Will not
guarantee the effectiveness of the clause where the doctrine of non est factum applies,
2. Incorporates the exemption clause into the contract, and 3. Will be effective even if
one party did not read the contract.

47. Which of the following is not a requirement of a valid collateral contract? The
consideration must be of higher value than the main contract. True: 1. The statement
must be promissory, 2. The parties must have intended the promise to be binding, and 3.
The promise must be supported by consideration.

48. In Victoria Laundry (Windsor) Ltd v Newman Industries Ltd [1949] 2 KB 528, only the
ordinary loss related to the delay in installation of the boiler was recoverable because
the defendant was not made aware that additional lucrative contracts had been
negotiated.

49. Which of the following statements is correct with respect to mistake? Rectification is a
remedy for certain types of mistakes in written contracts.

50. The definition of “consumer” is limited by what amount? Recently increased to


$100,000.

51. Which of the following are sources of illegality? Statute and common law.

52. When a contract is made orally, the question of what the express terms are is a question
of fact.

53. Which of the following would not constitute the unfair practice of harassment or
coercion under the Australian Consumer Law? Where there is a debt in connection with
the possible of goods and the creditor, Tells the debtor, among repeated demands for
payment give the debtor a discount for early payment. Could constitute the unfair
practice of harassment or coercion under the Australian Consumer Law: 1. Consciously
calculates to intimidate the debtor with the content of her demands 2. Makes demands
constantly in order to exhaust the debtor, and 3. Chooses particularly threatening
demands in order to demoralise the debtor.

54. Smithy Builders have a contract with Big Bank Pty Ltd which contains the following
clause: “Where Smithy Builders fails to complete the contract by 7 December, it will pay
a sum of $300,000 in full and final satisfaction of its liability.” Smithy Builders fails to
complete by 7 December. If Smithy Builders wishes to avoid paying the sum of $300,000,
what must it prove? That the clause is a penalty clause (to punish the builders).
55. Which unfair practice involves participation in a trading scheme where persons at the
top receive most of the benefits? Pyramid selling.

56. Sharma was selling his secondhand vacuum cleaner to Natasha and told Natasha that it
could “do all the rooms in the house ten times over without the filter needing to be
changed”. Natasha spent a few weeks shopping for other vacuum cleaners but
eventually returned and purchased Sharma’s, telling Sharma it was the cheapest she had
seen in two weeks and she liked the colour. When Natasha took it home she soon
discovered that the vacuum cleaner could only do one room at a time and then the filter
would need changing. Natasha cannot rely on Sharma’s statement for breach of contract
because Natasha did not attach any importance to the statement when it was made.

57. Which of the following is not a circumstance where the court would award specific
performance? Where the contract would require constant supervision by the court.
True: 1. Mitigation, 2. Contributory negligence, and 3. Remoteness.

58. In which case did the court recognise that damages for disappointment and distress
could be available subject to certain limitations? Baltic Shipping Co v Dillion (1993) 176
CLR 344.

59. Which of the following is not an element required for acceptable quality of goods? That
the goods are acceptably fit for the consumer’s purpose. True: 1. Free from defects, 2.
Durable, and 3. Safe.

60. Which of the following is not a type of contract illegal at common law on the grounds of
public policy? Contracts to oust the jurisdiction of the courts. True: 1. Contracts to
commit a tort, 2. Contracts prejudicial to the public safety, and 3. Champertous
contracts.

61. Non est factum means it is not [my] deed.

62. Which of the following is an element required for the doctrine of frustration to operate?
No fault on the part of either party.
63. Sandy is selling her horse, Flossy. Miranda is interested in buying Flossy. Sandy tells
Miranda that Flossy is the best little racehorse in Australia and has all the registrations,
licences and permits required to race in the current season but that Miranda should
probably get this checked herself. These two statements are mere puff and
representation.

64. Where a contract has not been completed within the specified time, the innocent party
will be entitled to terminate the contract upon giving notice that time is of the
essence.

65. Which of the following statements is correct? An exemption clause is a term that
excludes or limits the liability of one or more parties.

66. Which of the following actions is unlikely to constitute duress? A threat to contact
police if stolen moneys were not repaid to the employer. Likely: 1. A threat to destroy
the house of a party’s sister, 2. A physical beating to a party until they signed a contract
that they would have signed without the beating, and 3. Physical confinement of an
otherwise unharmed party to pressure them to enter into a contract that would be for
their benefit.

67. How may a court view substantial performance of a contract? The contract is complete,
but the uncomplete portion may be allowed for as a reduction of the full contract
price.

68. Where a contract is made in writing, the express terms of that contract are generally to
be found in the writing.

69. Which of the following matters is least likely to contribute to a finding of unconscionable
conduct in connection with goods or services? Where the supplier makes a commercial
decision to breach the contract knowing that the customer will seek a legal remedy for
the breach. Likely: 1. Where conditions imposed on a customer were not necessary for
the protection of the supplier’s interests, 2. Where the supplier acted in bad faith, and 3.
Where unfair tactics were used against the customer.
70. Which of the following is incorrect? Repudiation is the same as termination; the
contract is at an end.

71. Which of the following statements is not correct? A breach of a warranty entitles the
innocent party to be compensated with damages and termination of the contract.
Correct: 1. A condition is a major term of the contract; a breach renders the contract
substantially different from the agreed term, 2. A breach of a condition entitles the
innocent party to be compensated with damages, and 3. A warranty is a minor term of
the contract; a breach renders the contract insubstantially different from the agreed
term.

72. Which of the following statements is not correct? With respect to legality of object,
illegal objectives will not affect a contract provided both parties have agreed to it.

73. Chris and Tama buy a noodle bar from Udon4U Pty Ltd. In the course of negotiations,
Udon4U Pty Ltd’s agent, Nadia, misrepresents the turnover of the noodle bar, stating an
amount that is double the actually turnover. Nadia herself received this information on
turnover from Geoff, a director of Udon4U Pty Ltd. In making the misrepresentation,
what sections has Nadia likely contravened? As she merely passed on the information
with no intention to procure a contravention, she probably will not be personally
liable.

74. Which of the following is not correct? With respect to s 18 of the Australian Consumer
Law, silence is not relevant in considering the contravention.

75. A divisible contract is one that provides expressly or impliedly that performance is due
after the other party has performed stages of the contract.

76. Which is not an enforcement measure under the Australian Consumer Law?
Disqualification from managing corporations for a period of time for a breach of s 18.
An enforcement measure under the Australian Consumer Law: 1. An order of the court
requiring a supplier to establish a training regime for its employees, 2. A one-year
probation order of the court, and 3. An order directing that a supplier comply with the
terms of an undertaking they have given as well as pay compensation for the breach.

77. In which of the following cases would a plea of frustration not succeed? Where the
event should have been foreseen. Succeed: 1. Where no specific provision was made
for the event in the contract, 2. Where performance is still possible but would be
pointless, and 3. Where performance has become fundamentally different from what
was contemplated by the parties.

78. Ben planned the perfect surprise birthday party for his mother. He hired a hall, he hired
a violinist and he organised catering. Two weeks prior to the event, the violinist called to
cancel. Ben was so upset at the loss of what he thought would be the highlight of the
evening that he called the whole event off. Ben lost both the deposit on the hall and the
deposit for the catering. Which of the following is correct? Ben has a duty to mitigate
his losses.

79. Which of the following constitutes pyramid selling? Jaz is approached by her neighbour
to join the sales team of Thames Beauty Products. Thames sells its products
door-to-door through agents and is a close-knit family type of company. Agents that
introduce other agents to “the family” are rewarded with a finders’ fee. It is only $500
to join the Thames family, and Jaz only has to spend $400 on products in her first
month.

80. What is the main difference between the misleading or deceptive conduct provisions in
the Trade Practices Act 1974 (Cth) and in the Australian Consumer Law? The Australian
Consumer Law refers to a person.

81. The plaintiff in an action under s18 of the Australian Consumer Law is required to prove
what? The conduct is misleading or deceptive or likely to mislead or deceive.

82. Which of the following guarantees applies to a sale by auction? Guarantee as to title.
83. When are nominal damages awarded? A legal right has been infringed but there is no
actual loss.

84. A unilateral mistake does not include mistake as to capacity of the parties. Include: 1.
Mistake as to the nature of the document signed, 2. Mistake as to the identity of the
parties, and 3. Mistake as to the terms of the contract.

85. A consumer contract with respect to unfair contract terms does not involve wholly for
personal, domestic or household use or resupply. Involve: 1. The supply of goods or
services, 2. The sale of land, and 3. An individual who acquires goods or services or land.

86. Which of the following is a way that a contract may be terminated by operation of law?
Where a party to a delivery of goods contract becomes bankrupt.

87. Which of the following statements is incorrect? The remedies available for breach of an
intermediate term depend on their classification as an intermediate term, not the
effect of the particular breach. Correct: 1. Innominate terms are neither conditions nor
warranties, 2. Intermediate terms are the same as innominate terms, and 3.
Intermediate terms are terms that are capable of being breached in minor and major
ways.

88. Which of the following is not correct? An injunction is sometimes granted to compel a
party to do something they would not have been ordered to do by specific
performance. Correct: 1. is an order of the court, 2. Restrains a person from doing
something, and 3. Is an equitable remedy.

89. Which of the following is not correct? Parol evidence is not admissible to evidence
whether a document constitutes a contract or merely a note. Correct: 1. Will always be
admissible to evidence duress, 2. Is admissible to evidence a collateral oral agreement,
and 3. Will always be admissible to evidence fraud.

90. Exemplary damages might be ordered as a penalty where there has been an
intentional breach.
91. In considering whether a contract is a standard form contract, which of the following is
not something that the court must consider? Whether the price payable takes into
account the specifics of the party and the transaction. Must consider: 1. If one of the
parties has most of the bargaining power, 2. If one party prepared the contract prior to
discussing it with the other party, and 3. If another party was given opportunity to
negotiate the terms.

92. Which of the following is not correct? The Australian Consumer does not have any
provision for unconscionable conduct. Correct: 1. The Australian Consumer Law
contains provisions on unconscionable conduct, 2. The Australian Consumer Law
contains provisions on misrepresentation, and 3. The Australian Consumer Law contains
provisions on misrepresentation in relation to the sale of land.

93. John tells Cara that the car he has for sale is “as is, where is” and “only needs the oil
filter changed and she’s good to go”. John made this statement based on the fact he has
had the car checked by a mechanic a few weeks ago. Unfortunately, the car required
more significant repairs before it was roadworthy. What remedies are available for Cara
under the common law? Cara has no remedy under the common law (if John’s
statement was not a term in the contract).

94. Which of the following is correct? When the parties’ obligations under a contract have
been discharged then so has the contract.

95. In which one of the following cases was it held that damages are not recoverable for
distress or disappointment unless the contract promised to provide pleasure, enjoyment
or relaxation or where the distress or disappointment arises from physical injury or
physical inconvenience? Baltic Shipping Co v Dillon (1993) 176 CLR 344.

96. Which of the following is not an equitable remedy? Exemplary damages. Equitable
remedy: 1. Injunction, 2. Specific performance, and 3. Restitution.
97. In which one of the following cases was it held that for a loss in the contemplation of the
parties to be recoverable it must be “a serious possibility”, “a real danger”, “liable to
result” or “not unlikely” to occur? Koufos v Czarnikow Ltd [1969] 1 AC 350.

98. How is repudiation determined by the court? Objectively.

99. Tom agrees to fix Richie’s car for $1,000. Richie pays a deposit of $200 and Tom
guarantees the job will be completed within two weeks. Before Tom commences the
repairs, the car is stolen from Richie’s locked garage and destroyed by fire. Richie
demands the return of his $200 deposit and Tom refuses to give it to him. Which of the
following is not correct? Under the Australian Consumer Law and Fair Trading Act 2012
(Vic), Tom is entitled to retain the $200 deposit. Correct: 1. The contract is
automatically terminated, 2. Tom is no longer obligated to fix Richie’s car, and 3. Under
the Australian Consumer Law and Fair Trading Act 2012 (Vic), Tom is entitled to retain
the portion of the $200 deposit he spent on parts and other relevant expenses, if any.

100. Which of the following statements about damages is not correct? Damages are awarded
to compensate a party for all losses caused by breach of contract.

101. Where one party believes the contract refers to the Bonny Lass, a freight ship based out
of Norway, and the other party believes the contract refers to the Bonny Lass, a freight
ship based out of Hong Kong, this is an example of a mutual mistake.

102. Which of the following is not one of the three elements of an unfair term under s 24(1)
of the Australian Consumer Law? Where the term is not transparent. Three elements of
an unfair term: 1. It would cause a significant imbalance in the parties’ rights and
obligations, 2. It is not reasonably necessary to protect the interests of the
disadvantaged party, and 3. It would cause detriment to a party if it were applied.

103. In Shevill v Builders Licensing Board (1982) 149 CLR 620 at 625, Gibbs CJ listed all but the
following various ways that a contract may be repudiated “[I]f one party discharges
[themself] from performance by agreeing to a new agreement that supersedes the
previous one”.
104. In Barton v Armstrong [1976] AC 104, the court held that the threats to the plaintiff’s
life contributed to the plaintiff’s decision to sign the contract and therefore
constituted duress.

105. Which of the following is not correct with respect to mitigation of loss? The burden of
proving that losses were mitigated falls upon the party claiming the damages ie. the
defendant.

106. Which of the following is the general position of the courts when awarding damages?
Where it is possible to place an innocent part in the position he/she would have been
in if the breach had not occurred.

107. With respect to consumer guarantees, which of the following is not correct? With
respect to fitness for purpose, a “disclosed purpose” relates to the supplier only - the
manufacturer may make representations to the consumer, but the consumer cannot
disclose a purpose to the manufacture and receive a guarantee in return. Correct: 1. It
is possible to exclude the consumer guarantees in very limited circumstances, 2.
Remedies for non-compliance with the consumer guarantees depend in whether there
was a “major failure” or not, and 3. A manufacturer does not need to provide repair
facilities or spare parts after a certain period.

108. Consumer guarantees as to title are contained within sections 51-56 and ss 60-61 of the
Australian Consumer Law.

109. Which of the following is not an unfair practice under the Australian Consumer Law?
Where a television advertisement for a chocolate bar shows teddy bears working the
manufacturing lines.

110. Which of the following is not a type of contract void at common law? An agreement that
offends the Australian Consumer Law (ACL). A type of contract void at common law: 1.
A contract to oust the jurisdiction of the courts, 2. A “marriage brokerage” agreement
whereby money is paid in order to procure a marriage, and 3. A price maintenance
agreement that restrains trade.
111. What does the concept “quantum meruit” mean? The innocent party would be unjustly
enriched if they were able to retain the benefit without compensating the party in
breach for the “amount he deserves”.

112. In Hadley v Baxendale (1854) 9 Exch 341; 156 ER 145 the court recognized “reasonably
foreseeable losses” as being

a) Losses that arise naturally from a breach of contract, and

b) Losses that are actually contemplated by the parties.

113. Which of the following is correct with respect to privity of contract? Despite the
doctrine, a new owner of land is able to enforce a covenant between the previous
owner and another party.

114. Why are late payment charges on credit cards not penalties? Banks penalties on cards
are legitimate to cover their costs.

115. In Curtis v Chemical Cleaning & Dyeing Co Ltd [1951] 1 KB 805, the exemption clause in
the sales agreement for the drycleaning was not sufficiently explained to the
customer.

116. Which of the following is not a situation where frustration would occur? Where the
performance of the contract is rendered illegal by the destruction of the subject
matter of the contract by one of the parties. A situation where frustration would occur:
1. Long term serious illness of a person contracted to perform a personal services
contract, 2. Where the government in exercising its powers has made completion of the
contract impossible, and 3. Where a change in the law renders performance of the
contract illegal.

117. When considering whether a contract is a standard form contract, the court does not
have to consider whether one party’s vicarious liability for its agents is limited.
Consider: 1. The bargaining power of the parties as unequal bargaining power does not
constitute a special disadvantage, 2. Whether both parties had the opportunity to
negotiate the terms of the contract, and 3. Whether the terms of the contract take into
account the specifics of the particular transaction.

118. A party repudiates a contract when they are no longer able or are unwilling to perform
their obligations

119. Chung is looking to purchase a carwash business from Terry. Terry tells Chung that she
serves 100-150 cars per day and that she has just concluded an agreement to service the
fleet of cars of a local business. In reality, Terry has only served around 15 cars per day
since her customer service received a bad review in the local newspaper and a
competing carwash business opened one street away. Terry intends for Chung to buy her
business based on what she has stated. This is an example of fraudulent
misrepresentation

120. Undue influence differs from duress in that with undue influence, no unlawful act is
required

121. If one party has completed its obligations under the contract, and the other has not: The
contract cannot be terminated by simple agreement without consideration

122. In ACCC v Turi Foods Pty Ltd [2012] FCA 19: The statement that the chickens were “free
to roam” was misleading as there were too many chickens pr square metre for the
chickens to easily move.

123. Can a person use an official position they hold for their financial advantage? No because
if a contract is involved, this could lead to the promoting of corruption in public life.

124. Marnie loans Bill $1000 so that Bill can pay a local politician to ensure his tender will be
the winning tender. The contract between Marnie and Bill is an illegal contract.

125. Which of the following is not a valid assignment of a contract? A contract for personal
services.
126. Which of the following is correct? When the parties’ obligations under the contract
have been discharge, then so has the contract

127. Matt sells his car to Tabitha, who purchases it on the condition that she can continue to:
keep it in his garage at no cost for the next six months. Tabitha pays in full. Two months
later, Matt tells Tabitha she will have to remove her car from his garage as he has bought
a: new car and wants to keep it in the garage. Which of the following statements is
correct? If Tabitha promises to remove her car, Matt will not be able to enforce the
promise: unless the promise is supported by consideration or made under seal.

128. In which case was it that held that where an exemption clause is in a non-contractual
document, the clause must be brought to the other party’s attention for it to be
effective? Causer v Browne [1952] VLR 1.

129. A liquidated damages clause in a written contract: Sets out the amount agreed payable
to the innocent party if there is a breach of contract

130. In which case did the court find that passing off had been engaged in and the misleading
and deceptive conduct provisions had been contravened? Apand Pty Ltd v The Kettle
Chip Co Pty Ltd (1994) 52 FCR 474.

131. Which of the following sections of the Australian Consumer Law are relevant to
misrepresentation? Sections 18 and 29.

132. Terms may not be implied into a contract by representations

133. In Hong Kong Fir Shipping Co Ltd v Kawasaki Kisen Kaisha Ltd [1962] 2 QB 26, the court
recognised which of the following terms which later became part of Australian law in
Koompahtoo Local Aboriginal Land Council v Sanpine Pty Ltd (2007) 233 CLR 115?
Innominate term

134. Fraser crashes his car with his friend Angus as a passenger. Angus is injured. Angus was
not wearing a seatbelt. Which of the following is not correct? Damages cannot be
apportioned for breach of contractual duty of care where there is contributory
negligence. Correct: 1. Liability would likely be apportioned between Fraser and Angus,
2. Damages awarded to Angus, if any, would be reduced by his contributory negligence,
and 3. Angus’ contributory negligence relates only to his own safety, not to that of
others.

135. Which of the following is not a restraint of trade? An agreement by which Steve agrees
to repay his daughter’s loan debt in return for the person who advanced the loan to
his daughter agreeing not to report Steve’s daughter to the police for fraud. A restraint
of trade: 1. A price maintenance agreement, 2. A contract restraining Laura from
working as a welder within Ballarat for two years after her termination with Welders R
Us, and 3. An agreement between John and Liz that John will not open a brew shop
within 2km of the brew shop he just sold to Liz.

136. Which of the following constitutes referral selling? Jaz purchases a painting through a
new art gallery that has just opened in town. The price is high and Jaz is at first uncertain
about the expense, but the art dealer promises Jaz a commission on sales to any of Jaz’s
friends who she sends along to the gallery. Jaz has a lot of art collector friends so is sure
that she will receive some commission from the gallery in the near future.

137. Ella contracted Anasaki to build a balcony for her house to certain specifications. When
Anasaki had finished, Ella was shocked to see that the balcony was much bigger than she
had specified. Ella refused to pay Anasaki. Which of the following is incorrect? It would
be unjust for Ella to have to pay any money for a balcony that was not built to her
specifications under the contract.

138. Cathy and Mel entered into a contract where Cathy was to steam clean seven rooms in
Mel’s house at $200 per room. After cleaning three rooms, Cathy abandoned the job for
a more valuable contract. Which of the following is true? If the contract is divisible,
Cathy is entitled to payment for the work she has done.
139. Chang wants the court to imply an engineering custom into a term of a contract to which
he is a party. The other party has no knowledge of this custom. It is not contrary to any
of the express terms of the contract. The court may imply the term.

140. Jong complains that the barbecue he bought is not fit for purpose because the metal
sides melted the first time he cooked with it. The manufacturer maintains that Jong
should not have used the barbecue for slow cooking and that it never represented that
the barbecue would be fit for the purpose of slow cooking. What are the circumstances
in which the guarantee as to fitness for purpose would apply? Where Jong made known
to the supplier that he was going to use the barbecue for slow cooking meals.

141. It is important to distinguish between representations and terms because damages can
only be awarded for fraudulent or negligent misrepresentation.

142. Which of the following is not an enforcement measure under the Australian Consumer
Law? A disclosure order issued by the ACCC. An enforcement measure under the
Australian Consumer Law: 1. An order of the court requiring a supplier to publish an
advertisement, 2. A two-year probation order of the court, and 3. A notice issued by the
ACCC requiring a supplier to substantiate a claim about a product they are selling.

143. Which of the following is not an exception to privity? Employment. Exception: 1.


Insurance, 2. Agency and trust, and 3. Property Law.

144. James is going through a difficult time and does not feel he can confide in anyone.
Eventually he confides in the priest of his church and asks for support. The priest
forcefully tells James to turn over all of his money to the church or he will be struck
down by lightening. The priest does not believe that James will be struck down. James
makes the payments. Which of the following is most likely legal grounds for James to
avoid the transaction? Undue influence.

145. In which High Court decision was it said: “[T]here are two relevant circumstances in
which a breach of contract by one party may entitle the other to terminate. The first is
where the obligation [is] essential … The second relevant circumstance is where there
has been a sufficiently serious breach of a non-essential term …we rest our decision in
the appeal not upon the ground of breach of an essential obligation, but upon
application of the doctrine respecting intermediate terms.” Koompahtoo Local
Aboriginal Land Council v Sanpine Pty Ltd (2007) 233 CLR 115.

146. A written contract allows Joseph, the neighbour of Sonny, to take “as many strawberries
as he likes each strawberry season”. This contract is valid as the parties’ intention is
clear.

147. Which of the following is not a way that a person seeking to rely on an exemption clause
can show that the clause has become part of the contract? By showing it was brought
to the notice of the other party at the time of, or subsequent to, entering into the
contract.

148. Which of the following is not correct? It is prohibited to offer goods at a special price
and not have a sufficient amount of those goods available for a long period of time.
Correct: 1. It is prohibited to offer free gifts with purchase then factor the cost of the gift
into the purchase, 2. It is prohibited to take a customer’s money knowing that the
product they want is no longer available even where the product the supplier intends to
sell to the customer in its place is similar, and 3. It is not prohibited for a bank to send a
credit card to a customer at their written request.

149. Which of the following is not a way to discharge a contract? Through breach.

150. Who/what are exempt from the provisions of s18 of the Australian Consumer Law?
Information providers such as media outlets.

151. Krell v Henry [1903] 2 KB 740 illustrated what point? That the parties are discharged if
an event occurs which forms the basis of the contract, but which prevents
performance at no fault of either party.

152. Contingent conditions include conditions subsequent and conditions precedent.


153. Which of the following is not one of the special relationships in which the onus of
proving that undue influence was not employed shifts to the denying Party? Paren and
child where the child is living independently. Special relationships: 1. Solicitor and
client, 2. Trustee and beneficiary, and 3. Doctor and patient.

154. Which of the following is not correct with respect to termination by subsequent
agreement? A subsequent simple contract can cancel a contract where one party has
completed their obligations and the other has not. Correct: 1. A subsequent agreement
may cancel an original contract, 2. A subsequent agreement must be valid to rescind an
earlier contract, and 3. A subsequent simple contract can cancel a contract where there
is still something to be done by each party.
155. The parol evidence rule performs much the same function as a merger clause.
156. A party repudiates contract when the party is able but unwilling to perform their
obligations under it.
157. Where a loan contract allows for termination by the loan provider in the event of default
by the borrower, this is an express power to terminate.
158. If the court held the sum the parties inserted into their contract ($100,000) to be a
penalty, and the actual loss determined by a court was $30,000, the plaintiff would be
able to recover $30,000 only.
159. Which of the following statement is not correct? A breach of a warranty entitles the
innocent party to be compensated with damages and termination of the contract.
Correct: 1. A condition is a major term of the contract; a breach renders the contract
substantially different from the agreed term, 2. A breach of a condition entitles the
innocent party to be compensated with damages, and 3. A warranty is a minor term of
the contract; a breach renders the contract in substantially different from the agreed
term.
160. Which of the following is not correct with respect to misleading conduct under the
Australian Consumer Law as compared with the common law of misrepresentation? The
person does not necessarily have to engage in the conduct. Correct: 1. An opinion will
not contravene s 18 unless the person did not genuinely hold that opinion, 2. Silence
may contravene section 18, and 3. Mere puffs will not contravene s 18.
161. Which of the following is not a remedy for unconscionable conduct? A criminal
prosecution. A remedy for unconscionable conduct: 1. An injunction, 2. Ordinary
damages, and 3. A pecuniary penalty.
162. Which of the following is not correct? A criminal penalty can be sought for a breach of s
18. Correct: 1. The ACCC can seek both criminal and pecuniary penalties for breaches of
the unfair practices provisions of the Australian Consumer Law, 2. The new maximum
penalties may vary depending on turnover of a corporation, and 3. Injunctions can be
sought as orders
163. Which Australian case is most similar to H Parsons (Livestock) Ltd v Uttley Ingham & Co
Ltd [1978] QB 791 in relation to applying foreseeability to limit claims for damages? Day
v O’Leary (1992) 57 SASR 206
164. Which of the following is not correct? Restitution was not awarded in the case of Pavey
& Matthews Pty Ltd v Paul (1987) 162 CLR 221 as the contract was not in writing as
required by statute and therefore not enforceable. Correct: 1. May be awarded where
there is less than substantial performance, 2. Is said to be based on unjust enrichment,
and 3. Is subject to a number of defences, such as estoppel.
165. Coulls v Bagot’s Executor & Trustee Co Ltd (1967) 119 CLR 460; Beswick v Beswick [1968]
AC 58 and Tweddle v Atkinson (1861) 1 B & S 393 all relate to the doctrine of privity.
166. The Australian Consumer Law definition of “supply” does not include gift. Include: 1.
Re-supply, 2. Exchange, and 3. Lease.
167. When it was said in Re Hall & Barker [1878] 9 Ch D 538, “if a shoemaker agrees to make
a pair of shoes, he cannot offer you one shoe and ask you to pay one half the price”, this
is referring to the court’s tendency to regard contracts as entire.
168. Which of the following is most correct? If an amount of $5,000 is specified in a written
contract as the agreed genuine pre-estimate of damages in the event the contract is
breached then this is commonly referred to as liquidated damages.
169. Which of the following is not true with respect to misleading or deceptive conduct? A
person acting as a “mere conduit” in passing on representations may also be liable.
True: 1. The plaitiff has to prove that conduct was misleading or decetive, 2. The plaitiff
has to prove the causal link between the conduct and the loss, and 3. A corporation is
not liable where it is clear that the corporation is not the source of the information.
170. Which of the following constitutes bait advertising? Jaz approaches a salesperson on
the forecourt to buy the car package she saw advertised on television and is told that
there were only two cars for sale as a part of that deal and there are now none left.
The salesperson tells Jaz that she would look much better in the convertible model

and although there is no deal on that car, it is only $50,000 more.


Quiz 14-15

1. To claim damages a causal link must be established between what two things? The
defendants breach and the plaintiff’s injury.

2. Which of the following is not correct? The reform to the law of negligence that took
place in Australia in the early 2000s does not apply to claims in contract law. Correct: 1.
Limits the scope of potential liability for negligence, 2. Was enacted in all Australian
states and territories, and 3. Covers personal injury.

3. In Tame v State of New South Wales (2002) 211 CLR 317 where Tame was given a false
blood alcohol reading, part of the test of reasonable foreseeability is a question of fact.

4. In Australian Safeway Stores v Zaluzna (1987) 162 CLR 479, the respondent was a lawful
entrant upon the land of the respondent, establishing a relationship between them
and there was therefore a duty of care owed by the appellant to avoid a foreseeable
risk of injury.

5. Which of the following is not a provision of the Wrongs Act 1958 (Vic) relating to
professional liability? The court must rely on peer professional opinion even where it
considers that opinion irrational.

6. Where a person gives advice, that advice is relied upon and the advice is incorrect, the
person giving the advice may be liable in negligence. Which of the following is incorrect?
The duty of care only arises where the “special relationship” involves a person seeking
advice from the advice-giver, not where the advice was given unrequested and merely
accepted.

7. The “neighbour principle” is often viewed as a test of the foreseeability of harm.

8. Why was the plaintiff successful in Overseas Tankship (UK) Ltd v Miller Steamship Co Pty
Ltd (The Wagon Mound No 2) [1967] AC 617 (PC) when another plaintiff failed in the
earlier related Wagon Mound case? In the first case, the plaintiffs failed to show that a
reasonable man would have foreseen the risk of damage from the oil spill.

9. Nicola drives the forklift at her place of work, Rooze’s Roofing. Nicola always leaves the
forklift in a certain place where she has been told to leave it, with the forks up off the
ground. One afternoon a customer who is collecting goods from the workshop reverses
his car into the forks on the forklift. He is injured and his car is damaged. What is the
principle that would make Nicola’s employer liable for her actions? Vicarious liability.

10. Which of the following is not correct? In The Wagon Mound No 1 and The Wagon
Mound No 2, where there was an unfortunate combination of an oil spill, welding
sparks and floating cotton waste, the loss was found to be not reasonably foreseeable
in the circumstances. Correct: 1. If damages are too remote will not be recoverable, 2.
The remoteness test will be satisfied where the damage suffered is of the same type of
kind as foreseeable damage, and 3. In order for damage to not be too remote, it must be
reasonably foreseeable.

Quiz 16-17

1. Which of the following statements regarding the dissolution of a partnership is


incorrect? A firm’s assets as contributed by partners to make up capital losses need
not be applied to pay the firm’s debts/liabilities to non-partners. Correct: 1. In the
event of dissolution, losses must be met first out of profits, followed by capital, then by
partners in proportion to their share of profits, 2. After dissolution, each partner is
entitled to advances and residue by the firm, and 3. After the dissolution, each partner’s
authority to bind the firm continues, so far as necessary for winding up partnership
affairs and completing unfinished transactions.

2. The incorrect statement is? The owners of a corporation have unlimited liability.
Correct: 1. A separate legal entity is separate from its owners and managers, 2. A
corporation can sue and be sued in its own name, and 3. Complex business structures
are more expensive to establish.
3. The incorrect statement? If the giving of investment advice is within the scope of the
firm’s business then all partners are jointly and severally liable for advice that is not in
a client’s best interests. Correct: 1. Partners are not bound when another partner
misappropriates monies, 2. The client should seek a personal indemnity from a partner,
and 3. Special skill is required to bind all partners.

4. Find a statement that is incorrect. A wrongful act/omission includes breach of contract


or of fiduciary duty, negligent misrepresentation, and misleading or deceptive
conduct. Correct: 1. A retiring partner should give specific notice of their retirement to
persons with whom the firm has had dealings to avoid being made liable for debts
incurred after retirement, 2. Persons who represent themselves, or knowingly allow
themselves to be represented, as a partner, are liable as a partner to anyone who has on
the faith of such representation given credit to the firm, and 3. Innocent partners are
liable for wrongful acts of other partners made in the ordinary conduct of business.

5. Which finding is correct?: In Popat v Schonchhatra (1997) 3 All ER 800, the Court decided
Popat was entitled to half the profits on the sale of the business and to a share of the
profits that had accrued after the dissolution of the partnership, but before the final
settlement of accounts.

6. Which statement regarding partnership is incorrect? Partnerships do not avoid taking


on the formality and expense of an incorporated company. Correct: 1. A partnership (or
firm) is a way of gathering resources or expertise for major projects, 2. Partners in a
partnership complement each other with their skills and bring in new capital and
broader funding options, 3. A partnership in law may exist without the partners being
aware of it.

7. Which below is incorrect? A firm is not bound by the acts of a partner if the person with
whom the partner is dealing knows or believes him/her to be a partner. Correct: 1. the
act is not of the usual business of the kind carried on by the firm, 2. the partner exceeds
his/her actual authority in the particular matter, and 3. the person with whom the
partner is dealing knows that he/she has exceeded his/her authority.
8. Which of the below statement is incorrect? Partnership property is liable to be seized
for the private (personal) debt of a partner and made liable on a judgment against the
partnership. Correct: 1. A creditor who has obtained a judgment in respect of the
separate debt of a partner may obtain an order charging that partner’s interest in the
partnership property and profits with the amount of the debt and interest, 2. A creditor
who has obtained a judgment in respect of the separate debt of a partner may obtain by
order the appointment of a receiver of that partner’s share of profits and of any other
money which may be coming to the partner in respect of the partnership, and 3. The
two sources of law relevant when considering how partners bind their partners when
dealing with third parties are the common law (including equity) of agency and the
Partnership Act. The common law complements the provisions of the Partnership Act in
relation to the authority that an agent has.

9. Which legislation regulates partnerships in NSW? Partnership Act 1892 (NSW) [or
Partnership Act 1958 (Vic) if there is no (NSW) after Partnership Act 1892]

10. Legislation in Victoria: What Act regulates partnerships in Victoria? Partnership Act 1958
(Vic)

11. Which below statement is incorrect? In a partnership, partners have limited liability for
the business debts. Correct: 1. Partners share the profits in a partnership, 2. A partner
is also an agent of the partnership, and 3. The agreement may be implied in a
partnership.

12. Which of the below statements is incorrect? A written and signed notice of a partner’s
intention to dissolve the partnership is required where no fixed term has been agreed
upon for the duration of the partnership. Correct: 1. a continuing guarantee given by or
to a partnership is revoked as to future transactions if there is a change in the
constitution of the partnership, 2. where a partnership continues after a fixed term has
expired, the rights and duties of the partners remain the same, but the partnership
becomes a partnership at will, and 3. partnership property must be used exclusively for
the purposes of the partnership and in the manner set out in the partnership
agreement.

13. Which of the statements below is incorrect? A partner’s express authority must be in
writing. Correct: 1. A partner’s authority to buy/sell goods of a kind necessary for or
usually employed in the business on account of the firm is implied, 2. A partner cannot
pledge the firm’s credit for purposes not connected with its ordinary course of business
without express authority, and 3. A partner cannot pledge the firm’s assets for private
debts without express authority.

14. Which of the below does not apply to the Partnership Act? The partnership under the
Act must be registered with ASIC. Apply: 1. The rules in the Partnership Acts assist in
determining whether a business is being carried on in common, 2. Each rule states a
negative: i.e. that a certain fact does not of itself create a partnership, but is only
indicative of a partnership, and 3. Joint or part ownership or joint tenancy, or tenancy in
common whether or not the owners/tenants share the profits, does not of itself create a
partnership as to anything soheld/owned.

15. How would a partner pledge the firm’s credit? Only for a purpose directly connected to
the firm’s business but must have express authority to do so.

16. Which of the following is NOT a ground for termination of a partnership? Partner leaves
the jurisdiction. A ground for termination of a partnership: 1. Court order, 2. Expiry of a
fixed term, and 3. Bankruptcy of a partner.

17. The incorrect statement is? A partner cannot pledge/sell partnership property, incur
and pay debts on partnership accounts or hire employees. Correct: 1. A partner has
express actual and implied actual authority to engage in certain activities with third
parties, 2. Contravention of an agreement to restrict a partner’s authority is not binding
on the firm if notice of the agreement has been given, and 3. In Polkinghorne v Holland
(1934) 51 CLR 143, the firm was held liable because Holland provided the advice in his
role as solicitor; thus in the ordinary course of the firm’s business.
18. Which of the following statements regarding limited partners is incorrect? If a limited
partner has suffered his/her share of the partnership property to be charged for a
separate debt, the other partners are entitled to dissolve the partnership. Correct: 1. A
limited partner may assign his/her share in the partnership with the consent of the
general partners, 2. The consent of a limited partner is not required to admit a person as
a partner, and 3. A limited partner is not entitled to dissolve the partnership by notice.

19. Which of the following statements regarding partnerships is incorrect? Each partner
must take an active part in the direction and management of the firm. Correct: 1. The
second element of a partnership is carrying out a business in common, 2. To be a
partnership there must be a mutuality of rights and obligations, and 3. In Degiorgio v
Dunn [2004] NSWSC 767 it was held that there was no partnership because the business
was not run “in common”.

20. Find incorrect statement. A partnership cannot be dissolved because the business is
carried on at a loss. Correct: 1. A term often included in a partnership agreement that
allows other partners to purchase a retiring/deceased partner’s interest at an agreed
valuation avoids the disruption of a formal winding, 2. Partnerships can be dissolved
because of mutual incompatibility, making it impossible for partners to carry on a
business, and 3. On dissolution, partnership property can be applied towards the
payment of partnership liabilities/debts and any surplus can be distributed among the
partners.

21. What constitutes an exception to the usual number of a partnership being 20 partners?
The Corporations Regulations 2001 (Cth) provide greater numbers in certain specified
professions.

22. Which court decision regarding fiduciary duties is incorrect? In Harvey v Harvey (1970)
120 CLR 529, the Court held that if a partner receives an additional annual fee to be
“on call” for that client, the fee belongs to the partnership. Correct: 1. In Chan v
Zacharia (1984) 154 CLR 178, the court decided that that the fiduciary relationship
continued until the partnership had finally been wound up, 2. In United Dominion Corp
Ltd v Brian Pty Ltd (1985) 157 CLR 1, the Court found that a fiduciary relationship, with
attendant fiduciary obligations ordinarily exist between prospective partners who have
embarked upon the conduct of the partnership business or venture before the precise
terms of any partnership agreement have been settled, and 3. In Popat v Schonchhatra
(1997) 3 ALLER 800 in the absence of a statement to the contrary, partners are entitled
to an equal share of profits.

23. Which is incorrect statement? If a member of a firm of solicitors acting for a vendor in a
sale absconds with the deposit, his/her partners are not liable to refund the money.
Correct: 1. In Lloyd v Grace, Smith & Co [1912] AC 716, the firm was held responsible for
the fraud committed by a managing clerk of a firm, who misappropriated property while
acting within the scope of his authority, 2. In SJ Mackie Pty Ltd v Dalziell Medical Practice
Pty Ltd [1989] 2 Qd it was held that the transfer of a share to a non-partner breaks the
continuity of the firm, constituting a new firm/partnership of the remaining former
partners and the new member, and 3. Partnership agreements can contain provisions to
enable the transition from one firm to another to be effected without the disruption of a
formal winding up.

24. Which of the following statements is incorrect? a partnership must be in writing.


Correct: 1. a separate legal entity can commit civil wrongs and engage in criminal
conduct, 2. a public company has serious compliance obligations, and 3. a separate legal
entity is entitled to own property, pay tax and enter into contracts.

25. Find one incorrect statement? A creditor can enforce liability against an incoming
partner whether or not he/she is a party to the contract. Correct: 1. In the absence of
special statutory provision, although each partner is liable with the others for the whole
of the debts of the firm, their liability is only joint, 2. A creditor can bring only one action
against members of a partnership and any partner can insist that the action be stayed
until all other partners are joined as parties, and 3. A person admitted into an existing
firm, liability may be incurred where it is specially agreed upon.
26. Which statement regarding dissolution of a partnership is incorrect? After payment of
the firm’s liabilities, partners do not have the right to have surplus assets applied in
payment of what may be due to the partners respectively / A firm’s asset as
contributed by partners to make up capital losses need not to be applied to pay the
firm’s debt/liabilities to non-partners Correct: (1) Partners are not entitled to have
partnership property applied towards the payment of partnership liabilities, (2) On the
termination of the partnership, any partner can apply to the court for a decree to
dissolve the partnership and appoint a receiver to wind up the firm’s business/affairs,
and (3) Should a sequestration order be made against a partner, a creditor of the firm
cannot receive a dividend out of the bankrupt’s separate property until all separate
creditors of the bankrupt partner have been paid in full, (4) In the event of dissolution,
losses must be met first out of profits, followed by capital, then by partners in
proportion to their share of profits, (5) After dissolution, each partner is entitled to
advances and residue by the firm, and (6) After the dissolution, each partner’s authority
to bind the firm continues, so far as necessary for winding up partnership affairs and
completing unfinished transactions.

27. Cribb v Korn (1911) 12 CLR 205 established the sharing of joint returns does not in itself
create a partnership.

28. Regarding partnerships, which of the following statements is incorrect? Partnerships are
created with a view to profit, so partners must make a profit b A partner does not
have to have a direct claim to a share of the profits. Correct: 1. Associations and
charities are not partnerships, as profits come from ancillary business activities and are
reinvested, not distributed as dividends to their members, and 2. The Partnership Acts
do not govern members of a corporation incorporated under the provisions of the
Corporations Act 2001 (Cth), a special Act of Parliament, or Royal Charter.

29. Which statements is incorrect? More complex business organisations are unsuitable for
larger businesses. Correct: 1. Like most partners, sole traders have unlimited personal
liability of the business/firm, 2. Upon incorporation, a corporation becomes a separate
legal entity, and 3. Diverse ownership requires proper management and expertise in
numerous areas.

30. Which statements is not correct? A partnership agreement must be in writing; it cannot
be reached orally or by a course of conduct. Correct: 1. if the agreement is that a
person should be paid a fixed sum by the firm, he/she may be a partner and jointly liable
to creditors of the firm, 2. the rights and obligations of partners to each other arise from
the partnership agreement, the statute and the equitable concept of the fiduciary, and
3. the Partnership Act determines partners’ rights, duties and interests, if not included in
the partnership agreement.

31. Which statement regarding persons of unsound mind and minors is incorrect? if a
partner who is a minor enters into a contract with a third party on behalf of the firm,
the minor is liable as far as private assets are concerned. Correct: 1. a partner of
unsound mind is capable of binding the firm and of being bound by co-partners, unless
proven that he/she was of unsound mind when the partnership was entered into and
the other partners knew this, 2. a creditor who has obtained judgment against the firm
may not seize the minor’s separate property, and 3. a minor will become liable as an
ordinary partner when he/she attains majority the partnership is not repudiated within a
reasonable time.

32. Which of the following statements is not correct? Incorporated limited partnerships
have been introduced in all Australian States and Territories. Correct: 1. as an agent, a
partner is able to bind the other partners and, as principal, be bound by the actions of
the other partners, 2. in New South Wales, Victoria, Queensland, South Australia,
Western Australia and Tasmania provision for limited partnerships is made in the
Partnership Act, and 3. the Partnership Act provides that the rules of the common law
and equity are to continue in force except insofar as they are inconsistent with the Act.

33. Which of the following statements regarding the written law is incorrect? The
Corporations Regulations 2001 (Cth), reg 2A.1.01 does not set a maxima of partners
for architects and accountants. Correct: 1. Under s 115 of the Corporations Act 2001
(Cth), the maximum number of persons who may form a partnership for the acquisition
of gain is 20, 2. Under the Corporations Regulations 2001 (Cth), reg 2A.1.01,
partnerships of more than 20 partners may be formed for certain professions/callings,
and 3. The Corporations Regulations 2001 (Cth), reg 2A.1.01 sets a maxima of partners
for medical and legal practitioners, veterinary surgeons, patent and trademark
attorneys, sharebrokers and stockbrokers and pharmaceutical chemists.

34. Which is incorrect statements regarding limited partnerships? A limited partner does
not have the right to inspect the books of the firm. Correct: 1. A limited partner must
not take part in the management of the business and does not have power to bind the
firm, 2. If a limited partner partakes in the management of the business, he/she is liable
as a general partner, and 3. Any differences arising as to ordinary matters connected
with the firm’s business are to be decided by a majority of the general partners.

35. Which of the following statements regarding dissolution of partnership is incorrect? A


firm’s assets as contributed by partners to make up capital losses need not be applied
to pay the firm’s debts/liabilities to non-partners. Correct: 1. In the event of
dissolution, losses must be met first out of profits, followed by capital, then by partners
in proportion to their share of profits, 2. After dissolution, each partner is entitled to
advances and residue by the firm, and 3. After the dissolution, each partner’s authority
to bind the firm continues, so far as necessary for winding up partnership affairs and
completing unfinished transactions.

36. Which statement regarding an outsize partnership is incorrect? Its partnership


agreement is invalid. Correct: 1. it has more than 20 partners, 2. it is liable to a criminal
penalty ($500), and 3. its agreement does not affect the enforceability of contracts or
other arrangements made.

37. Which statement is incorrect? Partnership may be formed in order to undertake a


single business transaction. Correct: 1. Carrying on a business implies repetition (Smith
v Anderson (1880) 15 Ch D 247), 2. In Khan v Miah [2000] 1 WLR 2123 it was held that
work, such as finding, acquiring and fitting out a shop/restaurant, is undertaken with a
view to profit, and 3. In Keith Spicer Ltd v Mansell [1970] 1 All ER 462 it was held that
ordering goods and opening a joint bank account in contemplation of a business are
insufficient for a partnership.

38. Which legislation controls the actual registration of the firm name? Business Names
Registration Act 2011 (Cth).

39. Which court finding is incorrect? In Mercantile Credit Co Ltd v Garrod [1962] 3 All ER
1103, damages were not recovered even though, from the plaintiff’s perspective, the
sale of the car was within the usual course of business. Correct: 1. In Goldberg v Jenkins
(1889) 15 VLR 36, the firm was not bound to the transaction because borrowing money
on behalf of the firm at over 60% interest when comparable rates were between 6% and
10% was beyond ‘the usual way’, 2. In Construction Engineering Pty Ltd v Hexyl Pty Ltd
(1985) 155 CLR 541, Hexyl was not liable because the construction contract appeared to
be between Construction Engineering and Tembel, 3. If judgment is obtained against one
or more partners of a firm, no action may be taken against the other partners, even if
satisfaction cannot be obtained from the partner(s) sued.
4/4/22, 2:12 PM Quiz 1_Chapters 1 -4_Session 3: Business Law-T122WSB-2

Quiz 1_Chapters 1 -4_Session 3


Due
Jan 17 at 12:15pm
Points
15
Questions
15
Available
Jan 17 at 11:59am - Jan 17 at 12:25pm
26 minutes
Time Limit
15 Minutes

This quiz was locked Jan 17 at 12:25pm.

Attempt History
Attempt Time Score
LATEST Attempt 1
15 minutes 8 out of 15

Score for this quiz:


8 out of 15
Submitted Jan 17 at 12:16pm
This attempt took 15 minutes.

Question 1 1
/ 1 pts

Which of the following statements about acceptance is false?

 
Acceptance can be by telephone or email or conduct

Correct!  
Acceptance can be in any manner chosen by the offeree

 
Acceptance must be unconditional

 
Acceptance can only be made by the person/s to whom the offer was made

Question 2 1
/ 1 pts

Section 15AA of the Acts Interpretation Act: Section 15AA of the Acts
Interpretation Act 1901 (Cth)

https://lms.westernsydney.edu.vn/courses/287/quizzes/3357?module_item_id=11129 1/8
4/4/22, 2:12 PM Quiz 1_Chapters 1 -4_Session 3: Business Law-T122WSB-2

 
Is an inflexible section

 
Provides that express mention of one matter suggests that other matters are
excluded

 
Provides that where there is a conflict between a specific and a general
provision, the : specific provision should be applied

Correct!  
Provides that the purpose of the statute or its object should be the preferred
method of : determining the statute’s meaning.

Question 3 1
/ 1 pts

Separation of powers: Which of the following statements about separation of


powers in Australia is correct?

 
The judiciary is the body that makes statute law.

 
The legislature is the branch that declares what the law is and interprets the
law.

Correct!  
The executive is the body that administers the law.

 
The legislature is the body that resolves disputes concerning the application of
law and : polices the law.

Question 4 1
/ 1 pts

https://lms.westernsydney.edu.vn/courses/287/quizzes/3357?module_item_id=11129 2/8
4/4/22, 2:12 PM Quiz 1_Chapters 1 -4_Session 3: Business Law-T122WSB-2

Simon and Stella, both of full legal capacity, agree to go on a date. Stella is
to pay for the : dinner, but she is running late and does not meet Simon at all.
Simon is embarrassed and : angry and calls Stella the next day to threaten to
sue her for his taxi fares and dinner : expenses. Will Simon succeed in his
claims?

 
Yes, because there is agreement

 
Yes, because there was consideration in that Simon incurred taxi and dinner
expenses

Correct!  
No, because Simon and Stella did not intend the agreement to create legally
enforceable : obligations

 
No, because there was no genuine consent of either party

Question 5 1
/ 1 pts

Equitable remedies: Which of the following is incorrect?: Equitable remedies:

 
Are discretionary

 
Include specific performance

Correct!  
Must be awarded in a separate proceeding to common law remedies such as
damages

 
Include injunctions

Question 6 0
/ 1 pts

https://lms.westernsydney.edu.vn/courses/287/quizzes/3357?module_item_id=11129 3/8
4/4/22, 2:12 PM Quiz 1_Chapters 1 -4_Session 3: Business Law-T122WSB-2

Civil and criminal law: Which of the following is not correct with respect to
civil law?

 
The typical purpose of a civil action is to obtain damages.

 
Under civil law, one person may sue another who has committed a wrongful
act.

You Answered  
Interrogatories and discovery are procedures that are available in civil
proceedings.

orrect Answer  
The document filed by the defendant is called a writ.

Question 7 0
/ 1 pts

Federal system: Which of the following statements is correct?

 
Australia is a federal system, with one Constitution and a number of non-law-
making : States and Territories.

 
Australia is a unitary system, where the States and Territories are constrained
in their : law-making powers by the Commonwealth.

orrect Answer  
Australia is a federal system, with two legal systems for each citizen.

You Answered  
Australia is a federal system with three branches of government: the judiciary,
the : executive and the Crown.

https://lms.westernsydney.edu.vn/courses/287/quizzes/3357?module_item_id=11129 4/8
4/4/22, 2:12 PM Quiz 1_Chapters 1 -4_Session 3: Business Law-T122WSB-2

Question 8 1
/ 1 pts

In Crown Melbourne Ltd v Cosmopolitan Hotel (Vic) Pty Ltd [2016] HCA 26
why did the : High Court disagree with the decision of the Victorian Court of
Appeal in finding there was : no case for promissory estoppel:

 
Even though the elements of estoppel had been proven no remedy was
provided as the : initial claim was brought too late and equity does not assist
claims lodged with unreasonable : delay

 
The plaintiff could not establish that the statement that it would be “looked
after at : renewal time” was capable of conveying to a reasonable person that
it was a genuine offer : of a further lease

 
There was a need for certainty as to what the parties had agreed on at the end
of the : negotiations to found a claim for estoppel

Correct!  
b & c

Question 9 1
/ 1 pts

A contract dividing the proceeds of a cocaine importing venture that has no


technical : defects and that is not overly harsh/unfair to either party is:

 
Valid

 
Voidable

Correct!  
Void

 
Unenforceable

https://lms.westernsydney.edu.vn/courses/287/quizzes/3357?module_item_id=11129 5/8
4/4/22, 2:12 PM Quiz 1_Chapters 1 -4_Session 3: Business Law-T122WSB-2

Question 10 1
/ 1 pts

If one party has threatened another party to enter into a contract, the element
that is : missing is:

 
Acceptance

 
Intention to create legal relations

Correct!  
Real/genuine consent

 
Legality

Question 11 0
/ 1 pts

Substantive and procedural law: Substantive law:

orrect Answer  
Refers to actual rights under the law

You Answered  
Is subsidiary to procedural law

 
Refers to the formal steps to enforcement of rights and duties under the law

 
Includes the rules of evidence

Question 12 0
/ 1 pts

The Age of Majority Act 1977 (Vic) reduced the age for contractual capacity
of a minor

https://lms.westernsydney.edu.vn/courses/287/quizzes/3357?module_item_id=11129 6/8
4/4/22, 2:12 PM Quiz 1_Chapters 1 -4_Session 3: Business Law-T122WSB-2

 
16

orrect Answer  
18

You Answered  
21

 
25

Question 13 0
/ 1 pts

Delegated legislation: Which of the following is not correct?: Delegated


legislation:

 
Is subordinate legislation

You Answered  
Is made under the authority of an Act of Parliament

 
Often contains more detailed rules than those of the associated Act

orrect Answer  
Can only be made by Government Ministers

Question 14 0
/ 1 pts

Delegated legislation: An example of delegated legislation is the:

 
Partnership Act 1958 (Vic)

 
Local Government Act 1993 (Qld), s 25

You Answered  
Corporations Bill 1988 (Cth)

orrect Answer  
Corporations Regulations 2001 (Cth)

https://lms.westernsydney.edu.vn/courses/287/quizzes/3357?module_item_id=11129 7/8
4/4/22, 2:12 PM Quiz 1_Chapters 1 -4_Session 3: Business Law-T122WSB-2

Question 15 0
/ 1 pts

Separation of powers: Which of the following is not one of the three branches
of the Commonwealth government : in Australia:

orrect Answer  
The Governor

 
The executive

 
The judiciary

You Answered  
The legislature

Quiz Score:
8 out of 15

https://lms.westernsydney.edu.vn/courses/287/quizzes/3357?module_item_id=11129 8/8
4/15/22, 8:14 PM Quiz 1_Chapters 1 -4_Session 3: Business Law-T122WSB-2

Quiz 1_Chapters 1 -4_Session 3


Due
Jan 17 at 12:15pm
Points
15
Questions
15
Available
Jan 17 at 11:59am - Jan 17 at 12:25pm
26 minutes
Time Limit
15 Minutes

This quiz was locked Jan 17 at 12:25pm.

Attempt History
Attempt Time Score
LATEST Attempt 1 15 minutes 9 out of 15

Score for this quiz:


9 out of 15
Submitted Jan 17 at 12:15pm
This attempt took 15 minutes.

Question 1 0
/ 1 pts

The rule of law: Which of the following concepts is closely related to the rule of
law?

 
Representative government

 
Responsible government

orrect Answer  
Due process

You Answered  
Separation of powers

Question 2 0
/ 1 pts

Which of the following is not a way in which an offer can be terminated?

https://lms.westernsydney.edu.vn/courses/287/quizzes/3357?module_item_id=11129 1/8
4/15/22, 8:14 PM Quiz 1_Chapters 1 -4_Session 3: Business Law-T122WSB-2

 
Lapse

You Answered  
Revocation before acceptance

orrect Answer  
Revocation after acceptance has been mailed where the postal rule applies

 
Lapse where no time was stipulated

Question 3 1
/ 1 pts

Changing the Constitution : Section 128 of the Commonwealth Constitution


provides that the Constitution can be: changed by referendum that requires a
“yes” vote:

 
in at least 2 States

 
in at least 3 States

 
in the ACT and NT (the Territories)

Correct!  
by the majority of voters and in a majority of States

Question 4 0
/ 1 pts

The main legal issue in Felthouse v Bindley (1862) 11CB (NS) 869 was:

 
the contract was formed due to offer and acceptance being proven

 
An offer to sell a horse was simply an invitation to treat

https://lms.westernsydney.edu.vn/courses/287/quizzes/3357?module_item_id=11129 2/8
4/15/22, 8:14 PM Quiz 1_Chapters 1 -4_Session 3: Business Law-T122WSB-2

orrect Answer  
generally silence /lack of action does not constitute acceptance of an offer

You Answered  
An offer can be revoked at any time prior to acceptance

Question 5 1
/ 1 pts

Which of the following is not a class of persons regarded by the law as wholly or
partly : incapable of entering into legally binding contracts?

 
Intoxicated persons

 
Minors

Correct!  
Women

 
Mentally incapacitated persons

Question 6 1
/ 1 pts

Ratio decidendi*: The ratio decidendi of a case:

 
Is the full judgment of all judges

 
Is a statement of principle that is important, yet not crucial to the decision

Correct!  
Is the reason given for deciding the case

 
Is contained within the last published judgment

https://lms.westernsydney.edu.vn/courses/287/quizzes/3357?module_item_id=11129 3/8
4/15/22, 8:14 PM Quiz 1_Chapters 1 -4_Session 3: Business Law-T122WSB-2

Question 7 1
/ 1 pts

Reporting obligations: What is a binding precedent?

 
A decision of another court that is of persuasive authority

 
With respect to Australia, it is a decision of the UK Supreme Court

Correct!  
A decision of a court that binds judges in a lower court in the same court
hierarchy

 
A decision of a different court on the same subject matter

Question 8 0
/ 1 pts

Separation of powers: Which of the following is not one of the three branches of
the Commonwealth government : in Australia:

orrect Answer  
The Governor

 
The executive

 
The judiciary

You Answered  
The legislature

Question 9 0
/ 1 pts

Criminal offences: Which of the following is incorrect with respect to criminal


offences?

https://lms.westernsydney.edu.vn/courses/287/quizzes/3357?module_item_id=11129 4/8
4/15/22, 8:14 PM Quiz 1_Chapters 1 -4_Session 3: Business Law-T122WSB-2

 
Indictable offences are generally the more serious offences.

You Answered  
The prosecution must prove its case beyond reasonable doubt.

 
Summary offences are determined by a magistrate without a jury.

orrect Answer  
A committal hearing is held before most summary offence matters.

Question 10 1
/ 1 pts

Australian Constitution : Which Act established the federal legal and political
system and converted the separate : colonies into states?

Correct!  
Commonwealth of Australia Constitution Act 1900

 
The Australia Act 1986

 
Statute of Westminster Adoption Act 1942 (Cth)

 
Statute of Westminster 1931 (IMP)

Question 11 1
/ 1 pts

Public and private law: Which of the following is not classified as private law?

 
The law of contract

Correct!  
Criminal law

 
The law of property

 
Corporations law

https://lms.westernsydney.edu.vn/courses/287/quizzes/3357?module_item_id=11129 5/8
4/15/22, 8:14 PM Quiz 1_Chapters 1 -4_Session 3: Business Law-T122WSB-2

Question 12 1
/ 1 pts

Farah agreed to take care of an elderly woman Marge and in return was
provided with a : house to live in that was promised by Marge to be Farah’s
after she died. Farah cared for the : woman for 23 years, but upon Marge’s
death Farah discovered that their oral agreement : was never put into writing.
The woman’s son moved into the house and Farah made a claim : to the house.
The court:

 
Is likely to reject Farah’s claim to the house as she provided no consideration
of any legal: value

 
Is likely to reject Farah’s claim to the house as the contract was not set down
in writing as : required

 
Is likely to order specific performance of the oral agreement because simple
contracts do : not need to be in writing

Correct!  
Is likely to apply the doctrine of part performance

Question 13 1
/ 1 pts

Andy promises Ellie $100 on her 21st birthday. If this promise was contained in
a simple : contract, the legal position is that such a promise is:

Correct!  
Never binding as it must have consideration supplied by Ellie

 
Binding only if Ellie and Andy had contractual capacity

https://lms.westernsydney.edu.vn/courses/287/quizzes/3357?module_item_id=11129 6/8
4/15/22, 8:14 PM Quiz 1_Chapters 1 -4_Session 3: Business Law-T122WSB-2

 
Sometimes binding but only after Ellie turned 21

 
Always binding because promises should not be broken

Question 14 0
/ 1 pts

Why are illusory terms in a contract problematic?

orrect Answer  
they are vague or ambiguous so they fail to create a legal obligation

You Answered  
They exist in one party’s imaginary belief only

 
Courts have difficulty interpreting them

 
Interpreting them in any way at all could result in an unfair decision

Question 15 1
/ 1 pts

The High Court’s decision in Waltons Stores (Interstate) Ltd v Maher (1988) 164
CLR 387:

 
Extended the doctrine of promissory estoppel such that it applies even where
the : promisor does not create or encourage the false assumption of the
promisee

Correct!  
Extended the doctrine of equitable estoppels such that it applies even where
there is no : pre-existing contractual relationship between the parties

https://lms.westernsydney.edu.vn/courses/287/quizzes/3357?module_item_id=11129 7/8
4/15/22, 8:14 PM Quiz 1_Chapters 1 -4_Session 3: Business Law-T122WSB-2

 
Restricted the doctrine of equitable estoppel such that, where the contract is
of a : commercial nature, the doctrine will not apply

 
Restricted the doctrine of promissory estoppel such that where an innocent
third party : has been affected, no remedy can be ordered

Quiz Score:
9 out of 15

https://lms.westernsydney.edu.vn/courses/287/quizzes/3357?module_item_id=11129 8/8
21:42, 11/04/2022 Quiz 1_Chapters 1 -4_Session 3: Business Law-T122WSB-2

Quiz 1_Chapters 1 -4_Session 3


Due
Jan 17 at 12:15pm
Points
15
Questions
15
Available
Jan 17 at 11:59am - Jan 17 at 12:25pm
26 minutes
Time Limit
15 Minutes

This quiz was locked Jan 17 at 12:25pm.

Attempt History
Attempt Time Score
LATEST Attempt 1
15 minutes 8 out of 15

Score for this quiz:


8 out of 15
Submitted Jan 17 at 12:14pm
This attempt took 15 minutes.

Question 1 1
/ 1 pts

In which of the following types of agreements is there a presumption that


parties intend : to be legally bound?

Correct!  
Partnership between husband and wife

 
Living arrangement between mother and daughter

 
Agreement between father and daughter

 
Dinner arrangement between friends

Question 2 0
/ 1 pts

Court hierarchy: Local or Magistrates Courts:

orrect Answer  
Are the lowest courts in the state hierarchy

You Answered  
Are the lowest courts in the federal hierarchy

https://lms.westernsydney.edu.vn/courses/287/quizzes/3357 1/7
21:42, 11/04/2022 Quiz 1_Chapters 1 -4_Session 3: Business Law-T122WSB-2

 
Are an intermediate court

 
Are above Country Courts in the hierarchy

Question 3 0
/ 1 pts

Which of the following statements about bilateral and unilateral contracts is


not correct?

 
More than one party is required for both bilateral and unilateral contracts

orrect Answer  
A unilateral contract does not require consideration but simply a promise to
perform an : act is sufficient

 
Both parties are obligated to perform their promises in a bilateral contract

You Answered  
Only one of the parties is obligated to perform an action in a unilateral
contract

Question 4 0
/ 1 pts

Section 15AA of the Acts Interpretation Act: Section 15AA of the Acts
Interpretation Act 1901 (Cth)

 
Is an inflexible section

 
Provides that express mention of one matter suggests that other matters are
excluded

You Answered  
Provides that where there is a conflict between a specific and a general
provision, the : specific provision should be applied

https://lms.westernsydney.edu.vn/courses/287/quizzes/3357 2/7
21:42, 11/04/2022 Quiz 1_Chapters 1 -4_Session 3: Business Law-T122WSB-2

orrect Answer  
Provides that the purpose of the statute or its object should be the preferred
method of : determining the statute’s meaning.

Question 5 1
/ 1 pts

Criminal proceedings: An indictable offence is:

 
A more serious civil wrong

 
A less serious civil wrong

Correct!  
A more serious criminal offence

 
A less serious criminal offence

Question 6 0
/ 1 pts

Law Reports: Where are decisions of the High Court of Australia found?: only
online

You Answered  
In the High Court reports

orrect Answer  
In the Commonwealth Law Reports

 
In the Common Law Reports

Question 7 1
/ 1 pts

Public and private law: Which of the following is not classified as private law?

 
The law of contract

https://lms.westernsydney.edu.vn/courses/287/quizzes/3357 3/7
21:42, 11/04/2022 Quiz 1_Chapters 1 -4_Session 3: Business Law-T122WSB-2

Correct!  
Criminal law

 
The law of property

 
Corporations law

Question 8 1
/ 1 pts

Which of the following circumstances are likely to affect the consent of one or
both : parties to a contract:

 
Mistake

 
Duress and undue influence

 
Lack of writing

Correct!  
a & b

Question 9 1
/ 1 pts

Reception of English law in Australia: Which latin phrase explained the


rationale for applying English laws to the new Colony of : New South Wales?

 
actus reus

Correct!  
terra nullius

 
ratio decidendi

 
prima facie

Question 10 1
/ 1 pts

https://lms.westernsydney.edu.vn/courses/287/quizzes/3357 4/7
21:42, 11/04/2022 Quiz 1_Chapters 1 -4_Session 3: Business Law-T122WSB-2

With respect to revocation, which of the following statements is not correct?

 
Revocation need not be in words

Correct!  
The offeror must personally communicate the revocation to the offeree

 
The offeree may accept the offer until such time as they become aware of the
revocation

 
Where an offer has been made to the world at large, revocation does not need
to be seen : by everyone in order to be effective

Question 11 0
/ 1 pts

Sources of law: The two main types of law in Australia are:

 
Common law and judge-made law

orrect Answer  
Statute law and judge-made law

You Answered  
Statute law and Acts of Parliament

 
Common law and equity

Question 12 0
/ 1 pts

A contract dividing the proceeds of a cocaine importing venture that has no


technical : defects and that is not overly harsh/unfair to either party is:

 
Valid

 
Voidable

https://lms.westernsydney.edu.vn/courses/287/quizzes/3357 5/7
21:42, 11/04/2022 Quiz 1_Chapters 1 -4_Session 3: Business Law-T122WSB-2

orrect Answer  
Void

You Answered  
Unenforceable

Question 13 1
/ 1 pts

In Ashton v Pratt [2015] NSWCA 12 what was the main reason for the court
deciding that : there was no intention to create a legally binding contract?

 
The agreement was not reduced to writing

Correct!  
The verbal language of the agreement greatly lacked detail from either party
and did not : indicate definite obligations

 
Pratt lacked the necessary contractual capacity due to his age and ill health

 
The plaintiff had been a worker in the escort business and not morally entitled
to any : monies under the verbal agreement

Question 14 1
/ 1 pts

Simone advertises a car for $4,000. John responds to the advertisement and
asks : whether Simone will take $3,000. Simone says ‘no’, but that she will
hold the offer open for : a week. The next day she sells it to Samantha. :
Which of the following statements is correct?

 
Simone has to hold the offer open as she made a promise

Correct!  
Simone does not have to hold the offer open as no consideration was given

 
John’s question is an invitation to treat

https://lms.westernsydney.edu.vn/courses/287/quizzes/3357 6/7
21:42, 11/04/2022 Quiz 1_Chapters 1 -4_Session 3: Business Law-T122WSB-2

 
Simone does not have to hold the offer open as John made a counter-offer

Question 15 0
/ 1 pts

Which of the following is not one of Brennan J’s six criteria for estoppel from
Waltons: Stores (Interstate) Ltd v Maher (1988) 164 CLR 387?

 
The promisor induced an assumption

You Answered  
The promisee acted in reliance on that assumption

orrect Answer  
The promisor knew or was reckless as to whether the promisee intended to
act in that

 
The promisee will suffer a material loss if the assumption is not fulfilled

Quiz Score:
8 out of 15

https://lms.westernsydney.edu.vn/courses/287/quizzes/3357 7/7
08:33, 19/04/2022 Quiz 1_Chapters 1 -4_Session 3: Business Law-T122WSB-3

Quiz 1_Chapters 1 -4_Session 3


Due
Jan 18 at 8:15am
Points
15
Questions
15
Available
Jan 18 at 7:59am - Jan 18 at 8:25am
26 minutes
Time Limit
15 Minutes

This quiz was locked Jan 18 at 8:25am.

Attempt History
Attempt Time Score
LATEST Attempt 1
11 minutes 9 out of 15

Score for this quiz:


9 out of 15
Submitted Jan 18 at 8:10am
This attempt took 11 minutes.

Question 1 0
/ 1 pts

Which of the following statements about acceptance is false?

 
Acceptance can be by telephone or email or conduct

orrect Answer  
Acceptance can be in any manner chosen by the offeree

You Answered  
Acceptance must be unconditional

 
Acceptance can only be made by the person/s to whom the offer was made

Question 2 0
/ 1 pts

Civil and criminal law: Which of the following is not correct with respect to
civil law?

https://lms.westernsydney.edu.vn/courses/286/quizzes/3358 1/8
08:33, 19/04/2022 Quiz 1_Chapters 1 -4_Session 3: Business Law-T122WSB-3

You Answered  
The typical purpose of a civil action is to obtain damages.

 
Under civil law, one person may sue another who has committed a wrongful
act.

 
Interrogatories and discovery are procedures that are available in civil
proceedings.

orrect Answer  
The document filed by the defendant is called a writ.

Question 3 1
/ 1 pts

The Age of Majority Act 1977 (Vic) reduced the age for contractual capacity
of a minor

 
16

Correct!  
18

 
21

 
25

Question 4 1
/ 1 pts

Ratio decidendi*: The ratio decidendi of a case:

 
Is the full judgment of all judges

 
Is a statement of principle that is important, yet not crucial to the decision

https://lms.westernsydney.edu.vn/courses/286/quizzes/3358 2/8
08:33, 19/04/2022 Quiz 1_Chapters 1 -4_Session 3: Business Law-T122WSB-3

Correct!  
Is the reason given for deciding the case

 
Is contained within the last published judgment

Question 5 1
/ 1 pts

Simon and Stella, both of full legal capacity, agree to go on a date. Stella is
to pay for the : dinner, but she is running late and does not meet Simon at all.
Simon is embarrassed and : angry and calls Stella the next day to threaten to
sue her for his taxi fares and dinner : expenses. Will Simon succeed in his
claims?

 
Yes, because there is agreement

 
Yes, because there was consideration in that Simon incurred taxi and dinner
expenses

Correct!  
No, because Simon and Stella did not intend the agreement to create legally
enforceable : obligations

 
No, because there was no genuine consent of either party

Question 6 1
/ 1 pts

Australian Constitution : Which Act established the federal legal and political
system and converted the separate : colonies into states?

Correct!  
Commonwealth of Australia Constitution Act 1900

 
The Australia Act 1986

https://lms.westernsydney.edu.vn/courses/286/quizzes/3358 3/8
08:33, 19/04/2022 Quiz 1_Chapters 1 -4_Session 3: Business Law-T122WSB-3

 
Statute of Westminster Adoption Act 1942 (Cth)

 
Statute of Westminster 1931 (IMP)

Question 7 1
/ 1 pts

Public and private law: Which of the following is not classified as private law?

 
The law of contract

Correct!  
Criminal law

 
The law of property

 
Corporations law

Question 8 1
/ 1 pts

Andy promises Ellie $100 on her 21st birthday. If this promise was contained
in a simple : contract, the legal position is that such a promise is:

Correct!  
Never binding as it must have consideration supplied by Ellie

 
Binding only if Ellie and Andy had contractual capacity

 
Sometimes binding but only after Ellie turned 21

 
Always binding because promises should not be broken

Question 9 1
/ 1 pts

https://lms.westernsydney.edu.vn/courses/286/quizzes/3358 4/8
08:33, 19/04/2022 Quiz 1_Chapters 1 -4_Session 3: Business Law-T122WSB-3

If one party has threatened another party to enter into a contract, the element
that is : missing is:

 
Acceptance

 
Intention to create legal relations

Correct!  
Real/genuine consent

 
Legality

Question 10 0
/ 1 pts

A qualified acceptance may also be known as a/an:

orrect Answer  
Counteroffer

 
Invitation to treat

 
Condition subsequent

You Answered  
Conditional agreement

Question 11 0
/ 1 pts

With respect to revocation, which of the following statements is not correct?

 
Revocation need not be in words

orrect Answer  
The offeror must personally communicate the revocation to the offeree

https://lms.westernsydney.edu.vn/courses/286/quizzes/3358 5/8
08:33, 19/04/2022 Quiz 1_Chapters 1 -4_Session 3: Business Law-T122WSB-3

 
The offeree may accept the offer until such time as they become aware of the
revocation

You Answered  
Where an offer has been made to the world at large, revocation does not
need to be seen : by everyone in order to be effective

Question 12 0
/ 1 pts

In Elizabeth City Centre Pty Ltd v Corralyn Pty Ltd (1995) 63 SASR 235, the
option to : exercise renewal of the lease:

 
Was effective because of the operation of the postal rule

You Answered  
Effective communication of the renewal requirement had not been made

orrect Answer  
Negated the postal rule because the notification was not sent by certified mail
as : required

 
b and c

Question 13 0
/ 1 pts

Which of the following statements about consideration is correct?

 
A gratuitous promise is enforceable if contained in a simple contract

You Answered  
Good consideration may consist of performing an existing legal obligation

https://lms.westernsydney.edu.vn/courses/286/quizzes/3358 6/8
08:33, 19/04/2022 Quiz 1_Chapters 1 -4_Session 3: Business Law-T122WSB-3

 
Good consideration can be present, future and even “past”

orrect Answer  
Consideration need not be adequate

Question 14 1
/ 1 pts

Business law: Which of the following no longer regulates business or


commercial law?

Correct!  
Trade Practices Act 1974 (Cth)

 
Competition and Consumer Act 2010 (Cth)

 
Corporations Act 2001 (Cth)

 
The law of contract

Question 15 1
/ 1 pts

Section 15AA of the Acts Interpretation Act: Section 15AA of the Acts
Interpretation Act 1901 (Cth)

 
Is an inflexible section

 
Provides that express mention of one matter suggests that other matters are
excluded

 
Provides that where there is a conflict between a specific and a general
provision, the : specific provision should be applied

https://lms.westernsydney.edu.vn/courses/286/quizzes/3358 7/8
08:33, 19/04/2022 Quiz 1_Chapters 1 -4_Session 3: Business Law-T122WSB-3

Correct!  
Provides that the purpose of the statute or its object should be the preferred
method of : determining the statute’s meaning.

Quiz Score:
9 out of 15

https://lms.westernsydney.edu.vn/courses/286/quizzes/3358 8/8
4/10/22, 10:18 PM Quiz 1_Chapters 1 -4_Session 3: Business Law-T122WSB-2

Quiz 1_Chapters 1 -4_Session 3


Due
Jan 17 at 12:15pm
Points
15
Questions
15
Available
Jan 17 at 11:59am - Jan 17 at 12:25pm
26 minutes
Time Limit
15 Minutes

This quiz was locked Jan 17 at 12:25pm.

Attempt History
Attempt Time Score
LATEST Attempt 1
15 minutes 7 out of 15

Score for this quiz:


7 out of 15
Submitted Jan 17 at 12:16pm
This attempt took 15 minutes.

Question 1 1
/ 1 pts

Constitution: Section 51 Commonwealth Constitution grants what type of law


making power to the : Commonwealth Parliament?

Correct!  
Concurrent powers

 
exclusive powers

 
residual

 
inherent

Question 2 0
/ 1 pts

Which of the following is not one of Brennan J’s six criteria for estoppel from
Waltons: Stores (Interstate) Ltd v Maher (1988) 164 CLR 387?

https://lms.westernsydney.edu.vn/courses/287/quizzes/3357?module_item_id=11129 1/8
4/10/22, 10:18 PM Quiz 1_Chapters 1 -4_Session 3: Business Law-T122WSB-2

 
The promisor induced an assumption

You Answered  
The promisee acted in reliance on that assumption

orrect Answer  
The promisor knew or was reckless as to whether the promisee intended to
act in that

 
The promisee will suffer a material loss if the assumption is not fulfilled

Question 3 1
/ 1 pts

Royal Assent: Who or what gives Royal Assent to an Act of the


Commonwealth Parliament?

 
The Queen

Correct!  
The Governor-General on the Queen’s behalf

 
The Prime Minister

 
Cabinet

Question 4 0
/ 1 pts

With respect to revocation, which of the following statements is not correct?

 
Revocation need not be in words

orrect Answer  
The offeror must personally communicate the revocation to the offeree

https://lms.westernsydney.edu.vn/courses/287/quizzes/3357?module_item_id=11129 2/8
4/10/22, 10:18 PM Quiz 1_Chapters 1 -4_Session 3: Business Law-T122WSB-2

 
The offeree may accept the offer until such time as they become aware of the
revocation

You Answered  
Where an offer has been made to the world at large, revocation does not
need to be seen : by everyone in order to be effective

Question 5 1
/ 1 pts

Court hierarchy: The highest court in Australia is:

 
The Privy Council

 
The Federal Court of Australia

Correct!  
The High Court of Australia

 
The Supreme Court of Australia

Question 6 0
/ 1 pts

The High Court’s decision in Waltons Stores (Interstate) Ltd v Maher (1988)
164 CLR 387:

You Answered  
Extended the doctrine of promissory estoppel such that it applies even where
the : promisor does not create or encourage the false assumption of the
promisee

https://lms.westernsydney.edu.vn/courses/287/quizzes/3357?module_item_id=11129 3/8
4/10/22, 10:18 PM Quiz 1_Chapters 1 -4_Session 3: Business Law-T122WSB-2

orrect Answer  
Extended the doctrine of equitable estoppels such that it applies even where
there is no : pre-existing contractual relationship between the parties

 
Restricted the doctrine of equitable estoppel such that, where the contract is of
a : commercial nature, the doctrine will not apply

 
Restricted the doctrine of promissory estoppel such that where an innocent
third party : has been affected, no remedy can be ordered

Question 7 1
/ 1 pts

Sally puts up a sign stating that she has lost her dogs and will pay a reward
of $100 for : their return to her within three days. Which of the following
statements is correct?

 
Jim will not get the reward because he returned the dog but did not verbally :
communicate acceptance of the offer to Sally

 
Nadia will get the reward as she returned the dog to the RSPCA

Correct!  
Chang will not get the reward because he returned the dog knowing it was
Sally’s, but : was unaware of her offer of a reward

 
Isabel will get the reward because she was only one day late in returning the
dogs (and to : not give her the reward would be unreasonable)

https://lms.westernsydney.edu.vn/courses/287/quizzes/3357?module_item_id=11129 4/8
4/10/22, 10:18 PM Quiz 1_Chapters 1 -4_Session 3: Business Law-T122WSB-2

Question 8 0
/ 1 pts

A qualified acceptance may also be known as a/an:

orrect Answer  
Counteroffer

 
Invitation to treat

 
Condition subsequent

You Answered  
Conditional agreement

Question 9 1
/ 1 pts

In Crown Melbourne Ltd v Cosmopolitan Hotel (Vic) Pty Ltd [2016] HCA 26
why did the : High Court disagree with the decision of the Victorian Court of
Appeal in finding there was : no case for promissory estoppel:

 
Even though the elements of estoppel had been proven no remedy was
provided as the : initial claim was brought too late and equity does not assist
claims lodged with unreasonable : delay

 
The plaintiff could not establish that the statement that it would be “looked
after at : renewal time” was capable of conveying to a reasonable person that
it was a genuine offer : of a further lease

 
There was a need for certainty as to what the parties had agreed on at the end
of the : negotiations to found a claim for estoppel

Correct!  
b & c

https://lms.westernsydney.edu.vn/courses/287/quizzes/3357?module_item_id=11129 5/8
4/10/22, 10:18 PM Quiz 1_Chapters 1 -4_Session 3: Business Law-T122WSB-2

Question 10 0
/ 1 pts

Interpretation – extrinsic materials: In interpretation, extrinsic materials:

You Answered  
Are required to be referred to by a court

 
Are only to be referred to where the meaning of the provision is ambiguous

orrect Answer  
Are allowed to be referred to by a court

 
Are to be referred to regardless of the length of delay this will cause
proceedings

Question 11 0
/ 1 pts

Native Title: Native Title was first recognised in which of the following cases?

orrect Answer  
Mabo v State of Queensland (No 2) (1992) 175 CLR 1

 
Wik Peoples v State of Queensland (1996) 187 CLR 1

You Answered  
Brandy v Human Rights and Equal Opportunity Commission (1995) 183 CLR
245

 
Commonwealth v Jones (1901) 174 CLR 25

Question 12 1
/ 1 pts

https://lms.westernsydney.edu.vn/courses/287/quizzes/3357?module_item_id=11129 6/8
4/10/22, 10:18 PM Quiz 1_Chapters 1 -4_Session 3: Business Law-T122WSB-2

Jack was subpoenaed to give evidence on Rods behalf. Jack claimed Rod
promised him : $2000 to give evidence. Would Jack recover that money?

Correct!  
No - Jack’s public duty is to give evidence in response to that subpoena.

 
No this is illegal

 
Yes if Rod put that in writing

 
Always binding because promises should not be broken

Question 13 0
/ 1 pts

Civil and criminal law: Which of the following is not correct with respect to
civil law?

 
The typical purpose of a civil action is to obtain damages.

You Answered  
Under civil law, one person may sue another who has committed a wrongful
act.

 
Interrogatories and discovery are procedures that are available in civil
proceedings.

orrect Answer  
The document filed by the defendant is called a writ.

Question 14 0
/ 1 pts

In Ashton v Pratt [2015] NSWCA 12 what was the main reason for the court
deciding that : there was no intention to create a legally binding contract?

https://lms.westernsydney.edu.vn/courses/287/quizzes/3357?module_item_id=11129 7/8
4/10/22, 10:18 PM Quiz 1_Chapters 1 -4_Session 3: Business Law-T122WSB-2

 
The agreement was not reduced to writing

orrect Answer  
The verbal language of the agreement greatly lacked detail from either party
and did not : indicate definite obligations

You Answered  
Pratt lacked the necessary contractual capacity due to his age and ill health

 
The plaintiff had been a worker in the escort business and not morally entitled
to any : monies under the verbal agreement

Question 15 1
/ 1 pts

Which of the following is not a way in which an offer can be terminated?

 
Lapse

 
Revocation before acceptance

Correct!  
Revocation after acceptance has been mailed where the postal rule applies

 
Lapse where no time was stipulated

Quiz Score:
7 out of 15

https://lms.westernsydney.edu.vn/courses/287/quizzes/3357?module_item_id=11129 8/8
Quiz 1_Chapters 1 -4_Session 3
Due
Jan 18 at 8:15am
Points
15
Questions
15
Available
Jan 18 at 7:59am - Jan 18 at 8:25am
26 minutes
Time Limit
15 Minutes

This quiz was locked Jan 18 at 8:25am.

Attempt History
Attempt Time Score
LATEST Attempt 1
15 minutes 6 out of 15

Score for this quiz:


6 out of 15
Submitted Jan 18 at 8:15am
This attempt took 15 minutes.

Question 1 1
/ 1 pts

Interpretation : Which of the following is not correct?: With respect to


interpretation, the main approach used by courts is:

 
The literal approach

Correct!  
The purposive approach

 
The requirement that the natural and ordinary meaning to be given to words

 
The courts are free to exercise judicial discretion

Question 2 0
/ 1 pts

The High Court’s decision in Waltons Stores (Interstate) Ltd v Maher (1988)
164 CLR 387:
 
Extended the doctrine of promissory estoppel such that it applies even where
the : promisor does not create or encourage the false assumption of the
promisee

orrect Answer  
Extended the doctrine of equitable estoppels such that it applies even where
there is no : pre-existing contractual relationship between the parties

 
Restricted the doctrine of equitable estoppel such that, where the contract is of
a : commercial nature, the doctrine will not apply

You Answered  
Restricted the doctrine of promissory estoppel such that where an innocent
third party : has been affected, no remedy can be ordered

Question 3 0
/ 1 pts

Royal Assent: Who or what gives Royal Assent to an Act of the


Commonwealth Parliament?

You Answered  
The Queen

orrect Answer  
The Governor-General on the Queen’s behalf

 
The Prime Minister

 
Cabinet

Question 4 1
/ 1 pts
Which of the following statements about acceptance is false?

 
Acceptance can be by telephone or email or conduct

Correct!  
Acceptance can be in any manner chosen by the offeree

 
Acceptance must be unconditional

 
Acceptance can only be made by the person/s to whom the offer was made

Question 5 1
/ 1 pts

Which of the following statements about the contractual obligations of


government : policy proposals is incorrect?

 
The courts tend not to assign contractual obligations to government policy
promises

 
Where the government enters into an ordinary commercial agreement to buy
services, : the law of contract regulates this agreement

Correct!  
Where the government makes a policy promise and a citizen relies on it the
court will : always regard that policy commitment as a binding contractual
obligation

 
In Australian Woollen Mills Pty Ltd v Commonwealth of Australia (1954) 92
CLR 424, the : government’s wool subsidy was found to be an administrative
scheme and not contractual : obligations
Question 6 0
/ 1 pts

Business law: Which of the following no longer regulates business or


commercial law?

orrect Answer  
Trade Practices Act 1974 (Cth)

You Answered  
Competition and Consumer Act 2010 (Cth)

 
Corporations Act 2001 (Cth)

 
The law of contract

Question 7 1
/ 1 pts

Sally puts up a sign stating that she has lost her dogs and will pay a reward
of $100 for : their return to her within three days. Which of the following
statements is correct?

 
Jim will not get the reward because he returned the dog but did not verbally :
communicate acceptance of the offer to Sally

 
Nadia will get the reward as she returned the dog to the RSPCA

Correct!  
Chang will not get the reward because he returned the dog knowing it was
Sally’s, but : was unaware of her offer of a reward

 
Isabel will get the reward because she was only one day late in returning the
dogs (and to : not give her the reward would be unreasonable)
Question 8 0
/ 1 pts

Delegated legislation: An example of delegated legislation is the:

 
Partnership Act 1958 (Vic)

You Answered  
Local Government Act 1993 (Qld), s 25

 
Corporations Bill 1988 (Cth)

orrect Answer  
Corporations Regulations 2001 (Cth)

Question 9 1
/ 1 pts

With respect to the contractual capacity of corporations, s 124 of the


Corporations Act : 2001 (Cth) gives them:

 
Only the capacity to make contracts approved by shareholders in general
meeting

 
The capacity to make contracts but only in the state or territory where the
corporation : conducts its business

Correct!  
All the legal capacity of a natural person (together with some additional
powers that can : only be exercised by a corporation)

 
Different capacity to contract depending on whether the corporation is public
or : proprietary
Question 10 0
/ 1 pts

Which of the following is not one of Brennan J’s six criteria for estoppel from
Waltons: Stores (Interstate) Ltd v Maher (1988) 164 CLR 387?

 
The promisor induced an assumption

 
The promisee acted in reliance on that assumption

orrect Answer  
The promisor knew or was reckless as to whether the promisee intended to
act in that

You Answered  
The promisee will suffer a material loss if the assumption is not fulfilled

Question 11 1
/ 1 pts

Which of the following statements about bilateral and unilateral contracts is


not correct?

 
More than one party is required for both bilateral and unilateral contracts

Correct!  
A unilateral contract does not require consideration but simply a promise to
perform an : act is sufficient

 
Both parties are obligated to perform their promises in a bilateral contract

 
Only one of the parties is obligated to perform an action in a unilateral contract
Unanswered Question 12 0
/ 1 pts

What was one of the “practical benefits” received by Roffey in Williams v


Roffey Bros &: Nicholls (Contractors) Ltd [1990] 1 All ER 512?

 
Williams did not sue Roffey for breach of contract

 
Roffey had no reason to doubt Williams would not complete his side of the
bargain

 
Roffey received additional payment from Williams

orrect Answer  
Roffey did not need to find another subcontractor

Question 13 0
/ 1 pts

Sources of law – statute: Which of the following is incorrect?: Statutes may:

 
Codify the law

You Answered  
Bring new laws into existence

 
Repeal laws

orrect Answer  
Not overrule existing common law

Question 14 0
/ 1 pts

Federal system: Which of the following statements is correct?


 
Australia is a federal system, with one Constitution and a number of non-law-
making : States and Territories.

You Answered  
Australia is a unitary system, where the States and Territories are constrained
in their : law-making powers by the Commonwealth.

orrect Answer  
Australia is a federal system, with two legal systems for each citizen.

 
Australia is a federal system with three branches of government: the judiciary,
the : executive and the Crown.

Question 15 0
/ 1 pts

Ratio decidendi*: The ratio decidendi of a case:

You Answered  
Is the full judgment of all judges

 
Is a statement of principle that is important, yet not crucial to the decision

orrect Answer  
Is the reason given for deciding the case

 
Is contained within the last published judgment

Quiz Score:
6 out of 15
4/4/22, 2:13 PM Quiz 2_Chapters 5 -13_Session 5: Business Law-T122WSB-2

Quiz 2_Chapters 5 -13_Session 5


Due
Feb 14 at 3:15pm
Points
15
Questions
15
Available
Feb 14 at 2:59pm - Feb 14 at 3:25pm
26 minutes
Time Limit
15 Minutes

This quiz was locked Feb 14 at 3:25pm.

Attempt History
Attempt Time Score
LATEST Attempt 1
15 minutes 7 out of 15

Score for this quiz:


7 out of 15
Submitted Feb 14 at 3:25pm
This attempt took 15 minutes.

Question 1 1
/ 1 pts

Compensatory damages are also known as:

 
Expectation damages

 
Nominal damages

Correct!  
Actual /ordinary damages

 
Punitive damages

Question 2 0
/ 1 pts

Which of the following is not a type of termination?

 
Termination by bankruptcy

https://lms.westernsydney.edu.vn/courses/287/quizzes/3367?module_item_id=11134 1/9
4/4/22, 2:13 PM Quiz 2_Chapters 5 -13_Session 5: Business Law-T122WSB-2

 
When the parties have fully and exactly performed their obligations to each
other under : the contract

You Answered  
When a deed displaces a simple contract

orrect Answer  
Where an event occurs that is not the fault of either party that causes a
fundamental : change to the nature of the contract and the parties obligations
and although the contract : covers that eventuality, it would cause hardship to
one party to enforce it

Question 3 0
/ 1 pts

Which of the following statements is correct?

orrect Answer  
A contract may be illegal as performed where only one party performs it in an
illegal : manner

You Answered  
A contract that is legal as formed will be legal as performed

 
The courts are most likely to find a contract illegal and unenforceable where a
statute : penalises the way it is performed

 
Any illegal conduct will make a contract illegal as performed

Question 4 0
/ 1 pts

Which of the following is not usually recoverable by way of damages?

https://lms.westernsydney.edu.vn/courses/287/quizzes/3367?module_item_id=11134 2/9
4/4/22, 2:13 PM Quiz 2_Chapters 5 -13_Session 5: Business Law-T122WSB-2

orrect Answer  
Compensation for mere inconvenience or disappointment

You Answered  
Loss that is difficult to estimate

 
Nominal loss

 
Expenses incurred in reliance on the other party’s promise to perform

Question 5 0
/ 1 pts

Which of the following is a circumstance where specific performance would


be ordered?

 
Where the subject matter of the contract is personal property that is not rare or
unique

 
Where a famous actor is to work as a Master of Ceremonies at a function

orrect Answer  
Where the subject matter of the contract is an apartment in a complex of 20
apartments

You Answered  
Where a non-famous actor is to wear a gorilla suit every day to advertise a
product

Question 6 1
/ 1 pts

The remedy of restitution is based on the concept of?

https://lms.westernsydney.edu.vn/courses/287/quizzes/3367?module_item_id=11134 3/9
4/4/22, 2:13 PM Quiz 2_Chapters 5 -13_Session 5: Business Law-T122WSB-2

 
Damages should be provided for part performance

 
There is such concept in equity

 
The parties apportion the performance and rewards are paid accordingly

Correct!  
Unjust enrichment where the defendant is unjustly enriched at the plaintiff’s
expense

Question 7 1
/ 1 pts

Which of the following are not usually compensated for with payment of
damages?

Correct!  
Injured feelings/disappointment

 
Nominal loss

 
Loss that is difficult for the court to estimate

 
Loss of profit expected to be received

Question 8 1
/ 1 pts

With respect to substantial performance and partial performance:

 
Partial performance is a lower level of performance than substantial
performance, and : there is no deduction of the contract price available where
substantial performance has : occurred

https://lms.westernsydney.edu.vn/courses/287/quizzes/3367?module_item_id=11134 4/9
4/4/22, 2:13 PM Quiz 2_Chapters 5 -13_Session 5: Business Law-T122WSB-2

 
Substantial performance is where the parties terminate the contract for less
than full : performance

Correct!  
Partial performance is where a party voluntarily accepts less than full
performance by : the other party

 
Partial performance is where the court terminates the contract for less than
exact : performance

Question 9 0
/ 1 pts

Which of the following is not correct with respect to the Australian Consumer
Law?

 
It is contained within Schedule 2 to the Competition and Consumer Act 2010
(Cth)

You Answered  
It applies to conduct engaged in outside of Australia

orrect Answer  
Owing to the Commonwealth’s limited lawmaking powers under s 51 of the :
Constitution, it applies only to corporations

 
The Australian Consumer Law is applied in each State and Territory

Question 10 1
/ 1 pts

https://lms.westernsydney.edu.vn/courses/287/quizzes/3367?module_item_id=11134 5/9
4/4/22, 2:13 PM Quiz 2_Chapters 5 -13_Session 5: Business Law-T122WSB-2

Which of the following is not one of the four ways a statute may render a
contract illegal : set out in Yango Pastoral Co v First Chicago Australia Ltd
(1978) 139 CLR 410? The contract:

 
May require an action that statute forbids

 
May be performed in a prohibited manner

 
May have been made to effect an unlawful purpose

Correct!  
May set out an unreasonable restraint

Question 11 1
/ 1 pts

Jacq and Jack robbed the Commercial Bank. Jacq thinks that Jack was too
aggressive : with the tellers and the robbery could have gone very wrong.
Jacq is now refusing to give : Jack his share of the takings. Which of the
following is correct?

 
Jack provided consideration in the form of performing the robbery and is
entitled to his : share

 
Under the principle of freedom of contract, Jacq and Jack are entitled to agree
to : whatever they wish and the contract will be enforceable

Correct!  
The contract involved the commission of a crime and is therefore illegal and :
unenforceable

 
The contract will be unenforceable if the courts deem it to be against public
policy

https://lms.westernsydney.edu.vn/courses/287/quizzes/3367?module_item_id=11134 6/9
4/4/22, 2:13 PM Quiz 2_Chapters 5 -13_Session 5: Business Law-T122WSB-2

Question 12 0
/ 1 pts

Janey takes her fur stole to the dry cleaner. When she returns to collect it,
there is a : black mark in the shape of an iron in the middle of her stole and it
is ruined. The dry cleaner : tells Janey that he is not liable for the damage to
her fur stole as there was an exemption : clause in their agreement. Which of
the following is not correct?

orrect Answer  
If the exemption clause was merely contained within a notice on the wall, the
dry : cleaner will be liable for the damage

 
If the exemption clause was on the dry cleaning docket Janey was given and
she was told : “don’t worry about this, it’s just the docket you have to give us
when you come to collect : your stole”, the dry cleaner will not be able to rely
on the exemption clause

You Answered  
In order for the exemption clause to be effective where it was not in a signed
document, : it had to have been brought to Janey’s attention

 
If the exemption clause was on the dry cleaning docket Janey was given
rather than a : formal written contract, the onus will be on the dry cleaner to
prove that Janey was aware it : contained conditions that would modify the
agreement

Question 13 0
/ 1 pts

Criminal proceedings may be brought for:

https://lms.westernsydney.edu.vn/courses/287/quizzes/3367?module_item_id=11134 7/9
4/4/22, 2:13 PM Quiz 2_Chapters 5 -13_Session 5: Business Law-T122WSB-2

You Answered  
A contravention of s 18 of the Australian Consumer Law

orrect Answer  
Unfair practices

 
Unfair contract terms

 
Unconscionable conduct

Question 14 0
/ 1 pts

In Trident General Insurance Co Ltd v McNiece Bros Pty Ltd (1988) 165 CLR
107:

 
Blue Circle was a sub-contractor whose worker was injured, and privity
precluded the : worker from getting an indemnity under the contract

orrect Answer  
It was held that the subcontractor could get an indemnity even though the :
subcontractor had given no consideration under the contract

 
In Deane J’s view Trident, the insurance company, had an obligation to the
subcontractor because otherwise Trident would be unjustly enriched

You Answered  
The majority were of the view that the “settled and fundamental” doctrine of
privity of : contract should not be overturned by the court

Question 15 1
/ 1 pts

https://lms.westernsydney.edu.vn/courses/287/quizzes/3367?module_item_id=11134 8/9
4/4/22, 2:13 PM Quiz 2_Chapters 5 -13_Session 5: Business Law-T122WSB-2

Chris maintains that a document that he and Kathy have signed is a binding
contract. : Kathy maintains that it is merely a receipt and does not include all
the terms of their : agreement. Which of the following statements is correct?

 
If both parties have signed the document then the court will look no further into
the : circumstances

 
Kathy should plead non est factum

 
Kathy will be able to establish the existence of an oral collateral contract even
where it is : inconsistent with the terms of the main binding contract

Correct!  
Parol evidence may be admissible

Quiz Score:
7 out of 15

https://lms.westernsydney.edu.vn/courses/287/quizzes/3367?module_item_id=11134 9/9
4/15/22, 8:14 PM Quiz 2_Chapters 5 -13_Session 5: Business Law-T122WSB-2

Quiz 2_Chapters 5 -13_Session 5


Due
Feb 14 at 3:15pm
Points
15
Questions
15
Available
Feb 14 at 2:59pm - Feb 14 at 3:25pm
26 minutes
Time Limit
15 Minutes

This quiz was locked Feb 14 at 3:25pm.

Attempt History
Attempt Time Score
LATEST Attempt 1 15 minutes 4 out of 15

Score for this quiz:


4 out of 15
Submitted Feb 14 at 3:16pm
This attempt took 15 minutes.

Question 1 0
/ 1 pts

Which of the following is NOT a requirement for an assignment of a debt or


other chose : in action under various State statutes?

 
The assignment is absolute and not by way of charge;

orrect Answer  
Fourteen days’ notice in writing is provided to debtor;

 
the assignment is in writing;

You Answered  
If not in writing then notice of the assignment is given by notice to the debtor.

Question 2 1
/ 1 pts

https://lms.westernsydney.edu.vn/courses/287/quizzes/3367?module_item_id=11134 1/9
4/15/22, 8:14 PM Quiz 2_Chapters 5 -13_Session 5: Business Law-T122WSB-2

Which of the following statements about the parol evidence rule is true.?

 
It applies only to oral (parol) evidence

 
It does not apply to written contracts

Correct!  
It does not apply where it can be shown that the written contract was not
intended to : be a complete record of the agreement

 
It applies only to previous drafts of a written contract

Question 3 1
/ 1 pts

In Jarvis v Swans Tours Ltd [1973] QB 233, where Jarvis was disappointed by,
among : other things, the little dry nut cakes on his holiday:

 
Damages were not awarded

 
Damages were awarded for anxiety and depression

 
Pleasure and enjoyment were not promises of the tour company with respect
to the : holiday in Switzerland

Correct!  
Damages were awarded partly for disappointment

Question 4 0
/ 1 pts

With respect to substantial performance and partial performance:

https://lms.westernsydney.edu.vn/courses/287/quizzes/3367?module_item_id=11134 2/9
4/15/22, 8:14 PM Quiz 2_Chapters 5 -13_Session 5: Business Law-T122WSB-2

You Answered  
Partial performance is a lower level of performance than substantial
performance, and : there is no deduction of the contract price available
where substantial performance has : occurred

 
Substantial performance is where the parties terminate the contract for less
than full : performance

orrect Answer  
Partial performance is where a party voluntarily accepts less than full
performance by : the other party

 
Partial performance is where the court terminates the contract for less than
exact : performance

Question 5 0
/ 1 pts

How is the Australian Consumer Law structured into the Competition and
Consumer Act : 2010 (Cth)?

 
They are separate statutes

You Answered  
The Australian Consumer Law is an amendment to the Trade Practices Act
1974 (Cth)

orrect Answer  
The Australian Consumer Law is Sch 2 to the Competition and Consumer Act
2010 (Cth)

https://lms.westernsydney.edu.vn/courses/287/quizzes/3367?module_item_id=11134 3/9
4/15/22, 8:14 PM Quiz 2_Chapters 5 -13_Session 5: Business Law-T122WSB-2

 
The Australian Consumer Law is App 3 to the Competition and Consumer Act
2010 (Cth)

Question 6 0
/ 1 pts

Century Dragon Pty Ltd is contracted to export 1000 beach balls to Minerva
Beach Surf : Club by 1 January, with payment due by 1 April. Which of the
following is correct?

orrect Answer  
The time of delivery is likely to be considered an essential term.

 
The time of payment is likely to be considered an essential term.

 
The time of delivery and the time of payment are likely to be considered
essential terms.

You Answered  
Neither the time of delivery nor the time of payment are likely to be
considered essential : terms unless time is expressly stated to be of the
essence.

Question 7 0
/ 1 pts

Which of the following is not true? Economic duress:

You Answered  
Is a threat to the innocent party’s economic interests

 
Permits the threatened party to avoid the contract

https://lms.westernsydney.edu.vn/courses/287/quizzes/3367?module_item_id=11134 4/9
4/15/22, 8:14 PM Quiz 2_Chapters 5 -13_Session 5: Business Law-T122WSB-2

 
Needs to be distinguished from a legitimate level of commercial pressure – an
example : of an actionable level of pressure is a threat to break a contract
without any legal : justification in order to extract money from an innocent
party

orrect Answer  
Was found in North Ocean Shipping Co Ltd v Hyundai Construction Co Ltd
[1979] 1 QB : 705 and the threatened party was able to recover the
payments made under duress

Question 8 1
/ 1 pts

A term that is vital to a contract is known as:

 
A key term

Correct!  
A condition

 
A warranty

 
An innominate term

Question 9 0
/ 1 pts

A party in breach of contract is liable to pay damages for:

orrect Answer  
Losses occurring in the usual course of things from the breach

You Answered  
All losses whether contemplated or not

 
Remote losses only

https://lms.westernsydney.edu.vn/courses/287/quizzes/3367?module_item_id=11134 5/9
4/15/22, 8:14 PM Quiz 2_Chapters 5 -13_Session 5: Business Law-T122WSB-2

 
Losses that should have been mitigated

Question 10 1
/ 1 pts

Which of the following is NOT a method of termination?

 
Agreement

 
Frustration

 
Breach

Correct!  
Rectification

Question 11 0
/ 1 pts

Parol evidence may not be admitted in which of the following circumstances?

 
To evidence an oral collateral contract

You Answered  
To evidence the identity of the parties to the contract

orrect Answer  
To contradict an unambiguous term

 
To explain a local custom

Question 12 0
/ 1 pts

https://lms.westernsydney.edu.vn/courses/287/quizzes/3367?module_item_id=11134 6/9
4/15/22, 8:14 PM Quiz 2_Chapters 5 -13_Session 5: Business Law-T122WSB-2

The test of whether a statement is a term or a representation is not dependent


on which : of the following?

You Answered  
The duration of time between the statement being made and the contract
being made

 
Whether the statement has been reduced to writing

orrect Answer  
Whether the party making the statement believed in the truth of the
statement or not

 
Reliance by one party on the skill of the other

Question 13 0
/ 1 pts

In Trident General Insurance Co Ltd v McNiece Bros Pty Ltd (1988) 165 CLR
107:

You Answered  
Blue Circle was a sub-contractor whose worker was injured, and privity
precluded the : worker from getting an indemnity under the contract

orrect Answer  
It was held that the subcontractor could get an indemnity even though the :
subcontractor had given no consideration under the contract

 
In Deane J’s view Trident, the insurance company, had an obligation to the
subcontractor because otherwise Trident would be unjustly enriched

https://lms.westernsydney.edu.vn/courses/287/quizzes/3367?module_item_id=11134 7/9
4/15/22, 8:14 PM Quiz 2_Chapters 5 -13_Session 5: Business Law-T122WSB-2

 
The majority were of the view that the “settled and fundamental” doctrine of
privity of : contract should not be overturned by the court

Question 14 0
/ 1 pts

Which of the following is not correct? An action in restitution:

 
Is often brought where one party has been unjustly enriched at the expense
of the other

orrect Answer  
Cannot be brought where there is no contract between the parties

 
Can be brought where the contract between the parties is unenforceable

You Answered  
Can be brought where there has been only part performance

Question 15 0
/ 1 pts

In Codelfa Construction Pty Ltd v State Rail Authority of New South Wales
(1982) 149 CLR : 337:

 
The High Court implied a term in the agreement granting a reasonable
extension of time: b Codelfa was able to prove that the term was necessary to
make the contract work

https://lms.westernsydney.edu.vn/courses/287/quizzes/3367?module_item_id=11134 8/9
4/15/22, 8:14 PM Quiz 2_Chapters 5 -13_Session 5: Business Law-T122WSB-2

You Answered  
Codelfa did not need to prove that the term was necessary to make the
contract work : for the term to be implied as it was sufficient that both parties
needed to be rescued from a : difficult position imposed upon them by the
injunction

orrect Answer  
The High Court found that the contract was frustrated

Quiz Score:
4 out of 15

https://lms.westernsydney.edu.vn/courses/287/quizzes/3367?module_item_id=11134 9/9
21:42, 11/04/2022 Quiz 2_Chapters 5 -13_Session 5: Business Law-T122WSB-2

Quiz 2_Chapters 5 -13_Session 5


Due
Feb 14 at 3:15pm
Points
15
Questions
15
Available
Feb 14 at 2:59pm - Feb 14 at 3:25pm
26 minutes
Time Limit
15 Minutes

This quiz was locked Feb 14 at 3:25pm.

Attempt History
Attempt Time Score
LATEST Attempt 1
15 minutes 3 out of 15

Score for this quiz:


3 out of 15
Submitted Feb 14 at 3:24pm
This attempt took 15 minutes.

Question 1 0
/ 1 pts

When a contract is made orally, the question of what the express terms are:

 
Is in most cases unascertainable

 
Depends on what a reasonable person would have said

orrect Answer  
Is a question of fact

You Answered  
Is not applicable; oral contracts only contain implied terms

Question 2 0
/ 1 pts

The definition of “consumer” is limited by what amount?

 
$40,000

orrect Answer  
Recently increased to $100,000

You Answered  
There is no limit

https://lms.westernsydney.edu.vn/courses/287/quizzes/3367 1/7
21:42, 11/04/2022 Quiz 2_Chapters 5 -13_Session 5: Business Law-T122WSB-2

 
A negotiated amount

Question 3 1
/ 1 pts

Which of the following would not constitute the unfair practice of harassment
or : coercion under the Australian Consumer Law? Where there is a debt in
connection with the : possible of goods and the creditor:

 
Consciously calculates to intimidate the debtor with the content of her
demands

Correct!  
Tells the debtor, among repeated demands for payment give the debtor a
discount for : early payment

 
Makes demands constantly in order to exhaust the debtor

 
Chooses particularly threatening demands in order to demoralise the debtor

Question 4 0
/ 1 pts

Smithy Builders have a contract with Big Bank Pty Ltd which contains the
following : clause: “Where Smithy Builders fails to complete the contract by 7
December, it will pay a : sum of $300,000 in full and final satisfaction of its
liability.” Smithy Builders fails to complete : by 7 December. If Smithy Builders
wishes to avoid paying the sum of $300,000, what must it : prove?

 
That the clause is an unliquidated damages clause

orrect Answer  
That the clause is a penalty clause (to punish the builders)

 
That the clause is a liquidated damages clause

You Answered  
That the clause is a genuine pre-estimate of the loss to Big Bank Pty Ltd

https://lms.westernsydney.edu.vn/courses/287/quizzes/3367 2/7
21:42, 11/04/2022 Quiz 2_Chapters 5 -13_Session 5: Business Law-T122WSB-2

Question 5 0
/ 1 pts

Which unfair practice involves participation in a trading scheme where


persons at the : top receive most of the benefits?

You Answered  
referral selling

orrect Answer  
pyramid selling

 
unconscionable conduct

 
misleading and deceptive conduct

Question 6 0
/ 1 pts

Sharma was selling his secondhand vacuum cleaner to Natasha and told
Natasha that it : could “do all the rooms in the house ten times over without
the filter needing to be : changed”. Natasha spent a few weeks shopping for
other vacuum cleaners but eventually : returned and purchased Sharma’s,
telling Sharma it was the cheapest she had seen in two : weeks and she
liked the colour. When Natasha took it home she soon discovered that the :
vacuum cleaner could only do one room at a time and then the filter would
need changing. : Natasha cannot rely on Sharma’s statement for breach of
contract because:

 
The statement was not put into writing

orrect Answer  
Natasha did not attach any importance to the statement when it was made

You Answered  
Natasha should have tested the vacuum cleaner before buying it

 
The statement was a merger clause

Question 7 0
/ 1 pts

https://lms.westernsydney.edu.vn/courses/287/quizzes/3367 3/7
21:42, 11/04/2022 Quiz 2_Chapters 5 -13_Session 5: Business Law-T122WSB-2

Which of the following is not a circumstance where the court would award
specific : performance?

orrect Answer  
Where the contract would require constant supervision by the court

 
Mitigation

 
Contributory negligence

You Answered  
Remoteness

Question 8 0
/ 1 pts

In which case did the court recognise that damages for disappointment and
distress : could be available subject to certain limitations?

You Answered  
Burns v MAN Automotive (Aust) Pty Ltd (1986) 161 CLR 653

 
Placer (Granny Smith) Pty Ltd v Thiess Contractors Pty Ltd (2003) 196 ALR
257

 
Howe v Teefy (1927) 27 SR (NSW) 301

orrect Answer  
Baltic Shipping Co v Dillon (1993) 176 CLR 344

Question 9 0
/ 1 pts

Chris maintains that a document that he and Kathy have signed is a binding
contract. : Kathy maintains that it is merely a receipt and does not include all
the terms of their : agreement. Which of the following statements is correct?

You Answered  
If both parties have signed the document then the court will look no further
into the : circumstances

https://lms.westernsydney.edu.vn/courses/287/quizzes/3367 4/7
21:42, 11/04/2022 Quiz 2_Chapters 5 -13_Session 5: Business Law-T122WSB-2

 
Kathy should plead non est factum

 
Kathy will be able to establish the existence of an oral collateral contract even
where it is : inconsistent with the terms of the main binding contract

orrect Answer  
Parol evidence may be admissible

Question 10 0
/ 1 pts

Which of the following is not an element required for acceptable quality of


goods? That : the goods are acceptably:

You Answered  
Free from defects

 
Durable

orrect Answer  
Fit for the consumer’s purpose

 
Safe

Question 11 1
/ 1 pts

Which of the following statements about the parol evidence rule is true?

 
It applies only to oral (parol) evidence

 
It does not apply to written contracts

Correct!  
It does not apply where it can be shown that the written contract was not
intended to : be a complete record of the agreement

 
It applies only to previous drafts of a written contract

https://lms.westernsydney.edu.vn/courses/287/quizzes/3367 5/7
21:42, 11/04/2022 Quiz 2_Chapters 5 -13_Session 5: Business Law-T122WSB-2

Question 12 0
/ 1 pts

Which of the following are sources of illegality?

 
Statute

orrect Answer  
Statute and common law

You Answered  
Statute, common law and equity

 
Statute, common law and parties’ subjective views on morality

Question 13 0
/ 1 pts

Which of the following is not a type of contract illegal at common law on the
grounds of : public policy?

orrect Answer  
Contracts to oust the jurisdiction of the courts

You Answered  
Contracts to commit a tort

 
Contracts prejudicial to the public safety

 
Champertous contracts

Question 14 1
/ 1 pts

Non est factum means:

Correct!  
It is not [my] deed

 
It is not true

 
Mistake of fact

https://lms.westernsydney.edu.vn/courses/287/quizzes/3367 6/7
21:42, 11/04/2022 Quiz 2_Chapters 5 -13_Session 5: Business Law-T122WSB-2

 
It is unknown

Question 15 0
/ 1 pts

Unliquidated damages are:

 
Non-monetary sums

You Answered  
Prohibited by law as they are uncertain and difficult to quantify

orrect Answer  
Damages where the court is to determine the amount

 
Damages where an amount has been fixed in the contract

Quiz Score:
3 out of 15

https://lms.westernsydney.edu.vn/courses/287/quizzes/3367 7/7
08:34, 19/04/2022 Quiz 2_Chapters 5 -13_Session 5: Business Law-T122WSB-3

Quiz 2_Chapters 5 -13_Session 5


Due
Feb 15 at 11:15am
Points
15
Questions
15
Available
Feb 15 at 10:59am - Feb 15 at 11:25am
26 minutes
Time Limit
15 Minutes

This quiz was locked Feb 15 at 11:25am.

Attempt History
Attempt Time Score
LATEST Attempt 1
15 minutes 9 out of 15

Score for this quiz:


9 out of 15
Submitted Feb 15 at 11:16am
This attempt took 15 minutes.

Question 1 1
/ 1 pts

Non est factum means:

Correct!  
It is not [my] deed

 
It is not true

 
Mistake of fact

 
It is unknown

Question 2 1
/ 1 pts

Which of the following statements is not correct? With respect to legality of


object:

https://lms.westernsydney.edu.vn/courses/286/quizzes/3368 1/8
08:34, 19/04/2022 Quiz 2_Chapters 5 -13_Session 5: Business Law-T122WSB-3

 
Illegality is not confined to criminal illegality

 
A contract may be illegal where it is made to effect a purpose that a statute
renders : unlawful

 
A contract may be illegal where its object is impliedly prohibited by statute

Correct!  
Illegal objectives will not affect a contract provided both parties have agreed to
it

Question 3 1
/ 1 pts

What does the concept “quantum meruit” mean?

 
The contract is terminated because of a breach by both parties

Correct!  
The innocent party would be unjustly enriched if they were able to retain the
benefit : without compensating the party in breach for the “amount he
deserves”

 
Voluntarily accepting a contractual benefit

 
Not my deed

Unanswered Question 4 0
/ 1 pts

Which of the following constitutes bait advertising?

https://lms.westernsydney.edu.vn/courses/286/quizzes/3368 2/8
08:34, 19/04/2022 Quiz 2_Chapters 5 -13_Session 5: Business Law-T122WSB-3

 
Jaz is approached by her neighbour to join the sales team of Thames Beauty
Products. : Thames sells its products door-to-door through agents and is a
close-knit family type of : company. Agents that introduce other agents to “the
family” are rewarded with a finders’: fee. It is only $500 to join the Thames
family, and Jaz only has to spend $400 on products in : her first month

 
Jaz starts work at a chemist and notices that the chemist, as retailers often do,
puts a : sale price on a sticker on most of the products in the shop. However,
the chemist does not : put the sale price sticker on top of the original price
sticker and when customers bring the : products to the counter, the chemist
tells them that unfortunately there has been a mistake, : the true price is the
higher price

orrect Answer  
Jaz approaches a salesperson on the forecourt to buy the car package she
saw : advertised on television and is told that there were only two cars for sale
as a part of that : deal and there are now none left. The salesperson tells Jaz
that she would look much better : in the convertible model and although there
is no deal on that car, it is only $50,000 more

 
Jaz purchases a painting through a new art gallery that has just opened in
town. The : price is high and Jaz is at first uncertain about the expense, but
the art dealer promises Jaz a : commission on sales to any of Jaz’s friends
who she sends along to the gallery. Jaz has a lot : of art collector friends so is
sure that she will receive some commissions from the gallery in : the near
future

Question 5 1
/ 1 pts

Which of the following is not correct? Parol evidence:

 
Will always be admissible to evidence duress

 
Is admissible to evidence a collateral oral agreement

https://lms.westernsydney.edu.vn/courses/286/quizzes/3368 3/8
08:34, 19/04/2022 Quiz 2_Chapters 5 -13_Session 5: Business Law-T122WSB-3

 
Will always be admissible to evidence fraud

Correct!  
Is not admissible to evidence whether a document constitutes a contract or
merely a : note

Question 6 1
/ 1 pts

Which of the following is incorrect? Repudiation:

 
Is where a party demonstrates an unwillingness to perform their obligations

Correct!  
Is the same as termination; the contract is at an end

 
Gives an innocent party the right to terminate the contract

 
Is where a party demonstrates an inability to perform their obligations

Question 7 1
/ 1 pts

Where a contract is made in writing, the express terms of that contract:

 
Are to be found in the writing

Correct!  
Are generally to be found in the writing

 
Are to be found in the writing unless it is an online agreement, in which case
terms and : conditions may be incorporated from the website of one or both
parties

 
Cannot include anything said orally

https://lms.westernsydney.edu.vn/courses/286/quizzes/3368 4/8
08:34, 19/04/2022 Quiz 2_Chapters 5 -13_Session 5: Business Law-T122WSB-3

Question 8 0
/ 1 pts

Which of the following is an element required for the doctrine of frustration to


operate?

 
Material loss to one party

orrect Answer  
No fault on the part of either party

 
Material loss to both parties

You Answered  
Hardship or inconvenience

Question 9 0
/ 1 pts

Sandy is selling her horse, Flossy. Miranda is interested in buying Flossy.


Sandy tells : Miranda that Flossy: : • Is the best little racehorse in Australia: •
Has all the registrations, licences and permits required to race in the current
season but : that Miranda should probably get this checked herself: These
two statements are:

 
Mere puff & term

 
Opinion & term

You Answered  
Opinion & representation

orrect Answer  
Mere puff & representation

Question 10 0
/ 1 pts
https://lms.westernsydney.edu.vn/courses/286/quizzes/3368 5/8
08:34, 19/04/2022 Quiz 2_Chapters 5 -13_Session 5: Business Law-T122WSB-3

The plaintiff in an action under s18 of the Australian Consumer Law is


required to prove : what?

 
the conduct is misleading or deceptive

orrect Answer  
the conduct is misleading or deceptive or likely to mislead or deceive

You Answered  
b, above, plus the intention of the defendant

 
The ACCC takes on the issues of proving the defendant’s intentions

Question 11 1
/ 1 pts

A party repudiates a contract when

 
They decide to terminate it

Correct!  
They are no longer able or are unwilling to perform their obligations

 
The other party decides to terminate it

 
They take too long to perform an obligation that has a time limit under the
contract, such : as payment of rent

Question 12 0
/ 1 pts

Sharma was selling his secondhand vacuum cleaner to Natasha and told
Natasha that it : could “do all the rooms in the house ten times over without
the filter needing to be : changed”. Natasha spent a few weeks shopping for
other vacuum cleaners but eventually : returned and purchased Sharma’s,

https://lms.westernsydney.edu.vn/courses/286/quizzes/3368 6/8
08:34, 19/04/2022 Quiz 2_Chapters 5 -13_Session 5: Business Law-T122WSB-3

telling Sharma it was the cheapest she had seen in two : weeks and she
liked the colour. When Natasha took it home she soon discovered that the :
vacuum cleaner could only do one room at a time and then the filter would
need changing. : Natasha cannot rely on Sharma’s statement for breach of
contract because:

 
The statement was not put into writing

orrect Answer  
Natasha did not attach any importance to the statement when it was made

You Answered  
Natasha should have tested the vacuum cleaner before buying it

 
The statement was a merger clause

Question 13 1
/ 1 pts

A party repudiates contract when

Correct!  
The party is able but unwilling to perform their obligations under it

 
The party decides to terminate the contract

 
The party breaches a representation

 
The party breaches a warranty

Question 14 0
/ 1 pts

Which of the following guarantees applies to a sale by auction?

You Answered  
Guarantee as to undisturbed possession

https://lms.westernsydney.edu.vn/courses/286/quizzes/3368 7/8
08:34, 19/04/2022 Quiz 2_Chapters 5 -13_Session 5: Business Law-T122WSB-3

 
Guarantee as to express warranties

 
Guarantee as to acceptable quality

orrect Answer  
Guarantee as to title

Question 15 1
/ 1 pts

Which of the following is not accurate? Duress involves:

 
Actual or threatened violence

Correct!  
Duress must be the only reason for entering into a contract

 
A party coerced into entering into a contract (against their will)

 
Coercion of a person or their immediate family or near relatives

Quiz Score:
9 out of 15

https://lms.westernsydney.edu.vn/courses/286/quizzes/3368 8/8
Quiz 2_Chapters 5 -13_Session 5
Due
Feb 15 at 11:15am
Points
15
Questions
15
Available
Feb 15 at 10:59am - Feb 15 at 11:25am
26 minutes
Time Limit
15 Minutes

This quiz was locked Feb 15 at 11:25am.

Attempt History
Attempt Time Score
LATEST Attempt 1
15 minutes 7 out of 15

Score for this quiz:


7 out of 15
Submitted Feb 15 at 11:16am
This attempt took 15 minutes.

Question 1 1
/ 1 pts

When it was said in Re Hall & Barker [1878] 9 Ch D 538, “if a shoemaker
agrees to make : a pair of shoes, he cannot offer you one shoe and ask you
to pay one half the price”, this is : referring to:

 
The court’s tendency to regard contracts as divisible

 
The court’s tendency to regard contracts as severable

Correct!  
The court’s tendency to regard contracts as entire

 
The court’s tendency to require substantial performance

Question 2 0
/ 1 pts

James is going through a difficult time and does not feel he can confide in
anyone. : Eventually he confides in the priest of his church and asks for
support. The priest forcefully : tells James to turn over all of his money to the
church or he will be struck down by : lightening. The priest does not believe
that James will be struck down. James makes the : payments. Which of the
following is most likely legal grounds for James to avoid the : transaction?

 
Mistake

orrect Answer  
Undue influence

 
Duress

You Answered  
Misrepresentation

Question 3 1
/ 1 pts

Sharma was selling his secondhand vacuum cleaner to Natasha and told
Natasha that it : could “do all the rooms in the house ten times over without
the filter needing to be : changed”. Natasha spent a few weeks shopping for
other vacuum cleaners but eventually : returned and purchased Sharma’s,
telling Sharma it was the cheapest she had seen in two : weeks and she
liked the colour. When Natasha took it home she soon discovered that the :
vacuum cleaner could only do one room at a time and then the filter would
need changing. : Natasha cannot rely on Sharma’s statement for breach of
contract because:

 
The statement was not put into writing

Correct!  
Natasha did not attach any importance to the statement when it was made

 
Natasha should have tested the vacuum cleaner before buying it

 
The statement was a merger clause

Question 4 0
/ 1 pts
Where a contract has not been completed within the specified time:

 
If no time limit was specified in the contract, there is nothing the innocent party
can do

You Answered  
The innocent party will be entitled to terminate the contract

orrect Answer  
The innocent party will be entitled to terminate the contract upon giving notice
that time : is of the essence

 
Only damages will be available no matter how crucial the time limit was to the
innocent : party

Question 5 1
/ 1 pts

Which of the following is not a circumstance where the court would award
specific : performance?

Correct!  
Where the contract would require constant supervision by the court

 
Mitigation

 
Contributory negligence

 
Remoteness

Question 6 0
/ 1 pts
Fraser crashes his car with his friend Angus as a passenger. Angus is
injured. Angus was : not wearing a seatbelt. Which of the following is not
correct?

 
Liability would likely be apportioned between Fraser and Angus

orrect Answer  
Damages cannot be apportioned for breach of contractual duty of care where
there is : contributory negligence

You Answered  
Damages awarded to Angus, if any, would be reduced by his contributory
negligence

 
Angus’ contributory negligence relates only to his own safety, not to that of
others

Question 7 0
/ 1 pts

Which of the following is not true with respect to misleading or deceptive


conduct?

 
The plaintiff has to prove that conduct was misleading or deceptive

 
The plaintiff has to prove the causal link between the conduct and the loss

orrect Answer  
A person acting as a ”mere conduit” in passing on representations may also
be liable
You Answered  
A corporation is not liable where it is clear that the corporation is not the
source of the : information

Question 8 1
/ 1 pts

Which of the following is NOT a method of termination?

 
Agreement

 
Frustration

 
Breach

Correct!  
Rectification

Question 9 1
/ 1 pts

In Codelfa Construction Pty Ltd v State Rail Authority of New South Wales
(1982) 149 CLR : 337:

 
The High Court implied a term in the agreement granting a reasonable
extension of time: b Codelfa was able to prove that the term was necessary to
make the contract work

 
Codelfa did not need to prove that the term was necessary to make the
contract work : for the term to be implied as it was sufficient that both parties
needed to be rescued from a : difficult position imposed upon them by the
injunction
Correct!  
The High Court found that the contract was frustrated

Question 10 1
/ 1 pts

Which of the following guarantees applies to a sale by auction?

 
Guarantee as to undisturbed possession

 
Guarantee as to express warranties

 
Guarantee as to acceptable quality

Correct!  
Guarantee as to title

Unanswered
Question 11 0
/ 1 pts

Matt sells his car to Tabitha, who purchases it on the condition that she can
continue to: keep it in his garage at no cost for the next six months. Tabitha
pays in full. Two months: later, Matt tells Tabitha she will have to remove her
car from his garage as he has bought a: new car and wants to keep it in the
garage. Which of the following statements is correct?

 
If Tabitha agrees to remove her car, the contract has been terminated by
partial: performance

 
The condition that Tabitha is allowed to keep her car in Matt’s garage is a
condition: precedent
 
The condition that Tabitha is allowed to keep her car in Matt’s garage is a
condition: subsequent

orrect Answer  
If Tabitha promises to remove her car, Matt will not be able to enforce the
promise: unless the promise is supported by consideration or made under seal

Unanswered Question 12 0
/ 1 pts

In which one of the following cases was it held that for a loss in the
contemplation of the : parties to be recoverable it must be “a serious
possibility”, “a real danger”, “liable to result” : or “not unlikely” to occur?

 
Hadley v Baxendale (1854) 9 Exch 341; 156 ER 145

 
Burns v MAN Automotive (Aust) Pty Ltd (1986) 161 CLR 653

orrect Answer  
Koufos v Czarnikow Ltd [1969] 1 AC 350

 
Tabcorp Holdings Ltd v Bowen Investments Pty Ltd (2009) 236 CLR 272

Question 13 1
/ 1 pts

Which of the following is not an equitable remedy?

 
Injunction

 
Specific performance

 
Restitution

Correct!  
Exemplary damages

Unanswered Question 14 0
/ 1 pts

How is repudiation determined by the court?

 
Subjectively

 
By considering the actions of the repudiating party

 
By viewing the circumstances of the repudiation

orrect Answer  
Objectively

Unanswered Question 15 0
/ 1 pts

Which of the following is not an unfair practice under the Australian


Consumer Law?

 
Where a free gift is offered with the purchase an electric toothbrush and the
cost of the : “gift” toothpaste is added to the price of the toothbrush

orrect Answer  
Where a television advertisement for a chocolate bar shows teddy bears
working the : manufacturing lines

 
Where a brochure advertising a beauty therapy treatment consisting of time in
an : oxygen tank lists the treatment as lasting two hours, when in fact the
treatment lasts just : under one hour
 
Where a shop assistant takes money from a customer knowing that the shirt
the : customer wants is sold out, but hoping that after a few days when the
customer is told that : there are no more shirts they will purchase something
else from the shop

Quiz Score:
7 out of 15
Quiz 3_Chap 14 & 15_Session 7
Due Mar 1 at 12:15pm Points 15 Questions 15
Available Mar 1 at 11:59am - Mar 1 at 12:25pm 26 minutes Time Limit 15 Minutes

This quiz was locked Mar 1 at 12:25pm.

A"empt History
Attempt Time Score
LATEST Attempt 1 15 minutes 10 out of 15

Score for this quiz: 10 out of 15


Submitted Mar 1 at 12:14pm
This attempt took 15 minutes.

Question 1 1 / 1 pts

Where a person gives advice, that advice is relied upon and the advice is
incorrect, the : person giving the advice may be liable in negligence. Which of
the following is incorrect?

The “special relationship” described in Mutual Life & Citizens’ Assurance Co


Ltd v Evatt : (1968) 122 CLR 556 has been accepted as the test for
determining the existence of a duty of : care with respect to negligent
misstatements

The test in Mutual Life & Citizens’ Assurance Co Ltd v Evatt (1968) 122 CLR
556 was : approved in San Sebastian Pty Ltd v Minister Administering
Environmental Planning &: Assessment Act 1979 (1986) 162 CLR 340

Correct!
The duty of care only arises where the “special relationship” involves a person
seeking : advice from the advice-giver, not where the advice was given
unrequested and merely : accepted

The advice-giver does not need to be in the business of giving advice

Question 2 1 / 1 pts

Which of the following is not correct? The reform to the law of negligence that
took : place in Australia in the early 2000s:

Limits the scope of potential liability for negligence

Correct! Does not apply to claims in contract law

Was enacted in all Australian states and territories

Covers personal injury

Question 3 1 / 1 pts

In Tame v State of New South Wales (2002) 211 CLR 317 where Tame was
given a false : blood alcohol reading:

Whether the police officer was found to have owed Tame a duty of care was
entirely a : question of community standards
The psychotic depressive illness Tame developed was found to flow from the
breach of : duty of the police sergeant because it was entirely caused by his
actions that could have : been avoided had he exercised sufficient care

Correct! Part of the test of reasonable foreseeability is a question of fact

Because of the “eggshell skull” principle, pre-existing knowledge of Tame’s


susceptibility : to suffering nervous shock was not required to prove a breach

Question 4 1 / 1 pts

Which of the following is not correct?

If damages are too remote they will not be recoverable

The remoteness test will be satisfied where the damage suffered is of the
same type or : kind as foreseeable damage

In order for damage to not be too remote, it must be reasonably foreseeable

Correct!
In The Wagon Mound No 1 and The Wagon Mound No 2, where there was an
: unfortunate combination of an oil spill, welding sparks and floating cotton
waste, the loss : was found to be not reasonably foreseeable in the
circumstances

Question 5 1 / 1 pts

In Australian Safeway Stores v Zaluzna (1987) 162 CLR 479:


Correct!
The respondent was a lawful entrant upon the land of the respondent,
establishing a : relationship between them and there was therefore a duty of
care owed by the appellant to : avoid a foreseeable risk of injury

The respondent was limited by his particular status as entrant without specific
consent of : the appellant

The respondent contributed to the accident thus limiting damages

There was no liability.

Question 6 1 / 1 pts

To claim damages a causal link must be established between what two


things?

Between the defendant’s conduct and the actual monetary value of the
loss/damage

The conduct and initiating court proceedings

The fact that the plaintiff suffered loss or damage is sufficient. Nothing else is
required

Correct! The defendants breach and the plaintiff’s injury

Question 7 1 / 1 pts
Why was the plaintiff successful in Overseas Tankship (UK) Ltd v Miller
Steamship Co Pty : Ltd (The Wagon Mound No 2) [1967] AC 617 (PC) when
another plaintiff failed in the earlier : related Wagon Mound case?

The plaintiffs in the second had better legal representation

The legal situation had changed between the two cases

The court was more disposed to think about the consequences of the damage
suffered by : so many as a result of the oil spill

Correct!
In the first case, the plaintiffs failed to show that a reasonable man would have
foreseen : the risk of damage from the oil spill

Question 8 1 / 1 pts

Nicola drives the forklift at her place of work, Rooze’s Roofing. Nicola always
leaves the : forklift in a certain place where she has been told to leave it, with
the forks up off the : ground. One afternoon a customer who is collecting
goods from the workshop reverses his : car into the forks on the forklift. He is
injured and his car is damaged.: What is the principle that would make
Nicola’s employer liable for her actions?

Contributory negligence

Strict liability

Correct! Vicarious liability

Voluntary assumption of risk


Question 9 1 / 1 pts

The “neighbour principle” is often viewed as:

An objective test

A test of the reasonable person

Correct! A test of the foreseeability of harm

An identity test

Question 10 1 / 1 pts

Which of the following is not a provision of the Wrongs Act 1958 (Vic) relating
to: professional liability?

If there are differing peer professional opinions across Australia, the court may
accept : just one of those opinions

Correct!
The court must rely on peer professional opinion even where it considers that
opinion : irrational

If there are differing peer professional opinions across Australia, the court may
accept all : of those opinions

Peer professional opinion can be considered widely accepted even where it is


not : universally accepted
Quiz Score: 10 out of 15
4/10/22, 10:19 PM Quiz 3_Chap 14 & 15_Session 7: Business Law-T122WSB-2

Quiz 3_Chap 14 & 15_Session 7


Due
Feb 28 at 12:15pm
Points
15
Questions
15
Available
Feb 28 at 11:59am - Feb 28 at 12:25pm
26 minutes
Time Limit
15 Minutes

This quiz was locked Feb 28 at 12:25pm.

Attempt History
Attempt Time Score
LATEST Attempt 1
11 minutes 6 out of 15

Score for this quiz:


6 out of 15
Submitted Feb 28 at 12:10pm
This attempt took 11 minutes.

Question 1 0
/ 1 pts

Which of the following is not a provision of the Wrongs Act 1958 (Vic) relating
to: professional liability?

 
If there are differing peer professional opinions across Australia, the court may
accept : just one of those opinions

orrect Answer  
The court must rely on peer professional opinion even where it considers that
opinion : irrational

 
If there are differing peer professional opinions across Australia, the court may
accept all : of those opinions

https://lms.westernsydney.edu.vn/courses/287/quizzes/3573?module_item_id=11540 1/6
4/10/22, 10:19 PM Quiz 3_Chap 14 & 15_Session 7: Business Law-T122WSB-2

You Answered  
Peer professional opinion can be considered widely accepted even where it is
not : universally accepted

Question 2 0
/ 1 pts

Why was the plaintiff successful in Overseas Tankship (UK) Ltd v Miller
Steamship Co Pty : Ltd (The Wagon Mound No 2) [1967] AC 617 (PC) when
another plaintiff failed in the earlier : related Wagon Mound case?

 
The plaintiffs in the second had better legal representation

 
The legal situation had changed between the two cases

You Answered  
The court was more disposed to think about the consequences of the damage
suffered by : so many as a result of the oil spill

orrect Answer  
In the first case, the plaintiffs failed to show that a reasonable man would have
foreseen : the risk of damage from the oil spill

Question 3 0
/ 1 pts

In Tame v State of New South Wales (2002) 211 CLR 317 where Tame was
given a false : blood alcohol reading:

You Answered  
Whether the police officer was found to have owed Tame a duty of care was
entirely a : question of community standards

https://lms.westernsydney.edu.vn/courses/287/quizzes/3573?module_item_id=11540 2/6
4/10/22, 10:19 PM Quiz 3_Chap 14 & 15_Session 7: Business Law-T122WSB-2

 
The psychotic depressive illness Tame developed was found to flow from the
breach of : duty of the police sergeant because it was entirely caused by his
actions that could have : been avoided had he exercised sufficient care

orrect Answer  
Part of the test of reasonable foreseeability is a question of fact

 
Because of the “eggshell skull” principle, pre-existing knowledge of Tame’s
susceptibility : to suffering nervous shock was not required to prove a breach

Question 4 1
/ 1 pts

Which of the following is not correct? The reform to the law of negligence that
took : place in Australia in the early 2000s:

 
Limits the scope of potential liability for negligence

Correct!  
Does not apply to claims in contract law

 
Was enacted in all Australian states and territories

 
Covers personal injury

Question 5 1
/ 1 pts

To claim damages a causal link must be established between what two


things?

 
Between the defendant’s conduct and the actual monetary value of the
loss/damage

https://lms.westernsydney.edu.vn/courses/287/quizzes/3573?module_item_id=11540 3/6
4/10/22, 10:19 PM Quiz 3_Chap 14 & 15_Session 7: Business Law-T122WSB-2

 
The conduct and initiating court proceedings

 
The fact that the plaintiff suffered loss or damage is sufficient. Nothing else is
required

Correct!  
The defendants breach and the plaintiff’s injury

Question 6 0
/ 1 pts

In Australian Safeway Stores v Zaluzna (1987) 162 CLR 479:

orrect Answer  
The respondent was a lawful entrant upon the land of the respondent,
establishing a : relationship between them and there was therefore a duty of
care owed by the appellant to : avoid a foreseeable risk of injury

You Answered  
The respondent was limited by his particular status as entrant without specific
consent of : the appellant

 
The respondent contributed to the accident thus limiting damages

 
There was no liability.

Question 7 1
/ 1 pts

Nicola drives the forklift at her place of work, Rooze’s Roofing. Nicola always
leaves the : forklift in a certain place where she has been told to leave it, with
the forks up off the : ground. One afternoon a customer who is collecting
goods from the workshop reverses his : car into the forks on the forklift. He is

https://lms.westernsydney.edu.vn/courses/287/quizzes/3573?module_item_id=11540 4/6
4/10/22, 10:19 PM Quiz 3_Chap 14 & 15_Session 7: Business Law-T122WSB-2

injured and his car is damaged.: What is the principle that would make
Nicola’s employer liable for her actions?

 
Contributory negligence

 
Strict liability

Correct!  
Vicarious liability

 
Voluntary assumption of risk

Question 8 1
/ 1 pts

The “neighbour principle” is often viewed as:

 
An objective test

 
A test of the reasonable person

Correct!  
A test of the foreseeability of harm

 
An identity test

Question 9 1
/ 1 pts

Where a person gives advice, that advice is relied upon and the advice is
incorrect, the : person giving the advice may be liable in negligence. Which of
the following is incorrect?

 
The “special relationship” described in Mutual Life & Citizens’ Assurance Co
Ltd v Evatt : (1968) 122 CLR 556 has been accepted as the test for
determining the existence of a duty of : care with respect to negligent
misstatements

https://lms.westernsydney.edu.vn/courses/287/quizzes/3573?module_item_id=11540 5/6
4/10/22, 10:19 PM Quiz 3_Chap 14 & 15_Session 7: Business Law-T122WSB-2

 
The test in Mutual Life & Citizens’ Assurance Co Ltd v Evatt (1968) 122 CLR
556 was : approved in San Sebastian Pty Ltd v Minister Administering
Environmental Planning &: Assessment Act 1979 (1986) 162 CLR 340

Correct!  
The duty of care only arises where the “special relationship” involves a person
seeking : advice from the advice-giver, not where the advice was given
unrequested and merely : accepted

 
The advice-giver does not need to be in the business of giving advice

Question 10 1
/ 1 pts

Which of the following is not correct?

 
If damages are too remote they will not be recoverable

 
The remoteness test will be satisfied where the damage suffered is of the
same type or : kind as foreseeable damage

 
In order for damage to not be too remote, it must be reasonably foreseeable

Correct!  
In The Wagon Mound No 1 and The Wagon Mound No 2, where there was an
: unfortunate combination of an oil spill, welding sparks and floating cotton
waste, the loss : was found to be not reasonably foreseeable in the
circumstances

Quiz Score:
6 out of 15

https://lms.westernsydney.edu.vn/courses/287/quizzes/3573?module_item_id=11540 6/6
Quiz 3_Chap 14 & 15_Session 7
Due
Mar 1 at 8:15am
Points
15
Questions
15
Available
Mar 1 at 7:59am - Mar 1 at 8:25am
26 minutes
Time Limit
15 Minutes

This quiz was locked Mar 1 at 8:25am.

Attempt History
Attempt Time Score
LATEST Attempt 1
15 minutes 7 out of 15

Score for this quiz:


7 out of 15
Submitted Mar 1 at 8:15am
This attempt took 15 minutes.

Question 1 0
/ 1 pts

Which of the following is not correct? The reform to the law of negligence that
took : place in Australia in the early 2000s:

You Answered  
Limits the scope of potential liability for negligence

orrect Answer  
Does not apply to claims in contract law

 
Was enacted in all Australian states and territories

 
Covers personal injury

Question 2 0
/ 1 pts

Where a person gives advice, that advice is relied upon and the advice is
incorrect, the : person giving the advice may be liable in negligence. Which of
the following is incorrect?

You Answered  
The “special relationship” described in Mutual Life & Citizens’ Assurance Co
Ltd v Evatt : (1968) 122 CLR 556 has been accepted as the test for
determining the existence of a duty of : care with respect to negligent
misstatements

 
The test in Mutual Life & Citizens’ Assurance Co Ltd v Evatt (1968) 122 CLR
556 was : approved in San Sebastian Pty Ltd v Minister Administering
Environmental Planning &: Assessment Act 1979 (1986) 162 CLR 340

orrect Answer  
The duty of care only arises where the “special relationship” involves a person
seeking : advice from the advice-giver, not where the advice was given
unrequested and merely : accepted

 
The advice-giver does not need to be in the business of giving advice

Question 3 1
/ 1 pts

Why was the plaintiff successful in Overseas Tankship (UK) Ltd v Miller
Steamship Co Pty : Ltd (The Wagon Mound No 2) [1967] AC 617 (PC) when
another plaintiff failed in the earlier : related Wagon Mound case?

 
The plaintiffs in the second had better legal representation

 
The legal situation had changed between the two cases

 
The court was more disposed to think about the consequences of the damage
suffered by : so many as a result of the oil spill

Correct!  
In the first case, the plaintiffs failed to show that a reasonable man would have
foreseen : the risk of damage from the oil spill
Question 4 1
/ 1 pts

Which of the following is not correct?

 
If damages are too remote they will not be recoverable

 
The remoteness test will be satisfied where the damage suffered is of the
same type or : kind as foreseeable damage

 
In order for damage to not be too remote, it must be reasonably foreseeable

Correct!  
In The Wagon Mound No 1 and The Wagon Mound No 2, where there was an
: unfortunate combination of an oil spill, welding sparks and floating cotton
waste, the loss : was found to be not reasonably foreseeable in the
circumstances

Question 5 1
/ 1 pts

In Tame v State of New South Wales (2002) 211 CLR 317 where Tame was
given a false : blood alcohol reading:

 
Whether the police officer was found to have owed Tame a duty of care was
entirely a : question of community standards

 
The psychotic depressive illness Tame developed was found to flow from the
breach of : duty of the police sergeant because it was entirely caused by his
actions that could have : been avoided had he exercised sufficient care
Correct!  
Part of the test of reasonable foreseeability is a question of fact

 
Because of the “eggshell skull” principle, pre-existing knowledge of Tame’s
susceptibility : to suffering nervous shock was not required to prove a breach

Question 6 1
/ 1 pts

Which of the following is not a provision of the Wrongs Act 1958 (Vic) relating
to: professional liability?

 
If there are differing peer professional opinions across Australia, the court may
accept : just one of those opinions

Correct!  
The court must rely on peer professional opinion even where it considers that
opinion : irrational

 
If there are differing peer professional opinions across Australia, the court may
accept all : of those opinions

 
Peer professional opinion can be considered widely accepted even where it is
not : universally accepted

Question 7 1
/ 1 pts

The “neighbour principle” is often viewed as:


 
An objective test

 
A test of the reasonable person

Correct!  
A test of the foreseeability of harm

 
An identity test

Question 8 1
/ 1 pts

In Australian Safeway Stores v Zaluzna (1987) 162 CLR 479:

Correct!  
The respondent was a lawful entrant upon the land of the respondent,
establishing a : relationship between them and there was therefore a duty of
care owed by the appellant to : avoid a foreseeable risk of injury

 
The respondent was limited by his particular status as entrant without specific
consent of : the appellant

 
The respondent contributed to the accident thus limiting damages

 
There was no liability.

Question 9 1
/ 1 pts

Nicola drives the forklift at her place of work, Rooze’s Roofing. Nicola always
leaves the : forklift in a certain place where she has been told to leave it, with
the forks up off the : ground. One afternoon a customer who is collecting
goods from the workshop reverses his : car into the forks on the forklift. He is
injured and his car is damaged.: What is the principle that would make
Nicola’s employer liable for her actions?
 
Contributory negligence

 
Strict liability

Correct!  
Vicarious liability

 
Voluntary assumption of risk

Question 10 0
/ 1 pts

To claim damages a causal link must be established between what two


things?

You Answered  
Between the defendant’s conduct and the actual monetary value of the
loss/damage

 
The conduct and initiating court proceedings

 
The fact that the plaintiff suffered loss or damage is sufficient. Nothing else is
required

orrect Answer  
The defendants breach and the plaintiff’s injury

Quiz Score:
7 out of 15
4/15/22, 8:15 PM Quiz 4_Chapters 16&17_Session 10: Business Law-T122WSB-2

Quiz 4_Chapters 16&17_Session 10


Due
Mar 21 at 12:15pm
Points
15
Questions
15
Available
Mar 21 at 11:59am - Mar 21 at 12:25pm
26 minutes
Time Limit
15 Minutes

This quiz was locked Mar 21 at 12:25pm.

Attempt History
Attempt Time Score
LATEST Attempt 1 15 minutes 7 out of 15

Score for this quiz:


7 out of 15
Submitted Mar 21 at 12:19pm
This attempt took 15 minutes.

Unanswered
Question 1 0
/ 1 pts

Find one incorrect statement?

 
In the absence of special statutory provision, although each partner is liable
with the others for : the whole of the debts of the firm, their liability is only joint

 
A creditor can bring only one action against members of a partnership and
any partner can insist : that the action be stayed until all other partners are
joined as parties.

 
A person admitted into an existing firm, liability may be incurred where it is
specially agreed upon

https://lms.westernsydney.edu.vn/courses/287/quizzes/3580 1/10
4/15/22, 8:15 PM Quiz 4_Chapters 16&17_Session 10: Business Law-T122WSB-2

orrect Answer  
A creditor can enforce liability against an incoming partner whether or not
he/she is a party to : the contract

Unanswered
Question 2 0
/ 1 pts

Which statement regarding dissolution of a partnership is incorrect?

 
Partners are not entitled to have partnership property applied towards the
payment of : partnership liabilities

orrect Answer  
After payment of the firm’s liabilities, partners do not have the right to have
surplus assets : applied in payment of what may be due to the partners
respectively

 
On the termination of the partnership, any partner can apply to the court for
a decree to dissolve : the partnership and appoint a receiver to wind up the
firm’s business/affairs

 
Should a sequestration order be made against a partner, a creditor of the
firm cannot receive a : dividend out of the bankrupt’s separate property until
all separate creditors of the bankrupt partner : have been paid in full.

Unanswered
Question 3 0
/ 1 pts

The incorrect statement is?

https://lms.westernsydney.edu.vn/courses/287/quizzes/3580 2/10
4/15/22, 8:15 PM Quiz 4_Chapters 16&17_Session 10: Business Law-T122WSB-2

 
A partner has express actual and implied actual authority to engage in
certain activities with third: parties

orrect Answer  
A partner cannot pledge/sell partnership property, incur and pay debts on
partnership : accounts or hire employees

 
Contravention of an agreement to restrict a partner’s authority is not binding
on the firm if notice : of the agreement has been given

 
In Polkinghorne v Holland (1934) 51 CLR 143, the firm was held liable
because Holland provided : the advice in his role as solicitor; thus in the
ordinary course of the firm’s business

Question 4 1
/ 1 pts

Which of the below does not apply to the Partnership Act?

 
The rules in the Partnership Acts assist in determining whether a business is
being carried on in : common

Correct!  
The partnership under the Act must be registered with ASIC

 
Each rule states a negative: i.e. that a certain fact does not of itself create a
partnership, but is : only indicative of a partnership

 
Joint or part ownership or joint tenancy, or tenancy in common whether or not
the : owners/tenants share the profits, does not of itself create a partnership
as to anything so : held/owned

https://lms.westernsydney.edu.vn/courses/287/quizzes/3580 3/10
4/15/22, 8:15 PM Quiz 4_Chapters 16&17_Session 10: Business Law-T122WSB-2

Question 5 1
/ 1 pts

Cribb v Korn (1911) 12 CLR 205 established:

 
Joint ownership creates a partnership

 
A joint tenancy is just another name for a partnership

Correct!  
The sharing of joint returns does not in itself create a partnership

 
The statutory rules can be excluded by agreement

Unanswered
Question 6 0
/ 1 pts

Which of the below statements is incorrect?

orrect Answer  
a written and signed notice of a partner’s intention to dissolve the partnership
is required, : where no fixed term has been agreed upon for the duration of
the partnership

 
a continuing guarantee given by or to a partnership is revoked as to future
transactions if there is : a change in the constitution of the partnership

 
where a partnership continues after a fixed term has expired, the rights and
duties of the partners : remain the same, but the partnership becomes a
partnership at will

https://lms.westernsydney.edu.vn/courses/287/quizzes/3580 4/10
4/15/22, 8:15 PM Quiz 4_Chapters 16&17_Session 10: Business Law-T122WSB-2

 
partnership property must be used exclusively for the purposes of the
partnership and in the : manner set out in the partnership agreement

Question 7 0
/ 1 pts

Find incorrect statement

 
A term often included in a partnership agreement that allows other partners
to purchase a : retiring/deceased partner’s interest at an agreed valuation
avoids the disruption of a formal winding

orrect Answer  
A partnership cannot be dissolved because the business is carried on at a
loss

 
Partnerships can be dissolved because of mutual incompatibility, making it
impossible for partners : to carry on a business

You Answered  
On dissolution, partnership property can be applied towards the payment of
partnership : liabilities/debts and any surplus can be distributed among the
partners

Unanswered
Question 8 0
/ 1 pts

Regarding partnerships, which of the following statements is incorrect?

https://lms.westernsydney.edu.vn/courses/287/quizzes/3580 5/10
4/15/22, 8:15 PM Quiz 4_Chapters 16&17_Session 10: Business Law-T122WSB-2

orrect Answer  
Partnerships are created with a view to profit, so partners must make a profit:
b A partner does not have to have a direct claim to a share of the profits

 
Associations and charities are not partnerships, as profits come from
ancillary business activities : and are reinvested, not distributed as dividends
to their members

 
The Partnership Acts do not govern members of a corporation incorporated
under the provisions : of the Corporations Act 2001 (Cth), a special Act of
Parliament, or Royal Charter

Question 9 1
/ 1 pts

Which legislation regulates partnerships in NSW?

Correct!  
Partnership Act 1892 (NSW)*

 
Competition & Consumer Act 2001 (Cth)

 
Partnership Act 1958 (Vic)

 
Partnership Act 1895

Unanswered
Question 10 0
/ 1 pts

Which of the following statements regarding partnerships is incorrect?

 
The second element of a partnership is carrying out a business in common

https://lms.westernsydney.edu.vn/courses/287/quizzes/3580 6/10
4/15/22, 8:15 PM Quiz 4_Chapters 16&17_Session 10: Business Law-T122WSB-2

 
To be a partnership there must be a mutuality of rights and obligations

orrect Answer  
Each partner must take an active part in the direction and management of
the firm

 
In Degiorgio v Dunn [2004] NSWSC 767 it was held that there was no
partnership because the : business was not run “in common”

Unanswered
Question 11 0
/ 1 pts

Which is incorrect statement?

orrect Answer  
If a member of a firm of solicitors acting for a vendor in a sale absconds with
the deposit, : his/her partners are not liable to refund the money

 
In Lloyd v Grace, Smith & Co [1912] AC 716, the firm was held responsible
for the fraud committed : by a managing clerk of a firm, who misappropriated
property while acting within the scope of his : authority

 
In SJ Mackie Pty Ltd v Dalziell Medical Practice Pty Ltd [1989] 2 Qd it was
held that the transfer of a : share to a non-partner breaks the continuity of
the firm, constituting a new firm/partnership of the : remaining former partners
and the new member

 
Partnership agreements can contain provisions to enable the transition from
one firm to another : to be effected without the disruption of a formal winding
up.

https://lms.westernsydney.edu.vn/courses/287/quizzes/3580 7/10
4/15/22, 8:15 PM Quiz 4_Chapters 16&17_Session 10: Business Law-T122WSB-2

Question 12 1
/ 1 pts

Which finding is correct?: In Popat v Schonchhatra (1997) 3 All ER 800, the


Court decided:

Correct!  
Popat was entitled to half the profits on the sale of the business and to a
share of the profits : that had accrued after the dissolution of the partnership,
but before the final settlement of : accounts.

 
Popat was not entitled to any profits on the sale of the business, as there was
no partnership : agreement and he was further not entitled to a share of the
profits accrued after the dissolution of : the partnership, but before the final
settlement of accounts.

 
Popat was entitled to half the profits on the sale of the business, but not to a
share of the profits : that had accrued after the dissolution of the partnership,
but before the final settlement of : accounts.

 
Popat was not entitled to half the profits on the sale of the business, but he
was entitled to a : share of the profits that had accrued after the dissolution
of the partnership but before the final : settlement of accounts.

Question 13 1
/ 1 pts

Which statements is incorrect?

 
Like most partners, sole traders have unlimited personal liability of the
business/firm

https://lms.westernsydney.edu.vn/courses/287/quizzes/3580 8/10
4/15/22, 8:15 PM Quiz 4_Chapters 16&17_Session 10: Business Law-T122WSB-2

Correct!  
More complex business organisations are unsuitable for larger businesses

 
Upon incorporation, a corporation becomes a separate legal entity

 
Diverse ownership requires proper management and expertise in numerous
areas

Question 14 1
/ 1 pts

Which statements is not correct?

 
if the agreement is that a person should be paid a fixed sum by the firm,
he/she may be a partner : and jointly liable to creditors of the firm

 
the rights and obligations of partners to each other arise from the partnership
agreement, the : statute and the equitable concept of the fiduciary

Correct!  
a partnership agreement must be in writing; it cannot be reached orally or by
a course of : conduct

 
the Partnership Act determines partners’ rights, duties and interests, if not
included in the : partnership agreement

Question 15 1
/ 1 pts

Find a statement that is incorrect


https://lms.westernsydney.edu.vn/courses/287/quizzes/3580 9/10
4/15/22, 8:15 PM Quiz 4_Chapters 16&17_Session 10: Business Law-T122WSB-2

 
A retiring partner should give specific notice of their retirement to persons
with whom the firm : has had dealings to avoid being made liable for debts
incurred after retirement.

 
Persons who represents themselves, or knowingly allow themselves to be
represented, as a : partner, are liable as a partner to anyone who has on the
faith of such representation given credit to : the firm

Correct!  
A wrongful act/omission includes breach of contract or of fiduciary duty,
negligent : misrepresentation and misleading or deceptive conduct

 
Innocent partners are liable for wrongful acts of other partners made in the
ordinary conduct of : business

Quiz Score:
7 out of 15

https://lms.westernsydney.edu.vn/courses/287/quizzes/3580 10/10
4/10/22, 10:19 PM Quiz 4_Chapters 16&17_Session 10: Business Law-T122WSB-2

Quiz 4_Chapters 16&17_Session 10


Due
Mar 21 at 12:15pm
Points
15
Questions
15
Available
Mar 21 at 11:59am - Mar 21 at 12:25pm
26 minutes
Time Limit
15 Minutes

This quiz was locked Mar 21 at 12:25pm.

Attempt History
Attempt Time Score
LATEST Attempt 1
15 minutes 9 out of 15

Score for this quiz:


9 out of 15
Submitted Mar 21 at 12:14pm
This attempt took 15 minutes.

Question 1 1
/ 1 pts

Which legislation regulates partnerships in NSW?

Correct!  
Partnership Act 1892 (NSW)*

 
Competition & Consumer Act 2001 (Cth)

 
Partnership Act 1958 (Vic)

 
Partnership Act 1895

Question 2 1
/ 1 pts

Which statement regarding an outsize partnership is incorrect?

 
it has more than 20 partners

https://lms.westernsydney.edu.vn/courses/287/quizzes/3580?module_item_id=11542 1/9
4/10/22, 10:19 PM Quiz 4_Chapters 16&17_Session 10: Business Law-T122WSB-2

Correct!  
its partnership agreement is invalid

 
it is liable to a criminal penalty ($500)

 
its agreement does not affect the enforceability of contracts or other
arrangements made

Question 3 1
/ 1 pts

Legislation in Victoria: What Act regulates partnerships in Victoria?

 
Partnership Act 1892

Correct!  
Partnership Act 1958 (Vic)

 
Corporations Act 2001 (Cth), s 20

 
Competition & Consumer Act 2010 (Cth)

Question 4 0
/ 1 pts

What constitutes an exception to the usual number of a partnership being 20


partners?

 
If one of the partners is married-his/her husband or wife is automatically a
partner

 
The exception relates only to limited partnerships

You Answered  
No more than 20 are allowed

https://lms.westernsydney.edu.vn/courses/287/quizzes/3580?module_item_id=11542 2/9
4/10/22, 10:19 PM Quiz 4_Chapters 16&17_Session 10: Business Law-T122WSB-2

orrect Answer  
The Corporations Regulations 2001 (Cth) provide greater numbers in certain
specified : professions

Question 5 0
/ 1 pts

Which statement regarding partnership is incorrect?

 
A partnerships (or firm) is a way of gathering resources or expertise for major
projects

orrect Answer  
Partnerships do not avoid taking on the formality and expense of an
incorporated company

 
Partners in a partnership complement each other with their skills and bring in
new capital and : broader funding options

You Answered  
A partnership in law may exist without the partners being aware of it

Question 6 1
/ 1 pts

Cribb v Korn (1911) 12 CLR 205 established:

 
Joint ownership creates a partnership

 
A joint tenancy is just another name for a partnership

Correct!  
The sharing of joint returns does not in itself create a partnership

https://lms.westernsydney.edu.vn/courses/287/quizzes/3580?module_item_id=11542 3/9
4/10/22, 10:19 PM Quiz 4_Chapters 16&17_Session 10: Business Law-T122WSB-2

 
The statutory rules can be excluded by agreement

Question 7 1
/ 1 pts

Which statement is incorrect?

 
carrying on a business implies repetition (Smith v Anderson (1880) 15 Ch D
247)

Correct!  
partnership may be formed in order to undertake a single business transaction

 
in Khan v Miah [2000] 1 WLR 2123 it was held that work, such as finding,
acquiring and fitting out : a shop/restaurant, is undertaken with a view to profit

 
in Keith Spicer Ltd v Mansell [1970] 1 All ER 462 it was held that ordering
goods and opening a : joint bank account in contemplation of a business are
insufficient for a partnership

Question 8 0
/ 1 pts

Which legislation controls the actual registration of the firm name?

You Answered  
Corporations Act 2001 (Cth)

 
Partnership Act

 
Statute of Frauds 1677

https://lms.westernsydney.edu.vn/courses/287/quizzes/3580?module_item_id=11542 4/9
4/10/22, 10:19 PM Quiz 4_Chapters 16&17_Session 10: Business Law-T122WSB-2

orrect Answer  
Business Names Registration Act 2011 (Cth)

Question 9 0
/ 1 pts

Which of the following statements regarding the written law is incorrect?

 
Under s 115 of the Corporations Act 2001 (Cth), the maximum number of
persons who may form a : partnership for the acquisition of gain is 20

You Answered  
Under the Corporations Regulations 2001 (Cth), reg 2A.1.01, partnerships of
more than 20 : partners may be formed for certain professions/callings

 
The Corporations Regulations 2001 (Cth), reg 2A.1.01 sets a maxima of
partners for medical and : legal practitioners, veterinary surgeons, patent and
trademark attorneys, sharebrokers and : stockbrokers and pharmaceutical
chemists

orrect Answer  
The Corporations Regulations 2001 (Cth), reg 2A.1.01 does not set a maxima
of partners for : architects and accountants

Question 10 1
/ 1 pts

Which statements is incorrect?

 
Like most partners, sole traders have unlimited personal liability of the
business/firm

https://lms.westernsydney.edu.vn/courses/287/quizzes/3580?module_item_id=11542 5/9
4/10/22, 10:19 PM Quiz 4_Chapters 16&17_Session 10: Business Law-T122WSB-2

Correct!  
More complex business organisations are unsuitable for larger businesses

 
Upon incorporation, a corporation becomes a separate legal entity

 
Diverse ownership requires proper management and expertise in numerous
areas

Question 11 0
/ 1 pts

Which statements is not correct?

 
if the agreement is that a person should be paid a fixed sum by the firm,
he/she may be a partner : and jointly liable to creditors of the firm

You Answered  
the rights and obligations of partners to each other arise from the partnership
agreement, the : statute and the equitable concept of the fiduciary

orrect Answer  
a partnership agreement must be in writing; it cannot be reached orally or by a
course of : conduct

 
the Partnership Act determines partners’ rights, duties and interests, if not
included in the : partnership agreement

Question 12 1
/ 1 pts

https://lms.westernsydney.edu.vn/courses/287/quizzes/3580?module_item_id=11542 6/9
4/10/22, 10:19 PM Quiz 4_Chapters 16&17_Session 10: Business Law-T122WSB-2

Which finding is correct?: In Popat v Schonchhatra (1997) 3 All ER 800, the


Court decided:

Correct!  
Popat was entitled to half the profits on the sale of the business and to a share
of the profits : that had accrued after the dissolution of the partnership, but
before the final settlement of : accounts.

 
Popat was not entitled to any profits on the sale of the business, as there was
no partnership : agreement and he was further not entitled to a share of the
profits accrued after the dissolution of : the partnership, but before the final
settlement of accounts.

 
Popat was entitled to half the profits on the sale of the business, but not to a
share of the profits : that had accrued after the dissolution of the partnership,
but before the final settlement of : accounts.

 
Popat was not entitled to half the profits on the sale of the business, but he
was entitled to a : share of the profits that had accrued after the dissolution of
the partnership but before the final : settlement of accounts.

Question 13 0
/ 1 pts

Which of the below statement is incorrect?

orrect Answer  
Partnership property is liable to be seized for the private (personal) debt of a
partner and : made liable on a judgment against the partnership

https://lms.westernsydney.edu.vn/courses/287/quizzes/3580?module_item_id=11542 7/9
4/10/22, 10:19 PM Quiz 4_Chapters 16&17_Session 10: Business Law-T122WSB-2

You Answered  
A creditor who has obtained judgment in respect of the separate debt of a
partner may obtain an : order charging that partner’s interest in the
partnership property and profits with the amount of the : debt and interest.

 
A creditor who has obtained judgment in respect of the separate debt of a
partner may obtain by : an order the appointment of a receiver of that partner’s
share of profits and of any other money : which may be coming to the partner
in respect of the partnership.

 
The two sources of law relevant when considering how partners bind their
partners when dealing : with third parties are the common law (including
equity) of agency and the Partnership Act. The : common law complements
the provisions of the Partnership Act in relation to the authority that an : agent
has.

Question 14 1
/ 1 pts

Which statement regarding persons of unsound mind and minors is


incorrect?

 
a partner of unsound mind is capable of binding the firm and of being bound
by co-partners, : unless proven that he/she was of unsound mind when the
partnership was entered into and the : other partners knew this

Correct!  
if a partner who is a minor enters into a contract with a third party on behalf of
the firm, the : minor is liable as far as private assets are concerned

 
a creditor who has obtained judgment against the firm may not seize the
minor’s separate : property

https://lms.westernsydney.edu.vn/courses/287/quizzes/3580?module_item_id=11542 8/9
4/10/22, 10:19 PM Quiz 4_Chapters 16&17_Session 10: Business Law-T122WSB-2

 
a minor will become liable as an ordinary partner when he/she attains majority
the partnership is : not repudiated within a reasonable time

Question 15 1
/ 1 pts

The incorrect statement?

 
Partners are not bound when another partner misappropriates monies

Correct!  
If the giving of investment advice is within the scope of the firm’s business
then all partners : are jointly and severally liable for advice which is not in a
client’s best interests

 
the client should seek a personal indemnity from a partner

 
special skill is required to bind all partners

Quiz Score:
9 out of 15

https://lms.westernsydney.edu.vn/courses/287/quizzes/3580?module_item_id=11542 9/9
Quiz 4_Chapters 16&17_Session 10
Due
Mar 22 at 7:30pm
Points
15
Questions
15
Available
Mar 22 at 7:59am - Mar 22 at 7:30pm
about 12 hours
Time Limit
15 Minutes

This quiz was locked Mar 22 at 7:30pm.

Attempt History
Attempt Time Score
LATEST Attempt 1
15 minutes 8 out of 15

Score for this quiz:


8 out of 15
Submitted Mar 22 at 7:15pm
This attempt took 15 minutes.

Question 1 1
/ 1 pts

Which is incorrect statement?

Correct!  
If a member of a firm of solicitors acting for a vendor in a sale absconds with
the deposit, : his/her partners are not liable to refund the money

 
In Lloyd v Grace, Smith & Co [1912] AC 716, the firm was held responsible for
the fraud committed : by a managing clerk of a firm, who misappropriated
property while acting within the scope of his : authority

 
In SJ Mackie Pty Ltd v Dalziell Medical Practice Pty Ltd [1989] 2 Qd it was
held that the transfer of a : share to a non-partner breaks the continuity of the
firm, constituting a new firm/partnership of the : remaining former partners and
the new member
 
Partnership agreements can contain provisions to enable the transition from
one firm to another : to be effected without the disruption of a formal winding
up.

Question 2 1
/ 1 pts

Find incorrect statement

 
A term often included in a partnership agreement that allows other partners to
purchase a : retiring/deceased partner’s interest at an agreed valuation avoids
the disruption of a formal winding

Correct!  
A partnership cannot be dissolved because the business is carried on at a loss

 
Partnerships can be dissolved because of mutual incompatibility, making it
impossible for partners : to carry on a business

 
On dissolution, partnership property can be applied towards the payment of
partnership : liabilities/debts and any surplus can be distributed among the
partners

Question 3 1
/ 1 pts

Which of the following statements regarding dissolution of partnership is


incorrect?
 
In the event of dissolution, losses must be met first out of profits, followed by
capital, then by : partners in proportion to their share of profits

Correct!  
A firm’s assets as contributed by partners to make up capital losses need not
be applied to pay : the firm’s debts/liabilities to non-partners

 
After dissolution, each partner is entitled to advances and residue by the firm

 
After the dissolution, each partner’s authority to bind the firm continues, so far
as necessary for : winding up partnership affairs and completing unfinished
transactions

Question 4 0
/ 1 pts

Which of the following statements is not correct?

 
as an agent, a partner is able to bind the other partners and, as principal, be
bound by the actions : of the other partners

You Answered  
in New South Wales, Victoria, Queensland, South Australia, Western
Australia and: Tasmania provision for limited partnerships is made in the
Partnership Act

orrect Answer  
incorporated limited partnerships have been introduced in all Australian States
and Territories
 
the Partnership Act provides that the rules of the common law and equity are
to continue in force : except insofar as they are inconsistent with the Act

Question 5 0
/ 1 pts

Which of the below statements is incorrect?

orrect Answer  
a written and signed notice of a partner’s intention to dissolve the partnership
is required, : where no fixed term has been agreed upon for the duration of the
partnership

 
a continuing guarantee given by or to a partnership is revoked as to future
transactions if there is : a change in the constitution of the partnership

You Answered  
where a partnership continues after a fixed term has expired, the rights and
duties of the partners : remain the same, but the partnership becomes a
partnership at will

 
partnership property must be used exclusively for the purposes of the
partnership and in the : manner set out in the partnership agreement

Question 6 1
/ 1 pts

Which of the following statements is incorrect?


 
a separate legal entity can commit civil wrongs and engage in criminal conduct

 
a public company has serious compliance obligations

Correct!  
a partnership must be in writing

 
a separate legal entity is entitled to own property, pay tax and enter into
contracts

Question 7 1
/ 1 pts

Which below statements is incorrect?

 
Partners share the profits in a partnership

Correct!  
In a partnership, partners have limited liability for the business debts

 
A partner is also an agent of the partnership

 
Agreement may be implied in a partnership

Question 8 0
/ 1 pts

Which of the following is NOT a ground for termination of a partnership?

 
Court order

orrect Answer  
Partner leaves the jurisdiction

 
Expiry of a fixed term

You Answered  
Bankruptcy of a partner

Question 9 1
/ 1 pts

Which is incorrect statements regarding limited partnerships?

 
A limited partner must not take part in the management of the business and
does not have power : to bind the firm

 
If a limited partner partakes in the management of the business, he/she is
liable as a general : partner.

Correct!  
A limited partner does not have the right to inspect the books of the firm

 
Any differences arising as to ordinary matters connected with the firm’s
business are to be : decided by a majority of the general partners.

Question 10 1
/ 1 pts

Which of the following statements regarding partnerships is incorrect?

 
The second element of a partnership is carrying out a business in common

 
To be a partnership there must be a mutuality of rights and obligations

Correct!  
Each partner must take an active part in the direction and management of the
firm

 
In Degiorgio v Dunn [2004] NSWSC 767 it was held that there was no
partnership because the : business was not run “in common”

Question 11 0
/ 1 pts

Find one incorrect statement?

You Answered  
In the absence of special statutory provision, although each partner is liable
with the others for : the whole of the debts of the firm, their liability is only joint

 
A creditor can bring only one action against members of a partnership and any
partner can insist : that the action be stayed until all other partners are joined
as parties.

 
A person admitted into an existing firm, liability may be incurred where it is
specially agreed upon

orrect Answer  
A creditor can enforce liability against an incoming partner whether or not
he/she is a party to : the contract

Unanswered Question 12 0
/ 1 pts
How would a partner pledge the firm’s credit?

 
By deed

orrect Answer  
Only for a purpose directly connected to the firm’s business but must have
express authority : to do so

 
By agreement with other partners

 
By agreement with a majority of partners plus an appropriate credit application

Question 13 0
/ 1 pts

Which finding is correct?: In Popat v Schonchhatra (1997) 3 All ER 800, the


Court decided:

orrect Answer  
Popat was entitled to half the profits on the sale of the business and to a share
of the profits : that had accrued after the dissolution of the partnership, but
before the final settlement of : accounts.

 
Popat was not entitled to any profits on the sale of the business, as there was
no partnership : agreement and he was further not entitled to a share of the
profits accrued after the dissolution of : the partnership, but before the final
settlement of accounts.

 
Popat was entitled to half the profits on the sale of the business, but not to a
share of the profits : that had accrued after the dissolution of the partnership,
but before the final settlement of : accounts.
You Answered  
Popat was not entitled to half the profits on the sale of the business, but he
was entitled to a : share of the profits that had accrued after the dissolution of
the partnership but before the final : settlement of accounts.

Question 14 1
/ 1 pts

Which court finding is incorrect?

Correct!  
In Mercantile Credit Co Ltd v Garrod [1962] 3 All ER 1103, damages were not
recovered even : though, from the plaintiff’s perspective, the sale of the car
was within the usual course of : business.

 
In Goldberg v Jenkins (1889) 15 VLR 36, the firm was not bound to the
transaction because : borrowing money on behalf of the firm at over 60%
interest when comparable rates were between : 6% and 10% was beyond ‘the
usual way’.

 
In Construction Engineering Pty Ltd v Hexyl Pty Ltd (1985) 155 CLR 541,
Hexyl was not liable : because the construction contract appeared to be
between Construction Engineering and Tembel

 
If judgment is obtained against one or more partners of a firm, no action may
be taken against the : other partners, even if satisfaction cannot be obtained
from the partner(s) sued

Question 15 0
/ 1 pts
Which statement regarding persons of unsound mind and minors is
incorrect?

You Answered  
a partner of unsound mind is capable of binding the firm and of being bound
by co-partners, : unless proven that he/she was of unsound mind when the
partnership was entered into and the : other partners knew this

orrect Answer  
if a partner who is a minor enters into a contract with a third party on behalf of
the firm, the : minor is liable as far as private assets are concerned

 
a creditor who has obtained judgment against the firm may not seize the
minor’s separate : property

 
a minor will become liable as an ordinary partner when he/she attains majority
the partnership is : not repudiated within a reasonable time

Quiz Score:
8 out of 15
QUIZ 1:
Reporting obligations: What is a binding precedent?
A decision of another court that is of persuasive authority
With respect to Australia, it is a decision of the UK Supreme Court
A decision of a court that binds judges in a lower court in the same court hierarchy
A decision of a different court on the same subject matter

Which are some of the main features of the Electronic Transactions Act 1999
(Cth):
validity of electronic transactions
recognition of writing by electronic means
Recognition of retaining information in electronic form
All of the above

Peter verbally offers Tara his car for $5000. Tara refuses saying it’s not worth that,
but : she will pay $4000. What is the status of Peter’s first offer?
It can still be accepted by Tara
Peter should have made it in writing so Tara had proof he made it
Peter needs to make it again
It has lapsed because of Tara’s counteroffer

What was one of the “practical benefits” received by Roffey in Williams v Roffey
Bros &: Nicholls (Contractors) Ltd [1990] 1 All ER 512?
Williams did not sue Roffey for breach of contract
Roffey had no reason to doubt Williams would not complete his side of the bargain
Roffey received additional payment from Williams
Roffey did not need to find another subcontractor

Changing the Constitution : Section 128 of the Commonwealth Constitution


provides that the Constitution can be: changed by referendum that requires a
“yes” vote:
in at least 2 States
in at least 3 States
in the ACT and NT (the Territories)
by the majority of voters and in a majority of States

Substantive and procedural law: Substantive law:


Refers to actual rights under the law (bên t đáp án là câu này cơ)
Is subsidiary to procedural law
Refers to the formal steps to enforcement of rights and duties under the law
Includes the rules of evidence

In which of the following situations is the offer most likely not to have lapsed?
Where Wrench had an option to purchase land but, unknown to Wrench, the seller
died : prior to Wrench’s acceptance (bên t là câu này)
Where Carter, a prospective purchaser of land, makes a counter-offer that is rejected
and: then states that he will accept the earlier offer
Where no time was stated for acceptance by Jane, but Ben does not accept within a :
reasonable time
Where Jim has become overtaken by insanity prior to accepting

Reception of English law in Australia: Which Latin phrase explained the rationale
for applying English laws to the new Colony of : New South Wales?
actus reus
terra nullius
ratio decidendi
prima facie

Andy promises Ellie $100 on her 21st birthday. If this promise was contained in a
simple : contract, the legal position is that such a promise is:
Never binding as it must have consideration supplied by Ellie
Binding only if Ellie and Andy had contractual capacity
Sometimes binding but only after Ellie turned 21
Always binding because promises should not be broken

In Elizabeth City Center Pty Ltd v Corralyn Pty Ltd (1995) 63 SASR 235, the option
to : exercise renewal of the lease:
Was effective because of the operation of the postal rule
Effective communication of the renewal requirement had not been made
Negated the postal rule because the notification was not sent by certified mail as :
required
b and c

Interpretation – extrinsic materials: In interpretation, extrinsic materials:


Are required to be referred to by a court
Are only to be referred to where the meaning of the provision is ambiguous
Are allowed to be referred to by a court
Are to be referred to regardless of the length of delay this will cause proceedings

With respect to revocation, which of the following statements is not correct?


Revocation need not be in words
The offeror must personally communicate the revocation to the offeree
The offeree may accept the offer until such time as they become aware of the
revocation
Where an offer has been made to the world at large, revocation does not need to be
seen : by everyone in order to be effective

Which of the following circumstances are likely to affect the consent of one or
both : parties to a contract:
Mistake
Duress and undue influence
Lack of writing
a&b

Simon and Stella, both of full legal capacity, agree to go on a date. Stella is to pay
for the : dinner, but she is running late and does not meet Simon at all. Simon is
embarrassed and : angry and calls Stella the next day to threaten to sue her for
his taxi fares and dinner : expenses. Will Simon succeed in his claims?
Yes, because there is agreement
Yes, because there was consideration in that Simon incurred taxi and dinner expenses
No, because Simon and Stella did not intend the agreement to create legally
enforceable : obligations
No, because there was no genuine consent of either party

Royal Assent: Who or what gives Royal Assent to an Act of the Commonwealth
Parliament?
The Queen
The Governor-General on the Queen’s behalf
The Prime Minister
Cabinet

In Crown Melbourne Ltd v Cosmopolitan Hotel (Vic) Pty Ltd [2016] HCA 26 why
did the : High Court disagree with the decision of the Victorian Court of Appeal in
finding there was : no case for promissory estoppel:
Even though the elements of estoppel had been proven no remedy was provided as the
: initial claim was brought too late and equity does not assist claims lodged with
unreasonable : delay
The plaintiff could not establish that the statement that it would be “looked after at :
renewal time” was capable of conveying to a reasonable person that it was a genuine
offer : of a further lease
There was a need for certainty as to what the parties had agreed on at the end of the :
negotiations to found a claim for estoppel
B&C

Which of the following is not a way in which an offer can be terminated?


Revocation after acceptance has been mailed where the postal rule applies

Farah agreed to take care of an elderly woman Marge and in return was provided
with a : house to live in that was promised by Marge to be Farah’s after she died.
Farah cared for the : woman for 23 years, but upon Marge’s death Farah
discovered that their oral agreement : was never put into writing. The woman’s
son moved into the house and Farah made a claim : to the house. The court:
Is likely to reject Farah’s claim to the house as she provided no consideration of any
legal: value
Is likely to reject Farah’s claim to the house as the contract was not set down in writing
as : required
Is likely to order specific performance of the oral agreement because simple contracts
do : not need to be in writing
Is likely to apply the doctrine of part performance

Which of the following is most likely to be considered an offer?


A catalogue of books with discounted prices shown
A radio advertisement for drinks at “the coolest little pub in Victoria”
“I’ll pay you $3,000 if you complete a total rewrite of chapters 12 and 15 of this :
textbook.”
“Would you allow me to pay off the $2,000 over six weeks?”

Australian Constitution : Which Act established the federal legal and political
system and converted the separate : colonies into states?
Commonwealth of Australia Constitution Act 1900
The Australia Act 1986
Statute of Westminster Adoption Act 1942 (Cth)
statute of Westminster 1931 (IMP)

Contracts voidable by a minor do not include:


Those not binding unless ratified by the minor during their minority
Those binding unless repudiated by the minor during their minority
Those binding unless repudiated by the minor within a reasonable time after attaining :
their majority
Those not binding unless ratified by the minor within a reasonable time of attaining :
majority

Where an agreement has been made during the course of trade or commerce
between businesses, the situation will commonly indicate that the parties:
intended to create moral binding relations based on trust
intended to create binding legal relations
intended that there was no contract unless clearly specified in the agreement
intended to avoid court proceedings by inserting exclusion clauses in the agreement

Which of the following is not a class of persons regarded by the law as wholly or
partly: incapable of entering into legally binding contracts?
Intoxicated persons
Minors
Women
Mentally incapacitated persons

Separation of powers: Which of the following is not one of the three branches of the
Commonwealth government : in Australia:
A. The Governor
B. The executive
C. The judiciary
D. The legislature

Criminal offences: Which of the following is incorrect with respect to criminal offences?
A. Indictable offences are generally the more serious offences.
B. The prosecution must prove its case beyond reasonable doubt.
C. Summary offences are determined by a magistrate without a jury.
D. A committal hearing is held before most summary offence matters.

Federal system: Which of the following statements is correct?


A. Australia is a federal system, with one Constitution and a number of
non-law-making : States and Territories.
B. Australia is a unitary system, where the States and Territories are constrained in
their : law-making powers by the Commonwealth.
C. Australia is a federal system, with two legal systems for each citizen.
D. Australia is a federal system with three branches of government: the judiciary, the :
executive and the Crown.

Native Title: Native Title was first recognised in which of the following cases?
A. Mabo v State of Queensland (No 2) (1992) 175 CLR 1
B. Wik Peoples v State of Queensland (1996) 187 CLR 1
C. Brandy v Human Rights and Equal Opportunity Commission (1995) 183 CLR 245
D. Commonwealth v Jones (1901) 174 CLR 25

Separation of powers: Which of the following statements about separation of powers in


Australia is correct?
A. The judiciary is the body that makes statute law.
B. The legislature is the branch that declares what the law is and interprets the law.
C. The executive is the body that administers the law.
D. The legislature is the body that resolves disputes concerning the application of law
and : polices the law.

The postal acceptance rule:


A. Can be negated where the mailed offer was misdirected and the misdirection was
the : fault of the offeror
B. Can be impliedly excluded by offeror specifying actual receipt of acceptance
C. Can create a situation where a revocation is validly made days after the offer has
actually : been accepted
D. May apply to electronic communications, such as email, if the offeree chooses

Jack was subpoenaed to give evidence on Rods behalf. Jack claimed Rod promised him :
$2000 to give evidence. Would Jack recover that money?
A. No - Jack’s public duty is to give evidence in response to that subpoena.
B. No this is illegal
C. Yes if Rod put that in writing
D. Always binding because promises should not be broken

Law Reports: Where are decisions of the High Court of Australia found?: only online
A. In the High Court reports
B. In the Commonwealth Law Reports
C. In the Common Law Reports

The Age of Majority Act 1977 (Vic) reduced the age for contractual capacity of a minor
A. 16
B. 18
C. 21
D. 25

Delegated legislation: An example of delegated legislation is the:


A. Partnership Act 1958 (Vic)
B. Local Government Act 1993 (Qld), s 25
C. Corporations Bill 1988 (Cth)
D. Corporations Regulations 2001 (Cth)

The Executive Power : The Executive power is administered by:


A. The Senate and the House of Representatives
B. The Queen, the Prime Minister and the Cabinet
C. The Governor, the Senate and the House of Representatives
D. The Judiciary, the Senate and the House of Representatives

Delegated legislation: Which of the following is not correct?: Delegated


legislation:

A. Is subordinate legislation
B. Is made under the authority of an Act of Parliament
C. Often contains more detailed rules than those of the associated Act
D. Can only be made by Government Ministers

Common law: Which one of the following statements about common law is correct?

A. Common law is judge made law


B. Common law overrides statute law
C. Common law is made by Parliaments
D. Common law and equity are the same types of laws and provide the same
remedies

In Ashton v Pratt [2015] NSWCA 12 what was the main reason for the court deciding
that : there was no intention to create a legally binding contract?

A. The agreement was not reduced to writing


B. The verbal language of the agreement greatly lacked detail from either party
and did not : indicate definite obligations
C. Pratt lacked the necessary contractual capacity due to his age and ill health
D. The plaintiff had been a worker in the escort business and not morally entitled to
any : monies under the verbal agreement

Separation of powers: Which of the following is not one of the three branches of the
Commonwealth government : in Australia:

1. The Governor
2. The executive
3. The judiciary
4. The legislature

Public and private law: Which of the following is not classified as private law?

A. The law of contract


B. Criminal law
C. The law of property
D. Corporations law

Criminal proceedings: An indictable offense is:


A. A more serious civil wrong
B. A less serious civil wrong
C. A more serious criminal offence
D. A less serious criminal offence

Where an agreement has been made during the course of trade or commerce between
businesses, the situation will commonly indicate that the parties:
A. intended to create moral binding relations based on trust
B. intended to create binding legal relations
C. intended that there was no contract unless clearly specified in the agreement
D. intended to avoid court proceedings by inserting exclusion clauses in the agreement

A qualified acceptance may also be known as a/an:


A. Counteroffer
B. Invitation to treat
C. Condition subsequent
D. Conditional agreement

Court hierarchy: Local or Magistrates Courts:

A. Are the lowest courts in the state hierarchy


B. Are the lowest courts in the federal hierarchy
C. Are an intermediate court
D. Are above Country Courts in the hierarchy

The legislature: Exclusive powers are those that:

A. Can be exercised by either the Commonwealth or the States


B. Can be exercised only by the Commonwealth
C. Can be exercised only by the States
D. Where there is any inconsistency, the Commonwealth law will prevail

Contracts voidable by a minor do not include:

A. Those not binding unless ratified by the minor during their minority
B. Those binding unless repudiated by the minor during their minority
C. Those binding unless repudiated by the minor within a reasonable time after
attaining : their majority
You Answered
D. Those not binding unless ratified by the minor within a reasonable time of attaining
: majority

Which of the following statements about contracts is true?

A. A contract must be in writing to be enforceable


B. It is a common law requirement that certain contracts are in writing
C. A simple contract is another term for an oral contract
D. It is a statutory requirement throughout Australia that some contracts must be in
writing : and others must be evidenced in writing

In which of the following situations is the offer most likely not to have lapsed?

A. Where Wrench had an option to purchase land but, unknown to Wrench, the seller
died : prior to Wrench’s acceptance
B. Where Carter, a prospective purchaser of land, makes a counter-offer that is
rejected and: then states that he will accept the earlier offer
C. Where no time was stated for acceptance by Jane, but Ben does not accept within a

: reasonable time

D. Where Jim has become overtaken by insanity prior to accepting

Interpretation – extrinsic materials: In interpretation, extrinsic materials:

A. Are required to be referred to by a court


B. Are only to be referred to where the meaning of the provision is ambiguous
C. Are allowed to be referred to by a court
D. Are to be referred to regardless of the length of delay this will cause proceedings

Sources of law: Sources of law in Australia do not include:

A. Consolidating statutes
B. Judge-made law
C. Repealed statutes
D. Equity
Public and private law: Which of the following is not classified as private law?

A. The law of contract


B. Criminal law
C. The law of property
D. Corporations law

Which of the following statements about acceptance is false?

A. Acceptance can be by telephone or email or conduct


B. Acceptance can be in any manner chosen by the offeree
C. Acceptance must be unconditional
D. Acceptance can only be made by the person/s to whom the offer was made

The main legal issue in Felthouse v Bindley (1862) 11CB (NS) 869 was:

A. the contract was formed due to offer and acceptance being proven
B. An offer to sell a horse was simply an invitation to treat
C. generally silence /lack of action does not constitute acceptance of an offer
D. An offer can be revoked at any time prior to acceptance

Changing the Constitution : Section 128 of the Commonwealth Constitution provides


that the Constitution can be: changed by referendum that requires a “yes” vote:

A. in at least 2 States
B. in at least 3 States
C. in the ACT and NT (the Territories)
D. by the majority of voters and in a majority of States

Jack was subpoenaed to give evidence on Rods behalf. Jack claimed Rod promised him :
$2000 to give evidence. Would Jack recover that money?

A. No - Jack’s public duty is to give evidence in response to that subpoena.


B. No this is illegal
C. Yes if Rod put that in writing
D. Always binding because promises should not be broken
Separation of powers: Which of the following statements about separation of powers in
Australia is correct?

A. The judiciary is the body that makes statute law.


B. The legislature is the branch that declares what the law is and interprets the law.
C. The executive is the body that administers the law.
D. The legislature is the body that resolves disputes concerning the application of law
and : polices the law.

QUIZ 2:

1. Which of the following is not true? A signature on a contract containing an


exemption : clause:
Will not guarantee the effectiveness of the clause where the doctrine of non est factum :
applies
Incorporates the exemption clause into the contract
Guarantees that an exemption clause will be effective
Will be effective even if one party did not read the contract

2. Which of the following would not constitute the unfair practice of


harassment or : coercion under the Australian Consumer Law? Where
there is a debt in connection with the : possible of goods and the creditor:
Consciously calculates to intimidate the debtor with the content of her demands
Tells the debtor, among repeated demands for payment give the debtor a discount for :
early payment
Makes demands constantly in order to exhaust the debtor
Chooses particularly threatening demands in order to demoralise the debtor

3. When considering whether a contract is a standard form contract, the court


does not : have to consider:
The bargaining power of the parties as unequal bargaining power does not constitute a :
special disadvantage
Whether both parties had the opportunity to negotiate the terms of the contract
Whether one party’s vicarious liability for its agents is limited
Whether the terms of the contract take into account the specifics of the particular :
transaction
4. Non est factum means:
It is not [my] deed
It is not true
Mistake of fact
It is unknown

5. Tom agrees to fix Richie’s car for $1,000. Richie pays a deposit of $200 and
Tom : guarantees the job will be completed within two weeks. Before Tom
commences the repairs, : the car is stolen from Richie’s locked garage and
destroyed by fire. Richie demands the : return of his $200 deposit and Tom
refuses to give it to him. Which of the following is not : correct?
The contract is automatically terminated
Tom is no longer obligated to fix Richie’s car
Under the Australian Consumer Law and Fair Trading Act 2012 (Vic), Tom is entitled
to:retain the $200 deposit
Under the Australian Consumer Law and Fair Trading Act 2012 (Vic), Tom is entitled to :
retain the portion of the $200 deposit he spent on parts and other relevant expenses, if
any

6. Which of the following is not correct? An action in restitution:


Is often brought where one party has been unjustly enriched at the expense of the
other
Cannot be brought where there is no contract between the parties
Can be brought where the contract between the parties is unenforceable
Can be brought where there has been only part performance

7. Chris and Tama buy a noodle bar from Udon4U Pty Ltd. In the course of
negotiations, : Udon4U Pty Ltd’s agent, Nadia, misrepresents the turnover
of the noodle bar, stating an : amount that is double the actually turnover.
Nadia herself received this information on : turnover from Geoff, a director
of Udon4U Pty Ltd.: In making the misrepresentation, what sections has
Nadia likely contravened?
As she merely passed on the information with no intention to procure a contravention :
she probably will not be personally liable
Section 2(1)
Section 18
Section 52
8. Which of the following is not one of the three elements of an unfair term
under s 24(1) : of the Australian Consumer Law? Where:
It would cause a significant imbalance in the parties’ rights and obligations
The term is not transparent
It is not reasonably necessary to protect the interests of the disadvantaged party
It would cause detriment to a party if it were applied

9. Which unfair practice involves participation in a trading scheme where


persons at the : top receive most of the benefits?
referral selling
pyramid selling
unconscionable conduct
misleading and deceptive conduct

10. Janey takes her fur stole to the dry cleaner. When she returns to collect it,
there is a : black mark in the shape of an iron in the middle of her stole and
it is ruined. The dry cleaner : tells Janey that he is not liable for the damage
to her fur stole as there was an exemption : clause in their agreement.
Which of the following is not correct?
If the exemption clause was merely contained within a notice on the wall, the dry :
cleaner will be liable for the damage
If the exemption clause was on the dry cleaning docket Janey was given and she was
told : “don’t worry about this, it’s just the docket you have to give us when you come to
collect : your stole”, the dry cleaner will not be able to rely on the exemption clause
In order for the exemption clause to be effective where it was not in a signed document,
: it had to have been brought to Janey’s attention
If the exemption clause was on the dry cleaning docket Janey was given rather than a :
formal written contract, the onus will be on the dry cleaner to prove that Janey was
aware it : contained conditions that would modify the agreement

11. Which of the following statements is correct?


An exemption clause is a term that completely excludes one party’s liability
An exemption clause is a term that excludes or limits the liability of one or more parties
An exemption clause limits liability to a certain monetary amount
An exclusion clause limits one party’s liability whereas an exemption clause limits all:
parties’ liability

12. Which of the following is not usually recoverable by way of damages?


Compensation for mere inconvenience or disappointment
Loss that is difficult to estimate
Nominal loss
Expenses incurred in reliance on the other party’s promise to perform

13. A party repudiates a contract when


They decide to terminate it
They are no longer able or are unwilling to perform their obligations
The other party decides to terminate it
They take too long to perform an obligation that has a time limit under the contract, such
: as payment of rent

14. Chung is looking to purchase a carwash business from Terry. Terry tells
Chung that she : serves 100-150 cars per day and that she has just
concluded an agreement to service the : fleet of cars of a local business. In
reality, Terry has only served around 15 cars per day since : her customer
service received a bad review in the local newspaper and a competing
carwash : business opened one street away. Terry intends for Chung to buy
her business based on : what she has stated. This is an example of:
unilateral mistake
negligent misrepresentation
fraudulent misrepresentation
mutual mistake

15. Undue influence differs from duress in that:


With undue influence, the contract is not voidable but void
With undue influence, no unlawful act is required
Duress arises only within a closed list of special fiduciary relationships
With undue influence, actual physical violence is required

16. Which of the following constitutes referral selling?


Jaz is approached by her neighbor to join the sales team of Thames Beauty Products. :
Thames sells its products door-to-door through agents and is a close-knit family type of
: company. Agents that introduce other agents to “the family” are rewarded with a
finders’: fee. It is only $500 to join the Thames family, and Jaz only has to spend $400
on products in : her first month
Jaz starts work at a chemist and notices that the chemist, as retailers often do, puts a :
sale price on a sticker on most of the products in the shop. However, the chemist does
not : put the sale price sticker on top of the original price sticker and when customers
bring the : products to the counter, the chemist tells them that unfortunately there has
been a mistake, : the true price is the higher price
Jaz approaches a salesperson on the forecourt to buy the car package she saw :
advertised on television and is told that there were only two cars for sale as a part of
that : deal and there are now none left. The salesperson tells Jaz that she would look
much better : in the convertible model and although there is no deal on that car, it is only
$50,000 more
Jaz purchases a painting through a new art gallery that has just opened in town. The :
price is high and Jaz is at first uncertain about the expense, but the art dealer promises
Jaz a : commission on sales to any of Jaz’s friends who she sends along to the gallery.
Jaz has a lot : of art collector friends so is sure that she will receive some commission
from the gallery in : the near future.

17. In Trident General Insurance Co Ltd v McNiece Bros Pty Ltd (1988) 165 CLR
107:
Blue Circle was a sub-contractor whose worker was injured, and privity precluded the :
worker from getting an indemnity under the contract
It was held that the subcontractor could get an indemnity even though the :
subcontractor had given no consideration under the contract
In Deane J’s view Trident, the insurance company, had an obligation to the
subcontractor because otherwise Trident would be unjustly enriched
The majority were of the view that the “settled and fundamental” doctrine of privity of :
contract should not be overturned by the court

18. Jacq and Jack robbed the Commercial Bank. Jacq thinks that Jack was too
aggressive : with the tellers and the robbery could have gone very wrong.
Jacq is now refusing to give : Jack his share of the takings. Which of the
following is correct?
Jack provided consideration in the form of performing the robbery and is entitled to his :
share
Under the principle of freedom of contract, Jacq and Jack are entitled to agree to :
whatever they wish and the contract will be enforceable
The contract involved the commission of a crime and is therefore illegal and :
unenforceable
The contract will be unenforceable if the courts deem it to be against public policy

19. If one party has completed its obligations under the contract, and the other
has not:
The contract can be terminated by mutual agreement
The contract cannot be terminated by simple agreement without consideration
The contract can be terminated if it is put down in writing; no further consideration is :
required
The contract can be terminated where one party promises to abandon their rights under
: the contract and the other party promises to do the same

20. Ella contracted Anasaki to build a balcony for her house to certain
specifications. When : Anasaki had finished, Ella was shocked to see that
the balcony was much bigger than she : had specified. Ella refused to pay
Anasaki. Which of the following is incorrect?
Ella has received some benefit
The benefit Ella received was at Anasaki’s expense
It would be unjust for Ella to have to pay any money for a balcony that was not built to :
her specifications under the contract
Ella will probably not have to pay the contract price but must pay a reasonable amount

21. Which of the following is an element required for the doctrine of frustration
to operate?
Material loss to one party
No fault on the part of either party
Material loss to both parties
Hardship or inconvenience

22. Which of the following is not correct with respect to the prohibition of
unconscionable : conduct within the meaning of the unwritten law?
Unwritten law includes equity
There is no unconscionable conduct where both parties have made the same mistake in
: good faith
There must be a special disadvantage and an exploitation of that disadvantage
Unequal bargaining power of itself is enough to constitute a special disadvantage

23. In Jarvis v Swans Tours Ltd [1973] QB 233, where Jarvis was disappointed
by, among : other things, the little dry nut cakes on his holiday:
Damages were not awarded
Damages were awarded for anxiety and depression
Pleasure and enjoyment were not promises of the tour company with respect to the :
holiday in Switzerland
Damages were awarded partly for disappointment

24. Chung is looking to purchase a carwash business from Terry. Terry tells
Chung that she : serves 100-150 cars per day and that she has just
concluded an agreement to service the : fleet of cars of a local business. In
reality, Terry has only served around 15 cars per day since : her customer
service received a bad review in the local newspaper and a competing
carwash : business opened one street away. Terry intends for Chung to buy
her business based on : what she has stated. This is an example of:
unliteral mistake
negligent misrepresentation
fraudulent misrepresentation
mutual mistake

25. What does the concept “quantum meruit” mean?


The contract is terminated because of a breach by both parties
The innocent party would be unjustly enriched if they were able to retain the benefit :
without compensating the party in breach for the “amount he deserves”
Voluntarily accepting a contractual benefit
Not my deed

26. Cathy and Mel entered into a contract where Cathy was to steam clean
seven rooms in : Mel’s house at $200 per room. After cleaning three rooms,
Cathy abandoned the job for a : more valuable contract. Which of the
following is true?
As there has been substantial performance, Cathy will be able to enforce all the rights :
conferred by the contract
If the exact performance rule is applied, Cathy is entitled to payment for the work she
has : done
If the contract is divisible, Cathy is entitled to payment for the work she has done
Courts are inclined to treat contracts as indivisible and to require exact performance

27. Chang wants the court to imply an engineering custom into a term of a
contract to which : he is a party. The other party has no knowledge of this
custom. It is not contrary to any of : the express terms of the contract. The
court:
Will not imply the term in these circumstances
May imply the term
Will imply the term even if the custom is not very well known
Will not imply the term unless it is implied by statute

28. Jong complains that the barbecue he bought is not fit for purpose because
the metal : sides melted the first time he cooked with it. The manufacturer
maintains that Jong should : not have used the barbecue for slow cooking
and that it never represented that the : barbecue would be fit for the
purpose of slow cooking. What are the circumstances in which : the
guarantee as to fitness for purpose would apply?
Where Jong made known to the supplier that he was going to use the barbecue for slow
cooking meals
Where Jong bought the barbecue at auction
The guarantee always applies as it is not possible for a manufacturer or supplier to :
contract out of it
Where the unfitness for slow cooking of the barbecue was not drawn to Jong’s attention
: prior to his purchase by either the manufacturer or the supplier

29. Which of the following matters is least likely to contribute to a finding of


unconscionable : conduct in connection with goods or services?
Where conditions imposed on a customer were not necessary for the protection of the :
supplier’s interests
Where the supplier makes a commercial decision to breach the contract knowing that :
the customer will seek a legal remedy for the breach
Where the supplier acted in bad faith
Where unfair tactics were used against the customer

30. It is important to distinguish between representations and terms because:


A. Damages cannot be awarded for misrepresentation
B. Damages can only be awarded for fraudulent or negligent misrepresentation
C. There is no remedy for misrepresentation
D. The remedy of rescission is not available for misrepresentation

31. Which of the following is not an enforcement measure under the Australian
Consumer : Law?
A. An order of the court requiring a supplier to publish an advertisement
B. A disclosure order issued by the ACCC
C. A two-year probation order of the court
D. A notice issued by the ACCC requiring a supplier to substantiate a claim about
a product : they are selling

32. Which of the following is not an exception to privity?

A. Insurance
B. Agency and trust
C. Employment
D. Property law
33. James is going through a difficult time and does not feel he can confide in
anyone. : Eventually he confides in the priest of his church and asks for
support. The priest forcefully : tells James to turn over all of his money to
the church or he will be struck down by : lightening. The priest does not
believe that James will be struck down. James makes the : payments.
Which of the following is most likely legal grounds for James to avoid the :
transaction?
A. Mistake
B. Undue influence
C. Duress
D. Misrepresentation

34 In which High Court decision was it said: “[T]here are two relevant
circumstances in which : a breach of contract by one party may entitle the other
to terminate. The first is where the : obligation [is] essential … The second
relevant circumstance is where there has been a : sufficiently serious breach of a
non-essential term …we rest our decision in the appeal not : upon the ground of
breach of an essential obligation, but upon application of the doctrine :
respecting intermediate terms.”
A. Gumland Property Holdings Pty Ltd v Duffy Bros Fruit Market (Campbelltown)
Pty Ltd : (2008) 234 CLR 237
B. Koompahtoo Local Aboriginal Land Council v Sanpine Pty Ltd (2007) 233 CLR
115
C. Ankar Pty Ltd v National Westminster Finance (Australia) Ltd (1987) 162 CLR
549
D. Shevill v Builders Licensing Board (1982) 149 CLR 620

35. Which of the following are sources of illegality?


A. Statute
B. Statute and common law
C. Statute, common law and equity
D. Statute, common law and parties’ subjective views on morality

36. What is the main difference between the misleading or deceptive conduct
provisions in : the Trade Practices Act 1974 (Cth) and in the Australian Consumer
Law?
A. The Australian Consumer Law no longer refers to “trade or commerce”
B. The Australian Consumer Law refers to a body corporate
C. The Australian Consumer Law refers to a corporation
D. The Australian Consumer Law refers to a person
37. Non est factum means:
A. It is not [my] deed
B. It is not true
C. Mistake of fact
D. It is unknown

38. With respect to consumer guarantees, which of the following is not correct?
A. It is possible to exclude the consumer guarantees in very limited circumstances
B. Remedies for non-compliance with the consumer guarantees depend on
whether there : was a “major failure” or not
C. With respect to fitness for purpose, a “disclosed purpose” relates to the supplier
only –: the manufacturer may make representations to the consumer, but the
consumer cannot : disclose a purpose to the manufacturer and receive a
guarantee in return
D. A manufacturer does not need to provide repair facilities or spare parts after a
certain : period

39. Which of the following is not a type of contract illegal at common law on the
grounds of : public policy?
A. Contracts to oust the jurisdiction of the courts
B. Contracts to commit a tort
C. Contracts prejudicial to the public safety
D. Champertous contracts

40. The term “transparency” in unfair contracts means:


A. It is expressed in reasonably plain language that is readily available to the
affected party
B. It is easily interpretable by the court
C. It is easily interpretable by the industry
D. It is capable of only one meaning

41. Undue influence differs from duress in that:


A. With undue influence, the contract is not voidable but void
B. With undue influence, no unlawful act is required
C. Duress arises only within a closed list of special fiduciary relationships
D. With undue influence, actual physical violence is required

42. In Howe v Teefy (1927) 27 SR (NSW) 301 where a leased racehorse was
retaken three : months into a three-year lease:
A. The court was unable to assess the damages because they were uncertain and
there was : no evidence on which they could be assessed
B. The fact that assessment was difficult did not prevent a court from granting
damages : where the plaintiff had been deprived of something of value
C. Although there was a value attributable to the plaintiff’s loss, it was too remote
to be : recoverable
D. The court was unable to assess the damages because the process was difficult
and time : consuming

43. A written contract allows Joseph, the neighbour of Sonny, to take “as many
strawberries : as he likes each strawberry season”. This contract:
A. Is void for uncertainty
B. Is valid but unenforceable because it is too uncertain
C. Is valid as the parties’ intention is clear
D. Is voidable for uncertainty
44. Which of the following is not an equitable remedy?
A. Injunction
B. Specific performance
C. Restitution
D. Exemplary damages
45. Which of the following matters is least likely to contribute to a finding of
unconscionable : conduct in connection with goods or services?

A. Where conditions imposed on a customer were not necessary for the protection of
the : supplier’s interests
B. Where the supplier makes a commercial decision to breach the contract knowing
that : the customer will seek a legal remedy for the breach
C. Where the supplier acted in bad faith
D. Where unfair tactics were used against the customer

46. Consumer guarantees as to title are contained within:

A. The “General Protections” chapter of the Australian Consumer Law


B. Section 38 of the Trade Practices Act 1974 (Cth)
C. Chapter 2 of the Australian Consumer Law
D. Sections 51-56 and ss 60-61 of the Australian Consumer Law
47. Which of the following is not one of the four ways a statute may render a contract
illegal : set out in Yango Pastoral Co v First Chicago Australia Ltd (1978) 139 CLR 410?
The contract:

A. May require an action that statute forbids


B. May be performed in a prohibited manner
C. May have been made to effect an unlawful purpose
D. May set out an unreasonable restraint

48. Which of the following is not correct with respect to the Australian Consumer Law?

A. It is contained within Schedule 2 to the Competition and Consumer Act 2010


(Cth)
B. It applies to conduct engaged in outside of Australia
C. Owing to the Commonwealth’s limited lawmaking powers under s 51 of the
:Constitution, it applies only to corporations
D. The Australian Consumer Law is applied in each State and Territory

49. Which of the following is not one of the six elements required for a claim of
fraudulent : misrepresentation to succeed?

A. The representation must be fall


B. The representation must have been acted upon by the other party
C. The representation must be one of fact
D. The representation must be in writing (cannot be verbal only)

50. Which of the following is not a way that a person seeking to rely on an exemption
clause : can show that the clause has become part of the contract?

A. By showing it is in writing and the other party has signed the contract
B. By showing it is in writing in a document a reasonable person would expect to
contain : contractual terms and was brought to the other party’s notice
C. By showing that the parties had previously contracted on terms that included the :
exemption clause and that they intended to contract on the same basis
Correct!

D. By showing it was brought to the notice of the other party at the time of, or
subsequent : to, entering into the contract

51. Ben planned the perfect surprise birthday party for his mother: He hired a hall, he
hired : a violinist and he organised catering. Two weeks prior to the event, the violinist
called to : cancel. Ben was so upset at the loss of what he thought would be the highlight
of the : evening that he called the whole event off. Ben lost both the deposit on the hall
and the : deposit for the catering. Which of the following is correct?

A. Ben can sue the violinist for all losses after Ben cancelled the hiring of the hall
B. Ben can recover his hire fees and catering fees as it was not his fault
C. The caterers and hall owners can sue the violinist for losses
D. Ben has a duty to mitigate his losses

52. Sandy is selling her horse, Flossy. Miranda is interested in buying Flossy. Sandy tells :
Miranda that Flossy: : • Is the best little racehorse in Australia: • Has all the
registrations, licences and permits required to race in the current season but : that
Miranda should probably get this checked herself: These two statements are:

A. Mere puff & term


B. Opinion & term
C. Opinion & representation
D. Mere puff & representation

53. What does the concept “quantum meruit” mean?

A. The contract is terminated because of a breach by both parties


B. The innocent party would be unjustly enriched if they were able to retain the
benefit : without compensating the party in breach for the “amount he deserves”
C. Voluntarily accepting a contractual benefit
D. Not my deed
54. The term “transparency” in unfair contracts means:

A. It is expressed in reasonably plain language that is readily available to the affected


party
B. It is easily interpretable by the court
C. It is easily interpretable by the industry
D. It is capable of only one meaning

55. Can a person use an official position they hold for their financial advantage?

A. Of course, this is a side benefit


B. Depends on the circumstances
C. The courts are silent on this specific point
D. No because if a contract is involved, this could lead to the promoting of corruption
in : public life

56. Smithy Builders have a contract with Big Bank Pty Ltd which contains the following :
clause: “Where Smithy Builders fails to complete the contract by 7 December, it will pay
a : sum of $300,000 in full and final satisfaction of its liability.” Smithy Builders fails to
complete : by 7 December. If Smithy Builders wishes to avoid paying the sum of
$300,000, what must it : prove?

A. That the clause is an unliquidated damages clause


B. That the clause is a penalty clause (to punish the builders)
C. That the clause is a liquidated damages clause
D. That the clause is a genuine pre-estimate of the loss to Big Bank Pty Ltd

57. Which of the following is not correct? An action in restitution


A. Is often brought where one party has been unjustly enriched at the expense of the
other
B. Cannot be brought where there is no contract between the parties
C. Can be brought where the contract between the parties is unenforceable

D. Can be brought where there has been only part performance

58. Which of the following is correct?

A. It is prohibited to offer free gifts with purchase


B. It is prohibited to offer goods at a special price, not have a sufficient amount of
those : goods available and then attempt to upsell to customers who are drawn to
the shop for the : special price deal
C. It is not prohibited for a supplier to take a customer’s money when the supplier
believes : they won’t be able to supply the service to the customer within a
reasonable time
D. It is not prohibited for a bank to send an unsolicited credit card to its client; it is :
prohibited for a bank to send an unsolicited debit card to a person who is a
potential client : but not a yet a client

59. Which of the following is not true? A signature on a contract containing an


exemption : clause:
A. Will not guarantee the effectiveness of the clause where the doctrine of non est
factum : applies
B. Incorporates the exemption clause into the contract
C. Guarantees that an exemption clause will be effective
D. Will be effective even if one party did not read the contract

QUIZ 3:
To claim damages a causal link must be established between what two things?
A. Between the defendant’s conduct and the actual monetary value of the
loss/damage
B. The conduct and initiating court proceedings
C. The fact that the plaintiff suffered loss or damage is sufficient. Nothing else is
required
D. The defendants breach and the plaintiff’s injury
The “neighbour principle” is often viewed as:
A. An objective test
B. A test of the reasonable person
C. A test of the foreseeability of harm
D. An identity test
In Tame v State of New South Wales (2002) 211 CLR 317 where Tame was given a
false : blood alcohol reading:
A. Whether the police officer was found to have owed Tame a duty of care was
entirely a : question of community standards
B. The psychotic depressive illness Tame developed was found to flow from the
breach of : duty of the police sergeant because it was entirely caused by his
actions that could have : been avoided had he exercised sufficient care
C. Part of the test of reasonable foreseeability is a question of fact
D. Because of the “eggshell skull” principle, pre-existing knowledge of Tame’s
susceptibility : to suffering nervous shock was not required to prove a breach
Which of the following is not correct?
A. If damages are too remote they will not be recoverable
B. The remoteness test will be satisfied where the damage suffered is of the same
type or : kind as foreseeable damage
C. In order for damage to not be too remote, it must be reasonably foreseeable
D. In The Wagon Mound No 1 and The Wagon Mound No 2, where there was an :
unfortunate combination of an oil spill, welding sparks and floating cotton waste,
the loss : was found to be not reasonably foreseeable in the circumstances
Which of the following is not a provision of the Wrongs Act 1958 (Vic) relating to:
professional liability?
A. If there are differing peer professional opinions across Australia, the court may
accept : just one of those opinions
B. The court must rely on peer professional opinion even where it considers that
opinion : irrational
C. If there are differing peer professional opinions across Australia, the court may
accept all : of those opinions
D. Peer professional opinion can be considered widely accepted even where it is
not : universally accepted
Why was the plaintiff successful in Overseas Tankship (UK) Ltd v Miller
Steamship Co Pty : Ltd (The Wagon Mound No 2) [1967] AC 617 (PC) when
another plaintiff failed in the earlier : related Wagon Mound case?
A. The plaintiffs in the second had better legal representation
B. The legal situation had changed between the two cases
C. The court was more disposed to think about the consequences of the damage
suffered by : so many as a result of the oil spill
D. In the first case, the plaintiffs failed to show that a reasonable man would have
foreseen : the risk of damage from the oil spill
Nicola drives the forklift at her place of work, Rooze’s Roofing. Nicola always
leaves the : forklift in a certain place where she has been told to leave it, with the
forks up off the : ground. One afternoon a customer who is collecting goods from
the workshop reverses his : car into the forks on the forklift. He is injured and his
car is damaged.: What is the principle that would make Nicola’s employer liable
for her actions?
A. Contributory negligence
B. Strict liability
C. Vicarious liability
D. Voluntary assumption of risk
Where a person gives advice, that advice is relied upon and the advice is
incorrect, the : person giving the advice may be liable in negligence. Which of the
following is incorrect?
A. The “special relationship” described in Mutual Life & Citizens’ Assurance Co Ltd
v Evatt : (1968) 122 CLR 556 has been accepted as the test for determining the
existence of a duty of : care with respect to negligent misstatements
B. The test in Mutual Life & Citizens’ Assurance Co Ltd v Evatt (1968) 122 CLR 556
was : approved in San Sebastian Pty Ltd v Minister Administering Environmental
Planning &: Assessment Act 1979 (1986) 162 CLR 340
C. The duty of care only arises where the “special relationship” involves a person
seeking : advice from the advice-giver, not where the advice was given
unrequested and merely : accepted
D. The advice-giver does not need to be in the business of giving advice
Which of the following is not correct? The reform to the law of negligence that
took : place in Australia in the early 2000s:
A. Limits the scope of potential liability for negligence
B. Does not apply to claims in contract law
C. Was enacted in all Australian states and territories
D. Covers personal injury
In Australian Safeway Stores v Zaluzna (1987) 162 CLR 479:
A. The respondent was a lawful entrant upon the land of the respondent,
establishing a : relationship between them and there was therefore a duty of care
owed by the appellant to : avoid a foreseeable risk of injury
B. The respondent was limited by his particular status as entrant without specific
consent of : the appellant
C. The respondent contributed to the accident thus limiting damages
D. There was no liability.
Where a person gives advice, that advice is relied upon and the advice is incorrect, the :
person giving the advice may be liable in negligence. Which of the following is incorrect?
A. The “special relationship” described in Mutual Life & Citizens’ Assurance Co Ltd
v Evatt : (1968) 122 CLR 556 has been accepted as the test for determining the
existence of a duty of : care with respect to negligent misstatements
B. The test in Mutual Life & Citizens’ Assurance Co Ltd v Evatt (1968) 122 CLR 556
was : approved in San Sebastian Pty Ltd v Minister Administering Environmental
Planning &: Assessment Act 1979 (1986) 162 CLR 340
C. The duty of care only arises where the “special relationship” involves a
person seeking : advice from the advice-giver, not where the advice was
given unrequested and merely : accepted
D. The advice-giver does not need to be in the business of giving advice
Which of the following is not correct? The reform to the law of negligence that took :
place in Australia in the early 2000s:
A. Limits the scope of potential liability for negligence
B. Does not apply to claims in contract law
C. Was enacted in all Australian states and territories
D. Covers personal injury
Nicola drives the forklift at her place of work, Rooze’s Roofing. Nicola always leaves the
: forklift in a certain place where she has been told to leave it, with the forks up off the :
ground. One afternoon a customer who is collecting goods from the workshop reverses
his : car into the forks on the forklift. He is injured and his car is damaged.: What is the
principle that would make Nicola’s employer liable for her actions?
A. Contributory negligence
B. Strict liability
C. Vicarious liability
D. Voluntary assumption of risk
In Australian Safeway Stores v Zaluzna (1987) 162 CLR 479:
A. The respondent was a lawful entrant upon the land of the respondent,
establishing a : relationship between them and there was therefore a duty of care
owed by the appellant to : avoid a foreseeable risk of injury
B. The respondent was limited by his particular status as entrant without specific
consent of : the appellant
C. The respondent contributed to the accident thus limiting damages
D. There was no liability.
Which of the following is not a provision of the Wrongs Act 1958 (Vic) relating to:
professional liability?
A. If there are differing peer professional opinions across Australia, the court may
accept : just one of those opinions
B. The court must rely on peer professional opinion even where it considers that
opinion : irrational
C. If there are differing peer professional opinions across Australia, the court may
accept all : of those opinions
D. Peer professional opinion can be considered widely accepted even where it is not :
universally accepted
To claim damages a causal link must be established between what two things?
A. Between the defendant’s conduct and the actual monetary value of the
loss/damage
B. The conduct and initiating court proceedings
C. The fact that the plaintiff suffered loss or damage is sufficient. Nothing else is
required
D. The defendants breach and the plaintiff’s injury
Which of the following is not correct?
A. If damages are too remote they will not be recoverable
B. The remoteness test will be satisfied where the damage suffered is of the same
type or : kind as foreseeable damage
C. In order for damage to not be too remote, it must be reasonably foreseeable
D. In The Wagon Mound No 1 and The Wagon Mound No 2, where there was an :
unfortunate combination of an oil spill, welding sparks and floating cotton waste,
the loss : was found to be not reasonably foreseeable in the circumstances
The “neighbour principle” is often viewed as:
A. An objective test
B. A test of the reasonable person
C. A test of the foreseeability of harm
D. An identity test
Why was the plaintiff successful in Overseas Tankship (UK) Ltd v Miller Steamship Co
Pty : Ltd (The Wagon Mound No 2) [1967] AC 617 (PC) when another plaintiff failed in
the earlier : related Wagon Mound case?
A. The plaintiffs in the second had better legal representation
B. The legal situation had changed between the two cases
C. The court was more disposed to think about the consequences of the damage
suffered by : so many as a result of the oil spill
D. In the first case, the plaintiffs failed to show that a reasonable man would have
foreseen : the risk of damage from the oil spill
In Tame v State of New South Wales (2002) 211 CLR 317 where Tame was given a false :
blood alcohol reading:
A. Whether the police officer was found to have owed Tame a duty of care was
entirely a : question of community standards
B. The psychotic depressive illness Tame developed was found to flow from the
breach of : duty of the police sergeant because it was entirely caused by his actions
that could have : been avoided had he exercised sufficient care
C. Part of the test of reasonable foreseeability is a question of fact
D. Because of the “eggshell skull” principle, pre-existing knowledge of Tame’s
susceptibility : to suffering nervous shock was not required to prove a breach
In Australian Safeway Stores v Zaluzna (1987) 162 CLR 479:
A. The respondent was a lawful entrant upon the land of the respondent,
establishing a : relationship between them and there was therefore a duty of care
owed by the appellant to : avoid a foreseeable risk of injury
B. The respondent was limited by his particular status as entrant without specific
consent of : the appellant
C. The respondent contributed to the accident thus limiting damages
D. There was no liability.
The “neighbour principle” is often viewed as:
A. An objective test
B. A test of the reasonable person
C. A test of the foreseeability of harm
D. An identity test
Which of the following is not correct?
A. if damages are too remote they will not be recoverable
B. The remoteness test will be satisfied where the damage suffered is of the same
type or : kind as foreseeable damage
C. In order for damage to not be too remote, it must be reasonably foreseeable
D. In The Wagon Mound No 1 and The Wagon Mound No 2, where there was an : unfortunate
combination of an oil spill, welding sparks and floating cotton waste, the loss : was found to
be not reasonably foreseeable in the circumstances

Where a person gives advice, that advice is relied upon and the advice is incorrect, the :
person giving the advice may be liable in negligence. Which of the following is incorrect?
A. The “special relationship” described in Mutual Life & Citizens’ Assurance Co Ltd v
Evatt : (1968) 122 CLR 556 has been accepted as the test for determining the
existence of a duty of : care with respect to negligent misstatements
B. The test in Mutual Life & Citizens’ Assurance Co Ltd v Evatt (1968) 122 CLR 556
was : approved in San Sebastian Pty Ltd v Minister Administering Environmental
Planning &: Assessment Act 1979 (1986) 162 CLR 340
C. The duty of care only arises where the “special relationship” involves a person
seeking : advice from the advice-giver, not where the advice was given
unrequested and merely : accepted
D. The advice-giver does not need to be in the business of giving advice

To claim damages a causal link must be established between what two things?
A. Between the defendant’s conduct and the actual monetary value of the
loss/damage
B. The conduct and initiating court proceedings
C. The fact that the plaintiff suffered loss or damage is sufficient. Nothing else is
required
D. The defendants breach and the plaintiff’s injury

Nicola drives the forklift at her place of work, Rooze’s Roofing. Nicola always leaves the
: forklift in a certain place where she has been told to leave it, with the forks up off the :
ground. One afternoon a customer who is collecting goods from the workshop reverses
his : car into the forks on the forklift. He is injured and his car is damaged.: What is the
principle that would make Nicola’s employer liable for her actions?
A. Contributory negligence
B. Strict liability
C. Vicarious liability
D. Voluntary assumption of risk

In Tame v State of New South Wales (2002) 211 CLR 317 where Tame was given a false :
blood alcohol reading:
A. Whether the police officer was found to have owed Tame a duty of care was
entirely a : question of community standards
B. The psychotic depressive illness Tame developed was found to flow from the
breach of : duty of the police sergeant because it was entirely caused by his actions
that could have : been avoided had he exercised sufficient care
C. Part of the test of reasonable foreseeability is a question of fact
D. Because of the “eggshell skull” principle, pre-existing knowledge of Tame’s
susceptibility : to suffering nervous shock was not required to prove a breach

Which of the following is not a provision of the Wrongs Act 1958 (Vic) relating to:
professional liability?
A. If there are differing peer professional opinions across Australia, the court may
accept : just one of those opinions
B. The court must rely on peer professional opinion even where it considers that
opinion : irrational
C. If there are differing peer professional opinions across Australia, the court may
accept all : of those opinions
D. Peer professional opinion can be considered widely accepted even where it is not :
universally accepted

Which of the following is not correct? The reform to the law of negligence that took :
place in Australia in the early 2000s:

A. Limits the scope of potential liability for negligence


B. Does not apply to claims in contract law
C. Was enacted in all Australian states and territories
D. Covers personal injury

Why was the plaintiff successful in Overseas Tankship (UK) Ltd v Miller Steamship Co
Pty : Ltd (The Wagon Mound No 2) [1967] AC 617 (PC) when another plaintiff failed in
the earlier : related Wagon Mound case?

A. The plaintiffs in the second had better legal representation


B. The legal situation had changed between the two cases
C. The court was more disposed to think about the consequences of the damage
suffered by : so many as a result of the oil spill
D. In the first case, the plaintiffs failed to show that a reasonable man would have
foreseen : the risk of damage from the oil spill

The “neighbour principle” is often viewed as:

A. An objective test
B. A test of the reasonable person
C. A test of the foreseeability of harm
D. An identity test

Which of the following is not correct?


A. If damages are too remote they will not be recoverable
B. The remoteness test will be satisfied where the damage suffered is of the same
type or : kind as foreseeable damage
C. In order for damage to not be too remote, it must be reasonably foreseeable
D. In The Wagon Mound No 1 and The Wagon Mound No 2, where there was an :
unfortunate combination of an oil spill, welding sparks and floating cotton waste,
the loss : was found to be not reasonably foreseeable in the circumstances

Which of the following is not correct? The reform to the law of negligence that took :
place in Australia in the early 2000s:

A. Limits the scope of potential liability for negligence


B. Does not apply to claims in contract law
C. Was enacted in all Australian states and territories
D. Covers personal injury

In Australian Safeway Stores v Zaluzna (1987) 162 CLR 479:


A. The respondent was a lawful entrant upon the land of the respondent, establishing
a : relationship between them and there was therefore a duty of care owed by the
appellant to : avoid a foreseeable risk of injury
B. The respondent was limited by his particular status as entrant without specific
consent of : the appellant
C. The respondent contributed to the accident thus limiting damages
D. There was no liability.

To claim damages a causal link must be established between what two things?

A. Between the defendant’s conduct and the actual monetary value of the
loss/damage
B. The conduct and initiating court proceedings
C. The fact that the plaintiff suffered loss or damage is sufficient. Nothing else is
required
D. The defendants breach and the plaintiff’s injury

In Tame v State of New South Wales (2002) 211 CLR 317 where Tame was given a false :
blood alcohol reading:
A. Whether the police officer was found to have owed Tame a duty of care was
entirely a : question of community standards
B. The psychotic depressive illness Tame developed was found to flow from the
breach of : duty of the police sergeant because it was entirely caused by his actions
that could have : been avoided had he exercised sufficient care
C. Part of the test of reasonable foreseeability is a question of fact
D. Because of the “eggshell skull” principle, pre-existing knowledge of Tame’s
susceptibility : to suffering nervous shock was not required to prove a breach

Where a person gives advice, that advice is relied upon and the advice is incorrect, the :
person giving the advice may be liable in negligence. Which of the following is incorrect?

A. The “special relationship” described in Mutual Life & Citizens’ Assurance Co Ltd v
Evatt : (1968) 122 CLR 556 has been accepted as the test for determining the
existence of a duty of : care with respect to negligent misstatements
B. The test in Mutual Life & Citizens’ Assurance Co Ltd v Evatt (1968) 122 CLR 556
was : approved in San Sebastian Pty Ltd v Minister Administering Environmental
Planning &: Assessment Act 1979 (1986) 162 CLR 340
C. The duty of care only arises where the “special relationship” involves a person
seeking : advice from the advice-giver, not where the advice was given unrequested
and merely : accepted
D. The advice-giver does not need to be in the business of giving advice

Nicola drives the forklift at her place of work, Rooze’s Roofing. Nicola always leaves the
: forklift in a certain place where she has been told to leave it, with the forks up off the :
ground. One afternoon a customer who is collecting goods from the workshop reverses
his : car into the forks on the forklift. He is injured and his car is damaged.: What is the
principle that would make Nicola’s employer liable for her actions?

A. Contributory negligence
B. Strict liability
C. Vicarious liability
D. Voluntary assumption of risk

Which of the following is not a provision of the Wrongs Act 1958 (Vic) relating to:
professional liability?
A. If there are differing peer professional opinions across Australia, the court may
accept : just one of those opinions
B. The court must rely on peer professional opinion even where it considers that
opinion : irrational
C. If there are differing peer professional opinions across Australia, the court may
accept all : of those opinions
D. Peer professional opinion can be considered widely accepted even where it is not :
universally accepted

QUIZ 4:
The incorrect statement?
A. Partners are not bound when another partner misappropriates monies
B. If the giving of investment advice is within the scope of the firm’s business then
all partners : are jointly and severally liable for advice which is not in a client’s
best interests
C. the client should seek a personal indemnity from a partner
D. special skill is required to bind all partners
Which of the below statements is incorrect?
A. a written and signed notice of a partner’s intention to dissolve the partnership is
required, : where no fixed term has been agreed upon for the duration of the
partnership
B. a continuing guarantee given by or to a partnership is revoked as to future
transactions if there is : a change in the constitution of the partnership
C. where a partnership continues after a fixed term has expired, the rights and
duties of the partners : remain the same, but the partnership becomes a
partnership at will
D. partnership property must be used exclusively for the purposes of the
partnership and in the : manner set out in the partnership agreement
Which legislation controls the actual registration of the firm name?
A. Corporations Act 2001 (Cth)
B. Partnership Act
C. Statute of Frauds 1677
D. Business Names Registration Act 2011 (Cth)
Which of the following statements regarding partnerships is incorrect?
A. The second element of a partnership is carrying out a business in common
B. To be a partnership there must be a mutuality of rights and obligations
C. Each partner must take an active part in the direction and management of the
firm
D. In Degiorgio v Dunn [2004] NSWSC 767 it was held that there was no
partnership because the : business was not run “in common”
Which of the following statements regarding dissolution of partnership is
incorrect?
A. In the event of dissolution, losses must be met first out of profits, followed by
capital, then by : partners in proportion to their share of profits
B. A firm’s assets as contributed by partners to make up capital losses need not be
applied to pay : the firm’s debts/liabilities to non-partners
C. After dissolution, each partner is entitled to advances and residue by the firm
D. After the dissolution, each partner’s authority to bind the firm continues, so far
as necessary for : winding up partnership affairs and completing unfinished
transactions
Which is incorrect statement?
A. If a member of a firm of solicitors acting for a vendor in a sale absconds with the
deposit, : his/her partners are not liable to refund the money
B. In Lloyd v Grace, Smith & Co [1912] AC 716, the firm was held responsible for
the fraud committed : by a managing clerk of a firm, who misappropriated
property while acting within the scope of his : authority
C. In SJ Mackie Pty Ltd v Dalziell Medical Practice Pty Ltd [1989] 2 Qd it was held
that the transfer of a : share to a non-partner breaks the continuity of the firm,
constituting a new firm/partnership of the : remaining former partners and the
new member
D. Partnership agreements can contain provisions to enable the transition from
one firm to another : to be effected without the disruption of a formal winding up.
Which of the following statements is not correct?
A. as an agent, a partner is able to bind the other partners and, as principal, be
bound by the actions : of the other partners
B. in New South Wales, Victoria, Queensland, South Australia, Western Australia
and: Tasmania provision for limited partnerships is made in the Partnership Act
C. incorporated limited partnerships have been introduced in all Australian States
and Territories
D. the Partnership Act provides that the rules of the common law and equity are to
continue in force : except insofar as they are inconsistent with the Act
Find one incorrect statement?
A. In the absence of special statutory provision, although each partner is liable with
the others for : the whole of the debts of the firm, their liability is only joint
B. A creditor can bring only one action against members of a partnership and any
partner can insist : that the action be stayed until all other partners are joined as
parties.
C. A person admitted into an existing firm, liability may be incurred where it is
specially agreed upon
D. A creditor can enforce liability against an incoming partner whether or not
he/she is a party to : the contract
What constitutes an exception to the usual number of a partnership being 20
partners?
A. If one of the partners is married-his/her husband or wife is automatically a
partner
B. The exception relates only to limited partnerships
C. No more than 20 are allowed
D. The Corporations Regulations 2001 (Cth) provide greater numbers in certain
specified : professions
Which statement regarding an outsize partnership is incorrect?
A. it has more than 20 partners
B. its partnership agreement is invalid
C. it is liable to a criminal penalty ($500)
D. its agreement does not affect the enforceability of contracts or other
arrangements made
Which finding is correct?: In Popat v Schonchhatra (1997) 3 All ER 800, the Court
decided:
A. Popat was entitled to half the profits on the sale of the business and to a share
of the profits : that had accrued after the dissolution of the partnership, but before
the final settlement of : accounts.
B. Popat was not entitled to any profits on the sale of the business, as there was
no partnership : agreement and he was further not entitled to a share of the
profits accrued after the dissolution of : the partnership, but before the final
settlement of accounts.
C. Popat was entitled to half the profits on the sale of the business, but not to a
share of the profits : that had accrued after the dissolution of the partnership, but
before the final settlement of : accounts.
D. Popat was not entitled to half the profits on the sale of the business, but he was
entitled to a : share of the profits that had accrued after the dissolution of the
partnership but before the final : settlement of accounts.
Which of the following statements is incorrect?
A. a separate legal entity can commit civil wrongs and engage in criminal conduct
B. a public company has serious compliance obligations
C. a partnership must be in writing
D. a separate legal entity is entitled to own property, pay tax and enter into
contracts
Which of the below does not apply to the Partnership Act?
A. The rules in the Partnership Acts assist in determining whether a business is
being carried on in : common
B. The partnership under the Act must be registered with ASIC
C. Each rule states a negative: i.e. that a certain fact does not of itself create a
partnership, but is : only indicative of a partnership
D. Joint or part ownership or joint tenancy, or tenancy in common whether or not
the : owners/tenants share the profits, does not of itself create a partnership as to
anything so : held/owned
Which of the following statements regarding the written law is incorrect?
A. Under s 115 of the Corporations Act 2001 (Cth), the maximum number of
persons who may form a : partnership for the acquisition of gain is 20
B. Under the Corporations Regulations 2001 (Cth), reg 2A.1.01, partnerships of
more than 20 : partners may be formed for certain professions/callings
C. The Corporations Regulations 2001 (Cth), reg 2A.1.01 sets a maxima of
partners for medical and : legal practitioners, veterinary surgeons, patent and
trademark attorneys, sharebrokers and : stockbrokers and pharmaceutical
chemists
D. The Corporations Regulations 2001 (Cth), reg 2A.1.01 does not set a maxima
of partners for : architects and accountants
Which court finding is incorrect?
A. In Mercantile Credit Co Ltd v Garrod [1962] 3 All ER 1103, damages were not
recovered even : though, from the plaintiff’s perspective, the sale of the car was
within the usual course of : business.
B. In Goldberg v Jenkins (1889) 15 VLR 36, the firm was not bound to the
transaction because : borrowing money on behalf of the firm at over 60% interest
when comparable rates were between : 6% and 10% was beyond ‘the usual
way’.
C. In Construction Engineering Pty Ltd v Hexyl Pty Ltd (1985) 155 CLR 541, Hexyl
was not liable : because the construction contract appeared to be between
Construction Engineering and Tembel
D. If judgment is obtained against one or more partners of a firm, no action may be
taken against the : other partners, even if satisfaction cannot be obtained from
the partner(s) sued
Cribb v Korn (1911) 12 CLR 205 established:
A. Joint ownership creates a partnership
B. A joint tenancy is just another name for a partnership
C. The sharing of joint returns does not in itself create a partnership
D. The statutory rules can be excluded by agreement
The incorrect statement is?
A. A partner has express actual and implied actual authority to engage in certain
activities with third: parties
B. A partner cannot pledge/sell partnership property, incur and pay debts on
partnership : accounts or hire employees
C. Contravention of an agreement to restrict a partner’s authority is not binding on
the firm if notice : of the agreement has been given
D. In Polkinghorne v Holland (1934) 51 CLR 143, the firm was held liable because
Holland provided : the advice in his role as solicitor; thus in the ordinary course of
the firm’s business
Find one incorrect statement?
A. In the absence of special statutory provision, although each partner is liable with
the others for : the whole of the debts of the firm, their liability is only joint
B. A creditor can bring only one action against members of a partnership and any
partner can insist : that the action be stayed until all other partners are joined as
parties.
C. A person admitted into an existing firm, liability may be incurred where it is
specially agreed upon
D. A creditor can enforce liability against an incoming partner whether or not
he/she is a party to : the contract
Regarding partnerships, which of the following statements is incorrect?
A. Partnerships are created with a view to profit, so partners must make a profit: b A
partner does not have to have a direct claim to a share of the profits
B. Associations and charities are not partnerships, as profits come from ancillary
business activities : and are reinvested, not distributed as dividends to their
members
C. The Partnership Acts do not govern members of a corporation incorporated under
the provisions : of the Corporations Act 2001 (Cth), a special Act of Parliament, or
Royal Charter
Which of the below statements is incorrect?
A. a written and signed notice of a partner’s intention to dissolve the partnership is
required, : where no fixed term has been agreed upon for the duration of the
partnership
B. a continuing guarantee given by or to a partnership is revoked as to future
transactions if there is : a change in the constitution of the partnership
C. where a partnership continues after a fixed term has expired, the rights and duties
of the partners : remain the same, but the partnership becomes a partnership at
will
D. partnership property must be used exclusively for the purposes of the partnership
and in the : manner set out in the partnership agreement
Which of the following statements is incorrect?
A. a separate legal entity can commit civil wrongs and engage in criminal conduct
B. a public company has serious compliance obligations
C. a partnership must be in writing
D. a separate legal entity is entitled to own property, pay tax and enter into contracts
The incorrect statement?
A. Partners are not bound when another partner misappropriates monies
B. If the giving of investment advice is within the scope of the firm’s business then
all partners : are jointly and severally liable for advice which is not in a client’s
best interests
C. the client should seek a personal indemnity from a partner
D. special skill is required to bind all partners
Which of the following statements is not correct?
A. as an agent, a partner is able to bind the other partners and, as principal, be bound
by the actions : of the other partners
B. in New South Wales, Victoria, Queensland, South Australia, Western Australia and:
Tasmania provision for limited partnerships is made in the Partnership Act
C. incorporated limited partnerships have been introduced in all Australian States
and Territories
D. the Partnership Act provides that the rules of the common law and equity are to
continue in force : except insofar as they are inconsistent with the Act
Which of the below statement is incorrect?
A. Partnership property is liable to be seized for the private (personal) debt of
a partner and : made liable on a judgment against the partnership
B. A creditor who has obtained judgment in respect of the separate debt of a partner
may obtain an : order charging that partner’s interest in the partnership property
and profits with the amount of the : debt and interest.
C. A creditor who has obtained judgment in respect of the separate debt of a partner
may obtain by : an order the appointment of a receiver of that partner’s share of
profits and of any other money : which may be coming to the partner in respect of
the partnership.
D. The two sources of law relevant when considering how partners bind their
partners when dealing : with third parties are the common law (including equity)
of agency and the Partnership Act. The : common law complements the
provisions of the Partnership Act in relation to the authority that an : agent has.
What constitutes an exception to the usual number of a partnership being 20 partners?
A. If one of the partners is married-his/her husband or wife is automatically a partner
B. The exception relates only to limited partnerships
C. No more than 20 are allowed
D. The Corporations Regulations 2001 (Cth) provide greater numbers in certain
specified : professions
Which of the following statements regarding partnerships is incorrect?
A. The second element of a partnership is carrying out a business in common
B. To be a partnership there must be a mutuality of rights and obligations
C. Each partner must take an active part in the direction and management of
the firm
D. In Degiorgio v Dunn [2004] NSWSC 767 it was held that there was no
partnership because the : business was not run “in common”
Which is incorrect statement?
A. If a member of a firm of solicitors acting for a vendor in a sale absconds with the
deposit, : his/her partners are not liable to refund the money
B. In Lloyd v Grace, Smith & Co [1912] AC 716, the firm was held responsible for the
fraud committed : by a managing clerk of a firm, who misappropriated property
while acting within the scope of his : authority
C. In SJ Mackie Pty Ltd v Dalziell Medical Practice Pty Ltd [1989] 2 Qd it was held that
the transfer of a : share to a non-partner breaks the continuity of the firm,
constituting a new firm/partnership of the : remaining former partners and the new
member
D. Partnership agreements can contain provisions to enable the transition from one
firm to another : to be effected without the disruption of a formal winding up.
Which statement is incorrect?
A. carrying on a business implies repetition (Smith v Anderson (1880) 15 Ch D 247)
B. partnership may be formed in order to undertake a single business transaction
C. in Khan v Miah [2000] 1 WLR 2123 it was held that work, such as finding, acquiring
and fitting out : a shop/restaurant, is undertaken with a view to profit
D. in Keith Spicer Ltd v Mansell [1970] 1 All ER 462 it was held that ordering goods
and opening a : joint bank account in contemplation of a business are insufficient
for a partnership
Which court finding is incorrect?
A. In Mercantile Credit Co Ltd v Garrod [1962] 3 All ER 1103, damages were not
recovered even : though, from the plaintiff’s perspective, the sale of the car was
within the usual course of : business.
B. In Goldberg v Jenkins (1889) 15 VLR 36, the firm was not bound to the transaction
because : borrowing money on behalf of the firm at over 60% interest when
comparable rates were between : 6% and 10% was beyond ‘the usual way’.
C. In Construction Engineering Pty Ltd v Hexyl Pty Ltd (1985) 155 CLR 541, Hexyl
was not liable : because the construction contract appeared to be between
Construction Engineering and Tembel
D. If judgment is obtained against one or more partners of a firm, no action may be
taken against the : other partners, even if satisfaction cannot be obtained from the
partner(s) sued
Find incorrect statement
A. A term often included in a partnership agreement that allows other partners to
purchase a : retiring/deceased partner’s interest at an agreed valuation avoids the
disruption of a formal winding
B. A partnership cannot be dissolved because the business is carried on at a loss
C. Partnerships can be dissolved because of mutual incompatibility, making it
impossible for partners : to carry on a business
D. On dissolution, partnership property can be applied towards the payment of
partnership : liabilities/debts and any surplus can be distributed among the
partners

Which below is incorrect? A firm is not bound by the acts of a partner if:

A. the act is not of the usual business of the kind carried on by the firm
B. the partner exceeds his/her actual authority in the particular matter
C. the person with whom the partner is dealing knows that he/she has exceeded
his/her authority
D. the person with whom the partner is dealing knows or believes him/her to be a
partner

Find one incorrect statement?

A. In the absence of special statutory provision, although each partner is liable with
the others for : the whole of the debts of the firm, their liability is only joint
B. A creditor can bring only one action against members of a partnership and any
partner can insist : that the action be stayed until all other partners are joined as
parties.
C. A person admitted into an existing firm, liability may be incurred where it is
specially agreed upon
D. A creditor can enforce liability against an incoming partner whether or not
he/she is a party to : the contract
Legislation in Victoria: What Act regulates partnerships in Victoria?

A. Partnership Act 1892


B. Partnership Act 1958 (Vic)
C. Corporations Act 2001 (Cth), s 20
D. Competition & Consumer Act 2010 (Cth)

Which statement regarding an outsize partnership is incorrect?

A. it has more than 20 partners


B. its partnership agreement is invalid
C. it is liable to a criminal penalty ($500)
D. its agreement does not affect the enforceability of contracts or other
arrangements made

Which of the following statements is incorrect?

A. a separate legal entity can commit civil wrongs and engage in criminal conduct
B. a public company has serious compliance obligations
C. a partnership must be in writing
D. a separate legal entity is entitled to own property, pay tax and enter into contracts

Which court finding is incorrect?

A. In Mercantile Credit Co Ltd v Garrod [1962] 3 All ER 1103, damages were not
recovered even : though, from the plaintiff’s perspective, the sale of the car was
within the usual course of : business.
B. In Goldberg v Jenkins (1889) 15 VLR 36, the firm was not bound to the transaction
because : borrowing money on behalf of the firm at over 60% interest when
comparable rates were between : 6% and 10% was beyond ‘the usual way’.
C. In Construction Engineering Pty Ltd v Hexyl Pty Ltd (1985) 155 CLR 541, Hexyl
was not liable : because the construction contract appeared to be between
Construction Engineering and Tembel
D. If judgment is obtained against one or more partners of a firm, no action may be
taken against the : other partners, even if satisfaction cannot be obtained from the
partner(s) sued

Which statement regarding persons of unsound mind and minors is incorrect?

A. a partner of unsound mind is capable of binding the firm and of being bound by
co-partners, : unless proven that he/she was of unsound mind when the partnership
was entered into and the : other partners knew this
B. if a partner who is a minor enters into a contract with a third party on behalf of
the firm, the : minor is liable as far as private assets are concerned
C. a creditor who has obtained judgment against the firm may not seize the minor’s
separate : property
D. a minor will become liable as an ordinary partner when he/she attains majority the
partnership is : not repudiated within a reasonable time

Which below statements is incorrect?

A. Partners share the profits in a partnership


B. In a partnership, partners have limited liability for the business debts
C. A partner is also an agent of the partnership
D. Agreement may be implied in a partnership

Which statement is incorrect?

A. carrying on a business implies repetition (Smith v Anderson (1880) 15 Ch D 247)


B. partnership may be formed in order to undertake a single business transaction
C. in Khan v Miah [2000] 1 WLR 2123 it was held that work, such as finding,
acquiring and fitting out : a shop/restaurant, is undertaken with a view to profit
D. in Keith Spicer Ltd v Mansell [1970] 1 All ER 462 it was held that ordering goods
and opening a : joint bank account in contemplation of a business are insufficient
for a partnership

Which statement regarding dissolution of a partnership is incorrect?


A. Partners are not entitled to have partnership property applied towards the
payment of : partnership liabilities
B. After payment of the firm’s liabilities, partners do not have the right to have
surplus assets : applied in payment of what may be due to the partners
respectively
C. On the termination of the partnership, any partner can apply to the court for a
decree to dissolve : the partnership and appoint a receiver to wind up the firm’s
business/affairs
D. Should a sequestration order be made against a partner, a creditor of the firm
cannot receive a : dividend out of the bankrupt’s separate property until all
separate creditors of the bankrupt partner : have been paid in full.

Which of the below statement is incorrect?

A. Partnership property is liable to be seized for the private (personal) debt of a


partner and : made liable on a judgment against the partnership
B. A creditor who has obtained judgment in respect of the separate debt of a partner
may obtain an : order charging that partner’s interest in the partnership property
and profits with the amount of the : debt and interest.
C. A creditor who has obtained judgment in respect of the separate debt of a partner
may obtain by : an order the appointment of a receiver of that partner’s share of
profits and of any other money : which may be coming to the partner in respect of
the partnership.
D. The two sources of law relevant when considering how partners bind their
partners when dealing : with third parties are the common law (including equity) of
agency and the Partnership Act. The : common law complements the provisions of
the Partnership Act in relation to the authority that an : agent has.

Which court decision regarding fiduciary duties is incorrect?

A. In Chan v Zacharia (1984) 154 CLR 178, the court decided that that the fiduciary
relationship : continued until the partnership had finally been wound up.
B. In United Dominion Corp Ltd v Brian Pty Ltd (1985) 157 CLR 1, the Court found
that a fiduciary : relationship, with attendant fiduciary obligations ordinarily exist
between prospective partners who : have embarked upon the conduct of the
partnership business or venture before the precise terms of : any partnership
agreement have been settled
C. In Popat v Schonchhatra (1997) 3 ALLER 800 in the absence of a statement to the
contrary, : partners are entitled to an equal share of profits
D. in Harvey v Harvey (1970) 120 CLR 529, the Court held that if a partner receives
an additional : annual fee to be “on call” for that client, the fee belongs to the
partnership.

Which of the following statements regarding limited partners is incorrect?

A. A limited partner may assign his/her share in the partnership with the consent of
the general : partners
B. The consent of a limited partner is not required to admit a person as a partner
C. If a limited partner has suffered his/her share of the partnership property to be
charged for a : separate debt, the other partners are entitled to dissolve the
partnership
D. A limited partner is not entitled to dissolve the partnership by notice.

Which legislation regulates partnerships in NSW?

A. Partnership Act 1892 (NSW)*


B. Competition & Consumer Act 2001 (Cth)
C. Partnership Act 1958 (Vic)
D. Partnership Act 1895

Find incorrect statement

A. A term often included in a partnership agreement that allows other partners to


purchase a : retiring/deceased partner’s interest at an agreed valuation avoids the
disruption of a formal winding
B. A partnership cannot be dissolved because the business is carried on at a loss
C. Partnerships can be dissolved because of mutual incompatibility, making it
impossible for partners : to carry on a business
D. On dissolution, partnership property can be applied towards the payment of
partnership : liabilities/debts and any surplus can be distributed among the
partners

Quiz 1:
1. Sources of law: Sources of law in Australia do not include:
A. Consolidating statutes
B. Judge-made law
C. Repealed statutes
D. Equity
2. Criminal offences: Which of the following is incorrect with respect to criminal
offences?
A. Indictable offences are generally the more serious offences.
B. The prosecution must prove its case beyond reasonable doubt.
C. Summary offences are determined by a magistrate without a jury.
D. A committal hearing is held before most summary offence matters. orrect Answer
3. Common law: Which one of the following statements about common law is
A. Common law is judge made law Correct!
B. Common law overrides statute law
C. Common law is made by Parliaments
D. Common law and equity are the same types of laws and provide the same remedies
4. The High Court’s decision in Waltons Stores (Interstate) Ltd v Maher (1988)
164 CLR 387:
A. Extended the doctrine of promissory estoppel such that it applies even where
the : promisor does not create or encourage the false assumption of the
promisee
B. Extended the doctrine of equitable estoppels such that it applies even where
there is no : pre-existing contractual relationship between the parties
C. Restricted the doctrine of equitable estoppel such that, where the contract is of
a : commercial nature, the doctrine will not apply
D. Restricted the doctrine of promissory estoppel such that where an innocent
third party : has been affected, no remedy can be ordered
5.Which are some of the main features of the Electronic Transactions Act 1999
(Cth)
A. validity of electronic transactions
B. recognition of writing by electronic means
C. Recognition of retaining information in electronic form
D. All of the above
6. In Ashton v Pratt [2015] NSWCA 12 what was the main reason for the court
deciding that : there was no intention to create a legally binding contract?
A. The agreement was not reduced to writing
B. The verbal language of the agreement greatly lacked detail from either party and
did not : indicate definite obligations
C. Pratt lacked the necessary contractual capacity due to his age and ill health
7. Which of the following statements about bilateral and unilateral contracts is
not correct?
A. More than one party is required for both bilateral and unilateral contracts
B. A unilateral contract does not require consideration but simply a promise to
perform an : act is sufficient
C. Both parties are obligated to perform their promises in a bilateral contract
D. Only one of the parties is obligated to perform an action in a unilateral contract
8.Which of the following is not a class of persons regarded by the law as wholly
or partly : incapable of entering into legally binding contracts?
A. Intoxicated persons
B. Minors You Answered
C. Women
D. Mentally incapacitated persons
9. The legislature: Exclusive powers are those that:
A. Can be exercised by either the Commonwealth or the States
B. Can be exercised only by the Commonwealth
C. Can be exercised only by the States
D. Where there is any inconsistency, the Commonwealth law will prevail
10. Why are illusory terms in a contract problematic?
A. they are vague or ambiguous so they fail to create a legal obligation
B. They exist in one party’s imaginary belief only
C. Courts have difficulty interpreting them
D. Interpreting them in any way at all could result in an unfair decision
11. Delegated legislation: Which of the following is not correct?: Delegated Legislation:
A. Is subordinate legislation
B. Is made under the authority of an Act of Parliament
C. Often contains more detailed rules than those of the associated Act
D. Can only be made Government Ministers
12.Native Title: Native Title was first recognised in which of the following cases?
A. Mabo v State of Queensland (No 2) (1992) 175 CLR 1
B. Wik Peoples v State of Queensland (1996) 187 CLR 1
C. Brandy v Human Rights and Equal Opportunity Commission (1995) 183 CLR 245
D. Commonwealth v Jones (1901) 174 CLR 25
13. Jack was subpoenaed to give evidence on Rods behalf. Jack claimed Rod promised him
: $2000 to give evidence. Would Jack recover that money?
A. No - Jack’s public duty is to give evidence in response to that subpoena. Correct!
B. No this is illegal
C. Yes if Rod put that in writing
D. Always binding because promises should not be broken
14. Business law: Which of the following no longer regulates business or commercial law?
A. Trade Practices Act 1974 (Cth)
B. Competition and Consumer act 2010 (cth)
C. Corporations Act 2001 (cth)
D. The law of contract
15. The main legal issue in Felthouse v Bindley (1862) 11CB (NS) 869 was:
A. the contract was formed due to offer and acceptance being proven
B. An offer to sell a horse was simply an invitation to treat
C. generally silence /lack of action does not constitute acceptance of an offer
D. An offer can be revoked at any time prior to acceptance

Quiz 2:
1. Who/what are exempt from the provisions of s18 of the Australian Consumer Law?
A. No-one is exempt
B. Corporations
C. Information providers such as media outlets
D. Advertisers
2. Which of the following is not a way to discharge a contract?
A. By performance
B. By operation of law
C. Through breach
D. By agreement
3. Which of the following statements about the parol evidence rule is true.?
A. It applies only to oral (parol) evidence
B. It does not apply to written contracts
C. It does not apply where it can be shown that the written contract was not
intended to : be a complete record of the agreement
D. It applies only to previous drafts of a written contract
4, In Curtis v Chemical Cleaning & Dyeing Co Ltd [1951] 1 KB 805:
A. The exemption clause in the sales agreement for the drycleaning was effective
B. The exemption clause in the sales agreement for the drycleaning was not
sufficiently : explained to the customer
C. The exemption clause in the sales agreement for the drycleaning was signed and :
therefore effective
5. Which of the following is not correct with respect to mitigation of loss?
A. There is a duty on the person claiming the damages to take all reasonable steps to :
mitigate their losses
B. Where loss could have been mitigated and was not, a person cannot claim that loss
C. The burden of proving that losses were mitigated falls upon the party claiming the :
damages ie the defendant
D. The question of whether a person took all reasonable steps to mitigate is one of
fact : dependent upon the particular circumstances
6. Which of the following is not correct with respect to termination by
subsequent : agreement?
A. A subsequent simple contract can cancel a contract where one party has completed
their : obligations and the other has not
B. A subsequent agreement may cancel an original contract
C. A subsequent agreement must be valid to rescind an earlier contract
D. A subsequent simple contract can cancel a contract where there is still something
to be : done by each party
7. In the case of restraint of trade with respect to employment:

A. The court does not easily allow parties to contract out of their means of
employment
B. Whatever the parties determine will be accepted by the court
C. There are no restraints on employees unless set out in a contract with their
employer
D. These agreements are construed as strictly as restraints of trade between
purchaser and : vendor of a business
8. Krell v Henry [1903] 2 KB 740 illustrated what point?
A. That death can be regarded as frustration
B. That the parties were entitled to a discharge
C. That the parties must perform regardless of circumstance
D. That the parties are discharged if an event occurs which forms the basis of the
contract, : but which prevents performance at no fault of either party
9. Contingent conditions include:
A. Accord and satisfaction and consideration
B. Conditions subsequent and conditions precedent
C. Warranties and innominate terms
D. Releases and mutual terminations
10. Fraser crashes his car with his friend Angus as a passenger. Angus is
injured. Angus was : not wearing a seatbelt. Which of the following is not
correct?
A. Liability would likely be apportioned between Fraser and Angus
B. Damages cannot be apportioned for breach of contractual duty of care where
there is : contributory negligence
C. Damages awarded to Angus, if any, would be reduced by his contributory
negligence
D. Angus’ contributory negligence relates only to his own safety, not to that of others
11.Which of the following is NOT a requirement for an assignment of a debt or other
chose : in action under various State statutes?
A. The assignment is absolute and not by way of charge;
B. Fourteen days’ notice in writing is provided to debtor;
C. the assignment is in writing;
D. If not in writing then notice of the assignment is given by notice to the debtor.
12. The parol evidence rule performs much the same function as:
A. A collateral warranty
B. An oral collateral contract
C. Consideration
D. A merger clause
13. Which of the following is not one of the special relationships in which the
onus of : proving that undue influence was not employed shifts to the denying Party?
A. Paren and child where the child is living independently
B. Solicitor and client
C. Trustee and beneficiary
D. Doctor and patient
14. How may a court view substantial performance of a contract?:
A. The contract is complete, but the uncomplete portion may be allowed for as a
reduction : of the full contract price
B. it will not grant a discharge
C. It will sever the incomplete parts
D. It will treat the contract as breached
15. How is repudiation determined by the court?
A. Subjectively
B. By considering the actions of the repudiating party
C. By viewing the circumstances of the repudiation
D. Objectively

QUIZ 3
Question 1: To claim damages a causal link must be established between what two things?
A. Between the defendant’s conduct and the actual monetary value of the
loss/damage
B. The conduct and initiating court proceedings
C. The fact that the plaintiff suffered loss or damage is sufficient. Nothing else is
required
D. The defendants breach and the plaintiff’s injury

Question 2:Which of the following is not correct?


A. If damages are too remote they will not be recoverable
B. The remoteness test will be satisfied where the damage suffered is of the same
type or : kind as foreseeable damage
C. In order for damage to not be too remote, it must be reasonably foreseeable
D. In The Wagon Mound No 1 and The Wagon Mound No 2, where there was an :
unfortunate combination of an oil spill, welding sparks and floating cotton waste, the loss
: was found to be not reasonably foreseeable in the circumstances

Question 3: Which of the following is not a provision of the Wrongs Act 1958 (Vic)
relating to:
professional liability?
A. If there are differing peer professional opinions across Australia, the court may
accept : just one of those opinions
B. The court must rely on peer professional opinion even where it considers that
opinion : irrational
C. If there are differing peer professional opinions across Australia, the court may
accept all : of those opinions
D. Peer professional opinion can be considered widely accepted even where it is not :
universally accepted

Question 4 : Why was the plaintiff successful in Overseas Tankship (UK) Ltd v Miller
Steamship Co Pty : Ltd (The Wagon Mound No 2) [1967] AC 617 (PC) when another
plaintiff failed in the earlier : related Wagon Mound case?
A. The plaintiffs in the second had better legal representation
B. The legal situation had changed between the two cases
C. The court was more disposed to think about the consequences of the damage
suffered by : so many as a result of the oil spill
D. In the first case, the plaintiffs failed to show that a reasonable man would have
foreseen : the risk of damage from the oil spill

Question 5 Where a person gives advice, that advice is relied upon and the advice is
incorrect, the : person giving the advice may be liable in negligence. Which of the following
is incorrect?
A. The “special relationship” described in Mutual Life & Citizens’ Assurance Co Ltd v
Evatt : (1968) 122 CLR 556 has been accepted as the test for determining the
existence of a duty of : care with respect to negligent misstatements
B. The test in Mutual Life & Citizens’ Assurance Co Ltd v Evatt (1968) 122 CLR 556
was : approved in San Sebastian Pty Ltd v Minister Administering Environmental
Planning &: Assessment Act 1979 (1986) 162 CLR 340
C. The duty of care only arises where the “special relationship” involves a person
seeking : advice from the advice-giver, not where the advice was given unrequested
and merely : accepted
D. The advice-giver does not need to be in the business of giving advice

Question 6: The “neighbour principle” is often viewed as:


A. An objective test
B. A test of the reasonable person
C. A test of the foreseeability of harm
D. An identity test

Question 7: In Tame v State of New South Wales (2002) 211 CLR 317 where Tame was
given a false : blood alcohol reading:
A. Whether the police officer was found to have owed Tame a duty of care was
entirely a : question of community standards
B. The psychotic depressive illness Tame developed was found to flow from the
breach of : duty of the police sergeant because it was entirely caused by his actions
that could have : been avoided had he exercised sufficient care
C. Part of the test of reasonable foreseeability is a question of fact
D. Because of the “eggshell skull” principle, pre-existing knowledge of Tame’s
susceptibility: to suffering nervous shock was not required to prove a breach

Question 8:Nicola drives the forklift at her place of work, Rooze’s Roofing. Nicola always
leaves the : forklift in a certain place where she has been told to leave it, with the forks up
off the : ground. One afternoon a customer who is collecting goods from the workshop
reverses his : car into the forks on the forklift. He is injured and his car is damaged.: What
is the principle that would make Nicola’s employer liable for her actions?
A. Contributory negligence
B. Strict liabilityCorrect!
C. Vicarious liability
D. Voluntary assumption of risk

Question 9: Which of the following is not correct? The reform to the law of negligence that
took : place in Australia in the early 2000s:
A. Limits the scope of potential liability for negligence
B. Does not apply to claims in contract law
C. Was enacted in all Australian states and territories
D. Covers personal injury
Question 10: In Australian Safeway Stores v Zaluzna (1987) 162 CLR 479:
A. The respondent was a lawful entrant upon the land of the respondent, establishing
a : relationship between them and there was therefore a duty of care owed by the
appellant to : avoid a foreseeable risk of injury
B. The respondent was limited by his particular status as entrant without specific
consent of : the appellant
C. The respondent contributed to the accident thus limiting damages
D. There was no liability.

Quiz 4:
Question 1:Which is incorrect statement?
A. If a member of a firm of solicitors acting for a vendor in a sale absconds with
the deposit, : his/her partners are not liable to refund the money
B. In Lloyd v Grace, Smith & Co [1912] AC 716, the firm was held responsible for
the fraud committed : by a managing clerk of a firm, who misappropriated
property while acting within the scope of his : authority
C. In SJ Mackie Pty Ltd v Dalziell Medical Practice Pty Ltd [1989] 2 Qd it was held that
the transfer of a : share to a non-partner breaks the continuity of the firm,
constituting a new firm/partnership of the : remaining former partners and the new
member
D. Partnership agreements can contain provisions to enable the transition from one
firm to another : to be effected without the disruption of a formal winding up.
Question 2: Find incorrect statement
A. A term often included in a partnership agreement that allows other partners to
purchase a : retiring/deceased partner’s interest at an agreed valuation avoids the
disruption of a formal winding
B. A partnership cannot be dissolved because the business is carried on at a loss
C. Partnerships can be dissolved because of mutual incompatibility, making it
impossible for partners : to carry on a business
D. On dissolution, partnership property can be applied towards the payment of
partnership : liabilities/debts and any surplus can be distributed among the
partners
Q3: Find one incorrect statement?
A. In the absence of special statutory provision, although each partner is liable with
the others for : the whole of the debts of the firm, their liability is only joint
B. A creditor can bring only one action against members of a partnership and any
partner can insist : that the action be stayed until all other partners are joined as
parties.
C. A person admitted into an existing firm, liability may be incurred where it is
specially agreed upon
D. A creditor can enforce liability against an incoming partner whether or not he/she
is a party to : the contract
Q4: Find a statement that is incorrect
A. A retiring partner should give specific notice of their retirement to persons with
whom the firm : has had dealings to avoid being made liable for debts incurred after
retirement.
B. Persons who represents themselves, or knowingly allow themselves to be
represented, as a : partner, are liable as a partner to anyone who has on the faith of
such representation given credit to : the firm
C. A wrongful act/omission includes breach of contract or of fiduciary duty, negligent :
misrepresentation and misleading or deceptive conduct
D. Innocent partners are liable for wrongful acts of other partners made in the
ordinary conduct of : business
Q5: Which statement regarding partnership is incorrect?

A. A partnerships (or firm) is a way of gathering resources or expertise for major


projects
B. Partnerships do not avoid taking on the formality and expense of an incorporated
company
C. Partners in a partnership complement each other with their skills and bring in new
capital and : broader funding options
Q6: Which of the following statements regarding the written law is incorrect?
A. Under s 115 of the Corporations Act 2001 (Cth), the maximum number of persons
who may form a : partnership for the acquisition of gain is 20
B. Under the Corporations Regulations 2001 (Cth), reg 2A.1.01, partnerships of more
than 20 : partners may be formed for certain professions/callings
C. The Corporations Regulations 2001 (Cth), reg 2A.1.01 sets a maxima of partners
for medical and : legal practitioners, veterinary surgeons, patent and trademark
attorneys, sharebrokers and : stockbrokers and pharmaceutical chemists
D. The Corporations Regulations 2001 (Cth), reg 2A.1.01 does not set a maxima of
partners for : architects and accountants
Q7: Which statement regarding persons of unsound mind and minors is
incorrect?
A. a partner of unsound mind is capable of binding the firm and of being bound by
co-partners, : unless proven that he/she was of unsound mind when the partnership
was entered into and the : other partners knew this
B. if a partner who is a minor enters into a contract with a third party on behalf of the
firm, the : minor is liable as far as private assets are concerned
C. a creditor who has obtained judgment against the firm may not seize the minor’s
separate : property
D. a minor will become liable as an ordinary partner when he/she attains majority the
partnership is : not repudiated within a reasonable time
Q8: Which of the following statements is not correct?
A. as an agent, a partner is able to bind the other partners and, as principal, be bound
by the actions : of the other partners
B. in New South Wales, Victoria, Queensland, South Australia, Western Australia and:
Tasmania provision for limited partnerships is made in the Partnership Act
C. incorporated limited partnerships have been introduced in all Australian States and
Territories
D. the Partnership Act provides that the rules of the common law and equity are to
continue in force : except insofar as they are inconsistent with the Act
Q9: Which finding is correct?: In Popat v Schonchhatra (1997) 3 All ER 800, the
Court decided:

A. Popat was entitled to half the profits on the sale of the business and to a share
of the profits : that had accrued after the dissolution of the partnership, but
before the final settlement of : accounts.
B. was not entitled to any profits on the sale of the business, as there was
no partnership : agreement and he was further not entitled to a share of the
profits accrued after the dissolution of : the partnership, but before the final
settlement of accounts.
C. Popat was entitled to half the profits on the sale of the business, but not to a
share of the profits : that had accrued after the dissolution of the partnership,
but before the final settlement of : accounts.
D. Popat was not entitled to half the profits on the sale of the business, but he
was entitled to a : share of the profits that had accrued after the dissolution of
the partnership but before the final : settlement of accounts.
Q10: What constitutes an exception to the usual number of a partnership being 20
partners?
A. If one of the partners is married-his/her husband or wife is automatically a
partner
B. The exception relates only to limited partnerships
C. No more than 20 are allowed
D. The Corporations Regulations 2001 (Cth) provide greater numbers in certain
specified : professions
Q11: Which court decision regarding fiduciary duties is incorrect?
A. In Chan v Zacharia (1984) 154 CLR 178, the court decided that that the
fiduciary relationship : continued until the partnership had finally been wound
up.
B. In United Dominion Corp Ltd v Brian Pty Ltd (1985) 157 CLR 1, the Court
found that a fiduciary : relationship, with attendant fiduciary obligations
ordinarily exist between prospective partners who : have embarked upon the
conduct of the partnership business or venture before the precise terms of :
any partnership agreement have been settled
C. In Popat v Schonchhatra (1997) 3 ALLER 800 in the absence of a statement
to the contrary, : partners are entitled to an equal share of profits
D. in Harvey v Harvey (1970) 120 CLR 529, the Court held that if a partner
receives an additional : annual fee to be “on call” for that client, the fee
belongs to the partnership.
Q12: Which below is incorrect? A firm is not bound by the acts of a partner if:
A. the act is not of the usual business of the kind carried on by the firm
B. the partner exceeds his/her actual authority in the particular matter

C. the person with whom the partner is dealing knows that he/she has exceeded
his/her authority
D. the person with whom the partner is dealing knows or believes him/her to be
partner
Q13: How would a partner pledge the firm’s credit?
A. By deed
B. Only for a purpose directly connected to the firm’s business but must have express
authority : to do so
C. By agreement with other partners
D. By agreement with a majority of partners plus an appropriate credit application
Q14: Which statements is not correct?
A. if the agreement is that a person should be paid a fixed sum by the firm, he/she may
be a partner : and jointly liable to creditors of the firm
B. the rights and obligations of partners to each other arise from the partnership
agreement, the : statute and the equitable concept of the fiduciary
C. a partnership agreement must be in writing; it cannot be reached orally or by a
course of : conduct
D. the Partnership Act determines partners’ rights, duties and interests, if not
included in the : partnership agreement
Q15:Which of the statements below is incorrect?
A. A partner’s express authority must be in writing
B. A partner’s authority to buy/sell goods of a kind necessary for or usually employed
in the business : on account of the firm is implied
C. A partner cannot pledge the firm’s credit for purposes not connected with its
ordinary course of : business without express authority
D. A partner cannot pledge the firm’s assets for private debts without express
authority
QUIZ 1:
The rule of law: Which of the following concepts is closely related to the rule of law?

A. Representative government
B. Responsible government
C. Due process
D. Separation of powers

If one party has threatened another party to enter into a contract, the element that is
: missing is:

A. Acceptance
B. Intention to create legal relations
C. Real/genuine consent
D. Legality

Section 15AA of the Acts Interpretation Act: Section 15AA of the Acts Interpretation
Act 1901 (Cth)

A. Is an inflexible section
B. Provides that express mention of one matter suggests that other matters are
excluded
C. Provides that where there is a conflict between a specific and a general
provision, the : specific provision should be applied
D. Provides that the purpose of the statute or its object should be the preferred
method of : determining the statute’s meaning.

Criminal proceedings: An indictable offence is:

A. A more serious civil wrong


B. A less serious civil wrong
C. A more serious criminal offence
D. A less serious criminal offence

Sources of law: Sources of law in Australia do not include:


A. Consolidating statutes
B. Judge-made law
C. Repealed statutes
D. Equity

Criminal offences: Which of the following is incorrect with respect to criminal


offences?

A. Indictable offences are generally the more serious offences.


B. The prosecution must prove its case beyond reasonable doubt.
C. Summary offences are determined by a magistrate without a jury.
D. A committal hearing is held before most summary offence matters.

Common law: Which one of the following statements about common law is

A. Common law is judge made law Correct!


B. Common law overrides statute law
C. Common law is made by Parliaments
D. Common law and equity are the same types of laws and provide the same
remedies

The High Court’s decision in Waltons Stores (Interstate) Ltd v Maher (1988) 164 CLR
387:

A. Extended the doctrine of promissory estoppel such that it applies even where
the : promisor does not create or encourage the false assumption of the
promisee
B. Extended the doctrine of equitable estoppels such that it applies even where
there is no : pre-existing contractual relationship between the parties
C. Restricted the doctrine of equitable estoppel such that, where the contract is of
a : commercial nature, the doctrine will not apply
D. Restricted the doctrine of promissory estoppel such that where an innocent
third party : has been affected, no remedy can be ordered

Which are some of the main features of the Electronic Transactions Act 1999 (Cth)

A. validity of electronic transactions


B. recognition of writing by electronic means
C. Recognition of retaining information in electronic form
D. All of the above
In Ashton v Pratt [2015] NSWCA 12 what was the main reason for the court deciding
that : there was no intention to create a legally binding contract?

A. The agreement was not reduced to writing


B. The verbal language of the agreement greatly lacked detail from either party
and did not : indicate definite obligations
C. Pratt lacked the necessary contractual capacity due to his age and ill health
D. The plaintiff had been a worker in the escort business and not morally entitled
to any : monies under the verbal agreement

Which of the following statements about bilateral and unilateral contracts is not
correct?

A. More than one party is required for both bilateral and unilateral contracts
B. A unilateral contract does not require consideration but simply a promise to
perform an : act is sufficient
C. Both parties are obligated to perform their promises in a bilateral contract
D. Only one of the parties is obligated to perform an action in a unilateral contract

Which of the following is not a class of persons regarded by the law as wholly or
partly : incapable of entering into legally binding contracts?

A. Intoxicated persons
B. Minors You Answered
C. Women
D. Mentally incapacitated persons

Which of the following is not one of Brennan J’s six criteria for estoppel from
Waltons: Stores (Interstate) Ltd v Maher (1988) 164 CLR 387?

A. The promisor induced an assumption


B. The promisee acted in reliance on that assumption
C. The promisor knew or was reckless as to whether the promisee intended to act
in that
D. The promisee will suffer a material loss if the assumption is not fulfilled

Interpretation : Which of the following is not correct?: With respect to


interpretation, the main approach used by courts is:

A. The literal approach


B. The purposive approach
C. The requirement that the natural and ordinary meaning to be given to words
D. The courts are free to exercise judicial discretion

Simone advertises a car for $4,000. John responds to the advertisement and asks :
whether Simone will take $3,000. Simone says ‘no’, but that she will hold the offer
open for : a week. The next day she sells it to Samantha. : Which of the following
statements is correct?

A. Simone has to hold the offer open as she made a promise


B. Simone does not have to hold the offer open as no consideration was given
C. John’s question is an invitation to treat
D. Simone does not have to hold the offer open as John made a counter-offer

Ratio decidendi*: The ratio decidendi of a case:

A. Is the full judgment of all judges


B. Is a statement of principle that is important, yet not crucial to the decision
C. Is the reason given for deciding the case
D. Is contained within the last published judgment

The legislature: Exclusive powers are those that:

A. Can be exercised by either the Commonwealth or the States


B. Can be exercised only by the Commonwealth
C. Can be exercised only by the States
D. Where there is any inconsistency, the Commonwealth law will prevail

Why are illusory terms in a contract problematic?

A. they are vague or ambiguous so they fail to create a legal obligation


B. They exist in one party’s imaginary belief only
C. Courts have difficulty interpreting them
D. Interpreting them in any way at all could result in an unfair decision

Delegated legislation: Which of the following is not correct?: Delegated Legislation:

A. Is subordinate legislation
B. Is made under the authority of an Act of Parliament
C. Often contains more detailed rules than those of the associated Act
D. Can only be made Government Ministers

Native Title: Native Title was first recognised in which of the following cases?

A. Mabo v State of Queensland (No 2) (1992) 175 CLR 1


B. Wik Peoples v State of Queensland (1996) 187 CLR 1
C. Brandy v Human Rights and Equal Opportunity Commission (1995) 183 CLR 245
D. Commonwealth v Jones (1901) 174 CLR 25

Jack was subpoenaed to give evidence on Rods behalf. Jack claimed Rod promised
him : $2000 to give evidence. Would Jack recover that money?

A. No - Jack’s public duty is to give evidence in response to that subpoena. Correct!


B. No this is illegal
C. Yes if Rod put that in writing
D. Always binding because promises should not be broken

Business law: Which of the following no longer regulates business or commercial


law?

A. Trade Practices Act 1974 (Cth)


B. Competition and Consumer act 2010 (cth)
C. Corporations Act 2001 (cth)
D. The law of contract

The main legal issue in Felthouse v Bindley (1862) 11CB (NS) 869 was:

A. the contract was formed due to offer and acceptance being proven
B. An offer to sell a horse was simply an invitation to treat
C. generally silence /lack of action does not constitute acceptance of an offer
D. An offer can be revoked at any time prior to acceptance

Sally puts up a sign stating that she has lost her dogs and will pay a reward of $100
for : their return to her within three days. Which of the following statements is
correct?
A. Jim will not get the reward because he returned the dog but did not verbally :
communicate acceptance of the offer to Sally
B. Nadia will get the reward as she returned the dog to the RSPCA
C. Chang will not get the reward because he returned the dog knowing it was
Sally’s, but : was unaware of her offer of a reward
D. Isabel will get the reward because she was only one day late in returning the
dogs (and to : not give her the reward would be unreasonable)
Reporting obligations: What is a binding precedent?
A. A decision of another court that is of persuasive authority
B. With respect to Australia, it is a decision of the UK Supreme Court
C. A decision of a court that binds judges in a lower court in the same court
hierarchy
D. A decision of a different court on the same subject matter

Which are some of the main features of the Electronic Transactions Act 1999 (Cth):
validity of electronic transactions
recognition of writing by electronic means
Recognition of retaining information in electronic form
All of the above

Peter verbally offers Tara his car for $5000. Tara refuses saying it’s not worth that,
but : she will pay $4000. What is the status of Peter’s first offer?
It can still be accepted by Tara
Peter should have made it in writing so Tara had proof he made it
Peter needs to make it again
It has lapsed because of Tara’s counteroffer

What was one of the “practical benefits” received by Roffey in Williams v Roffey Bros
&: Nicholls (Contractors) Ltd [1990] 1 All ER 512?
A. Williams did not sue Roffey for breach of contract
B. Roffey had no reason to doubt Williams would not complete his side of the
bargain
C. Roffey received additional payment from Williams
D. Roffey did not need to find another subcontractor
Sources of law – statute: Which of the following is incorrect?: Statutes may:
a. Codify the law
b. Bring new laws into existence
c. Repeal laws
d. Not overrule existing common law
Changing the Constitution : Section 128 of the Commonwealth Constitution provides
that the Constitution can be: changed by referendum that requires a “yes” vote:
A. in at least 2 States
B. in at least 3 States
C. in the ACT and NT (the Territories)
D. by the majority of voters and in a majority of States

Substantive and procedural law: Substantive law:


A. Refers to actual rights under the law
B. Is subsidiary to procedural law
C. Refers to the formal steps to enforcement of rights and duties under the law
D. Includes the rules of evidence

In which of the following situations is the offer most likely not to have lapsed?
Where Wrench had an option to purchase land but, unknown to Wrench, the seller
died : prior to Wrench’s acceptance (bên t là câu này)
Where Carter, a prospective purchaser of land, makes a counter-offer that is rejected
and: then states that he will accept the earlier offer
Where no time was stated for acceptance by Jane, but Ben does not accept within a :
reasonable time
Where Jim has become overtaken by insanity prior to accepting

Reception of English law in Australia: Which Latin phrase explained the rationale for
applying English laws to the new Colony of : New South Wales?
A. actus reus
B. terra nullius
C. ratio decidendi
D. prima facie

Andy promises Ellie $100 on her 21st birthday. If this promise was contained in a
simple : contract, the legal position is that such a promise is:
A. Never binding as it must have consideration supplied by Ellie
B. Binding only if Ellie and Andy had contractual capacity
C. Sometimes binding but only after Ellie turned 21
D. Always binding because promises should not be broken
In Elizabeth City Center Pty Ltd v Corralyn Pty Ltd (1995) 63 SASR 235, the option to :
exercise renewal of the lease:
Was effective because of the operation of the postal rule
Effective communication of the renewal requirement had not been made
Negated the postal rule because the notification was not sent by certified mail as :
required
b and c

Interpretation – extrinsic materials: In interpretation, extrinsic materials:


Are required to be referred to by a court
Are only to be referred to where the meaning of the provision is ambiguous
Are allowed to be referred to by a court
Are to be referred to regardless of the length of delay this will cause proceedings

With respect to revocation, which of the following statements is not correct?


A. Revocation need not be in words
B. The offeror must personally communicate the revocation to the offeree
C. The offeree may accept the offer until such time as they become aware of the
revocation
D. Where an offer has been made to the world at large, revocation does not need to
be seen : by everyone in order to be effective

Which of the following circumstances are likely to affect the consent of one or both :
parties to a contract:
A. Mistake
B. Duress and undue influence
C. Lack of writing
D. a & b
Civil and criminal law: Which of the following is not correct with respect to
civil law?
A. The typical purpose of a civil action is to obtain damages.
B. Under civil law, one person may sue another who has committed a wrongful
C. act.
D. Interrogatories and discovery are procedures that are available in civil
proceedings.
E. The document filed by the defendant is called a writ
Equitable remedies: Which of the following is incorrect?: Equitable remedies:
A. Are discretionary
B. Include specific performance
C. Must be awarded in a separate proceeding to common law remedies such as
damages
D. Include injunctions

Simon and Stella, both of full legal capacity, agree to go on a date. Stella is to pay for
the : dinner, but she is running late and does not meet Simon at all. Simon is
embarrassed and : angry and calls Stella the next day to threaten to sue her for his
taxi fares and dinner : expenses. Will Simon succeed in his claims?
A. Yes, because there is agreement
B. Yes, because there was consideration in that Simon incurred taxi and dinner
expenses
C. No, because Simon and Stella did not intend the agreement to create legally
enforceable : obligations
D. No, because there was no genuine consent of either party
Royal Assent: Who or what gives Royal Assent to an Act of the Commonwealth
Parliament?
A. The Queen
B. The Governor-General on the Queen’s behalf
C. The Prime Minister
D. Cabinet

In Crown Melbourne Ltd v Cosmopolitan Hotel (Vic) Pty Ltd [2016] HCA 26 why did
the : High Court disagree with the decision of the Victorian Court of Appeal in
finding there was : no case for promissory estoppel:
A. Even though the elements of estoppel had been proven no remedy was provided
as the : initial claim was brought too late and equity does not assist claims
lodged with unreasonable : delay
B. The plaintiff could not establish that the statement that it would be “looked after
at : renewal time” was capable of conveying to a reasonable person that it was a
genuine offer : of a further lease
C. There was a need for certainty as to what the parties had agreed on at the end of
the : negotiations to found a claim for estoppel
D. B&C
A contract dividing the proceeds of a cocaine importing venture that has no technical
: defects and that is not overly harsh/unfair to either party is:
A. Valid
B. Voidable
C. Void
D. Unenforceable
Sources of law: The two main types of law in Australia are:
A. Common law and judge-made law
B. Statute law and judge-made law
C. Statute law and Acts of Parliament
D. Common law and equity

Farah agreed to take care of an elderly woman Marge and in return was provided
with a : house to live in that was promised by Marge to be Farah’s after she died.
Farah cared for the : woman for 23 years, but upon Marge’s death Farah discovered
that their oral agreement : was never put into writing. The woman’s son moved into
the house and Farah made a claim : to the house. The court:
A. Is likely to reject Farah’s claim to the house as she provided no consideration of
any legal: value
B. Is likely to reject Farah’s claim to the house as the contract was not set down in
writing as : required
C. Is likely to order specific performance of the oral agreement because simple
contracts do : not need to be in writing
D. Is likely to apply the doctrine of part performance

Which of the following is most likely to be considered an offer?


A catalogue of books with discounted prices shown
A radio advertisement for drinks at “the coolest little pub in Victoria”
“I’ll pay you $3,000 if you complete a total rewrite of chapters 12 and 15 of this :
textbook.”
“Would you allow me to pay off the $2,000 over six weeks?”

Australian Constitution : Which Act established the federal legal and political system
and converted the separate : colonies into states?
A. Commonwealth of Australia Constitution Act 1900
B. The Australia Act 1986
C. Statute of Westminster Adoption Act 1942 (Cth)
D. statute of Westminster 1931 (IMP)

Contracts voidable by a minor do not include:


Those not binding unless ratified by the minor during their minority
Those binding unless repudiated by the minor during their minority
Those binding unless repudiated by the minor within a reasonable time after attaining
: their majority
Those not binding unless ratified by the minor within a reasonable time of attaining :
majority

Where an agreement has been made during the course of trade or commerce
between businesses, the situation will commonly indicate that the parties:
intended to create moral binding relations based on trust
intended to create binding legal relations
intended that there was no contract unless clearly specified in the agreement
intended to avoid court proceedings by inserting exclusion clauses in the agreement

Which of the following is not a class of persons regarded by the law as wholly or
partly: incapable of entering into legally binding contracts?
Intoxicated persons
Minors
Women
Mentally incapacitated persons
Separation of powers: Which of the following is not one of the three branches of the
Commonwealth government : in Australia:
A. The Governor
B. The executive
C. The judiciary
D. The legislature

Federal system: Which of the following statements is correct?

A. Australia is a federal system, with one Constitution and a number of


non-law-making : States and Territories.
B. Australia is a unitary system, where the States and Territories are constrained
in their : law-making powers by the Commonwealth.
C. Australia is a federal system, with two legal systems for each citizen.
D. Australia is a federal system with three branches of government: the judiciary,
the : executive and the Crown.

Native Title: Native Title was first recognised in which of the following cases?

A. Mabo v State of Queensland (No 2) (1992) 175 CLR 1


B. Wik Peoples v State of Queensland (1996) 187 CLR 1
C. Brandy v Human Rights and Equal Opportunity Commission (1995) 183 CLR 245
D. Commonwealth v Jones (1901) 174 CLR 25

Separation of powers: Which of the following statements about separation of powers


in Australia is correct?

A. The judiciary is the body that makes statute law.


B. The legislature is the branch that declares what the law is and interprets the
law.
C. The executive is the body that administers the law .
D. The legislature is the body that resolves disputes concerning the application of
law and : polices the law.

The postal acceptance rule:


A. Can be negated where the mailed offer was misdirected and the misdirection
was the : fault of the offeror
B. Can be impliedly excluded by offeror specifying actual receipt of acceptance
C. Can create a situation where a revocation is validly made days after the offer
has actually : been accepted
D. May apply to electronic communications, such as email, if the offeree chooses

Jack was subpoenaed to give evidence on Rods behalf. Jack claimed Rod promised
him : $2000 to give evidence. Would Jack recover that money?

A. No - Jack’s public duty is to give evidence in response to that subpoena.


B. No this is illegal
C. Yes if Rod put that in writing
D. Always binding because promises should not be broken

Law Reports: Where are decisions of the High Court of Australia found?: only online

A. In the High Court reports


B. In the Commonwealth Law Reports
C. In the Common Law Reports

The Age of Majority Act 1977 (Vic) reduced the age for contractual capacity of a minor

A. 16
B. 18
C. 21
D. 25

With respect to the contractual capacity of corporations, s 124 of the Corporations


Act : 2001 (Cth) gives them:

A. Only the capacity to make contracts approved by shareholders in general


meeting
B. The capacity to make contracts but only in the state or territory where the
corporation : conducts its business
C. All the legal capacity of a natural person (together with some additional powers
that can : only be exercised by a corporation)
D. Different capacity to contract depending on whether the corporation is public
or : proprietary

Delegated legislation: An example of delegated legislation is the:

A. Partnership Act 1958 (Vic) (ban dau)


B. Local Government Act 1993 (Qld), s 25
C. Corporations Bill 1988 (Cth)
D. Corporations Regulations 2001 (Cth)

In which of the following types of agreements is there a presumption that

parties intend : to be legally bound?

A. Partnership between husband and wife


B. Living arrangement between mother and daughter
C. Agreement between father and daughter
D. Dinner arrangement between friends

The Executive Power : The Executive power is administered by:

A. The Senate and the House of Representatives


B. The Queen, the Prime Minister and the Cabinet
C. The Governor, the Senate and the House of Representatives
D. The Judiciary, the Senate and the House of Representatives

Common law: Which one of the following statements about common law is correct?

A. Common law is judge made law


B. Common law overrides statute law
C. Common law is made by Parliaments
D. Common law and equity are the same types of laws and provide the same
remedies
Public and private law: Which of the following is not classified as private law?
A. The law of contract
B. Criminal law
C. The law of property
D. Corporations law

Criminal proceedings: An indictable offense is:

A. A more serious civil wrong


B. A less serious civil wrong
C. A more serious criminal offence
D. A less serious criminal offence

Where an agreement has been made during the course of trade or commerce
between businesses, the situation will commonly indicate that the parties:
A. intended to create moral binding relations based on trust
B. intended to create binding legal relations
C. intended that there was no contract unless clearly specified in the agreement
D. intended to avoid court proceedings by inserting exclusion clauses in the agreement

A qualified acceptance may also be known as a/an:

A. Counteroffer
B. Invitation to treat
C. Condition subsequent
D. Conditional agreement

Court hierarchy: Local or Magistrates Courts:

A. Are the lowest courts in the state hierarchy


B. Are the lowest courts in the federal hierarchy
C. Are an intermediate court
D. Are above Country Courts in the hierarchy
The legislature: Exclusive powers are those that:

A. Can be exercised by either the Commonwealth or the States


B. Can be exercised only by the Commonwealth
C. Can be exercised only by the States
D. Where there is any inconsistency, the Commonwealth law will prevail

Contracts voidable by a minor do not include:

A. Those not binding unless ratified by the minor during their minority
B. Those binding unless repudiated by the minor during their minority
C. Those binding unless repudiated by the minor within a reasonable time after
attaining : their majority

You Answered

D. Those not binding unless ratified by the minor within a reasonable time of
attaining : majority

Which of the following statements about contracts is true?

A. A contract must be in writing to be enforceable


B. It is a common law requirement that certain contracts are in writing
C. A simple contract is another term for an oral contract
D. It is a statutory requirement throughout Australia that some contracts must be
in writing : and others must be evidenced in writing

In which of the following situations is the offer most likely not to have lapsed?

A. Where Wrench had an option to purchase land but, unknown to Wrench, the
seller died : prior to Wrench’s acceptance
B. Where Carter, a prospective purchaser of land, makes a counter-offer that is
rejected and: then states that he will accept the earlier offer
C. Where no time was stated for acceptance by Jane, but Ben does not accept

within a : reasonable time

D. Where Jim has become overtaken by insanity prior to accepting

Interpretation – extrinsic materials: In interpretation, extrinsic materials:

A. Are required to be referred to by a court


B. Are only to be referred to where the meaning of the provision is ambiguous
C. Are allowed to be referred to by a court
D. Are to be referred to regardless of the length of delay this will cause proceedings

Sources of law: Sources of law in Australia do not include:

A. Consolidating statutes
B. Judge-made law
C. Repealed statutes
D. Equity

Which of the following statements about acceptance is false?

A. Acceptance can be by telephone or email or conduct


B. Acceptance can be in any manner chosen by the offeree
C. Acceptance must be unconditional
D. Acceptance can only be made by the person/s to whom the offer was made

The main legal issue in Felthouse v Bindley (1862) 11CB (NS) 869 was:

A. the contract was formed due to offer and acceptance being proven
B. An offer to sell a horse was simply an invitation to treat
C. generally silence /lack of action does not constitute acceptance of an offer
D. An offer can be revoked at any time prior to acceptance
Which of the following statements about the contractual obligations of government :
policy proposals is incorrect?

A. The courts tend not to assign contractual obligations to government policy


promises
B. Where the government enters into an ordinary commercial agreement to buy
services, : the law of contract regulates this agreement
C. Where the government makes a policy promise and a citizen relies on it the
court will : always regard that policy commitment as a binding contractual
obligation
D. In Australian Woollen Mills Pty Ltd v Commonwealth of Australia (1954) 92 CLR
424, the : government’s wool subsidy was found to be an administrative scheme
and not contractual : obligations

Jack was subpoenaed to give evidence on Rods behalf. Jack claimed Rod promised
him : $2000 to give evidence. Would Jack recover that money?

A. No - Jack’s public duty is to give evidence in response to that subpoena.


B. No this is illegal
C. Yes if Rod put that in writing
D. Always binding because promises should not be broken

QUIZ 2:

1. Which of the following is not true? A signature on a contract containing an


exemption : clause:
A. Will not guarantee the effectiveness of the clause where the doctrine of non est
factum : applies
B. Incorporates the exemption clause into the contract
C. Guarantees that an exemption clause will be effective
D. Will be effective even if one party did not read the contract
2. Which of the following would not constitute the unfair practice of harassment
or : coercion under the Australian Consumer Law? Where there is a debt in
connection with the : possible of goods and the creditor:
A. Consciously calculates to intimidate the debtor with the content of her demands
B. Tells the debtor, among repeated demands for payment give the debtor a
discount for : early payment
C. Makes demands constantly in order to exhaust the debtor
D. Chooses particularly threatening demands in order to demoralise the debtor

3. When considering whether a contract is a standard form contract, the court


does not : have to consider:
A. The bargaining power of the parties as unequal bargaining power does not
constitute a : special disadvantage
B. Whether both parties had the opportunity to negotiate the terms of the contract
C. Whether one party’s vicarious liability for its agents is limited
D. Whether the terms of the contract take into account the specifics of the
particular : transaction

4. The definition of “consumer” is limited by what amount?


A. $40,000
B. Recently increased to $100,000\
C. There is no limit
D. A negotiated amount
5. When a contract is made orally, the question of what the express terms are:
A. Is in most cases unascertainable
B. Depends on what a reasonable person would have said
C. Is a question of fact
D. Is not applicable; oral contracts only contain implied terms
6. Non est factum means:
A. It is not [my] deed
B. It is not true
C. Mistake of fact
D. It is unknown
7. Tom agrees to fix Richie’s car for $1,000. Richie pays a deposit of $200 and
Tom : guarantees the job will be completed within two weeks. Before Tom
commences the repairs, : the car is stolen from Richie’s locked garage and
destroyed by fire. Richie demands the : return of his $200 deposit and Tom
refuses to give it to him. Which of the following is not : correct?
A. The contract is automatically terminated
B. Tom is no longer obligated to fix Richie’s car
C. Under the Australian Consumer Law and Fair Trading Act 2012 (Vic), Tom is
entitled to:retain the $200 deposit
D. Under the Australian Consumer Law and Fair Trading Act 2012 (Vic), Tom is
entitled to : retain the portion of the $200 deposit he spent on parts and other
relevant expenses, if any

8. Sharma was selling his secondhand vacuum cleaner to Natasha and told
Natasha that it : could “do all the rooms in the house ten times over without
the filter needing to be : changed”. Natasha spent a few weeks shopping for
other vacuum cleaners but eventually : returned and purchased Sharma’s,
telling Sharma it was the cheapest she had seen in two : weeks and she liked
the colour. When Natasha took it home she soon discovered that the : vacuum
cleaner could only do one room at a time and then the filter would need
changing. : Natasha cannot rely on Sharma’s statement for breach of contract
because:
A. The statement was not put into writing
B. Natasha did not attach any importance to the statement when it was
made
C. Natasha should have tested the vacuum cleaner before buying it
D. The statement was a merger clause
9. Where a contract has not been completed within the specified time:
a. If no time limit was specified in the contract, there is nothing the innocent party
can do
b. The innocent party will be entitled to terminate the contract You Answered
c. The innocent party will be entitled to terminate the contract upon giving notice
that time : is of the essence
d. Only damages will be available no matter how crucial the time limit was to the
innocent : party
10. In which case did the court recognise that damages for disappointment and
distress : could be available subject to certain limitations?
a. Burns v MAN Automotive (Aust) Pty Ltd (1986) 161 CLR 653
b. Placer (Granny Smith) Pty Ltd v Thiess Contractors Pty Ltd (2003) 196 ALR
257
c. Howe v Teefy (1927) 27 SR (NSW) 301
d. Baltic Shipping Co v Dillon (1993) 176 CLR 344
11. Chris maintains that a document that he and Kathy have signed is a binding
contract. : Kathy maintains that it is merely a receipt and does not include all
the terms of their : agreement. Which of the following statements is correct?
a. If both parties have signed the document then the court will look no further into
the : circumstances
b. Kathy should plead non est factum
c. Kathy will be able to establish the existence of an oral collateral contract even
where it is : inconsistent with the terms of the main binding contract
d. Parol evidence may be admissible
12. Which of the following is not an element required for acceptable quality of
goods? That : the goods are acceptably:
a. Free from defects
b. Durable
c. Fit for the consumer’s purpose
d. Safe

13. When it was said in Re Hall & Barker [1878] 9 Ch D 538, “if a shoemaker agrees
to make : a pair of shoes, he cannot offer you one shoe and ask you to pay one
half the price”, this is : referring to:
a. The court’s tendency to regard contracts as divisible
b. The court’s tendency to regard contracts as severable
c. The court’s tendency to regard contracts as entire
d. The court’s tendency to require substantial performance
14. Which of the following statements about the parol evidence rule is true?
a. It applies only to oral (parol) evidence
b. It does not apply to written contracts
c. It does not apply where it can be shown that the written contract was not
intended to : be a complete record of the agreement
d. It applies only to previous drafts of a written contract
15. Which of the following is not a circumstance where the court would award
specific : performance?
a. Where the contract would require constant supervision by the court
b. Mitigation
c. Contributory negligence
d. Remoteness
16. Fraser crashes his car with his friend Angus as a passenger. Angus is injured.
Angus was : not wearing a seatbelt. Which of the following is not correct?
a. Liability would likely be apportioned between Fraser and Angus
b. Damages cannot be apportioned for breach of contractual duty of care where
there is : contributory negligence
c. Damages awarded to Angus, if any, would be reduced by his contributory
negligence
d. Angus’ contributory negligence relates only to his own safety, not to that of
others
17. Which of the following is not correct? An action in restitution:
A. Is often brought where one party has been unjustly enriched at the expense of
the other
B. Cannot be brought where there is no contract between the parties
C. Can be brought where the contract between the parties is unenforceable
D. Can be brought where there has been only part performance

18. Which of the following is not true with respect to misleading or deceptive
conduct?
a. The plaintiff has to prove that conduct was misleading or deceptive
b. The plaintiff has to prove the causal link between the conduct and the loss
c. A person acting as a ”mere conduit” in passing on representations may also be
liable
d. A corporation is not liable where it is clear that the corporation is not the source
of the : information
19. Chris and Tama buy a noodle bar from Udon4U Pty Ltd. In the course of
negotiations, : Udon4U Pty Ltd’s agent, Nadia, misrepresents the turnover of
the noodle bar, stating an : amount that is double the actually turnover. Nadia
herself received this information on : turnover from Geoff, a director of
Udon4U Pty Ltd.: In making the misrepresentation, what sections has Nadia
likely contravened?
A. As she merely passed on the information with no intention to procure a
contravention : she probably will not be personally liable
B. Section 2(1)
C. Section 18
D. Section 52

20.Which of the following is not a type of termination?


a. Termination by bankruptcy
b. When the parties have fully and exactly performed their obligations to each
other under : the contract
c. When a deed displaces a simple contract
d. Where an event occurs that is not the fault of either party that causes a
fundamental : change to the nature of the contract and the parties obligations
and although the contract : covers that eventuality, it would cause hardship to
one party to enforce it
21. Which of the following statements is correct?
a. A contract may be illegal as performed where only one party performs it in an
illegal : manner
b. A contract that is legal as formed will be legal as performed
c. The courts are most likely to find a contract illegal and unenforceable where a
statute : penalises the way it is performed
d. Any illegal conduct will make a contract illegal as performed
22. Compensatory damages are also known as:
a. Expectation damages
b. Nominal damages
c. Actual /ordinary damages
d. Punitive damages
23. In Codelfa Construction Pty Ltd v State Rail Authority of New South Wales
(1982) 149 CLR : 337:
a. The High Court implied a term in the agreement granting a reasonable
extension of time:
b. Codelfa was able to prove that the term was necessary to make the contract
work
c. Codelfa did not need to prove that the term was necessary to make the contract
work : for the term to be implied as it was sufficient that both parties needed to
be rescued from a : difficult position imposed upon them by the injunction
d. The High Court found that the contract was frustrated
24. Which of the following guarantees applies to a sale by auction?
a. Guarantee as to undisturbed possession
b. Guarantee as to express warranties
c. Guarantee as to acceptable quality
d. Guarantee as to title
25. Unliquidated damages are:
a. Non-monetary sums
b. Prohibited by law as they are uncertain and difficult to quantify
c. Damages where the court is to determine the amount
d. Damages where an amount has been fixed in the contract

26. In which one of the following cases was it held that for a loss in the
contemplation of the : parties to be recoverable it must be “a serious
possibility”, “a real danger”, “liable to result” : or “not unlikely” to occur?
A. Hadley v Baxendale (1854) 9 Exch 341; 156 ER 145
B. Burns v MAN Automotive (Aust) Pty Ltd (1986) 161 CLR 653
C. Koufos v Czarnikow Ltd [1969] 1 AC 350
D. Tabcorp Holdings Ltd v Bowen Investments Pty Ltd (2009) 236 CLR 272
27.
28. Matt sells his car to Tabitha, who purchases it on the condition that she can
continue to: keep it in his garage at no cost for the next six months. Tabitha
pays in full. Two months: later, Matt tells Tabitha she will have to remove her
car from his garage as he has bought a: new car and wants to keep it in the
garage. Which of the following statements is correct?
a. If Tabitha agrees to remove her car, the contract has been terminated by partial:
performance
b. The condition that Tabitha is allowed to keep her car in Matt’s garage is a
condition: precedent
c. The condition that Tabitha is allowed to keep her car in Matt’s garage is a
condition: subsequent
d. If Tabitha promises to remove her car, Matt will not be able to enforce the
promise: unless the promise is supported by consideration or made under seal
29. Which of the following is not one of the three elements of an unfair term
under s 24(1) : of the Australian Consumer Law? Where:
A. It would cause a significant imbalance in the parties’ rights and obligations
B. The term is not transparent
C. It is not reasonably necessary to protect the interests of the disadvantaged party
D. It would cause detriment to a party if it were applied

30.Which unfair practice involves participation in a trading scheme where


persons at the : top receive most of the benefits?
A. referral selling
B. pyramid selling
C. unconscionable conduct
D. misleading and deceptive conduct

31. Janey takes her fur stole to the dry cleaner. When she returns to collect it,
there is a : black mark in the shape of an iron in the middle of her stole and it
is ruined. The dry cleaner : tells Janey that he is not liable for the damage to
her fur stole as there was an exemption : clause in their agreement. Which of
the following is not correct?
A. If the exemption clause was merely contained within a notice on the wall, the
dry : cleaner will be liable for the damage
B. If the exemption clause was on the dry cleaning docket Janey was given and she
was told : “don’t worry about this, it’s just the docket you have to give us when
you come to collect : your stole”, the dry cleaner will not be able to rely on the
exemption clause
C. In order for the exemption clause to be effective where it was not in a signed
document, : it had to have been brought to Janey’s attention
D. If the exemption clause was on the dry cleaning docket Janey was given rather
than a : formal written contract, the onus will be on the dry cleaner to prove
that Janey was aware it : contained conditions that would modify the agreement

32. Criminal proceedings may be brought for:


a. A contravention of s 18 of the Australian Consumer Law
b. Unfair practices
c. Unfair contract terms
d. Unconscionable conduct
33. Which of the following statements is correct?
A. An exemption clause is a term that completely excludes one party’s liability
B. An exemption clause is a term that excludes or limits the liability of one or more
parties
C. An exemption clause limits liability to a certain monetary amount
D. An exclusion clause limits one party’s liability whereas an exemption clause
limits all: parties’ liability

34. Which of the following is a circumstance where specific performance would


be ordered?
a. Where the subject matter of the contract is personal property that is not rare or
unique
b. Where a famous actor is to work as a Master of Ceremonies at a function
c. Where the subject matter of the contract is an apartment in a complex of 20
apartments
d. Where a non-famous actor is to wear a gorilla suit every day to advertise a
product
35. The remedy of restitution is based on the concept of?
a. Damages should be provided for part performance
b. There is such concept in equity
c. The parties apportion the performance and rewards are paid accordingly
d. Unjust enrichment where the defendant is unjustly enriched at the plaintiff’s
expense
36. Which of the following are not usually compensated for with payment of
damages?
a. Injured feelings/disappointment
b. Nominal loss
c. Loss that is difficult for the court to estimate
d. Loss of profit expected to be received
37. Which of the following is not usually recoverable by way of damages?
A. Compensation for mere inconvenience or disappointment
B. Loss that is difficult to estimate
C. Nominal loss
D. Expenses incurred in reliance on the other party’s promise to perform

38. A party repudiates a contract when


They decide to terminate it
They are no longer able or are unwilling to perform their obligations
The other party decides to terminate it
They take too long to perform an obligation that has a time limit under the contract,
such : as payment of rent

39. Chung is looking to purchase a carwash business from Terry. Terry tells
Chung that she : serves 100-150 cars per day and that she has just concluded
an agreement to service the : fleet of cars of a local business. In reality, Terry
has only served around 15 cars per day since : her customer service received a
bad review in the local newspaper and a competing carwash : business opened
one street away. Terry intends for Chung to buy her business based on : what
she has stated. This is an example of:
unilateral mistake
negligent misrepresentation
fraudulent misrepresentation
mutual mistake

40.Undue influence differs from duress in that:


With undue influence, the contract is not voidable but void
With undue influence, no unlawful act is required
Duress arises only within a closed list of special fiduciary relationships
With undue influence, actual physical violence is required

41. Which of the following constitutes referral selling?


Jaz is approached by her neighbor to join the sales team of Thames Beauty Products. :
Thames sells its products door-to-door through agents and is a close-knit family type
of : company. Agents that introduce other agents to “the family” are rewarded with a
finders’: fee. It is only $500 to join the Thames family, and Jaz only has to spend $400
on products in : her first month
Jaz starts work at a chemist and notices that the chemist, as retailers often do, puts a :
sale price on a sticker on most of the products in the shop. However, the chemist does
not : put the sale price sticker on top of the original price sticker and when customers
bring the : products to the counter, the chemist tells them that unfortunately there has
been a mistake, : the true price is the higher price
Jaz approaches a salesperson on the forecourt to buy the car package she saw :
advertised on television and is told that there were only two cars for sale as a part of
that : deal and there are now none left. The salesperson tells Jaz that she would look
much better : in the convertible model and although there is no deal on that car, it is
only $50,000 more
Jaz purchases a painting through a new art gallery that has just opened in town. The :
price is high and Jaz is at first uncertain about the expense, but the art dealer promises
Jaz a : commission on sales to any of Jaz’s friends who she sends along to the gallery.
Jaz has a lot : of art collector friends so is sure that she will receive some commission
from the gallery in : the near future.

42. In Trident General Insurance Co Ltd v McNiece Bros Pty Ltd (1988) 165 CLR
107:
A. Blue Circle was a sub-contractor whose worker was injured, and privity
precluded the : worker from getting an indemnity under the contract
B. It was held that the subcontractor could get an indemnity even though the :
subcontractor had given no consideration under the contract
C. In Deane J’s view Trident, the insurance company, had an obligation to the
subcontractor because otherwise Trident would be unjustly enriched
D. The majority were of the view that the “settled and fundamental” doctrine of
privity of : contract should not be overturned by the court
43. Jacq and Jack robbed the Commercial Bank. Jacq thinks that Jack was too
aggressive : with the tellers and the robbery could have gone very wrong. Jacq
is now refusing to give : Jack his share of the takings. Which of the following is
correct?
A. Jack provided consideration in the form of performing the robbery and is
entitled to his : share
B. Under the principle of freedom of contract, Jacq and Jack are entitled to agree
to : whatever they wish and the contract will be enforceable
C. The contract involved the commission of a crime and is therefore illegal and :
unenforceable
D. The contract will be unenforceable if the courts deem it to be against public
policy
44. If one party has completed its obligations under the contract, and the other
has not:
The contract can be terminated by mutual agreement
The contract cannot be terminated by simple agreement without consideration
The contract can be terminated if it is put down in writing; no further consideration is
: required
The contract can be terminated where one party promises to abandon their rights
under : the contract and the other party promises to do the same

45. Ella contracted Anasaki to build a balcony for her house to certain
specifications. When : Anasaki had finished, Ella was shocked to see that the
balcony was much bigger than she : had specified. Ella refused to pay Anasaki.
Which of the following is incorrect?
Ella has received some benefit
The benefit Ella received was at Anasaki’s expense
It would be unjust for Ella to have to pay any money for a balcony that was not built to :
her specifications under the contract
Ella will probably not have to pay the contract price but must pay a reasonable amount

46. Which of the following is an element required for the doctrine of frustration to
operate?
Material loss to one party
No fault on the part of either party
Material loss to both parties
Hardship or inconvenience

47. Which of the following is not correct with respect to the prohibition of
unconscionable : conduct within the meaning of the unwritten law?
Unwritten law includes equity
There is no unconscionable conduct where both parties have made the same mistake
in : good faith
There must be a special disadvantage and an exploitation of that disadvantage
Unequal bargaining power of itself is enough to constitute a special disadvantage

48. In Jarvis v Swans Tours Ltd [1973] QB 233, where Jarvis was disappointed by,
among : other things, the little dry nut cakes on his holiday:
A. Damages were not awarded
B. Damages were awarded for anxiety and depression
C. Pleasure and enjoyment were not promises of the tour company with respect to
the : holiday in Switzerland
D. Damages were awarded partly for disappointment

49. With respect to substantial performance and partial performance:


a. Partial performance is a lower level of performance than substantial
performance, and : there is no deduction of the contract price available where
substantial performance has : occurred
b. Substantial performance is where the parties terminate the contract for less
than full : performance
c. Partial performance is where a party voluntarily accepts less than full
performance by : the other party
d. Partial performance is where the court terminates the contract for less than
exact : performance
50. How is the Australian Consumer Law structured into the Competition and
Consumer Act : 2010 (Cth)?
a. They are separate statutes
b. The Australian Consumer Law is an amendment to the Trade Practices Act 1974
(Cth)
c. The Australian Consumer Law is Sch 2 to the Competition and Consumer Act
2010 (Cth)
d. The Australian Consumer Law is App 3 to the Competition and Consumer Act
2010 (Cth)
51. Century Dragon Pty Ltd is contracted to export 1000 beach balls to Minerva
Beach Surf : Club by 1 January, with payment due by 1 April. Which of the
following is correct?
a. The time of delivery is likely to be considered an essential term .
b. The time of payment is likely to be considered an essential term .
c. The time of delivery and the time of payment are likely to be considered
essential terms.
d. Neither the time of delivery nor the time of payment are likely to be considered
essential : terms unless time is expressly stated to be of the essence.

52. Which of the following is not true? Economic duress:


a. Is a threat to the innocent party’s economic interests
b. Permits the threatened party to avoid the contract
c. Needs to be distinguished from a legitimate level of commercial pressure – an
example : of an actionable level of pressure is a threat to break a contract
without any legal : justification in order to extract money from an innocent
party
d. Was found in North Ocean Shipping Co Ltd v Hyundai Construction Co Ltd
[1979] 1 QB : 705 and the threatened party was able to recover the payments
made under duress
53. A term that is vital to a contract is known as:
a. A key term
b. A condition
c. A warranty
d. An innominate term
54. A party in breach of contract is liable to pay damages for:
a. Losses occurring in the usual course of things from the breach
b. All losses whether contemplated or not
c. Remote losses only
d. Losses that should have been mitigated
55. Which of the following is NOT a method of termination?
a. Agreement
b. Frustration
c. Breach
d. Rectification
56. Parol evidence may not be admitted in which of the following circumstances?
a. To evidence an oral collateral contract
b. To evidence the identity of the parties to the contract
c. To contradict an unambiguous term
d. To explain a local custom

57. The test of whether a statement is a term or a representation is not dependent


on which : of the following?
a. The duration of time between the statement being made and the contract
being made
b. Whether the statement has been reduced to writing
c. Whether the party making the statement believed in the truth of the
statement or not
d. Reliance by one party on the skill of the other
58. Chung is looking to purchase a carwash business from Terry. Terry tells
Chung that she : serves 100-150 cars per day and that she has just concluded
an agreement to service the : fleet of cars of a local business. In reality, Terry
has only served around 15 cars per day since : her customer service received a
bad review in the local newspaper and a competing carwash : business opened
one street away. Terry intends for Chung to buy her business based on : what
she has stated. This is an example of:
unliteral mistake
negligent misrepresentation
fraudulent misrepresentation
mutual mistake

59. What does the concept “quantum meruit” mean?


The contract is terminated because of a breach by both parties
The innocent party would be unjustly enriched if they were able to retain the benefit :
without compensating the party in breach for the “amount he deserves”
Voluntarily accepting a contractual benefit
Not my deed
60.Cathy and Mel entered into a contract where Cathy was to steam clean seven
rooms in : Mel’s house at $200 per room. After cleaning three rooms, Cathy
abandoned the job for a : more valuable contract. Which of the following is
true?
As there has been substantial performance, Cathy will be able to enforce all the rights
: conferred by the contract
If the exact performance rule is applied, Cathy is entitled to payment for the work she
has : done
If the contract is divisible, Cathy is entitled to payment for the work she has done
Courts are inclined to treat contracts as indivisible and to require exact performance

61. Chang wants the court to imply an engineering custom into a term of a
contract to which : he is a party. The other party has no knowledge of this
custom. It is not contrary to any of : the express terms of the contract. The
court:
Will not imply the term in these circumstances
May imply the term
Will imply the term even if the custom is not very well known
Will not imply the term unless it is implied by statute

62. Jong complains that the barbecue he bought is not fit for purpose because the
metal : sides melted the first time he cooked with it. The manufacturer
maintains that Jong should : not have used the barbecue for slow cooking and
that it never represented that the : barbecue would be fit for the purpose of
slow cooking. What are the circumstances in which : the guarantee as to
fitness for purpose would apply?
Where Jong made known to the supplier that he was going to use the barbecue for slow
cooking meals
Where Jong bought the barbecue at auction
The guarantee always applies as it is not possible for a manufacturer or supplier to :
contract out of it
Where the unfitness for slow cooking of the barbecue was not drawn to Jong’s
attention : prior to his purchase by either the manufacturer or the supplier
63. Which of the following matters is least likely to contribute to a finding of
unconscionable : conduct in connection with goods or services?
Where conditions imposed on a customer were not necessary for the protection of the
: supplier’s interests
Where the supplier makes a commercial decision to breach the contract knowing that :
the customer will seek a legal remedy for the breach
Where the supplier acted in bad faith
Where unfair tactics were used against the customer

64. It is important to distinguish between representations and terms because:


A. Damages cannot be awarded for misrepresentation
B. Damages can only be awarded for fraudulent or negligent misrepresentation
C. There is no remedy for misrepresentation
D. The remedy of rescission is not available for misrepresentation

65. Which of the following is not an enforcement measure under the Australian
Consumer : Law?
A. An order of the court requiring a supplier to publish an advertisement
B. A disclosure order issued by the ACCC
C. A two-year probation order of the court
D. A notice issued by the ACCC requiring a supplier to substantiate a claim about
a product : they are selling

66. Which of the following is not an exception to privity?

A. Insurance
B. Agency and trust
C. Employment
D. Property law

67. James is going through a difficult time and does not feel he can confide in
anyone. : Eventually he confides in the priest of his church and asks for
support. The priest forcefully : tells James to turn over all of his money to the
church or he will be struck down by : lightening. The priest does not believe
that James will be struck down. James makes the : payments. Which of the
following is most likely legal grounds for James to avoid the : transaction?
A. Mistake
B. Undue influence
C. Duress
D. Misrepresentation

34 In which High Court decision was it said: “[T]here are two relevant circumstances
in which : a breach of contract by one party may entitle the other to terminate. The
first is where the : obligation [is] essential … The second relevant circumstance is
where there has been a : sufficiently serious breach of a non-essential term …we rest
our decision in the appeal not : upon the ground of breach of an essential obligation,
but upon application of the doctrine : respecting intermediate terms.”
A. Gumland Property Holdings Pty Ltd v Duffy Bros Fruit Market (Campbelltown)
Pty Ltd : (2008) 234 CLR 237
B. Koompahtoo Local Aboriginal Land Council v Sanpine Pty Ltd (2007) 233 CLR
115
C. Ankar Pty Ltd v National Westminster Finance (Australia) Ltd (1987) 162 CLR 549
D. Shevill v Builders Licensing Board (1982) 149 CLR 620

35. Which of the following are sources of illegality?


A. Statute
B. Statute and common law
C. Statute, common law and equity
D. Statute, common law and parties’ subjective views on morality

36. What is the main difference between the misleading or deceptive conduct
provisions in : the Trade Practices Act 1974 (Cth) and in the Australian Consumer
Law?
A. The Australian Consumer Law no longer refers to “trade or commerce”
B. The Australian Consumer Law refers to a body corporate
C. The Australian Consumer Law refers to a corporation
D. The Australian Consumer Law refers to a person

37. Non est factum means:


A. It is not [my] deed
B. It is not true
C. Mistake of fact
D. It is unknown

38. With respect to consumer guarantees, which of the following is not correct?
A. It is possible to exclude the consumer guarantees in very limited
circumstances
B. Remedies for non-compliance with the consumer guarantees depend on
whether there : was a “major failure” or not
C. With respect to fitness for purpose, a “disclosed purpose” relates to the
supplier only –: the manufacturer may make representations to the consumer,
but the consumer cannot : disclose a purpose to the manufacturer and receive a
guarantee in return
D. A manufacturer does not need to provide repair facilities or spare parts after a
certain : period

39. Which of the following is not a type of contract illegal at common law on the
grounds of : public policy?
A. Contracts to oust the jurisdiction of the courts
B. Contracts to commit a tort
C. Contracts prejudicial to the public safety
D. Champertous contracts

40. The term “transparency” in unfair contracts means:


A. It is expressed in reasonably plain language that is readily available to the
affected party
B. It is easily interpretable by the court
C. It is easily interpretable by the industry
D. It is capable of only one meaning

41. Undue influence differs from duress in that:


A. With undue influence, the contract is not voidable but void
B. With undue influence, no unlawful act is required
C. Duress arises only within a closed list of special fiduciary relationships
D. With undue influence, actual physical violence is required

42. In Howe v Teefy (1927) 27 SR (NSW) 301 where a leased racehorse was retaken
three : months into a three-year lease:
A. The court was unable to assess the damages because they were uncertain and
there was : no evidence on which they could be assessed
B. The fact that assessment was difficult did not prevent a court from granting
damages : where the plaintiff had been deprived of something of value
C. Although there was a value attributable to the plaintiff’s loss, it was too remote
to be : recoverable
D. The court was unable to assess the damages because the process was difficult
and time : consuming

43. A written contract allows Joseph, the neighbour of Sonny, to take “as many
strawberries : as he likes each strawberry season”. This contract:
A. Is void for uncertainty
B. Is valid but unenforceable because it is too uncertain
C. Is valid as the parties’ intention is clear
D. Is voidable for uncertainty
44. Which of the following is not an equitable remedy?
A. Injunction
B. Specific performance
C. Restitution
D. Exemplary damages
45. Which of the following matters is least likely to contribute to a finding of
unconscionable : conduct in connection with goods or services?

A. Where conditions imposed on a customer were not necessary for the protection
of the : supplier’s interests
B. Where the supplier makes a commercial decision to breach the contract
knowing that : the customer will seek a legal remedy for the breach
C. Where the supplier acted in bad faith
D. Where unfair tactics were used against the customer

46. Consumer guarantees as to title are contained within:


A. The “General Protections” chapter of the Australian Consumer Law
B. Section 38 of the Trade Practices Act 1974 (Cth)
C. Chapter 2 of the Australian Consumer Law
D. Sections 51-56 and ss 60-61 of the Australian Consumer Law

47. Which of the following is not one of the four ways a statute may render a contract
illegal : set out in Yango Pastoral Co v First Chicago Australia Ltd (1978) 139 CLR 410?
The contract:

A. May require an action that statute forbids


B. May be performed in a prohibited manner
C. May have been made to effect an unlawful purpose
D. May set out an unreasonable restraint

48. Which of the following is not correct with respect to the Australian Consumer
Law?

A. It is contained within Schedule 2 to the Competition and Consumer Act 2010


(Cth)
B. It applies to conduct engaged in outside of Australia
C. Owing to the Commonwealth’s limited lawmaking powers under s 51 of the
:Constitution, it applies only to corporations
D. The Australian Consumer Law is applied in each State and Territory

49. Which of the following is not one of the six elements required for a claim of
fraudulent : misrepresentation to succeed?

A. The representation must be fall


B. The representation must have been acted upon by the other party
C. The representation must be one of fact
D. The representation must be in writing (cannot be verbal only)

50. Which of the following is not a way that a person seeking to rely on an exemption
clause : can show that the clause has become part of the contract?

A. By showing it is in writing and the other party has signed the contract
B. By showing it is in writing in a document a reasonable person would expect to
contain : contractual terms and was brought to the other party’s notice
C. By showing that the parties had previously contracted on terms that included
the : exemption clause and that they intended to contract on the same basis

Correct!

D. By showing it was brought to the notice of the other party at the time of, or
subsequent : to, entering into the contract

51. Ben planned the perfect surprise birthday party for his mother: He hired a hall,
he hired : a violinist and he organised catering. Two weeks prior to the event, the
violinist called to : cancel. Ben was so upset at the loss of what he thought would be
the highlight of the : evening that he called the whole event off. Ben lost both the
deposit on the hall and the : deposit for the catering. Which of the following is
correct?

A. Ben can sue the violinist for all losses after Ben cancelled the hiring of the hall
B. Ben can recover his hire fees and catering fees as it was not his fault
C. The caterers and hall owners can sue the violinist for losses
D. Ben has a duty to mitigate his losses

52. Sandy is selling her horse, Flossy. Miranda is interested in buying Flossy. Sandy
tells : Miranda that Flossy: : • Is the best little racehorse in Australia: • Has all the
registrations, licences and permits required to race in the current season but : that
Miranda should probably get this checked herself: These two statements are:
A. Mere puff & term
B. Opinion & term
C. Opinion & representation
D. Mere puff & representation

53. What does the concept “quantum meruit” mean?

A. The contract is terminated because of a breach by both parties


B. The innocent party would be unjustly enriched if they were able to retain the
benefit : without compensating the party in breach for the “amount he
deserves”
C. Voluntarily accepting a contractual benefit
D. Not my deed

Which of the following is not an unfair practice under the Australian


Consumer Law?
a. Where a free gift is offered with the purchase an electric toothbrush and the
cost of the : “gift” toothpaste is added to the price of the toothbrush
b. Where a television advertisement for a chocolate bar shows teddy bears working
the : manufacturing lines
c. Where a brochure advertising a beauty therapy treatment consisting of time in
an : oxygen tank lists the treatment as lasting two hours, when in fact the
treatment lasts just : under one hour
d. Where a shop assistant takes money from a customer knowing that the shirt the
: customer wants is sold out, but hoping that after a few days when the
customer is told that : there are no more shirts they will purchase something
else from the shop

54. The term “transparency” in unfair contracts means:

A. It is expressed in reasonably plain language that is readily available to the


affected party
B. It is easily interpretable by the court
C. It is easily interpretable by the industry
D. It is capable of only one meaning

55. Can a person use an official position they hold for their financial advantage?

A. Of course, this is a side benefit


B. Depends on the circumstances
C. The courts are silent on this specific point
D. No because if a contract is involved, this could lead to the promoting of
corruption in : public life

56. Smithy Builders have a contract with Big Bank Pty Ltd which contains the
following : clause: “Where Smithy Builders fails to complete the contract by 7
December, it will pay a : sum of $300,000 in full and final satisfaction of its liability.”
Smithy Builders fails to complete : by 7 December. If Smithy Builders wishes to avoid
paying the sum of $300,000, what must it : prove?

A. That the clause is an unliquidated damages clause


B. That the clause is a penalty clause (to punish the builders)
C. That the clause is a liquidated damages clause
D. That the clause is a genuine pre-estimate of the loss to Big Bank Pty Ltd

58. Which of the following is correct?

A. It is prohibited to offer free gifts with purchase


B. It is prohibited to offer goods at a special price, not have a sufficient amount of
those : goods available and then attempt to upsell to customers who are drawn
to the shop for the : special price deal
C. It is not prohibited for a supplier to take a customer’s money when the supplier
believes : they won’t be able to supply the service to the customer within a
reasonable time
D. It is not prohibited for a bank to send an unsolicited credit card to its client; it
is : prohibited for a bank to send an unsolicited debit card to a person who is a
potential client : but not a yet a client

59. Which of the following is not true? A signature on a contract containing an


exemption : clause:
A. Will not guarantee the effectiveness of the clause where the doctrine of non est
factum : applies
B. Incorporates the exemption clause into the contract
C. Guarantees that an exemption clause will be effective
D. Will be effective even if one party did not read the contract

Who/what are exempt from the provisions of s18 of the Australian Consumer Law?

A. No-one is exempt
B. Corporations
C. Information providers such as media outlets
D. Advertisers

Which of the following is not a way to discharge a contract?

A. By performance
B. By operation of law
C. Through breach
D. By agreement

3. Which of the following statements about the parol evidence rule is true.?

A. It applies only to oral (parol) evidence


B. It does not apply to written contracts
C. It does not apply where it can be shown that the written contract was not
intended to : be a complete record of the agreement
D. It applies only to previous drafts of a written contract

4, In Curtis v Chemical Cleaning & Dyeing Co Ltd [1951] 1 KB 805:

A. The exemption clause in the sales agreement for the drycleaning was effective
B. The exemption clause in the sales agreement for the drycleaning was not
sufficiently : explained to the customer
C. The exemption clause in the sales agreement for the drycleaning was signed and
: therefore effective

5. Which of the following is not correct with respect to mitigation of loss?

A. There is a duty on the person claiming the damages to take all reasonable steps
to : mitigate their losses
B. Where loss could have been mitigated and was not, a person cannot claim that
loss
C. The burden of proving that losses were mitigated falls upon the party claiming
the : damages ie the defendant
D. The question of whether a person took all reasonable steps to mitigate is one of
fact : dependent upon the particular circumstances

6. Which of the following is not correct with respect to termination by

subsequent : agreement?

A. A subsequent simple contract can cancel a contract where one party has
completed their : obligations and the other has not
B. A subsequent agreement may cancel an original contract
C. A subsequent agreement must be valid to rescind an earlier contract
D. A subsequent simple contract can cancel a contract where there is still
something to be : done by each party

7. In the case of restraint of trade with respect to employment:

A. The court does not easily allow parties to contract out of their means of
employment
B. Whatever the parties determine will be accepted by the court
C. There are no restraints on employees unless set out in a contract with their
employer
D. These agreements are construed as strictly as restraints of trade between
purchaser and : vendor of a business

8. Krell v Henry [1903] 2 KB 740 illustrated what point?

A. That death can be regarded as frustration


B. That the parties were entitled to a discharge
C. That the parties must perform regardless of circumstance
D. That the parties are discharged if an event occurs which forms the basis of the
contract, : but which prevents performance at no fault of either party

9. Contingent conditions include:

A. Accord and satisfaction and consideration


B. Conditions subsequent and conditions precedent
C. Warranties and innominate terms
D. Releases and mutual terminations

10. 11.Which of the following is NOT a requirement for an assignment of a debt or


other chose : in action under various State statutes?

A. The assignment is absolute and not by way of charge;


B. Fourteen days’ notice in writing is provided to debtor;
C. the assignment is in writing;
D. If not in writing then notice of the assignment is given by notice to the debtor.

12. The parol evidence rule performs much the same function as:

A. A collateral warranty
B. An oral collateral contract
C. Consideration
D. A merger clause

13. Which of the following is not one of the special relationships in which the

onus of : proving that undue influence was not employed shifts to the denying Party?

A. Paren and child where the child is living independently


B. Solicitor and client
C. Trustee and beneficiary
D. Doctor and patient

14. How may a court view substantial performance of a contract?:

A. The contract is complete, but the uncomplete portion may be allowed for as a
reduction : of the full contract price
B. it will not grant a discharge
C. It will sever the incomplete parts
D. It will treat the contract as breached

15. How is repudiation determined by the court?


A. Subjectively
B. By considering the actions of the repudiating party
C. By viewing the circumstances of the repudiation
D. Objectively

QUIZ 3:
To claim damages a causal link must be established between what two things?
A. Between the defendant’s conduct and the actual monetary value of the
loss/damage
B. The conduct and initiating court proceedings
C. The fact that the plaintiff suffered loss or damage is sufficient. Nothing else is
required
D. The defendants breach and the plaintiff’s injury
The “neighbour principle” is often viewed as:
A. An objective test
B. A test of the reasonable person
C. A test of the foreseeability of harm
D. An identity test
In Tame v State of New South Wales (2002) 211 CLR 317 where Tame was given a false :
blood alcohol reading:
A. Whether the police officer was found to have owed Tame a duty of care was
entirely a : question of community standards
B. The psychotic depressive illness Tame developed was found to flow from the
breach of : duty of the police sergeant because it was entirely caused by his
actions that could have : been avoided had he exercised sufficient care
C. Part of the test of reasonable foreseeability is a question of fact
D. Because of the “eggshell skull” principle, pre-existing knowledge of Tame’s
susceptibility : to suffering nervous shock was not required to prove a breach
Which of the following is not correct?
A. If damages are too remote they will not be recoverable
B. The remoteness test will be satisfied where the damage suffered is of the same
type or : kind as foreseeable damage
C. In order for damage to not be too remote, it must be reasonably foreseeable
D. In The Wagon Mound No 1 and The Wagon Mound No 2, where there was an :
unfortunate combination of an oil spill, welding sparks and floating cotton
waste, the loss : was found to be not reasonably foreseeable in the
circumstances

Why was the plaintiff successful in Overseas Tankship (UK) Ltd v Miller Steamship
Co Pty : Ltd (The Wagon Mound No 2) [1967] AC 617 (PC) when another plaintiff failed
in the earlier : related Wagon Mound case?
A. The plaintiffs in the second had better legal representation
B. The legal situation had changed between the two cases
C. The court was more disposed to think about the consequences of the damage
suffered by : so many as a result of the oil spill
D. In the first case, the plaintiffs failed to show that a reasonable man would have
foreseen : the risk of damage from the oil spill
Nicola drives the forklift at her place of work, Rooze’s Roofing. Nicola always leaves
the : forklift in a certain place where she has been told to leave it, with the forks up
off the : ground. One afternoon a customer who is collecting goods from the
workshop reverses his : car into the forks on the forklift. He is injured and his car is
damaged.: What is the principle that would make Nicola’s employer liable for her
actions?
A. Contributory negligence
B. Strict liability
C. Vicarious liability
D. Voluntary assumption of risk
Where a person gives advice, that advice is relied upon and the advice is incorrect,
the : person giving the advice may be liable in negligence. Which of the following is
incorrect?
A. The “special relationship” described in Mutual Life & Citizens’ Assurance Co
Ltd v Evatt : (1968) 122 CLR 556 has been accepted as the test for determining the
existence of a duty of : care with respect to negligent misstatements
B. The test in Mutual Life & Citizens’ Assurance Co Ltd v Evatt (1968) 122 CLR 556
was : approved in San Sebastian Pty Ltd v Minister Administering
Environmental Planning &: Assessment Act 1979 (1986) 162 CLR 340
C. The duty of care only arises where the “special relationship” involves a person
seeking : advice from the advice-giver, not where the advice was given
unrequested and merely : accepted
D. The advice-giver does not need to be in the business of giving advice

In Australian Safeway Stores v Zaluzna (1987) 162 CLR 479:


A. The respondent was a lawful entrant upon the land of the respondent,
establishing a : relationship between them and there was therefore a duty of
care owed by the appellant to : avoid a foreseeable risk of injury
B. The respondent was limited by his particular status as entrant without specific
consent of : the appellant
C. The respondent contributed to the accident thus limiting damages
D. There was no liability.
Which of the following is not correct? The reform to the law of negligence that took:
place in Australia in the early 2000s:
A. Limits the scope of potential liability for negligence
B. Does not apply to claims in contract law
C. Was enacted in all Australian states and territories
D. Covers personal injury

Which of the following is not correct? The reform to the law of negligence that
took : place in Australia in the early 2000s:
a. Limits the scope of potential liability for negligence
b. Does not apply to claims in contract law
c. Was enacted in all Australian states and territories
d. Covers personal injury
Which of the following is not a provision of the Wrongs Act 1958 (Vic) relating to:
professional liability?
A. If there are differing peer professional opinions across Australia, the court may
accept : just one of those opinions
B. The court must rely on peer professional opinion even where it considers that
opinion : irrational
C. If there are differing peer professional opinions across Australia, the court may
accept all : of those opinions
D. Peer professional opinion can be considered widely accepted even where it is
not : universally accepted

Question 2:Which of the following is not correct?


A. If damages are too remote they will not be recoverable
B. The remoteness test will be satisfied where the damage suffered is of the same
type or : kind as foreseeable damage
C. In order for damage to not be too remote, it must be reasonably foreseeable
D. In The Wagon Mound No 1 and The Wagon Mound No 2, where there was an :
unfortunate combination of an oil spill, welding sparks and floating cotton waste, the
loss
: was found to be not reasonably foreseeable in the circumstances

Question 3: Which of the following is not a provision of the Wrongs Act 1958 (Vic)
relating to:
professional liability?
A. If there are differing peer professional opinions across Australia, the court may
accept : just one of those opinions
B. The court must rely on peer professional opinion even where it considers that
opinion : irrational
C. If there are differing peer professional opinions across Australia, the court may
accept all : of those opinions
D. Peer professional opinion can be considered widely accepted even where it is
not : universally accepted

QUIZ 4:
Which statements is incorrect?
a. Like most partners, sole traders have unlimited personal liability of the
business/firm
b. More complex business organisations are unsuitable for larger businesses
c. Upon incorporation, a corporation becomes a separate legal entity
d. Diverse ownership requires proper management and expertise in numerous
areas
Which statement regarding dissolution of a partnership is incorrect?
A. Partners are not entitled to have partnership property applied towards the
payment of : partnership liabilities
B. After payment of the firm’s liabilities, partners do not have the right to have
surplus assets : applied in payment of what may be due to the partners
respectively
C. On the termination of the partnership, any partner can apply to the court for a
decree to dissolve : the partnership and appoint a receiver to wind up the firm’s
business/affairs
D. Should a sequestration order be made against a partner, a creditor of the firm
cannot receive a : dividend out of the bankrupt’s separate property until all
separate creditors of the bankrupt partner : have been paid in full.

Find one incorrect statement?


A. In the absence of special statutory provision, although each partner is liable
with the others for : the whole of the debts of the firm, their liability is only joint
B. A creditor can bring only one action against members of a partnership and any
partner can insist : that the action be stayed until all other partners are joined
as parties.
C. A person admitted into an existing firm, liability may be incurred where it is
specially agreed upon
D. A creditor can enforce liability against an incoming partner whether or not
he/she is a party to : the contract

The incorrect statement?


A. Partners are not bound when another partner misappropriates monies
B. If the giving of investment advice is within the scope of the firm’s business
then all partners : are jointly and severally liable for advice which is not in a
client’s best interests
C. the client should seek a personal indemnity from a partner
D. special skill is required to bind all partners

Which of the below statements is incorrect?


A. a written and signed notice of a partner’s intention to dissolve the partnership
is required, : where no fixed term has been agreed upon for the duration of the
partnership
B. a continuing guarantee given by or to a partnership is revoked as to future
transactions if there is : a change in the constitution of the partnership
C. where a partnership continues after a fixed term has expired, the rights and
duties of the partners : remain the same, but the partnership becomes a
partnership at will
D. partnership property must be used exclusively for the purposes of the
partnership and in the : manner set out in the partnership agreement
Which legislation controls the actual registration of the firm name?
A. Corporations Act 2001 (Cth)
B. Partnership Act
C. Statute of Frauds 1677
D. Business Names Registration Act 2011 (Cth)

Which of the following statements regarding partnerships is incorrect?


A. The second element of a partnership is carrying out a business in common
B. To be a partnership there must be a mutuality of rights and obligations
C. Each partner must take an active part in the direction and management of the
firm
D. In Degiorgio v Dunn [2004] NSWSC 767 it was held that there was no
partnership because the : business was not run “in common”
Which of the following statements regarding dissolution of partnership is incorrect?
A. In the event of dissolution, losses must be met first out of profits, followed by
capital, then by : partners in proportion to their share of profits
B. A firm’s assets as contributed by partners to make up capital losses need not
be applied to pay : the firm’s debts/liabilities to non-partners
C. After dissolution, each partner is entitled to advances and residue by the firm
D. After the dissolution, each partner’s authority to bind the firm continues, so
far as necessary for : winding up partnership affairs and completing unfinished
transactions

Which is incorrect statement?


A. If a member of a firm of solicitors acting for a vendor in a sale absconds with
the deposit, : his/her partners are not liable to refund the money
B. In Lloyd v Grace, Smith & Co [1912] AC 716, the firm was held responsible for
the fraud committed : by a managing clerk of a firm, who misappropriated
property while acting within the scope of his : authority
C. In SJ Mackie Pty Ltd v Dalziell Medical Practice Pty Ltd [1989] 2 Qd it was held
that the transfer of a : share to a non-partner breaks the continuity of the firm,
constituting a new firm/partnership of the : remaining former partners and the
new member
D. Partnership agreements can contain provisions to enable the transition from
one firm to another : to be effected without the disruption of a formal winding
up.

Which of the following statements is not correct?


A. as an agent, a partner is able to bind the other partners and, as principal, be
bound by the actions : of the other partners
B. in New South Wales, Victoria, Queensland, South Australia, Western Australia
and: Tasmania provision for limited partnerships is made in the Partnership Act
C. incorporated limited partnerships have been introduced in all Australian
States and Territories
D. the Partnership Act provides that the rules of the common law and equity are
to continue in force : except insofar as they are inconsistent with the Act

What constitutes an exception to the usual number of a partnership being 20


partners?
A. If one of the partners is married-his/her husband or wife is automatically a
partner
B. The exception relates only to limited partnerships
C. No more than 20 are allowed
D. The Corporations Regulations 2001 (Cth) provide greater numbers in certain
specified : professions
Which statement regarding an outsize partnership is incorrect?
A. it has more than 20 partners
B. its partnership agreement is invalid
C. it is liable to a criminal penalty ($500)
D. its agreement does not affect the enforceability of contracts or other
arrangements made
Which finding is correct?: In Popat v Schonchhatra (1997) 3 All ER 800, the Court
decided:
A. Popat was entitled to half the profits on the sale of the business and to a share
of the profits : that had accrued after the dissolution of the partnership, but
before the final settlement of : accounts.
B. Popat was not entitled to any profits on the sale of the business, as there was
no partnership : agreement and he was further not entitled to a share of the
profits accrued after the dissolution of : the partnership, but before the final
settlement of accounts.
C. Popat was entitled to half the profits on the sale of the business, but not to a
share of the profits : that had accrued after the dissolution of the partnership,
but before the final settlement of : accounts.
D. Popat was not entitled to half the profits on the sale of the business, but he was
entitled to a : share of the profits that had accrued after the dissolution of the
partnership but before the final : settlement of accounts.

Which of the following statements is incorrect?


A. a separate legal entity can commit civil wrongs and engage in criminal conduct
B. a public company has serious compliance obligations
C. a partnership must be in writing
D. a separate legal entity is entitled to own property, pay tax and enter into
contracts

Which of the below does not apply to the Partnership Act?


A. The rules in the Partnership Acts assist in determining whether a business is
being carried on in : common
B. The partnership under the Act must be registered with ASIC
C. Each rule states a negative: i.e. that a certain fact does not of itself create a
partnership, but is : only indicative of a partnership
D. Joint or part ownership or joint tenancy, or tenancy in common whether or not
the : owners/tenants share the profits, does not of itself create a partnership as
to anything so : held/owned
Which of the following statements regarding the written law is incorrect?
A. Under s 115 of the Corporations Act 2001 (Cth), the maximum number of
persons who may form a : partnership for the acquisition of gain is 20
B. Under the Corporations Regulations 2001 (Cth), reg 2A.1.01, partnerships of
more than 20 : partners may be formed for certain professions/callings
C. The Corporations Regulations 2001 (Cth), reg 2A.1.01 sets a maxima of partners
for medical and : legal practitioners, veterinary surgeons, patent and trademark
attorneys, sharebrokers and : stockbrokers and pharmaceutical chemists
D. The Corporations Regulations 2001 (Cth), reg 2A.1.01 does not set a maxima of
partners for : architects and accountants

Which court finding is incorrect?


A. In Mercantile Credit Co Ltd v Garrod [1962] 3 All ER 1103, damages were not
recovered even : though, from the plaintiff’s perspective, the sale of the car was
within the usual course of : business.
B. In Goldberg v Jenkins (1889) 15 VLR 36, the firm was not bound to the
transaction because : borrowing money on behalf of the firm at over 60%
interest when comparable rates were between : 6% and 10% was beyond ‘the
usual way’.
C. In Construction Engineering Pty Ltd v Hexyl Pty Ltd (1985) 155 CLR 541, Hexyl
was not liable : because the construction contract appeared to be between
Construction Engineering and Tembel
D. If judgment is obtained against one or more partners of a firm, no action may
be taken against the : other partners, even if satisfaction cannot be obtained
from the partner(s) sued

Cribb v Korn (1911) 12 CLR 205 established:


A. Joint ownership creates a partnership
B. A joint tenancy is just another name for a partnership
C. The sharing of joint returns does not in itself create a partnership
D. The statutory rules can be excluded by agreement

The incorrect statement is?


A. A partner has express actual and implied actual authority to engage in certain
activities with third: parties
B. A partner cannot pledge/sell partnership property, incur and pay debts on
partnership : accounts or hire employees
C. Contravention of an agreement to restrict a partner’s authority is not binding
on the firm if notice : of the agreement has been given
D. In Polkinghorne v Holland (1934) 51 CLR 143, the firm was held liable because
Holland provided : the advice in his role as solicitor; thus in the ordinary course
of the firm’s business

Which is incorrect statements regarding limited partnerships?


a. A limited partner must not take part in the management of the business and
does not have power : to bind the firm
b. If a limited partner partakes in the management of the business, he/she is liable
as a general : partner.
c. A limited partner does not have the right to inspect the books of the firm
d. Any differences arising as to ordinary matters connected with the firm’s
business are to be : decided by a majority of the general partners.

Regarding partnerships, which of the following statements is incorrect?


A. Partnerships are created with a view to profit, so partners must make a profit: b
A partner does not have to have a direct claim to a share of the profits
B. Associations and charities are not partnerships, as profits come from ancillary
business activities : and are reinvested, not distributed as dividends to their
members
C. The Partnership Acts do not govern members of a corporation incorporated
under the provisions : of the Corporations Act 2001 (Cth), a special Act of
Parliament, or Royal Charter

Which of the below statement is incorrect?


A. Partnership property is liable to be seized for the private (personal) debt of a
partner and : made liable on a judgment against the partnership
B. A creditor who has obtained judgment in respect of the separate debt of a
partner may obtain an : order charging that partner’s interest in the
partnership property and profits with the amount of the : debt and interest.
C. A creditor who has obtained judgment in respect of the separate debt of a
partner may obtain by : an order the appointment of a receiver of that partner’s
share of profits and of any other money : which may be coming to the partner in
respect of the partnership.
D. The two sources of law relevant when considering how partners bind their
partners when dealing : with third parties are the common law (including
equity) of agency and the Partnership Act. The : common law complements the
provisions of the Partnership Act in relation to the authority that an : agent has.
What constitutes an exception to the usual number of a partnership being 20
partners?
A. If one of the partners is married-his/her husband or wife is automatically a
partner
B. The exception relates only to limited partnerships
C. No more than 20 are allowed
D. The Corporations Regulations 2001 (Cth) provide greater numbers in certain
specified : professions

Which statement is incorrect?


A. carrying on a business implies repetition (Smith v Anderson (1880) 15 Ch D 247)
B. partnership may be formed in order to undertake a single business transaction
C. in Khan v Miah [2000] 1 WLR 2123 it was held that work, such as finding,
acquiring and fitting out : a shop/restaurant, is undertaken with a view to profit
D. in Keith Spicer Ltd v Mansell [1970] 1 All ER 462 it was held that ordering goods
and opening a : joint bank account in contemplation of a business are
insufficient for a partnership

Find incorrect statement


A. A term often included in a partnership agreement that allows other partners to
purchase a : retiring/deceased partner’s interest at an agreed valuation avoids
the disruption of a formal winding
B. A partnership cannot be dissolved because the business is carried on at a loss
C. Partnerships can be dissolved because of mutual incompatibility, making it
impossible for partners : to carry on a business
D. On dissolution, partnership property can be applied towards the payment of
partnership : liabilities/debts and any surplus can be distributed among the
partners
Which below is incorrect? A firm is not bound by the acts of a partner if:

A. the act is not of the usual business of the kind carried on by the firm
B. the partner exceeds his/her actual authority in the particular matter
C. the person with whom the partner is dealing knows that he/she has exceeded
his/her authority
D. the person with whom the partner is dealing knows or believes him/her to be a
partner

Legislation in Victoria: What Act regulates partnerships in Victoria?

A. Partnership Act 1892


B. Partnership Act 1958 (Vic)
C. Corporations Act 2001 (Cth), s 20
D. Competition & Consumer Act 2010 (Cth)

Which below statements is incorrect?

A. Partners share the profits in a partnership


B. In a partnership, partners have limited liability for the business debts
C. A partner is also an agent of the partnership
D. Agreement may be implied in a partnership

Which of the below statement is incorrect?

A. Partnership property is liable to be seized for the private (personal) debt of a


partner and : made liable on a judgment against the partnership
B. A creditor who has obtained judgment in respect of the separate debt of a
partner may obtain an : order charging that partner’s interest in the
partnership property and profits with the amount of the : debt and interest.
C. A creditor who has obtained judgment in respect of the separate debt of a
partner may obtain by : an order the appointment of a receiver of that partner’s
share of profits and of any other money : which may be coming to the partner in
respect of the partnership.
D. The two sources of law relevant when considering how partners bind their
partners when dealing : with third parties are the common law (including
equity) of agency and the Partnership Act. The : common law complements the
provisions of the Partnership Act in relation to the authority that an : agent has.

Which of the following statements regarding limited partners is incorrect?

A. A limited partner may assign his/her share in the partnership with the consent
of the general : partners
B. The consent of a limited partner is not required to admit a person as a partner
C. If a limited partner has suffered his/her share of the partnership property to
be charged for a : separate debt, the other partners are entitled to dissolve the
partnership
D. A limited partner is not entitled to dissolve the partnership by notice.

Which legislation regulates partnerships in NSW?

A. Partnership Act 1892 (NSW)*


B. Competition & Consumer Act 2001 (Cth)
C. Partnership Act 1958 (Vic)
D. Partnership Act 1895

Which is incorrect statement?


A. If a member of a firm of solicitors acting for a vendor in a sale absconds with
the deposit, : his/her partners are not liable to refund the money
B. In Lloyd v Grace, Smith & Co [1912] AC 716, the firm was held responsible for
the fraud committed : by a managing clerk of a firm, who misappropriated
property while acting within the scope of his : authority
C. In SJ Mackie Pty Ltd v Dalziell Medical Practice Pty Ltd [1989] 2 Qd it was held
that the transfer of a : share to a non-partner breaks the continuity of the firm,
constituting a new firm/partnership of the : remaining former partners and the
new member
D. Partnership agreements can contain provisions to enable the transition from
one firm to another : to be effected without the disruption of a formal winding
up.

Q4: Find a statement that is incorrect


A. A retiring partner should give specific notice of their retirement to persons with
whom the firm : has had dealings to avoid being made liable for debts incurred
after retirement.
B. Persons who represents themselves, or knowingly allow themselves to be
represented, as a : partner, are liable as a partner to anyone who has on the faith
of such representation given credit to : the firm
C. A wrongful act/omission includes breach of contract or of fiduciary duty,
negligent : misrepresentation and misleading or deceptive conduct
D. Innocent partners are liable for wrongful acts of other partners made in the
ordinary conduct of : business
Q5: Which statement regarding partnership is incorrect?

A. A partnerships (or firm) is a way of gathering resources or expertise for major


projects
B. Partnerships do not avoid taking on the formality and expense of an
incorporated company
C. Partners in a partnership complement each other with their skills and bring in
new capital and : broader funding options
D. A partnership in law may exist without the partners being aware of it
Q7: Which statement regarding persons of unsound mind and minors is
incorrect?
A. a partner of unsound mind is capable of binding the firm and of being bound by
co-partners, : unless proven that he/she was of unsound mind when the
partnership was entered into and the : other partners knew this
B. if a partner who is a minor enters into a contract with a third party on behalf of
the firm, the : minor is liable as far as private assets are concerned
C. a creditor who has obtained judgment against the firm may not seize the
minor’s separate : property
D. a minor will become liable as an ordinary partner when he/she attains majority
the partnership is : not repudiated within a reasonable time
Q10: What constitutes an exception to the usual number of a partnership being 20
partners?
A. If one of the partners is married-his/her husband or wife is automatically a
partner
B. The exception relates only to limited partnerships
C. No more than 20 are allowed
D. The Corporations Regulations 2001 (Cth) provide greater numbers in certain
specified : professions
Q11: Which court decision regarding fiduciary duties is incorrect?
A. In Chan v Zacharia (1984) 154 CLR 178, the court decided that that the
fiduciary relationship : continued until the partnership had finally been wound up.
B. In United Dominion Corp Ltd v Brian Pty Ltd (1985) 157 CLR 1, the Court found
that a fiduciary : relationship, with attendant fiduciary obligations ordinarily
exist between prospective partners who : have embarked upon the conduct of
the partnership business or venture before the precise terms of : any
partnership agreement have been settled
C. In Popat v Schonchhatra (1997) 3 ALLER 800 in the absence of a statement to the
contrary, : partners are entitled to an equal share of profits
D. in Harvey v Harvey (1970) 120 CLR 529, the Court held that if a partner receives
an additional : annual fee to be “on call” for that client, the fee belongs to the
partnership.

Q13: How would a partner pledge the firm’s credit?


A. By deed
B. Only for a purpose directly connected to the firm’s business but must have
express authority : to do so
C. By agreement with other partners
D. By agreement with a majority of partners plus an appropriate credit application
Q14: Which statements is not correct?
A. if the agreement is that a person should be paid a fixed sum by the firm, he/she
may be a partner : and jointly liable to creditors of the firm
B. the rights and obligations of partners to each other arise from the partnership
agreement, the : statute and the equitable concept of the fiduciary
C. a partnership agreement must be in writing; it cannot be reached orally or by a
course of : conduct
D. the Partnership Act determines partners’ rights, duties and interests, if not
included in the : partnership agreement
Q15: Which of the statements below is incorrect?
A. A partner’s express authority must be in writing
B. A partner’s authority to buy/sell goods of a kind necessary for or usually
employed in the business : on account of the firm is implied
C. A partner cannot pledge the firm’s credit for purposes not connected with its
ordinary course of : business without express authority
D. A partner cannot pledge the firm’s assets for private debts without express
authority

Question 3: Which of the following statements regarding limited partners is incorrect?


A. A limited partner may assign his/her share in the partnership with the consent
of the general : partners
B. The consent of a limited partner is not required to admit a person as a partner
C. If a limited partner has suffered his/her share of the partnership property to
be charged for a : separate debt, the other partners are entitled to dissolve the
partnership
D. A limited partner is not entitled to dissolve the partnership by notice.

Question 7: Which of the following is NOT a ground for termination of a partnership?


A. Court order
B. Partner leaves the jurisdiction
C. Expiry of a fixed term
D. Bankruptcy of a partner

Question 10: Which of the below statement is incorrect?


A. Partnership property is liable to be seized for the private (personal) debt of a
partner and : made liable on a judgment against the partnership
B. A creditor who has obtained judgment in respect of the separate debt of a
partner may obtain an : order charging that partner’s interest in the
partnership property and profits with the amount of the : debt and interest.
C. A creditor who has obtained judgment in respect of the separate debt of a
partner may obtain by : an order the appointment of a receiver of that partner’s
share of profits and of any other money : which may be coming to the partner in
respect of the partnership.
D. The two sources of law relevant when considering how partners bind their
partners when dealing : with third parties are the common law (including
equity) of agency and the Partnership Act. The : common law complements the
provisions of the Partnership Act in relation to the authority that an : agent has.

Question 13: Incorrect statement is?


A. The owners of a corporation have unlimited liability
B. A separate legal entity is separate from its owners and managers
C. A corporation can sue and be sued in its own name
D. Complex business structures are more expensive to establish
Quiz 1_Chapters 1 -4_Session 3
Due
Jan 17 at 8:15am
Points
15
Questions
15
Available
Jan 17 at 7:59am - Jan 17 at 8:25am
26 minutes
Time Limit
15 Minutes

This quiz was locked Jan 17 at 8:25am.

Attempt History
Attempt Time Score
LATEST Attempt 1
15 minutes 5 out of 15

Score for this quiz:


5 out of 15
Submitted Jan 17 at 8:18am
This attempt took 15 minutes.

Question 1 1
/ 1 pts

In which of the following situations is the offer most likely not to have lapsed?

Correct!  
Where Wrench had an option to purchase land but, unknown to Wrench, the
seller died : prior to Wrench’s acceptance

 
Where Carter, a prospective purchaser of land, makes a counter-offer that is
rejected and: then states that he will accept the earlier offer

 
Where no time was stated for acceptance by Jane, but Ben does not accept
within a : reasonable time

 
Where Jim has become overtaken by insanity prior to accepting

Question 2 0
/ 1 pts

Extrinsic Materials: Which of the following is NOT an example of an extrinsic


source of material?

 
Law Reform Commission reports

You Answered  
Parliamentary Committee reports

orrect Answer  
The internet

 
Parliamentary Debates

Question 3 1
/ 1 pts

Substantive and procedural law: Substantive law:

Correct!  
Refers to actual rights under the law

 
Is subsidiary to procedural law

 
Refers to the formal steps to enforcement of rights and duties under the law

 
Includes the rules of evidence

Question 4 0
/ 1 pts

Which of the following statements about bilateral and unilateral contracts is


not correct?
You Answered  
More than one party is required for both bilateral and unilateral contracts

orrect Answer  
A unilateral contract does not require consideration but simply a promise to
perform an : act is sufficient

 
Both parties are obligated to perform their promises in a bilateral contract

 
Only one of the parties is obligated to perform an action in a unilateral contract

Question 5 0
/ 1 pts

The Executive Power : The Executive power is administered by:

 
The Senate and the House of Representatives

orrect Answer  
The Queen, the Prime Minister and the Cabinet

You Answered  
The Governor, the Senate and the House of Representatives

 
The Judiciary, the Senate and the House of Representatives

Question 6 0
/ 1 pts

The legislature: Exclusive powers are those that:

You Answered  
Can be exercised by either the Commonwealth or the States

orrect Answer  
Can be exercised only by the Commonwealth

 
Can be exercised only by the States

 
Where there is any inconsistency, the Commonwealth law will prevail

Question 7 0
/ 1 pts

Section 51 powers: Which of the following is not an area that the


Commonwealth Parliament has power to : legislate on under s 51 of the
Commonwealth Constitution?

orrect Answer  
Education

 
Bankruptcy and insolvency

You Answered  
Foreign corporations

 
Banking and insurance

Question 8 0
/ 1 pts

Where an agreement has been made during the course of trade or


commerce between businesses, the situation will commonly indicate that the
parties:

 
intended to create moral binding relations based on trust

orrect Answer  
intended to create binding legal relations

 
intended that there was no contract unless clearly specified in the agreement

You Answered  
intended to avoid court proceedings by inserting exclusion clauses in the
agreement

Question 9 1
/ 1 pts

Business law: Which of the following no longer regulates business or


commercial law?

Correct!  
Trade Practices Act 1974 (Cth)

 
Competition and Consumer Act 2010 (Cth)

 
Corporations Act 2001 (Cth)

 
The law of contract

Question 10 0
/ 1 pts

The High Court’s decision in Waltons Stores (Interstate) Ltd v Maher (1988)
164 CLR 387:

You Answered  
Extended the doctrine of promissory estoppel such that it applies even where
the : promisor does not create or encourage the false assumption of the
promisee

orrect Answer  
Extended the doctrine of equitable estoppels such that it applies even where
there is no : pre-existing contractual relationship between the parties
 
Restricted the doctrine of equitable estoppel such that, where the contract is of
a : commercial nature, the doctrine will not apply

 
Restricted the doctrine of promissory estoppel such that where an innocent
third party : has been affected, no remedy can be ordered

Question 11 1
/ 1 pts

Criminal proceedings: An indictable offence is:

 
A more serious civil wrong

 
A less serious civil wrong

Correct!  
A more serious criminal offence

 
A less serious criminal offence

Question 12 0
/ 1 pts

Sally puts up a sign stating that she has lost her dogs and will pay a reward
of $100 for : their return to her within three days. Which of the following
statements is correct?

 
Jim will not get the reward because he returned the dog but did not verbally :
communicate acceptance of the offer to Sally

 
Nadia will get the reward as she returned the dog to the RSPCA

orrect Answer  
Chang will not get the reward because he returned the dog knowing it was
Sally’s, but : was unaware of her offer of a reward

You Answered  
Isabel will get the reward because she was only one day late in returning the
dogs (and to : not give her the reward would be unreasonable)

Question 13 0
/ 1 pts

Which of the following statements about consideration is correct?

You Answered  
A gratuitous promise is enforceable if contained in a simple contract

 
Good consideration may consist of performing an existing legal obligation

 
Good consideration can be present, future and even “past”

orrect Answer  
Consideration need not be adequate

Question 14 0
/ 1 pts

Constitution: Section 51 Commonwealth Constitution grants what type of law


making power to the : Commonwealth Parliament?

orrect Answer  
Concurrent powers

You Answered  
exclusive powers

 
residual

 
inherent

Question 15 1
/ 1 pts

Reporting obligations: What is a binding precedent?

 
A decision of another court that is of persuasive authority

 
With respect to Australia, it is a decision of the UK Supreme Court

Correct!  
A decision of a court that binds judges in a lower court in the same court
hierarchy

 
A decision of a different court on the same subject matter

Quiz Score:
5 out of 15
4/4/22, 9:41 PM Quiz 1_Chapters 1 -4_Session 3: Business Law-T122WSB-1

Quiz 1_Chapters 1 -4_Session 3


Due
Jan 17 at 8:15am
Points
15
Questions
15
Available
Jan 17 at 7:59am - Jan 17 at 8:25am
26 minutes
Time Limit
15 Minutes

This quiz was locked Jan 17 at 8:25am.

Attempt History
Attempt Time Score
LATEST Attempt 1
15 minutes 10 out of 15

Score for this quiz:


10 out of 15
Submitted Jan 17 at 8:18am
This attempt took 15 minutes.

Question 1 1
/ 1 pts

Which of the following is not a way in which an offer can be terminated?

 
Lapse

 
Revocation before acceptance

Correct!  
Revocation after acceptance has been mailed where the postal rule applies

 
Lapse where no time was stipulated

Question 2 1
/ 1 pts

The Age of Majority Act 1977 (Vic) reduced the age for contractual capacity
of a minor

 
16

Correct!  
18

https://lms.westernsydney.edu.vn/courses/288/quizzes/3356 1/7
4/4/22, 9:41 PM Quiz 1_Chapters 1 -4_Session 3: Business Law-T122WSB-1

 
21

 
25

Question 3 0
/ 1 pts

The rule of law: Which of the following concepts is closely related to the rule
of law?

 
Representative government

 
Responsible government

orrect Answer  
Due process

You Answered  
Separation of powers

Question 4 1
/ 1 pts

Equitable remedies: Which of the following is incorrect?: Equitable remedies:

 
Are discretionary

 
Include specific performance

Correct!  
Must be awarded in a separate proceeding to common law remedies such as
damages

 
Include injunctions

Question 5 1
/ 1 pts

https://lms.westernsydney.edu.vn/courses/288/quizzes/3356 2/7
4/4/22, 9:41 PM Quiz 1_Chapters 1 -4_Session 3: Business Law-T122WSB-1

Which of the following circumstances are likely to affect the consent of one or
both : parties to a contract:

 
Mistake

 
Duress and undue influence

 
Lack of writing

Correct!  
a & b

Question 6 1
/ 1 pts

The main legal issue in Felthouse v Bindley (1862) 11CB (NS) 869 was:

 
the contract was formed due to offer and acceptance being proven

 
An offer to sell a horse was simply an invitation to treat

Correct!  
generally silence /lack of action does not constitute acceptance of an offer

 
An offer can be revoked at any time prior to acceptance

Question 7 1
/ 1 pts

Hierarchy of courts: Which of the following is higher than the Federal Court of
Australia in the federal court : system?

 
Supreme Court

 
Magistrates Court

Correct!  
High Court

 
Federal Magistrates Court

https://lms.westernsydney.edu.vn/courses/288/quizzes/3356 3/7
4/4/22, 9:41 PM Quiz 1_Chapters 1 -4_Session 3: Business Law-T122WSB-1

Question 8 0
/ 1 pts

With respect to revocation, which of the following statements is not correct?

You Answered  
Revocation need not be in words

orrect Answer  
The offeror must personally communicate the revocation to the offeree

 
The offeree may accept the offer until such time as they become aware of the
revocation

 
Where an offer has been made to the world at large, revocation does not need
to be seen : by everyone in order to be effective

Question 9 0
/ 1 pts

Law Reports: Where are decisions of the High Court of Australia found?: only
online

You Answered  
In the High Court reports

orrect Answer  
In the Commonwealth Law Reports

 
In the Common Law Reports

Question 10 0
/ 1 pts

Delegated legislation: An example of delegated legislation is the:

 
Partnership Act 1958 (Vic)

You Answered  
Local Government Act 1993 (Qld), s 25

https://lms.westernsydney.edu.vn/courses/288/quizzes/3356 4/7
4/4/22, 9:41 PM Quiz 1_Chapters 1 -4_Session 3: Business Law-T122WSB-1

 
Corporations Bill 1988 (Cth)

orrect Answer  
Corporations Regulations 2001 (Cth)

Question 11 0
/ 1 pts

A qualified acceptance may also be known as a/an:

orrect Answer  
Counteroffer

 
Invitation to treat

 
Condition subsequent

You Answered  
Conditional agreement

Question 12 1
/ 1 pts

In which of the following types of agreements is there a presumption that


parties intend : to be legally bound?

Correct!  
Partnership between husband and wife

 
Living arrangement between mother and daughter

 
Agreement between father and daughter

 
Dinner arrangement between friends

Question 13 1
/ 1 pts

Reception of English law in Australia: Which latin phrase explained the


rationale for applying English laws to the new Colony of : New South Wales?

https://lms.westernsydney.edu.vn/courses/288/quizzes/3356 5/7
4/4/22, 9:41 PM Quiz 1_Chapters 1 -4_Session 3: Business Law-T122WSB-1

 
actus reus

Correct!  
terra nullius

 
ratio decidendi

 
prima facie

Question 14 1
/ 1 pts

Jack was subpoenaed to give evidence on Rods behalf. Jack claimed Rod
promised him : $2000 to give evidence. Would Jack recover that money?

Correct!  
No - Jack’s public duty is to give evidence in response to that subpoena.

 
No this is illegal

 
Yes if Rod put that in writing

 
Always binding because promises should not be broken

Question 15 1
/ 1 pts

Which of the following is most likely to be considered an offer?

 
A catalogue of books with discounted prices shown

 
A radio advertisement for drinks at “the coolest little pub in Victoria”

Correct!  
“I’ll pay you $3,000 if you complete a total rewrite of chapters 12 and 15 of this
: textbook.”

 
“Would you allow me to pay off the $2,000 over six weeks?”

https://lms.westernsydney.edu.vn/courses/288/quizzes/3356 6/7
4/4/22, 9:41 PM Quiz 1_Chapters 1 -4_Session 3: Business Law-T122WSB-1

Quiz Score:
10 out of 15

https://lms.westernsydney.edu.vn/courses/288/quizzes/3356 7/7
Quiz 1_Chapters 1 -4_Session 3: Business Law-T122WSB-1 07/04/2022, 15:24

Quiz 1_Chapters 1 -4_Session 3


Due Jan 17 at 8:15am Points 15 Questions 15
Available Jan 17 at 7:59am - Jan 17 at 8:25am 26 minutes Time Limit 15 Minutes

This quiz was locked Jan 17 at 8:25am.

Attempt History
Attempt Time Score
LATEST Attempt 1 14 minutes 8 out of 15

Score for this quiz: 8 out of 15


Submitted Jan 17 at 8:18am
This attempt took 14 minutes.

Question 1 0 / 1 pts

Which of the following statements about consideration is correct?

A gratuitous promise is enforceable if contained in a simple contract

You Answered Good consideration may consist of performing an existing legal obligation

Good consideration can be present, future and even “past”

Correct Answer
Consideration need not be adequate

Question 2 1 / 1 pts

Sources of law – statute: Which of the following is incorrect?: Statutes may:

https://lms.westernsydney.edu.vn/courses/288/quizzes/3356?module_item_id=11128 Page 1 of 8
Quiz 1_Chapters 1 -4_Session 3: Business Law-T122WSB-1 07/04/2022, 15:24

Codify the law

Bring new laws into existence

Repeal laws

Correct!
Not overrule existing common law

Question 3 1 / 1 pts

Interpretation – extrinsic materials: In interpretation, extrinsic materials:

Are required to be referred to by a court

Are only to be referred to where the meaning of the provision is ambiguous

Correct!
Are allowed to be referred to by a court

Are to be referred to regardless of the length of delay this will cause


proceedings

Question 4 1 / 1 pts

Australian Constitution : Which Act established the federal legal and political
system and converted the separate : colonies into states?

Correct!
Commonwealth of Australia Constitution Act 1900

The Australia Act 1986

Statute of Westminster Adoption Act 1942 (Cth)

https://lms.westernsydney.edu.vn/courses/288/quizzes/3356?module_item_id=11128 Page 2 of 8
Quiz 1_Chapters 1 -4_Session 3: Business Law-T122WSB-1 07/04/2022, 15:24

Statute of Westminster 1931 (IMP)

Question 5 0 / 1 pts

Which of the following is not one of Brennan J’s six criteria for estoppel from
Waltons: Stores (Interstate) Ltd v Maher (1988) 164 CLR 387?

The promisor induced an assumption

The promisee acted in reliance on that assumption

Correct Answer
The promisor knew or was reckless as to whether the promisee intended to
act in that

You Answered The promisee will suffer a material loss if the assumption is not fulfilled

Question 6 0 / 1 pts

The main legal issue in Felthouse v Bindley (1862) 11CB (NS) 869 was:

the contract was formed due to offer and acceptance being proven

An offer to sell a horse was simply an invitation to treat

Correct Answer generally silence /lack of action does not constitute acceptance of an offer

You Answered An offer can be revoked at any time prior to acceptance

https://lms.westernsydney.edu.vn/courses/288/quizzes/3356?module_item_id=11128 Page 3 of 8
Quiz 1_Chapters 1 -4_Session 3: Business Law-T122WSB-1 07/04/2022, 15:24

Question 7 0 / 1 pts

Which of the following is most likely to be considered an offer?

A catalogue of books with discounted prices shown

A radio advertisement for drinks at “the coolest little pub in Victoria”

Correct Answer
“I’ll pay you $3,000 if you complete a total rewrite of chapters 12 and 15 of this
: textbook.”

You Answered “Would you allow me to pay off the $2,000 over six weeks?”

Question 8 0 / 1 pts

Contracts voidable by a minor do not include:

Correct Answer Those not binding unless ratified by the minor during their minority

Those binding unless repudiated by the minor during their minority

You Answered
Those binding unless repudiated by the minor within a reasonable time after
attaining : their majority

Those not binding unless ratified by the minor within a reasonable time of
attaining : majority

Question 9 0 / 1 pts

https://lms.westernsydney.edu.vn/courses/288/quizzes/3356?module_item_id=11128 Page 4 of 8
Quiz 1_Chapters 1 -4_Session 3: Business Law-T122WSB-1 07/04/2022, 15:24

The legislature: Exclusive powers are those that:

You Answered
Can be exercised by either the Commonwealth or the States

Correct Answer Can be exercised only by the Commonwealth

Can be exercised only by the States

Where there is any inconsistency, the Commonwealth law will prevail

Question 10 1 / 1 pts

Separation of powers: Which of the following statements about separation of


powers in Australia is correct?

The judiciary is the body that makes statute law.

The legislature is the branch that declares what the law is and interprets the
law.

Correct! The executive is the body that administers the law.

The legislature is the body that resolves disputes concerning the application of
law and : polices the law.

Question 11 1 / 1 pts

https://lms.westernsydney.edu.vn/courses/288/quizzes/3356?module_item_id=11128 Page 5 of 8
Quiz 1_Chapters 1 -4_Session 3: Business Law-T122WSB-1 07/04/2022, 15:24

Simone advertises a car for $4,000. John responds to the advertisement and
asks : whether Simone will take $3,000. Simone says ‘no’, but that she will
hold the offer open for : a week. The next day she sells it to Samantha. :
Which of the following statements is correct?

Simone has to hold the offer open as she made a promise

Correct!

Simone does not have to hold the offer open as no consideration was given

John’s question is an invitation to treat

Simone does not have to hold the offer open as John made a counter-offer

Question 12 1 / 1 pts

Criminal offences: Which of the following is incorrect with respect to criminal


offences?

Indictable offences are generally the more serious offences.

The prosecution must prove its case beyond reasonable doubt.

Summary offences are determined by a magistrate without a jury.

Correct! A committal hearing is held before most summary offence matters.

Question 13 1 / 1 pts

Which are some of the main features of the Electronic Transactions Act 1999
(Cth):

https://lms.westernsydney.edu.vn/courses/288/quizzes/3356?module_item_id=11128 Page 6 of 8
Quiz 1_Chapters 1 -4_Session 3: Business Law-T122WSB-1 07/04/2022, 15:24

validity of electronic transactions

recognition of writing by electronic means

Recognition of retaining information in electronic form

Correct!
All of the above

Question 14 0 / 1 pts

Civil law and criminal law: Where an act is both a crime and a tort:

The offender may only be prosecuted for the crime

Correct Answer
The State may prosecute the offender for the crime, and the victim may bring
a civil : action.

The State will prosecute the offender for both the crime and the tort.

You Answered Only a civil action may be taken against the offender.

Question 15 1 / 1 pts

Reporting obligations: What is a binding precedent?

A decision of another court that is of persuasive authority

With respect to Australia, it is a decision of the UK Supreme Court

Correct!

https://lms.westernsydney.edu.vn/courses/288/quizzes/3356?module_item_id=11128 Page 7 of 8
Quiz 1_Chapters 1 -4_Session 3: Business Law-T122WSB-1 07/04/2022, 15:24

A decision of a court that binds judges in a lower court in the same court
hierarchy

A decision of a different court on the same subject matter

Quiz Score: 8 out of 15

https://lms.westernsydney.edu.vn/courses/288/quizzes/3356?module_item_id=11128 Page 8 of 8
Quiz 2_Chapters 5 -13_Session 5
Due
Feb 14 at 11:15am
Points
15
Questions
15
Available
Feb 14 at 10:59am - Feb 14 at 11:25am
26 minutes
Time Limit
15 Minutes

This quiz was locked Feb 14 at 11:25am.

Attempt History
Attempt Time Score
LATEST Attempt 1
15 minutes 6 out of 15

Score for this quiz:


6 out of 15
Submitted Feb 14 at 11:14am
This attempt took 15 minutes.

Question 1 0
/ 1 pts

Which of the following statements about the parol evidence rule is true?

 
It applies only to oral (parol) evidence

You Answered  
It does not apply to written contracts

orrect Answer  
It does not apply where it can be shown that the written contract was not
intended to : be a complete record of the agreement

 
It applies only to previous drafts of a written contract

Question 2 0
/ 1 pts
When considering whether a contract is a standard form contract, the court
does not : have to consider:

 
The bargaining power of the parties as unequal bargaining power does not
constitute a : special disadvantage

 
Whether both parties had the opportunity to negotiate the terms of the contract

orrect Answer  
Whether one party’s vicarious liability for its agents is limited

You Answered  
Whether the terms of the contract take into account the specifics of the
particular : transaction

Question 3 1
/ 1 pts

What does the concept “quantum meruit” mean?

 
The contract is terminated because of a breach by both parties

Correct!  
The innocent party would be unjustly enriched if they were able to retain the
benefit : without compensating the party in breach for the “amount he
deserves”

 
Voluntarily accepting a contractual benefit

 
Not my deed

Question 4 0
/ 1 pts

Which of the following is the general position of the courts when awarding
damages?

orrect Answer  
Where it is possible to place an innocent part in the position he/she would
have been in : if the breach had not occurred

 
Where it is difficult to quantify the loss

 
Where the plaintiff is unable to establish that they have suffered any actual
loss

You Answered  
Where the damages are unliquidated

Question 5 1
/ 1 pts

Can a person use an official position they hold for their financial advantage?

 
Of course, this is a side benefit

 
Depends on the circumstances

 
The courts are silent on this specific point

Correct!  
No because if a contract is involved, this could lead to the promoting of
corruption in : public life
Question 6 0
/ 1 pts

Unliquidated damages are:

 
Non-monetary sums

 
Prohibited by law as they are uncertain and difficult to quantify

orrect Answer  
Damages where the court is to determine the amount

You Answered  
Damages where an amount has been fixed in the contract

Question 7 0
/ 1 pts

How is repudiation determined by the court?

 
Subjectively

You Answered  
By considering the actions of the repudiating party

 
By viewing the circumstances of the repudiation

orrect Answer  
Objectively

Question 8 0
/ 1 pts

In determining whether a statement has become a term of the contract, the


key test : applied by the courts is:

You Answered  
Whether it is in writing

orrect Answer  
Contractual intention

 
The knowledge of the parties

 
The parties’ own beliefs

Question 9 0
/ 1 pts

Marnie loans Bill $1000 so that Bill can pay a local politician to ensure his
tender will be : the winning tender. The contract between Marnie and Bill is:

You Answered  
A simple loan contract that is enforceable

 
A contract voidable at Marnie’s option

orrect Answer  
An illegal contract

 
A contract voidable at Bill’s option

Question 10 1
/ 1 pts

In Hadley v Baxendale (1854) 9 Exch 341; 156 ER 145 the court recognized
“reasonably : foreseeable losses” as being:

 
All losses that a plaintiff can prove beyond reasonable doubt

 
Losses that arise naturally from a breach of contract

 
Losses that are actually contemplated by the parties

Correct!  
b & c
Question 11 1
/ 1 pts

A unilateral mistake does not include:

 
Mistake as to the nature of the document signed

 
Mistake as to the identity of the parties

 
Mistake as to the terms of the contract

Correct!  
Mistake as to capacity of the parties

Question 12 0
/ 1 pts

Parol evidence may not be admitted in which of the following circumstances?

You Answered  
To evidence an oral collateral contract

 
To evidence the identity of the parties to the contract

orrect Answer  
To contradict an unambiguous term

 
To explain a local custom

Question 13 0
/ 1 pts

Which unfair practice involves participation in a trading scheme where


persons at the : top receive most of the benefits?

 
referral selling

orrect Answer  
pyramid selling

You Answered  
unconscionable conduct

 
misleading and deceptive conduct

Question 14 1
/ 1 pts

Which of the following is not a situation where frustration would occur?

 
Long term serious illness of a person contracted to perform a personal
services contract

 
Where the government in exercising its powers has made completion of the
contract : impossible

 
Where a change in the law renders performance of the contract illegal

Correct!  
Where the performance of the contract is rendered illegal by the destruction of
the : subject matter of the contract by one of the parties

Question 15 1
/ 1 pts

Which of the following is not true? A signature on a contract containing an


exemption : clause:
 
Will not guarantee the effectiveness of the clause where the doctrine of non
est factum : applies

 
Incorporates the exemption clause into the contract

Correct!  
Guarantees that an exemption clause will be effective

 
Will be effective even if one party did not read the contract

Quiz Score:
6 out of 15
Quiz 2_Chapters 5 -13_Session 5: Business Law-T122WSB-1 07/04/2022, 15:24

Quiz 2_Chapters 5 -13_Session 5


Due Feb 14 at 11:15am Points 15 Questions 15
Available Feb 14 at 10:59am - Feb 14 at 11:25am 26 minutes Time Limit 15 Minutes

This quiz was locked Feb 14 at 11:25am.

Attempt History
Attempt Time Score
LATEST Attempt 1 15 minutes 4 out of 15

Score for this quiz: 4 out of 15


Submitted Feb 14 at 11:15am
This attempt took 15 minutes.

Question 1 0 / 1 pts

Which of the following actions is unlikely to constitute duress?

You Answered
A threat to destroy the house of a party’s sister

A physical beating to a party until they signed a contract that they would have
signed : without the beating

Correct Answer A threat to contact police if stolen moneys were not repaid to the employer

Physical confinement of an otherwise unharmed party to pressure them to


enter into a : contract that would be for their benefit

Question 2 1 / 1 pts

https://lms.westernsydney.edu.vn/courses/288/quizzes/3361?module_item_id=11133 Page 1 of 9
Quiz 2_Chapters 5 -13_Session 5: Business Law-T122WSB-1 07/04/2022, 15:24

Which of the following is not a requirement of a valid collateral contract?

The statement must be promissory

The parties must have intended the promise to be binding

The promise must be supported by consideration

Correct!
The consideration must be of higher value than the main contract

Question 3 0 / 1 pts

Terms may not be implied into a contract by:

The court

Correct Answer Representations

Trade usage

You Answered Custom

Question 4 0 / 1 pts

In Hong Kong Fir Shipping Co Ltd v Kawasaki Kisen Kaisha Ltd [1962] 2 QB
26, the court : recognised which of the following terms which later became
part of Australian law in : Koompahtoo Local Aboriginal Land Council v
Sanpine Pty Ltd (2007) 233 CLR 115?

Transitional term

https://lms.westernsydney.edu.vn/courses/288/quizzes/3361?module_item_id=11133 Page 2 of 9
Quiz 2_Chapters 5 -13_Session 5: Business Law-T122WSB-1 07/04/2022, 15:24

You Answered Warranty

Correct Answer Innominate term

Intervening term

Question 5 1 / 1 pts

Fraser crashes his car with his friend Angus as a passenger. Angus is
injured. Angus was : not wearing a seatbelt. Which of the following is not
correct?

Liability would likely be apportioned between Fraser and Angus

Correct!
Damages cannot be apportioned for breach of contractual duty of care where
there is : contributory negligence

Damages awarded to Angus, if any, would be reduced by his contributory


negligence

Angus’ contributory negligence relates only to his own safety, not to that of
others

Question 6 0 / 1 pts

When considering whether a contract is a standard form contract, the court


does not : have to consider:

https://lms.westernsydney.edu.vn/courses/288/quizzes/3361?module_item_id=11133 Page 3 of 9
Quiz 2_Chapters 5 -13_Session 5: Business Law-T122WSB-1 07/04/2022, 15:24

You Answered
The bargaining power of the parties as unequal bargaining power does not
constitute a : special disadvantage

Whether both parties had the opportunity to negotiate the terms of the contract

Correct Answer
Whether one party’s vicarious liability for its agents is limited

Whether the terms of the contract take into account the specifics of the
particular : transaction

Question 7 1 / 1 pts

John tells Cara that the car he has for sale is “as is, where is” and “only
needs the oil : filter changed and she’s good to go”. John made this
statement based on the fact he has had : the car checked by a mechanic a
few weeks ago. Unfortunately, the car required more : significant repairs
before it was roadworthy. What remedies are available for Cara under the :
common law?

Cara can seek damages for deceit

Cara can rescind the contract under the common law

Cara can seek damages under the tort of negligence

Correct!
Cara has no remedy under the common law (if John’s statement was not a
term in the : contract)

https://lms.westernsydney.edu.vn/courses/288/quizzes/3361?module_item_id=11133 Page 4 of 9
Quiz 2_Chapters 5 -13_Session 5: Business Law-T122WSB-1 07/04/2022, 15:24

Question 8 1 / 1 pts

Which of the following is not a restraint of trade?

A price maintenance agreement

A contract restraining Laura from working as a welder within Ballarat for two
years after : her termination with Welders R Us

An agreement between John and Liz that John will not open a brew shop
within 2km of : the brew shop he just sold to Liz

Correct!
An agreement by which Steve agrees to repay his daughter’s loan debt in
return for the : person who advanced the loan to his daughter agreeing not to
report Steve’s daughter to : the police for fraud

Question 9 0 / 1 pts

Which of the following statements is not correct?

You Answered
A condition is a major term of the contract; a breach renders the contract
substantially : different from the agreed term

Correct Answer
A breach of a warranty entitles the innocent party to be compensated with
damages and : termination of the contract

https://lms.westernsydney.edu.vn/courses/288/quizzes/3361?module_item_id=11133 Page 5 of 9
Quiz 2_Chapters 5 -13_Session 5: Business Law-T122WSB-1 07/04/2022, 15:24

A breach of a condition entitles the innocent party to be compensated with


damages

A warranty is a minor term of the contract; a breach renders the contract


insubstantially : different from the agreed term

Question 10 0 / 1 pts

Which of the following is not usually recoverable by way of damages?

Correct Answer Compensation for mere inconvenience or disappointment

Loss that is difficult to estimate

Nominal loss

You Answered Expenses incurred in reliance on the other party’s promise to perform

Question 11 0 / 1 pts

Which of the following statements about the parol evidence rule is true.?

It applies only to oral (parol) evidence

You Answered
It does not apply to written contracts

Correct Answer
It does not apply where it can be shown that the written contract was not
intended to : be a complete record of the agreement

https://lms.westernsydney.edu.vn/courses/288/quizzes/3361?module_item_id=11133 Page 6 of 9
Quiz 2_Chapters 5 -13_Session 5: Business Law-T122WSB-1 07/04/2022, 15:24

It applies only to previous drafts of a written contract

Question 12 0 / 1 pts

Sandy is selling her horse, Flossy. Miranda is interested in buying Flossy.


Sandy tells : Miranda that Flossy: : • Is the best little racehorse in Australia: •
Has all the registrations, licences and permits required to race in the current
season but : that Miranda should probably get this checked herself: These
two statements are:

Mere puff & term

You Answered Opinion & term

Opinion & representation

Correct Answer Mere puff & representation

Question 13 0 / 1 pts

Undue influence differs from duress in that:

With undue influence, the contract is not voidable but void

Correct Answer With undue influence, no unlawful act is required

You Answered Duress arises only within a closed list of special fiduciary relationships

With undue influence, actual physical violence is required

https://lms.westernsydney.edu.vn/courses/288/quizzes/3361?module_item_id=11133 Page 7 of 9
Quiz 2_Chapters 5 -13_Session 5: Business Law-T122WSB-1 07/04/2022, 15:24

Question 14 0 / 1 pts

Which of the following is not correct? With respect to s 18 of the Australian


Consumer : Law, silence:

You Answered
May constitute misleading or deceptive conduct where a statement is literally
true but is : misleading without further qualification

May constitute misleading or deceptive conduct where there is a “reasonable :


expectation” that disclosure will be made

Correct Answer Is not relevant in considering the contravention

May constitute misleading or deceptive conduct where there is failure to


disclose a : change in circumstances

Question 15 0 / 1 pts

Which of the following matters is least likely to contribute to a finding of


unconscionable : conduct in connection with goods or services?

You Answered
Where conditions imposed on a customer were not necessary for the
protection of the : supplier’s interests

Correct Answer
Where the supplier makes a commercial decision to breach the contract
knowing that : the customer will seek a legal remedy for the breach

Where the supplier acted in bad faith

https://lms.westernsydney.edu.vn/courses/288/quizzes/3361?module_item_id=11133 Page 8 of 9
Quiz 2_Chapters 5 -13_Session 5: Business Law-T122WSB-1 07/04/2022, 15:24

Where unfair tactics were used against the customer

Quiz Score: 4 out of 15

https://lms.westernsydney.edu.vn/courses/288/quizzes/3361?module_item_id=11133 Page 9 of 9
4/4/22, 9:57 PM Quiz 2_Chapters 5 -13_Session 5: Business Law-T122WSB-1

Quiz 2_Chapters 5 -13_Session 5


Due
Feb 14 at 11:15am
Points
15
Questions
15
Available
Feb 14 at 10:59am - Feb 14 at 11:25am
26 minutes
Time Limit
15 Minutes

This quiz was locked Feb 14 at 11:25am.

Attempt History
Attempt Time Score
LATEST Attempt 1
15 minutes 8 out of 15

Score for this quiz:


8 out of 15
Submitted Feb 14 at 11:21am
This attempt took 15 minutes.

Question 1 1
/ 1 pts

Which of the following is not a valid assignment of a contract?

 
A bankrupt’s estate passing to the Official Trustee in Bankruptcy

Correct!  
A contract for personal services

 
A contract under which Sharma and his friend Melanie were to concrete a
driveway : devolving on Melanie after Sharma’s death

 
A contract under which Tom was to receive goods devolving on Tom’s
administrator : after letters of administration were granted

Question 2 1
/ 1 pts

In Codelfa Construction Pty Ltd v State Rail Authority of New South Wales
(1982) 149 CLR : 337:

https://lms.westernsydney.edu.vn/courses/288/quizzes/3361 1/8
4/4/22, 9:57 PM Quiz 2_Chapters 5 -13_Session 5: Business Law-T122WSB-1

 
The High Court implied a term in the agreement granting a reasonable
extension of time: b Codelfa was able to prove that the term was necessary to
make the contract work

 
Codelfa did not need to prove that the term was necessary to make the
contract work : for the term to be implied as it was sufficient that both parties
needed to be rescued from a : difficult position imposed upon them by the
injunction

Correct!  
The High Court found that the contract was frustrated

Question 3 0
/ 1 pts

Matt sells his car to Tabitha, who purchases it on the condition that she can
continue to: keep it in his garage at no cost for the next six months. Tabitha
pays in full. Two months: later, Matt tells Tabitha she will have to remove her
car from his garage as he has bought a: new car and wants to keep it in the
garage. Which of the following statements is correct?

 
If Tabitha agrees to remove her car, the contract has been terminated by
partial: performance

 
The condition that Tabitha is allowed to keep her car in Matt’s garage is a
condition: precedent

You Answered  
The condition that Tabitha is allowed to keep her car in Matt’s garage is a
condition: subsequent

orrect Answer  
If Tabitha promises to remove her car, Matt will not be able to enforce the
promise: unless the promise is supported by consideration or made under seal

https://lms.westernsydney.edu.vn/courses/288/quizzes/3361 2/8
4/4/22, 9:57 PM Quiz 2_Chapters 5 -13_Session 5: Business Law-T122WSB-1

Question 4 0
/ 1 pts

In which case was it that held that where an exemption clause is in a non-
contractual : document, the clause must be brought to the other party’s
attention for it to be effective?

orrect Answer  
Causer v Browne [1952] VLR 1

 
Olley v Marlborough Court Ltd [1949] 1 KB 532

You Answered  
Thornton v Shoe Lane Parking Ltd [1971] 2 QB 163

 
Baltic Shipping Co v Dillon (1991) 22 NSWLR 1

Question 5 0
/ 1 pts

Smithy Builders have a contract with Big Bank Pty Ltd which contains the
following : clause: “Where Smithy Builders fails to complete the contract by 7
December, it will pay a : sum of $300,000 in full and final satisfaction of its
liability.” Smithy Builders fails to complete : by 7 December. If Smithy Builders
wishes to avoid paying the sum of $300,000, what must it : prove?

 
That the clause is an unliquidated damages clause

orrect Answer  
That the clause is a penalty clause (to punish the builders)

 
That the clause is a liquidated damages clause

You Answered  
That the clause is a genuine pre-estimate of the loss to Big Bank Pty Ltd

Question 6 0
/ 1 pts

James, a chef, tells Laura, who is looking to buy his antique lounge chair,
that the chair is : of Spanish origin. Three weeks later a written contract is
drafted that makes no mention of : the chair’s origin. The statement that the
chair is of Spanish origin is most likely to be a/an:: a Mere puff

https://lms.westernsydney.edu.vn/courses/288/quizzes/3361 3/8
4/4/22, 9:57 PM Quiz 2_Chapters 5 -13_Session 5: Business Law-T122WSB-1

orrect Answer  
Representation

You Answered  
Opinion

 
Term

Question 7 1
/ 1 pts

A party repudiates a contract when

 
They decide to terminate it

Correct!  
They are no longer able or are unwilling to perform their obligations

 
The other party decides to terminate it

 
They take too long to perform an obligation that has a time limit under the
contract, such : as payment of rent

Question 8 0
/ 1 pts

Where one party believes the contract refers to the Bonny Lass, a freight
ship based out : of Norway, and the other party believes the contract refers to
the Bonny Lass, a freight ship : based out of Hong Kong, this is an example
of a:

 
Mistake of law

 
Unilateral mistake

You Answered  
Common mistake

orrect Answer  
Mutual mistake

https://lms.westernsydney.edu.vn/courses/288/quizzes/3361 4/8
4/4/22, 9:57 PM Quiz 2_Chapters 5 -13_Session 5: Business Law-T122WSB-1

Question 9 1
/ 1 pts

John tells Cara that the car he has for sale is “as is, where is” and “only
needs the oil : filter changed and she’s good to go”. John made this
statement based on the fact he has had : the car checked by a mechanic a
few weeks ago. Unfortunately, the car required more : significant repairs
before it was roadworthy. What remedies are available for Cara under the :
common law?

 
Cara can seek damages for deceit

 
Cara can rescind the contract under the common law

 
Cara can seek damages under the tort of negligence

Correct!  
Cara has no remedy under the common law (if John’s statement was not a
term in the : contract)

Question 10 1
/ 1 pts

A liquidated damages clause in a written contract:

 
States that the parties have agreed to leave the calculation of damages to the
court

 
Sets out the price of the contract

 
Sets out the agreed amount payable to the innocent party for terminating the
contract

Correct!  
Sets out the amount agreed payable to the innocent party if there is a breach
of contract

https://lms.westernsydney.edu.vn/courses/288/quizzes/3361 5/8
4/4/22, 9:57 PM Quiz 2_Chapters 5 -13_Session 5: Business Law-T122WSB-1

Question 11 1
/ 1 pts

Why are late payment charges on credit cards not penalties?

Correct!  
banks penalties on cards are legitimate to cover their costs

 
They are considered penalties but in Paciocco v ANZ Banking Group Ltd the
court held : they were not

 
The Reserve Bank states they are not

 
They are considered just a consequence of the use of credit

Question 12 0
/ 1 pts

In which case did the court find that passing off had been engaged in and the
misleading : and deceptive conduct provisions had been contravened?

orrect Answer  
Apand Pty Ltd v The Kettle Chip Co Pty Ltd (1994) 52 FCR 474

You Answered  
McWilliam’s Wines Pty Ltd v McDonald’s System of Australia Pty Ltd (1980)
33 ALR 394

 
Parkdale Custom Built Furniture Pty Ltd v Puxu Pty Ltd (1982) 149 CLR 191

 
eBay International AG v Creative Festival Entertainment Pty Ltd (2006) 170
FCR 450

Question 13 1
/ 1 pts

A divisible contract:

https://lms.westernsydney.edu.vn/courses/288/quizzes/3361 6/8
4/4/22, 9:57 PM Quiz 2_Chapters 5 -13_Session 5: Business Law-T122WSB-1

 
Is one that provides expressly that performance is due after the other party
has : performed stages of the contract

 
Is one that provides impliedly that performance is due after the other party has
: performed stages of the contract

Correct!  
Is one that provides expressly or impliedly that performance is due after the
other party : has performed stages of the contract

 
Is one that provides that exact performance of the entire contract is required
and duties : under it are not severable

Question 14 1
/ 1 pts

Which of the following sections of the Australian Consumer Law are relevant
to : misrepresentation?

 
Sections 23-25

Correct!  
Sections 18 and 29

 
Sections 20-22

 
Sections 52 and 53

Question 15 0
/ 1 pts

Which of the following is not true? A signature on a contract containing an


exemption : clause:

https://lms.westernsydney.edu.vn/courses/288/quizzes/3361 7/8
4/4/22, 9:57 PM Quiz 2_Chapters 5 -13_Session 5: Business Law-T122WSB-1

You Answered  
Will not guarantee the effectiveness of the clause where the doctrine of non
est factum : applies

 
Incorporates the exemption clause into the contract

orrect Answer  
Guarantees that an exemption clause will be effective

 
Will be effective even if one party did not read the contract

Quiz Score:
8 out of 15

https://lms.westernsydney.edu.vn/courses/288/quizzes/3361 8/8
Quiz 3_Chap 14 & 15_Session 7
Due
Feb 28 at 8:15am
Points
15
Questions
15
Available
Feb 28 at 7:59am - Feb 28 at 8:25am
26 minutes
Time Limit
15 Minutes

This quiz was locked Feb 28 at 8:25am.

Attempt History
Attempt Time Score
LATEST Attempt 1
15 minutes 7 out of 15

Score for this quiz:


7 out of 15
Submitted Feb 28 at 8:16am
This attempt took 15 minutes.

Question 1 1
/ 1 pts

The “neighbour principle” is often viewed as:

 
An objective test

 
A test of the reasonable person

Correct!  
A test of the foreseeability of harm

 
An identity test

Question 2 1
/ 1 pts

To claim damages a causal link must be established between what two


things?
 
Between the defendant’s conduct and the actual monetary value of the
loss/damage

 
The conduct and initiating court proceedings

 
The fact that the plaintiff suffered loss or damage is sufficient. Nothing else is
required

Correct!  
The defendants breach and the plaintiff’s injury

Question 3 0
/ 1 pts

Which of the following is not a provision of the Wrongs Act 1958 (Vic) relating
to: professional liability?

 
If there are differing peer professional opinions across Australia, the court may
accept : just one of those opinions

orrect Answer  
The court must rely on peer professional opinion even where it considers that
opinion : irrational

 
If there are differing peer professional opinions across Australia, the court may
accept all : of those opinions

You Answered  
Peer professional opinion can be considered widely accepted even where it is
not : universally accepted
Question 4 0
/ 1 pts

In Tame v State of New South Wales (2002) 211 CLR 317 where Tame was
given a false : blood alcohol reading:

 
Whether the police officer was found to have owed Tame a duty of care was
entirely a : question of community standards

You Answered  
The psychotic depressive illness Tame developed was found to flow from the
breach of : duty of the police sergeant because it was entirely caused by his
actions that could have : been avoided had he exercised sufficient care

orrect Answer  
Part of the test of reasonable foreseeability is a question of fact

 
Because of the “eggshell skull” principle, pre-existing knowledge of Tame’s
susceptibility : to suffering nervous shock was not required to prove a breach

Question 5 0
/ 1 pts

Where a person gives advice, that advice is relied upon and the advice is
incorrect, the : person giving the advice may be liable in negligence. Which of
the following is incorrect?

You Answered  
The “special relationship” described in Mutual Life & Citizens’ Assurance Co
Ltd v Evatt : (1968) 122 CLR 556 has been accepted as the test for
determining the existence of a duty of : care with respect to negligent
misstatements
 
The test in Mutual Life & Citizens’ Assurance Co Ltd v Evatt (1968) 122 CLR
556 was : approved in San Sebastian Pty Ltd v Minister Administering
Environmental Planning &: Assessment Act 1979 (1986) 162 CLR 340

orrect Answer  
The duty of care only arises where the “special relationship” involves a person
seeking : advice from the advice-giver, not where the advice was given
unrequested and merely : accepted

 
The advice-giver does not need to be in the business of giving advice

Question 6 1
/ 1 pts

Why was the plaintiff successful in Overseas Tankship (UK) Ltd v Miller
Steamship Co Pty : Ltd (The Wagon Mound No 2) [1967] AC 617 (PC) when
another plaintiff failed in the earlier : related Wagon Mound case?

 
The plaintiffs in the second had better legal representation

 
The legal situation had changed between the two cases

 
The court was more disposed to think about the consequences of the damage
suffered by : so many as a result of the oil spill

Correct!  
In the first case, the plaintiffs failed to show that a reasonable man would have
foreseen : the risk of damage from the oil spill

Question 7 0
/ 1 pts
Which of the following is not correct? The reform to the law of negligence that
took : place in Australia in the early 2000s:

You Answered  
Limits the scope of potential liability for negligence

orrect Answer  
Does not apply to claims in contract law

 
Was enacted in all Australian states and territories

 
Covers personal injury

Question 8 0
/ 1 pts

Which of the following is not correct?

 
If damages are too remote they will not be recoverable

You Answered  
The remoteness test will be satisfied where the damage suffered is of the
same type or : kind as foreseeable damage

 
In order for damage to not be too remote, it must be reasonably foreseeable

orrect Answer  
In The Wagon Mound No 1 and The Wagon Mound No 2, where there was an
: unfortunate combination of an oil spill, welding sparks and floating cotton
waste, the loss : was found to be not reasonably foreseeable in the
circumstances

Question 9 1
/ 1 pts
Nicola drives the forklift at her place of work, Rooze’s Roofing. Nicola always
leaves the : forklift in a certain place where she has been told to leave it, with
the forks up off the : ground. One afternoon a customer who is collecting
goods from the workshop reverses his : car into the forks on the forklift. He is
injured and his car is damaged.: What is the principle that would make
Nicola’s employer liable for her actions?

 
Contributory negligence

 
Strict liability

Correct!  
Vicarious liability

 
Voluntary assumption of risk

Question 10 1
/ 1 pts

In Australian Safeway Stores v Zaluzna (1987) 162 CLR 479:

Correct!  
The respondent was a lawful entrant upon the land of the respondent,
establishing a : relationship between them and there was therefore a duty of
care owed by the appellant to : avoid a foreseeable risk of injury

 
The respondent was limited by his particular status as entrant without specific
consent of : the appellant

 
The respondent contributed to the accident thus limiting damages

 
There was no liability.

Question 11 0
/ 1 pts
Where a person gives advice, that advice is relied upon and the advice is
incorrect, the : person giving the advice may be liable in negligence. Which of
the following is incorrect?

You Answered  
The “special relationship” described in Mutual Life & Citizens’ Assurance Co
Ltd v Evatt : (1968) 122 CLR 556 has been accepted as the test for
determining the existence of a duty of : care with respect to negligent
misstatements

 
The test in Mutual Life & Citizens’ Assurance Co Ltd v Evatt (1968) 122 CLR
556 was : approved in San Sebastian Pty Ltd v Minister Administering
Environmental Planning &: Assessment Act 1979 (1986) 162 CLR 340

orrect Answer  
The duty of care only arises where the “special relationship” involves a person
seeking : advice from the advice-giver, not where the advice was given
unrequested and merely : accepted

 
The advice-giver does not need to be in the business of giving advice

Question 12 1
/ 1 pts

Nicola drives the forklift at her place of work, Rooze’s Roofing. Nicola always
leaves the : forklift in a certain place where she has been told to leave it, with
the forks up off the : ground. One afternoon a customer who is collecting
goods from the workshop reverses his : car into the forks on the forklift. He is
injured and his car is damaged.: What is the principle that would make
Nicola’s employer liable for her actions?

 
Contributory negligence

 
Strict liability

Correct!  
Vicarious liability

 
Voluntary assumption of risk

Question 13 0
/ 1 pts

Why was the plaintiff successful in Overseas Tankship (UK) Ltd v Miller
Steamship Co Pty : Ltd (The Wagon Mound No 2) [1967] AC 617 (PC) when
another plaintiff failed in the earlier : related Wagon Mound case?

You Answered  
The plaintiffs in the second had better legal representation

 
The legal situation had changed between the two cases

 
The court was more disposed to think about the consequences of the damage
suffered by : so many as a result of the oil spill

orrect Answer  
In the first case, the plaintiffs failed to show that a reasonable man would have
foreseen : the risk of damage from the oil spill

Question 14 0
/ 1 pts

In Tame v State of New South Wales (2002) 211 CLR 317 where Tame was
given a false : blood alcohol reading:

You Answered  
Whether the police officer was found to have owed Tame a duty of care was
entirely a : question of community standards

 
The psychotic depressive illness Tame developed was found to flow from the
breach of : duty of the police sergeant because it was entirely caused by his
actions that could have : been avoided had he exercised sufficient care
orrect Answer  
Part of the test of reasonable foreseeability is a question of fact

 
Because of the “eggshell skull” principle, pre-existing knowledge of Tame’s
susceptibility : to suffering nervous shock was not required to prove a breach

Question 15 1
/ 1 pts

The “neighbour principle” is often viewed as:

 
An objective test

 
A test of the reasonable person

Correct!  
A test of the foreseeability of harm

 
An identity test

Quiz Score:
7 out of 15
Quiz 3_Chap 14 & 15_Session 7: Business Law-T122WSB-1 07/04/2022, 15:24

Quiz 3_Chap 14 & 15_Session 7


Due Feb 28 at 8:15am Points 15 Questions 15
Available Feb 28 at 7:59am - Feb 28 at 8:25am 26 minutes Time Limit 15 Minutes

This quiz was locked Feb 28 at 8:25am.

Attempt History
Attempt Time Score
LATEST Attempt 1 15 minutes 6 out of 15

Score for this quiz: 6 out of 15


Submitted Feb 28 at 8:18am
This attempt took 15 minutes.

Question 1 0 / 1 pts

Which of the following is not correct? The reform to the law of negligence that
took : place in Australia in the early 2000s:

You Answered Limits the scope of potential liability for negligence

Correct Answer Does not apply to claims in contract law

Was enacted in all Australian states and territories

Covers personal injury

Question 2 1 / 1 pts

In Australian Safeway Stores v Zaluzna (1987) 162 CLR 479:

https://lms.westernsydney.edu.vn/courses/288/quizzes/3572?module_item_id=11528 Page 1 of 9
Quiz 3_Chap 14 & 15_Session 7: Business Law-T122WSB-1 07/04/2022, 15:24

Correct!
The respondent was a lawful entrant upon the land of the respondent,
establishing a : relationship between them and there was therefore a duty of
care owed by the appellant to : avoid a foreseeable risk of injury

The respondent was limited by his particular status as entrant without specific
consent of : the appellant

The respondent contributed to the accident thus limiting damages

There was no liability.

Question 3 0 / 1 pts

Nicola drives the forklift at her place of work, Rooze’s Roofing. Nicola always
leaves the : forklift in a certain place where she has been told to leave it, with
the forks up off the : ground. One afternoon a customer who is collecting
goods from the workshop reverses his : car into the forks on the forklift. He is
injured and his car is damaged.: What is the principle that would make
Nicola’s employer liable for her actions?

You Answered Contributory negligence

Strict liability

Correct Answer Vicarious liability

Voluntary assumption of risk

Question 4 1 / 1 pts

https://lms.westernsydney.edu.vn/courses/288/quizzes/3572?module_item_id=11528 Page 2 of 9
Quiz 3_Chap 14 & 15_Session 7: Business Law-T122WSB-1 07/04/2022, 15:24

In Tame v State of New South Wales (2002) 211 CLR 317 where Tame was
given a false : blood alcohol reading:

Whether the police officer was found to have owed Tame a duty of care was
entirely a : question of community standards

The psychotic depressive illness Tame developed was found to flow from the
breach of : duty of the police sergeant because it was entirely caused by his
actions that could have : been avoided had he exercised sufficient care

Correct!
Part of the test of reasonable foreseeability is a question of fact

Because of the “eggshell skull” principle, pre-existing knowledge of Tame’s


susceptibility : to suffering nervous shock was not required to prove a breach

Question 5 1 / 1 pts

To claim damages a causal link must be established between what two


things?

Between the defendant’s conduct and the actual monetary value of the
loss/damage

The conduct and initiating court proceedings

The fact that the plaintiff suffered loss or damage is sufficient. Nothing else is
required

Correct!
The defendants breach and the plaintiff’s injury

https://lms.westernsydney.edu.vn/courses/288/quizzes/3572?module_item_id=11528 Page 3 of 9
Quiz 3_Chap 14 & 15_Session 7: Business Law-T122WSB-1 07/04/2022, 15:24

Question 6 0 / 1 pts

Where a person gives advice, that advice is relied upon and the advice is
incorrect, the : person giving the advice may be liable in negligence. Which of
the following is incorrect?

You Answered
The “special relationship” described in Mutual Life & Citizens’ Assurance Co
Ltd v Evatt : (1968) 122 CLR 556 has been accepted as the test for
determining the existence of a duty of : care with respect to negligent
misstatements

The test in Mutual Life & Citizens’ Assurance Co Ltd v Evatt (1968) 122 CLR
556 was : approved in San Sebastian Pty Ltd v Minister Administering
Environmental Planning &: Assessment Act 1979 (1986) 162 CLR 340

Correct Answer
The duty of care only arises where the “special relationship” involves a person
seeking : advice from the advice-giver, not where the advice was given
unrequested and merely : accepted

The advice-giver does not need to be in the business of giving advice

Question 7 0 / 1 pts

Which of the following is not correct?

You Answered If damages are too remote they will not be recoverable

The remoteness test will be satisfied where the damage suffered is of the
same type or : kind as foreseeable damage

https://lms.westernsydney.edu.vn/courses/288/quizzes/3572?module_item_id=11528 Page 4 of 9
Quiz 3_Chap 14 & 15_Session 7: Business Law-T122WSB-1 07/04/2022, 15:24

In order for damage to not be too remote, it must be reasonably foreseeable

Correct Answer
In The Wagon Mound No 1 and The Wagon Mound No 2, where there was an
: unfortunate combination of an oil spill, welding sparks and floating cotton
waste, the loss : was found to be not reasonably foreseeable in the
circumstances

Question 8 1 / 1 pts

Why was the plaintiff successful in Overseas Tankship (UK) Ltd v Miller
Steamship Co Pty : Ltd (The Wagon Mound No 2) [1967] AC 617 (PC) when
another plaintiff failed in the earlier : related Wagon Mound case?

The plaintiffs in the second had better legal representation

The legal situation had changed between the two cases

The court was more disposed to think about the consequences of the damage
suffered by : so many as a result of the oil spill

Correct!
In the first case, the plaintiffs failed to show that a reasonable man would have
foreseen : the risk of damage from the oil spill

Question 9 0 / 1 pts

Which of the following is not a provision of the Wrongs Act 1958 (Vic) relating
to: professional liability?

https://lms.westernsydney.edu.vn/courses/288/quizzes/3572?module_item_id=11528 Page 5 of 9
Quiz 3_Chap 14 & 15_Session 7: Business Law-T122WSB-1 07/04/2022, 15:24

If there are differing peer professional opinions across Australia, the court may
accept : just one of those opinions

Correct Answer
The court must rely on peer professional opinion even where it considers that
opinion : irrational

You Answered
If there are differing peer professional opinions across Australia, the court
may accept all : of those opinions

Peer professional opinion can be considered widely accepted even where it is


not : universally accepted

Question 10 1 / 1 pts

The “neighbour principle” is often viewed as:

An objective test

A test of the reasonable person

Correct! A test of the foreseeability of harm

An identity test

Question 11 0 / 1 pts

https://lms.westernsydney.edu.vn/courses/288/quizzes/3572?module_item_id=11528 Page 6 of 9
Quiz 3_Chap 14 & 15_Session 7: Business Law-T122WSB-1 07/04/2022, 15:24

Which of the following is not a provision of the Wrongs Act 1958 (Vic) relating
to: professional liability?

You Answered
If there are differing peer professional opinions across Australia, the court
may accept : just one of those opinions

Correct Answer
The court must rely on peer professional opinion even where it considers that
opinion : irrational

If there are differing peer professional opinions across Australia, the court may
accept all : of those opinions

Peer professional opinion can be considered widely accepted even where it is


not : universally accepted

Question 12 0 / 1 pts

Which of the following is not correct?

You Answered If damages are too remote they will not be recoverable

The remoteness test will be satisfied where the damage suffered is of the
same type or : kind as foreseeable damage

In order for damage to not be too remote, it must be reasonably foreseeable

Correct Answer

https://lms.westernsydney.edu.vn/courses/288/quizzes/3572?module_item_id=11528 Page 7 of 9
Quiz 3_Chap 14 & 15_Session 7: Business Law-T122WSB-1 07/04/2022, 15:24

In The Wagon Mound No 1 and The Wagon Mound No 2, where there was an
: unfortunate combination of an oil spill, welding sparks and floating cotton
waste, the loss : was found to be not reasonably foreseeable in the
circumstances

Question 13 1 / 1 pts

Why was the plaintiff successful in Overseas Tankship (UK) Ltd v Miller
Steamship Co Pty : Ltd (The Wagon Mound No 2) [1967] AC 617 (PC) when
another plaintiff failed in the earlier : related Wagon Mound case?

The plaintiffs in the second had better legal representation

The legal situation had changed between the two cases

The court was more disposed to think about the consequences of the damage
suffered by : so many as a result of the oil spill

Correct!
In the first case, the plaintiffs failed to show that a reasonable man would have
foreseen : the risk of damage from the oil spill

Question 14 0 / 1 pts

In Tame v State of New South Wales (2002) 211 CLR 317 where Tame was
given a false : blood alcohol reading:

Whether the police officer was found to have owed Tame a duty of care was
entirely a : question of community standards

https://lms.westernsydney.edu.vn/courses/288/quizzes/3572?module_item_id=11528 Page 8 of 9
Quiz 3_Chap 14 & 15_Session 7: Business Law-T122WSB-1 07/04/2022, 15:24

The psychotic depressive illness Tame developed was found to flow from the
breach of : duty of the police sergeant because it was entirely caused by his
actions that could have : been avoided had he exercised sufficient care

Correct Answer Part of the test of reasonable foreseeability is a question of fact

You Answered
Because of the “eggshell skull” principle, pre-existing knowledge of Tame’s
susceptibility : to suffering nervous shock was not required to prove a breach

Question 15 0 / 1 pts

Which of the following is not correct? The reform to the law of negligence that
took : place in Australia in the early 2000s:

You Answered Limits the scope of potential liability for negligence

Correct Answer Does not apply to claims in contract law

Was enacted in all Australian states and territories

Covers personal injury

Quiz Score: 6 out of 15

https://lms.westernsydney.edu.vn/courses/288/quizzes/3572?module_item_id=11528 Page 9 of 9
4/4/22, 10:11 PM Quiz 3_Chap 14 & 15_Session 7: Business Law-T122WSB-1

Quiz 3_Chap 14 & 15_Session 7


Due
Feb 28 at 8:15am
Points
15
Questions
15
Available
Feb 28 at 7:59am - Feb 28 at 8:25am
26 minutes
Time Limit
15 Minutes

This quiz was locked Feb 28 at 8:25am.

Attempt History
Attempt Time Score
LATEST Attempt 1
15 minutes 7 out of 15

Score for this quiz:


7 out of 15
Submitted Feb 28 at 8:17am
This attempt took 15 minutes.

Question 1 1
/ 1 pts

To claim damages a causal link must be established between what two


things?

 
Between the defendant’s conduct and the actual monetary value of the
loss/damage

 
The conduct and initiating court proceedings

 
The fact that the plaintiff suffered loss or damage is sufficient. Nothing else is
required

Correct!  
The defendants breach and the plaintiff’s injury

Question 2 1
/ 1 pts

Nicola drives the forklift at her place of work, Rooze’s Roofing. Nicola always
leaves the : forklift in a certain place where she has been told to leave it, with

https://lms.westernsydney.edu.vn/courses/288/quizzes/3572 1/8
4/4/22, 10:11 PM Quiz 3_Chap 14 & 15_Session 7: Business Law-T122WSB-1

the forks up off the : ground. One afternoon a customer who is collecting
goods from the workshop reverses his : car into the forks on the forklift. He is
injured and his car is damaged.: What is the principle that would make
Nicola’s employer liable for her actions?

 
Contributory negligence

 
Strict liability

Correct!  
Vicarious liability

 
Voluntary assumption of risk

Question 3 1
/ 1 pts

The “neighbour principle” is often viewed as:

 
An objective test

 
A test of the reasonable person

Correct!  
A test of the foreseeability of harm

 
An identity test

Question 4 1
/ 1 pts

Why was the plaintiff successful in Overseas Tankship (UK) Ltd v Miller
Steamship Co Pty : Ltd (The Wagon Mound No 2) [1967] AC 617 (PC) when
another plaintiff failed in the earlier : related Wagon Mound case?

 
The plaintiffs in the second had better legal representation

 
The legal situation had changed between the two cases

https://lms.westernsydney.edu.vn/courses/288/quizzes/3572 2/8
4/4/22, 10:11 PM Quiz 3_Chap 14 & 15_Session 7: Business Law-T122WSB-1

 
The court was more disposed to think about the consequences of the damage
suffered by : so many as a result of the oil spill

Correct!  
In the first case, the plaintiffs failed to show that a reasonable man would have
foreseen : the risk of damage from the oil spill

Question 5 0
/ 1 pts

In Australian Safeway Stores v Zaluzna (1987) 162 CLR 479:

orrect Answer  
The respondent was a lawful entrant upon the land of the respondent,
establishing a : relationship between them and there was therefore a duty of
care owed by the appellant to : avoid a foreseeable risk of injury

You Answered  
The respondent was limited by his particular status as entrant without specific
consent of : the appellant

 
The respondent contributed to the accident thus limiting damages

 
There was no liability.

Question 6 0
/ 1 pts

Which of the following is not correct? The reform to the law of negligence that
took : place in Australia in the early 2000s:

You Answered  
Limits the scope of potential liability for negligence

orrect Answer  
Does not apply to claims in contract law

 
Was enacted in all Australian states and territories

https://lms.westernsydney.edu.vn/courses/288/quizzes/3572 3/8
4/4/22, 10:11 PM Quiz 3_Chap 14 & 15_Session 7: Business Law-T122WSB-1

 
Covers personal injury

Question 7 0
/ 1 pts

Which of the following is not correct?

 
If damages are too remote they will not be recoverable

You Answered  
The remoteness test will be satisfied where the damage suffered is of the
same type or : kind as foreseeable damage

 
In order for damage to not be too remote, it must be reasonably foreseeable

orrect Answer  
In The Wagon Mound No 1 and The Wagon Mound No 2, where there was an
: unfortunate combination of an oil spill, welding sparks and floating cotton
waste, the loss : was found to be not reasonably foreseeable in the
circumstances

Question 8 0
/ 1 pts

Where a person gives advice, that advice is relied upon and the advice is
incorrect, the : person giving the advice may be liable in negligence. Which of
the following is incorrect?

 
The “special relationship” described in Mutual Life & Citizens’ Assurance Co
Ltd v Evatt : (1968) 122 CLR 556 has been accepted as the test for
determining the existence of a duty of : care with respect to negligent
misstatements

You Answered  
The test in Mutual Life & Citizens’ Assurance Co Ltd v Evatt (1968) 122 CLR
556 was : approved in San Sebastian Pty Ltd v Minister Administering
Environmental Planning &: Assessment Act 1979 (1986) 162 CLR 340

https://lms.westernsydney.edu.vn/courses/288/quizzes/3572 4/8
4/4/22, 10:11 PM Quiz 3_Chap 14 & 15_Session 7: Business Law-T122WSB-1

orrect Answer  
The duty of care only arises where the “special relationship” involves a person
seeking : advice from the advice-giver, not where the advice was given
unrequested and merely : accepted

 
The advice-giver does not need to be in the business of giving advice

Question 9 0
/ 1 pts

Which of the following is not a provision of the Wrongs Act 1958 (Vic) relating
to: professional liability?

 
If there are differing peer professional opinions across Australia, the court may
accept : just one of those opinions

orrect Answer  
The court must rely on peer professional opinion even where it considers that
opinion : irrational

You Answered  
If there are differing peer professional opinions across Australia, the court
may accept all : of those opinions

 
Peer professional opinion can be considered widely accepted even where it is
not : universally accepted

Question 10 0
/ 1 pts

In Tame v State of New South Wales (2002) 211 CLR 317 where Tame was
given a false : blood alcohol reading:

https://lms.westernsydney.edu.vn/courses/288/quizzes/3572 5/8
4/4/22, 10:11 PM Quiz 3_Chap 14 & 15_Session 7: Business Law-T122WSB-1

 
Whether the police officer was found to have owed Tame a duty of care was
entirely a : question of community standards

You Answered  
The psychotic depressive illness Tame developed was found to flow from the
breach of : duty of the police sergeant because it was entirely caused by his
actions that could have : been avoided had he exercised sufficient care

orrect Answer  
Part of the test of reasonable foreseeability is a question of fact

 
Because of the “eggshell skull” principle, pre-existing knowledge of Tame’s
susceptibility : to suffering nervous shock was not required to prove a breach

Question 11 1
/ 1 pts

Nicola drives the forklift at her place of work, Rooze’s Roofing. Nicola always
leaves the : forklift in a certain place where she has been told to leave it, with
the forks up off the : ground. One afternoon a customer who is collecting
goods from the workshop reverses his : car into the forks on the forklift. He is
injured and his car is damaged.: What is the principle that would make
Nicola’s employer liable for her actions?

 
Contributory negligence

 
Strict liability

Correct!  
Vicarious liability

 
Voluntary assumption of risk

Question 12 0
/ 1 pts

Which of the following is not a provision of the Wrongs Act 1958 (Vic) relating
to: professional liability?

https://lms.westernsydney.edu.vn/courses/288/quizzes/3572 6/8
4/4/22, 10:11 PM Quiz 3_Chap 14 & 15_Session 7: Business Law-T122WSB-1

 
If there are differing peer professional opinions across Australia, the court may
accept : just one of those opinions

orrect Answer  
The court must rely on peer professional opinion even where it considers that
opinion : irrational

You Answered  
If there are differing peer professional opinions across Australia, the court
may accept all : of those opinions

 
Peer professional opinion can be considered widely accepted even where it is
not : universally accepted

Question 13 0
/ 1 pts

Which of the following is not correct?

 
If damages are too remote they will not be recoverable

You Answered  
The remoteness test will be satisfied where the damage suffered is of the
same type or : kind as foreseeable damage

 
In order for damage to not be too remote, it must be reasonably foreseeable

orrect Answer  
In The Wagon Mound No 1 and The Wagon Mound No 2, where there was an
: unfortunate combination of an oil spill, welding sparks and floating cotton
waste, the loss : was found to be not reasonably foreseeable in the
circumstances

Question 14 1
/ 1 pts

https://lms.westernsydney.edu.vn/courses/288/quizzes/3572 7/8
4/4/22, 10:11 PM Quiz 3_Chap 14 & 15_Session 7: Business Law-T122WSB-1

In Tame v State of New South Wales (2002) 211 CLR 317 where Tame was
given a false : blood alcohol reading:

 
Whether the police officer was found to have owed Tame a duty of care was
entirely a : question of community standards

 
The psychotic depressive illness Tame developed was found to flow from the
breach of : duty of the police sergeant because it was entirely caused by his
actions that could have : been avoided had he exercised sufficient care

Correct!  
Part of the test of reasonable foreseeability is a question of fact

 
Because of the “eggshell skull” principle, pre-existing knowledge of Tame’s
susceptibility : to suffering nervous shock was not required to prove a breach

Question 15 1
/ 1 pts

The “neighbour principle” is often viewed as:

 
An objective test

 
A test of the reasonable person

Correct!  
A test of the foreseeability of harm

 
An identity test

Quiz Score:
7 out of 15

https://lms.westernsydney.edu.vn/courses/288/quizzes/3572 8/8
Quiz 4_Chapters 16&17_Session 10
Due
Mar 21 at 8:15am
Points
15
Questions
15
Available
Mar 21 at 7:59am - Mar 21 at 8:25am
26 minutes
Time Limit
15 Minutes

This quiz was locked Mar 21 at 8:25am.

Attempt History
Attempt Time Score
LATEST Attempt 1
15 minutes 8 out of 15

Score for this quiz:


8 out of 15
Submitted Mar 21 at 8:19am
This attempt took 15 minutes.

Question 1 1
/ 1 pts

Which below statements is incorrect?

 
Partners share the profits in a partnership

Correct!  
In a partnership, partners have limited liability for the business debts

 
A partner is also an agent of the partnership

 
Agreement may be implied in a partnership

Question 2 1
/ 1 pts

Legislation in Victoria: What Act regulates partnerships in Victoria?

 
Partnership Act 1892

Correct!  
Partnership Act 1958 (Vic)

 
Corporations Act 2001 (Cth), s 20

 
Competition & Consumer Act 2010 (Cth)

Question 3 1
/ 1 pts

Which statement regarding persons of unsound mind and minors is


incorrect?

 
a partner of unsound mind is capable of binding the firm and of being bound
by co-partners, : unless proven that he/she was of unsound mind when the
partnership was entered into and the : other partners knew this

Correct!  
if a partner who is a minor enters into a contract with a third party on behalf of
the firm, the : minor is liable as far as private assets are concerned

 
a creditor who has obtained judgment against the firm may not seize the
minor’s separate : property

 
a minor will become liable as an ordinary partner when he/she attains majority
the partnership is : not repudiated within a reasonable time

Question 4 0
/ 1 pts

How would a partner pledge the firm’s credit?


 
By deed

orrect Answer  
Only for a purpose directly connected to the firm’s business but must have
express authority : to do so

You Answered  
By agreement with other partners

 
By agreement with a majority of partners plus an appropriate credit application

Question 5 1
/ 1 pts

Which of the below statements is incorrect?

Correct!  
a written and signed notice of a partner’s intention to dissolve the partnership
is required, : where no fixed term has been agreed upon for the duration of the
partnership

 
a continuing guarantee given by or to a partnership is revoked as to future
transactions if there is : a change in the constitution of the partnership

 
where a partnership continues after a fixed term has expired, the rights and
duties of the partners : remain the same, but the partnership becomes a
partnership at will

 
partnership property must be used exclusively for the purposes of the
partnership and in the : manner set out in the partnership agreement
Question 6 1
/ 1 pts

Which is incorrect statement?

Correct!  
If a member of a firm of solicitors acting for a vendor in a sale absconds with
the deposit, : his/her partners are not liable to refund the money

 
In Lloyd v Grace, Smith & Co [1912] AC 716, the firm was held responsible for
the fraud committed : by a managing clerk of a firm, who misappropriated
property while acting within the scope of his : authority

 
In SJ Mackie Pty Ltd v Dalziell Medical Practice Pty Ltd [1989] 2 Qd it was
held that the transfer of a : share to a non-partner breaks the continuity of the
firm, constituting a new firm/partnership of the : remaining former partners and
the new member

 
Partnership agreements can contain provisions to enable the transition from
one firm to another : to be effected without the disruption of a formal winding
up.

Question 7 0
/ 1 pts

Which of the below statement is incorrect?

orrect Answer  
Partnership property is liable to be seized for the private (personal) debt of a
partner and : made liable on a judgment against the partnership
 
A creditor who has obtained judgment in respect of the separate debt of a
partner may obtain an : order charging that partner’s interest in the partnership
property and profits with the amount of the : debt and interest.

You Answered  
A creditor who has obtained judgment in respect of the separate debt of a
partner may obtain by : an order the appointment of a receiver of that
partner’s share of profits and of any other money : which may be coming to
the partner in respect of the partnership.

 
The two sources of law relevant when considering how partners bind their
partners when dealing : with third parties are the common law (including
equity) of agency and the Partnership Act. The : common law complements
the provisions of the Partnership Act in relation to the authority that an : agent
has.

Question 8 1
/ 1 pts

Which of the following statements regarding limited partners is incorrect?

 
A limited partner may assign his/her share in the partnership with the consent
of the general : partners

 
The consent of a limited partner is not required to admit a person as a partner

Correct!  
If a limited partner has suffered his/her share of the partnership property to be
charged for a : separate debt, the other partners are entitled to dissolve the
partnership

 
A limited partner is not entitled to dissolve the partnership by notice.

Question 9 0
/ 1 pts

Which statement regarding partnership is incorrect?

 
A partnerships (or firm) is a way of gathering resources or expertise for major
projects

orrect Answer  
Partnerships do not avoid taking on the formality and expense of an
incorporated company

 
Partners in a partnership complement each other with their skills and bring in
new capital and : broader funding options

You Answered  
A partnership in law may exist without the partners being aware of it

Question 10 1
/ 1 pts

Regarding partnerships, which of the following statements is incorrect?

Correct!  
Partnerships are created with a view to profit, so partners must make a profit:
b A partner does not have to have a direct claim to a share of the profits

 
Associations and charities are not partnerships, as profits come from ancillary
business activities : and are reinvested, not distributed as dividends to their
members
 
The Partnership Acts do not govern members of a corporation incorporated
under the provisions : of the Corporations Act 2001 (Cth), a special Act of
Parliament, or Royal Charter

Question 11 1
/ 1 pts

Which of the following statements is not correct?

 
as an agent, a partner is able to bind the other partners and, as principal, be
bound by the actions : of the other partners

 
in New South Wales, Victoria, Queensland, South Australia, Western Australia
and: Tasmania provision for limited partnerships is made in the Partnership Act

Correct!  
incorporated limited partnerships have been introduced in all Australian States
and Territories

 
the Partnership Act provides that the rules of the common law and equity are
to continue in force : except insofar as they are inconsistent with the Act

Question 12 0
/ 1 pts

Which court finding is incorrect?


orrect Answer  
In Mercantile Credit Co Ltd v Garrod [1962] 3 All ER 1103, damages were not
recovered even : though, from the plaintiff’s perspective, the sale of the car
was within the usual course of : business.

You Answered  
In Goldberg v Jenkins (1889) 15 VLR 36, the firm was not bound to the
transaction because : borrowing money on behalf of the firm at over 60%
interest when comparable rates were between : 6% and 10% was beyond ‘the
usual way’.

 
In Construction Engineering Pty Ltd v Hexyl Pty Ltd (1985) 155 CLR 541,
Hexyl was not liable : because the construction contract appeared to be
between Construction Engineering and Tembel

 
If judgment is obtained against one or more partners of a firm, no action may
be taken against the : other partners, even if satisfaction cannot be obtained
from the partner(s) sued

Question 13 0
/ 1 pts

Which finding is correct?: In Popat v Schonchhatra (1997) 3 All ER 800, the


Court decided:

orrect Answer  
Popat was entitled to half the profits on the sale of the business and to a share
of the profits : that had accrued after the dissolution of the partnership, but
before the final settlement of : accounts.

 
Popat was not entitled to any profits on the sale of the business, as there was
no partnership : agreement and he was further not entitled to a share of the
profits accrued after the dissolution of : the partnership, but before the final
settlement of accounts.
 
Popat was entitled to half the profits on the sale of the business, but not to a
share of the profits : that had accrued after the dissolution of the partnership,
but before the final settlement of : accounts.

You Answered  
Popat was not entitled to half the profits on the sale of the business, but he
was entitled to a : share of the profits that had accrued after the dissolution of
the partnership but before the final : settlement of accounts.

Question 14 0
/ 1 pts

Which statement regarding an outsize partnership is incorrect?

 
it has more than 20 partners

orrect Answer  
its partnership agreement is invalid

You Answered  
it is liable to a criminal penalty ($500)

 
its agreement does not affect the enforceability of contracts or other
arrangements made

Question 15 0
/ 1 pts

Which of the below does not apply to the Partnership Act?

 
The rules in the Partnership Acts assist in determining whether a business is
being carried on in : common
orrect Answer  
The partnership under the Act must be registered with ASIC

 
Each rule states a negative: i.e. that a certain fact does not of itself create a
partnership, but is : only indicative of a partnership

You Answered  
Joint or part ownership or joint tenancy, or tenancy in common whether or not
the : owners/tenants share the profits, does not of itself create a partnership
as to anything so : held/owned

Quiz Score:
8 out of 15
Quiz 4_Chapters 16&17_Session 10: Business Law-T122WSB-1 07/04/2022, 15:24

Quiz 4_Chapters 16&17_Session 10


Due Mar 21 at 8:15am Points 15 Questions 15
Available Mar 21 at 7:59am - Mar 21 at 8:25am 26 minutes Time Limit 15 Minutes

This quiz was locked Mar 21 at 8:25am.

Attempt History
Attempt Time Score
LATEST Attempt 1 15 minutes 6 out of 15

Score for this quiz: 6 out of 15


Submitted Mar 21 at 8:16am
This attempt took 15 minutes.

Question 1 1 / 1 pts

Which of the following statements is incorrect?

a separate legal entity can commit civil wrongs and engage in criminal conduct

a public company has serious compliance obligations

Correct!
a partnership must be in writing

a separate legal entity is entitled to own property, pay tax and enter into
contracts

Question 2 1 / 1 pts

https://lms.westernsydney.edu.vn/courses/288/quizzes/3579?module_item_id=11541 Page 1 of 9
Quiz 4_Chapters 16&17_Session 10: Business Law-T122WSB-1 07/04/2022, 15:24

Which is incorrect statements regarding limited partnerships?

A limited partner must not take part in the management of the business and
does not have power : to bind the firm

If a limited partner partakes in the management of the business, he/she is


liable as a general : partner.

Correct!
A limited partner does not have the right to inspect the books of the firm

Any differences arising as to ordinary matters connected with the firm’s


business are to be : decided by a majority of the general partners.

Question 3 0 / 1 pts

Which statements is not correct?

if the agreement is that a person should be paid a fixed sum by the firm,
he/she may be a partner : and jointly liable to creditors of the firm

You Answered
the rights and obligations of partners to each other arise from the partnership
agreement, the : statute and the equitable concept of the fiduciary

Correct Answer
a partnership agreement must be in writing; it cannot be reached orally or by a
course of : conduct

https://lms.westernsydney.edu.vn/courses/288/quizzes/3579?module_item_id=11541 Page 2 of 9
Quiz 4_Chapters 16&17_Session 10: Business Law-T122WSB-1 07/04/2022, 15:24

the Partnership Act determines partners’ rights, duties and interests, if not
included in the : partnership agreement

Question 4 0 / 1 pts

Which court decision regarding fiduciary duties is incorrect?

In Chan v Zacharia (1984) 154 CLR 178, the court decided that that the
fiduciary relationship : continued until the partnership had finally been wound
up.

You Answered
In United Dominion Corp Ltd v Brian Pty Ltd (1985) 157 CLR 1, the Court
found that a fiduciary : relationship, with attendant fiduciary obligations
ordinarily exist between prospective partners who : have embarked upon the
conduct of the partnership business or venture before the precise terms of :
any partnership agreement have been settled

In Popat v Schonchhatra (1997) 3 ALLER 800 in the absence of a statement


to the contrary, : partners are entitled to an equal share of profits

Correct Answer
in Harvey v Harvey (1970) 120 CLR 529, the Court held that if a partner
receives an additional : annual fee to be “on call” for that client, the fee
belongs to the partnership.

Question 5 1 / 1 pts

Which court finding is incorrect?

https://lms.westernsydney.edu.vn/courses/288/quizzes/3579?module_item_id=11541 Page 3 of 9
Quiz 4_Chapters 16&17_Session 10: Business Law-T122WSB-1 07/04/2022, 15:24

Correct!
In Mercantile Credit Co Ltd v Garrod [1962] 3 All ER 1103, damages were not
recovered even : though, from the plaintiff’s perspective, the sale of the car
was within the usual course of : business.

In Goldberg v Jenkins (1889) 15 VLR 36, the firm was not bound to the
transaction because : borrowing money on behalf of the firm at over 60%
interest when comparable rates were between : 6% and 10% was beyond ‘the
usual way’.

In Construction Engineering Pty Ltd v Hexyl Pty Ltd (1985) 155 CLR 541,
Hexyl was not liable : because the construction contract appeared to be
between Construction Engineering and Tembel

If judgment is obtained against one or more partners of a firm, no action may


be taken against the : other partners, even if satisfaction cannot be obtained
from the partner(s) sued

Question 6 0 / 1 pts

Which of the following is NOT a ground for termination of a partnership?

You Answered Court order

Correct Answer Partner leaves the jurisdiction

Expiry of a fixed term

Bankruptcy of a partner

https://lms.westernsydney.edu.vn/courses/288/quizzes/3579?module_item_id=11541 Page 4 of 9
Quiz 4_Chapters 16&17_Session 10: Business Law-T122WSB-1 07/04/2022, 15:24

Question 7 1 / 1 pts

Find one incorrect statement?

In the absence of special statutory provision, although each partner is liable


with the others for : the whole of the debts of the firm, their liability is only joint

A creditor can bring only one action against members of a partnership and any
partner can insist : that the action be stayed until all other partners are joined
as parties.

A person admitted into an existing firm, liability may be incurred where it is


specially agreed upon

Correct!
A creditor can enforce liability against an incoming partner whether or not
he/she is a party to : the contract

Question 8 0 / 1 pts

Which statement regarding an outsize partnership is incorrect?

it has more than 20 partners

Correct Answer its partnership agreement is invalid

You Answered it is liable to a criminal penalty ($500)

https://lms.westernsydney.edu.vn/courses/288/quizzes/3579?module_item_id=11541 Page 5 of 9
Quiz 4_Chapters 16&17_Session 10: Business Law-T122WSB-1 07/04/2022, 15:24

its agreement does not affect the enforceability of contracts or other


arrangements made

Question 9 0 / 1 pts

Which of the below statements is incorrect?

Correct Answer
a written and signed notice of a partner’s intention to dissolve the partnership
is required, : where no fixed term has been agreed upon for the duration of the
partnership

a continuing guarantee given by or to a partnership is revoked as to future


transactions if there is : a change in the constitution of the partnership

You Answered
where a partnership continues after a fixed term has expired, the rights and
duties of the partners : remain the same, but the partnership becomes a
partnership at will

partnership property must be used exclusively for the purposes of the


partnership and in the : manner set out in the partnership agreement

Question 10 0 / 1 pts

Which below is incorrect? A firm is not bound by the acts of a partner if:

the act is not of the usual business of the kind carried on by the firm

https://lms.westernsydney.edu.vn/courses/288/quizzes/3579?module_item_id=11541 Page 6 of 9
Quiz 4_Chapters 16&17_Session 10: Business Law-T122WSB-1 07/04/2022, 15:24

You Answered the partner exceeds his/her actual authority in the particular matter

the person with whom the partner is dealing knows that he/she has exceeded
his/her authority

Correct Answer
the person with whom the partner is dealing knows or believes him/her to be a
partner

Question 11 0 / 1 pts

Cribb v Korn (1911) 12 CLR 205 established:

Joint ownership creates a partnership

You Answered A joint tenancy is just another name for a partnership

Correct Answer The sharing of joint returns does not in itself create a partnership

The statutory rules can be excluded by agreement

Question 12 0 / 1 pts

Which of the following statements regarding partnerships is incorrect?

The second element of a partnership is carrying out a business in common

To be a partnership there must be a mutuality of rights and obligations

Correct Answer

https://lms.westernsydney.edu.vn/courses/288/quizzes/3579?module_item_id=11541 Page 7 of 9
Quiz 4_Chapters 16&17_Session 10: Business Law-T122WSB-1 07/04/2022, 15:24

Each partner must take an active part in the direction and management of the
firm

You Answered
In Degiorgio v Dunn [2004] NSWSC 767 it was held that there was no
partnership because the : business was not run “in common”

Question 13 1 / 1 pts

Which below statements is incorrect?

Partners share the profits in a partnership

Correct!
In a partnership, partners have limited liability for the business debts

A partner is also an agent of the partnership

Agreement may be implied in a partnership

Question 14 1 / 1 pts

The incorrect statement?

Partners are not bound when another partner misappropriates monies

Correct!
If the giving of investment advice is within the scope of the firm’s business
then all partners : are jointly and severally liable for advice which is not in a
client’s best interests

the client should seek a personal indemnity from a partner

https://lms.westernsydney.edu.vn/courses/288/quizzes/3579?module_item_id=11541 Page 8 of 9
Quiz 4_Chapters 16&17_Session 10: Business Law-T122WSB-1 07/04/2022, 15:24

special skill is required to bind all partners

Question 15 0 / 1 pts

Which of the below does not apply to the Partnership Act?

The rules in the Partnership Acts assist in determining whether a business is


being carried on in : common

Correct Answer The partnership under the Act must be registered with ASIC

Each rule states a negative: i.e. that a certain fact does not of itself create a
partnership, but is : only indicative of a partnership

You Answered
Joint or part ownership or joint tenancy, or tenancy in common whether or not
the : owners/tenants share the profits, does not of itself create a partnership
as to anything so : held/owned

Quiz Score: 6 out of 15

https://lms.westernsydney.edu.vn/courses/288/quizzes/3579?module_item_id=11541 Page 9 of 9
4/4/22, 10:11 PM Quiz 4_Chapters 16&17_Session 10: Business Law-T122WSB-1

Quiz 4_Chapters 16&17_Session 10


Due
Mar 21 at 8:15am
Points
15
Questions
15
Available
Mar 21 at 7:59am - Mar 21 at 8:25am
26 minutes
Time Limit
15 Minutes

This quiz was locked Mar 21 at 8:25am.

Attempt History
Attempt Time Score
LATEST Attempt 1
15 minutes 6 out of 15

Score for this quiz:


6 out of 15
Submitted Mar 21 at 8:14am
This attempt took 15 minutes.

Question 1 0
/ 1 pts

Find a statement that is incorrect

 
A retiring partner should give specific notice of their retirement to persons with
whom the firm : has had dealings to avoid being made liable for debts incurred
after retirement.

 
Persons who represents themselves, or knowingly allow themselves to be
represented, as a : partner, are liable as a partner to anyone who has on the
faith of such representation given credit to : the firm

orrect Answer  
A wrongful act/omission includes breach of contract or of fiduciary duty,
negligent : misrepresentation and misleading or deceptive conduct

You Answered  
Innocent partners are liable for wrongful acts of other partners made in the
ordinary conduct of : business

https://lms.westernsydney.edu.vn/courses/288/quizzes/3579 1/9
4/4/22, 10:11 PM Quiz 4_Chapters 16&17_Session 10: Business Law-T122WSB-1

Question 2 0
/ 1 pts

Which statement regarding an outsize partnership is incorrect?

You Answered  
it has more than 20 partners

orrect Answer  
its partnership agreement is invalid

 
it is liable to a criminal penalty ($500)

 
its agreement does not affect the enforceability of contracts or other
arrangements made

Question 3 0
/ 1 pts

Which finding is correct?: In Popat v Schonchhatra (1997) 3 All ER 800, the


Court decided:

orrect Answer  
Popat was entitled to half the profits on the sale of the business and to a share
of the profits : that had accrued after the dissolution of the partnership, but
before the final settlement of : accounts.

You Answered  
Popat was not entitled to any profits on the sale of the business, as there was
no partnership : agreement and he was further not entitled to a share of the
profits accrued after the dissolution of : the partnership, but before the final
settlement of accounts.

 
Popat was entitled to half the profits on the sale of the business, but not to a
share of the profits : that had accrued after the dissolution of the partnership,
but before the final settlement of : accounts.

https://lms.westernsydney.edu.vn/courses/288/quizzes/3579 2/9
4/4/22, 10:11 PM Quiz 4_Chapters 16&17_Session 10: Business Law-T122WSB-1

 
Popat was not entitled to half the profits on the sale of the business, but he
was entitled to a : share of the profits that had accrued after the dissolution of
the partnership but before the final : settlement of accounts.

Question 4 0
/ 1 pts

The incorrect statement is?

 
A partner has express actual and implied actual authority to engage in certain
activities with third: parties

orrect Answer  
A partner cannot pledge/sell partnership property, incur and pay debts on
partnership : accounts or hire employees

 
Contravention of an agreement to restrict a partner’s authority is not binding
on the firm if notice : of the agreement has been given

You Answered  
In Polkinghorne v Holland (1934) 51 CLR 143, the firm was held liable
because Holland provided : the advice in his role as solicitor; thus in the
ordinary course of the firm’s business

Question 5 0
/ 1 pts

Which of the below statements is incorrect?

orrect Answer  
a written and signed notice of a partner’s intention to dissolve the partnership
is required, : where no fixed term has been agreed upon for the duration of the
partnership

https://lms.westernsydney.edu.vn/courses/288/quizzes/3579 3/9
4/4/22, 10:11 PM Quiz 4_Chapters 16&17_Session 10: Business Law-T122WSB-1

You Answered  
a continuing guarantee given by or to a partnership is revoked as to future
transactions if there is : a change in the constitution of the partnership

 
where a partnership continues after a fixed term has expired, the rights and
duties of the partners : remain the same, but the partnership becomes a
partnership at will

 
partnership property must be used exclusively for the purposes of the
partnership and in the : manner set out in the partnership agreement

Question 6 1
/ 1 pts

Which of the following statements is incorrect?

 
a separate legal entity can commit civil wrongs and engage in criminal conduct

 
a public company has serious compliance obligations

Correct!  
a partnership must be in writing

 
a separate legal entity is entitled to own property, pay tax and enter into
contracts

Question 7 0
/ 1 pts

Which court decision regarding fiduciary duties is incorrect?

https://lms.westernsydney.edu.vn/courses/288/quizzes/3579 4/9
4/4/22, 10:11 PM Quiz 4_Chapters 16&17_Session 10: Business Law-T122WSB-1

 
In Chan v Zacharia (1984) 154 CLR 178, the court decided that that the
fiduciary relationship : continued until the partnership had finally been wound
up.

You Answered  
In United Dominion Corp Ltd v Brian Pty Ltd (1985) 157 CLR 1, the Court
found that a fiduciary : relationship, with attendant fiduciary obligations
ordinarily exist between prospective partners who : have embarked upon the
conduct of the partnership business or venture before the precise terms of :
any partnership agreement have been settled

 
In Popat v Schonchhatra (1997) 3 ALLER 800 in the absence of a statement
to the contrary, : partners are entitled to an equal share of profits

orrect Answer  
in Harvey v Harvey (1970) 120 CLR 529, the Court held that if a partner
receives an additional : annual fee to be “on call” for that client, the fee
belongs to the partnership.

Question 8 1
/ 1 pts

Which of the below does not apply to the Partnership Act?

 
The rules in the Partnership Acts assist in determining whether a business is
being carried on in : common

Correct!  
The partnership under the Act must be registered with ASIC

 
Each rule states a negative: i.e. that a certain fact does not of itself create a
partnership, but is : only indicative of a partnership

 
Joint or part ownership or joint tenancy, or tenancy in common whether or not
the : owners/tenants share the profits, does not of itself create a partnership as
to anything so : held/owned

https://lms.westernsydney.edu.vn/courses/288/quizzes/3579 5/9
4/4/22, 10:11 PM Quiz 4_Chapters 16&17_Session 10: Business Law-T122WSB-1

Question 9 1
/ 1 pts

Which of the following statements regarding the written law is incorrect?

 
Under s 115 of the Corporations Act 2001 (Cth), the maximum number of
persons who may form a : partnership for the acquisition of gain is 20

 
Under the Corporations Regulations 2001 (Cth), reg 2A.1.01, partnerships of
more than 20 : partners may be formed for certain professions/callings

 
The Corporations Regulations 2001 (Cth), reg 2A.1.01 sets a maxima of
partners for medical and : legal practitioners, veterinary surgeons, patent and
trademark attorneys, sharebrokers and : stockbrokers and pharmaceutical
chemists

Correct!  
The Corporations Regulations 2001 (Cth), reg 2A.1.01 does not set a maxima
of partners for : architects and accountants

Question 10 0
/ 1 pts

Which below is incorrect? A firm is not bound by the acts of a partner if:

 
the act is not of the usual business of the kind carried on by the firm

 
the partner exceeds his/her actual authority in the particular matter

You Answered  
the person with whom the partner is dealing knows that he/she has exceeded
his/her authority

https://lms.westernsydney.edu.vn/courses/288/quizzes/3579 6/9
4/4/22, 10:11 PM Quiz 4_Chapters 16&17_Session 10: Business Law-T122WSB-1

orrect Answer  
the person with whom the partner is dealing knows or believes him/her to be a
partner

Question 11 1
/ 1 pts

Incorrect statement is?

Correct!  
The owners of a corporation have unlimited liability

 
A separate legal entity is separate from its owners and managers

 
A corporation can sue and be sued in its own name

 
Complex business structures are more expensive to establish

Question 12 1
/ 1 pts

Which of the following statements regarding dissolution of partnership is


incorrect?

 
In the event of dissolution, losses must be met first out of profits, followed by
capital, then by : partners in proportion to their share of profits

Correct!  
A firm’s assets as contributed by partners to make up capital losses need not
be applied to pay : the firm’s debts/liabilities to non-partners

 
After dissolution, each partner is entitled to advances and residue by the firm

 
After the dissolution, each partner’s authority to bind the firm continues, so far
as necessary for : winding up partnership affairs and completing unfinished
transactions

https://lms.westernsydney.edu.vn/courses/288/quizzes/3579 7/9
4/4/22, 10:11 PM Quiz 4_Chapters 16&17_Session 10: Business Law-T122WSB-1

Question 13 0
/ 1 pts

Which of the below statement is incorrect?

orrect Answer  
Partnership property is liable to be seized for the private (personal) debt of a
partner and : made liable on a judgment against the partnership

You Answered  
A creditor who has obtained judgment in respect of the separate debt of a
partner may obtain an : order charging that partner’s interest in the
partnership property and profits with the amount of the : debt and interest.

 
A creditor who has obtained judgment in respect of the separate debt of a
partner may obtain by : an order the appointment of a receiver of that partner’s
share of profits and of any other money : which may be coming to the partner
in respect of the partnership.

 
The two sources of law relevant when considering how partners bind their
partners when dealing : with third parties are the common law (including
equity) of agency and the Partnership Act. The : common law complements
the provisions of the Partnership Act in relation to the authority that an : agent
has.

Question 14 0
/ 1 pts

Which statement regarding partnership is incorrect?

You Answered  
A partnerships (or firm) is a way of gathering resources or expertise for major
projects

https://lms.westernsydney.edu.vn/courses/288/quizzes/3579 8/9
4/4/22, 10:11 PM Quiz 4_Chapters 16&17_Session 10: Business Law-T122WSB-1

orrect Answer  
Partnerships do not avoid taking on the formality and expense of an
incorporated company

 
Partners in a partnership complement each other with their skills and bring in
new capital and : broader funding options

 
A partnership in law may exist without the partners being aware of it

Question 15 1
/ 1 pts

What constitutes an exception to the usual number of a partnership being 20


partners?

 
If one of the partners is married-his/her husband or wife is automatically a
partner

 
The exception relates only to limited partnerships

 
No more than 20 are allowed

Correct!  
The Corporations Regulations 2001 (Cth) provide greater numbers in certain
specified : professions

Quiz Score:
6 out of 15

https://lms.westernsydney.edu.vn/courses/288/quizzes/3579 9/9
Upgrade
to
Home Explanations Your library Create …
Quizlet
Plus

BLAW final exam practice problems


Terms in this set (286)

When Sonya Celebrity reads an This is a civil lawsuit and Fame magazine is basing its
embarrassing article about herself argument on the First Amendment to the Constitution.
in Fame magazine, she becomes
very worried about damage to her
reputation. During her lawsuit
against Fame, the magazine's
lawyer argues that his client's rights
are protected by the Constitution,
specifically the right to free press.
Is this a civil or criminal lawsuit? On
what part of the Constitution is
Fame magazine's lawyer basing his
argument?

A state trial court held that Raynes the trial court's decision made a mistake in interpreting the
Inc. violated its contract with law.
Cranwell Management. Raynes
appealed the decision to the state
appellate court. If the appellate
court remands the case, it is most
likely because:
Ever since a large manufacturing injunction; court order
plant opened a quarter mile from
Stanley's house, he has been
bothered by air pollution and loud
noises coming from the plant. Tired
of the problems the plant is
causing, Stanley goes to civil court
seeking a(n) ________ which is a(n)
_________ requiring the plant to stop.

In the 1980s, the Supreme Court Common law


ruled that it was legal for
protesters to burn the American
flag because this activity counts as
free speech under the Constitution.
What kind of law is this?

In 1994, when the Arizona Supreme Yes, it will be used as precedent and it is binding.
Court decided the case of
Hernandez v. Arizona Board of
Regents, it found a duty of care to
avoid providing alcohol to
underage consumers. If in 2017 a
plaintiff from Flagstaff, Arizona
brings a lawsuit in an Arizona state
trial court against a student group
at an Arizona university for
providing alcohol to members
under the legal drinking age, will
the Hernandez v. Arizona Board of
Regents case serve as precedent
(assuming no court has overturned
it)?
Congress passes The Nylon-Free An administrative agency.
Water Act, a statute making it a
crime for commercial fishermen to
use nylon fishing nets in water
where manatees are found. Now
that the Act has passed Congress,
who will create regulations for
implementing it?

Pursuant to its power under the U.S. Federalism.


Constitution to enact laws affecting
the health and safety of its
residents, New Jersey lowers its
highway speed limits by 10 miles
per hour in hopes of reducing
traffic deaths. This is an example of
what aspect of our government?

The president has nominated Judge approved by the Senate; checks and balances.
Mukae to be a federal judge.
Before Judge Mukae can be
confirmed (that is, officially
appointed), she must be
______________________ as a result of our
system of _____________________.
Josh and Colin are driving down natural law for Josh, and legal positivism for Colin.
the highway in Josh's new
convertible. Josh steps on the gas
and accelerates to 110 miles per
hour, almost double the 60 miles
per hour speed limit. Colin urges
Josh to slow down, saying, "The law
is the law, and you should follow it
even if you think it's unfair." Josh
responds, "It's unjust to have a law
about how fast people can drive
their own cars. Let's see how fast I
can go!" In this scenario, Josh and
Colin's differing legal philosophies
seem to adhere most closely to:

In 1997, the Supreme Court Judicial power.


declared unconstitutional a
provision of a federal gun control
law that required state and local
governments to conduct a
background check on prospective
gun purchasers. Under what legal
authority did the Supreme Court
have the right to declare the
provision unconstitutional?
In the case of Hammer v. A case where a liquidated (pre-determined) damages
Breidenbach, 31 Mo. 49 (1860), Mr. payment in a contract was excessively disproportionate to
Breidenbach was hired to brew the injury.
beer in a cave for his employer, at a
salary of $1,000 per year. The
contract between the Bavarian
Brewery (which would later
become Anheuser-Busch) and Mr.
Breidenbach specified that any
violation of the agreement would
result in the breaching party paying
the sum of $500 to the injured
party. Because the cave was
dangerous, Mr. Breidenbach
refused to enter it to make the
beer, and his employer demanded
the $500. An appellate court later
determined that Mr. Breidenbach
should not be required to enter the
cave and endanger himself, and he
was not required to pay the $500
"penalty." In which of the following
modern cases could this case act
as an appropriate precedent?

Several U.S. southern states pass a No, because federal law preempts state law.
law that restricts the sale of alcohol
by businesses on Sunday between
the hours of 6AM and midnight. A
few weeks later, the federal
government passes a law that says
"no jurisdiction shall restrict the
sale of alcohol in any manner."
Would the states still be permitted
to restrict sales of alcohol within
their borders?
Which of the following is true of It is largely based on the English model with contributions
the basis of the U.S. legal system? from other societies.

No one school of jurisprudence is perfect.


likely to seem

____________ is the school of Legal realism


jurisprudence used on the
philosophy that what matter is not
what is written as law, but who
enforces the law and by what
process.

Which of the following statements Law is what the sovereign says.


best summarizes the philosophy of
legal positivism in jurisprudence?

In 1998, the United States Supreme True.


Court ruled in a case entitled,
Oncale v. Sundowner Offshore
Services, Inc., that same-sex sexual
harassment is illegal under federal
civil rights law. Amanda works for
Avali, Inc. in California. In 1999,
Amanda's supervisor, Jill,
repeatedly grabbed her breasts
and asked for sexual favors. If
Amanda decides to sue for sexual
harassment, the decision in Oncale
v. Sundowner Offshore Services,
Inc. will be precedent.

In a case, often both parties have legitimate


__________, opposing arguments.

The theory of natural law posits all law must have a foundation in morality.
that:
Martha sued her employer claiming False.
she was denied a promotion
because of her age. This is a
criminal lawsuit.

Law is _________. Essential.

Common law is also called judge-made law.


___________.

Congress passed the Civil Rights interpretive rules because they are clarifying an
Act of 1964, a statute that prohibits ambiguous statute.
employment discrimination based
on race, color, religion, sex, or
national origin. Congress also
established an administrative
agency, the Equal Employment
Opportunity Commission (EEOC)
to enforce this statute. The EEOC
created guidelines that define
terms such as "race/color
discrimination" and "sex
discrimination." These guidelines
are an example of:
The Aviation Security Improvement Yes, because the courts must defer to an agency's
Act (ASIA) states that the FAA can reasonable interpretation of the law.
refuse to divulge information about
airport security. The family
members of passengers who were
killed in a terrorist bombing of an
airplane over Lockerbie, Scotland,
requested certain airport security
information from the FAA under the
Freedom of Information Act (FOIA).
The FAA refused this request
because it interpreted ASIA to
mean that the agency had the right
to withhold data in spite of FOIA.
Must the courts uphold the FAA's
decision?

Armando is spending an autumn applied the duty of care exception under Tarasoff v.
weekend at Maria's Bed and Regents University of California.
Breakfast in Vermont. Armando has
a comfortable room on the second
floor, right next to the bedroom
where Maria herself sleeps every
night. On the third night of
Armando's stay, Maria wakes up to
the smell of smoke and realizes the
Bed and Breakfast may be in grave
danger. She walks past Armando's
bedroom door, goes down the
stairs and strolls out the front door
to safety. Armando does not wake
up and suffers severe burns when
the fire reaches his room ten
minutes later. If Armando
successfully sues Maria, the court
most likely:
The IRS promulgates several The IRS is an executive agency, and the president can fire
interpretive rules about corporate the head of any executive agency.
tax compliance, particularly for
executive compensation. The rules
prove to be highly controversial
and are a result of misconduct on
the part of the head of the IRS.
Which of the following statements
is correct?

Clark is outraged when the EPA No. Clark will not be able to have the administrative rule
promulgates a rule stating that reviewed in federal court because he has not suffered
lakefront buildings must be set direct harm.
back at least 30 feet from the
water. He believes the EPA has
exceeded its authority. Clark does
not own lake-front property, but
some of his best friends do, and he
is gravely concerned about this
encroachment on their individual
property rights. Will Clark be able
to bring an action to federal court?

The highest court in Montana must No, because the decision was made in a lower court.
decide a complicated case about
the custody of a child conceived
by using an egg donor and a
surrogate who was raised by an
unmarried, homosexual couple.
The highest court in Montana has
never heard a case like this before,
but there is relevant case law in a
family court in Montana. Is the
highest court bound by the
Montana family court's earlier
decision?
Congress has passed a new The president can veto the bill.
securities law. If the president
disagrees with this statute, what is
his/her best course of action?

Federal antitrust statutes are The congressman could introduce a bill overruling the
complex, but the basic goal is Supreme Court's interpretation of federal antitrust
straightforward: to prevent a major statutes.
industry from being so dominated
by a small group of corporations
that they destroy competition and
injure consumers. Does Major
League Baseball violate the
antitrust laws? Many observers say
that it does. A small group of
owners not only dominate the
industry but actually own it,
controlling the entry of new
owners into the game. This issue
went to the United States Supreme
Court in 1922. Justice Holmes ruled,
perhaps surprisingly, that baseball
is exempt from the antitrust laws,
holding that baseball is not "trade
or commerce." Suppose that a
congressman dislikes this ruling
and dislikes the current condition
of baseball. What could he/she
do?
Mark's state legislature passed a By using the plain meaning rule because "health
law that requires restaurant emergency" has an ordinary meaning to the general
employees to receive four hours of population.
first aid training that must include
instruction on the Heimlich
maneuver. The law also requires
that employees take reasonable
efforts to provide assistance to any
customers who experience a
health emergency at a restaurant.
Mark, a restaurant server, received
the required training but did
nothing to help a customer who
choked on his food at Mark's
restaurant. When Mark is charged
with violating the statute, he argues
that he did not believe that choking
qualified as a health emergency.
How will the court interpreting the
meaning of "health emergency" in
this statute?
Eighteen-year-old Alisha was No, because she wasn't babysitting Timmy that day.
driving through her neighborhood
one afternoon and recognized six-
year-old Timmy, a member of the
Caldwell family, who lived a block
away from her house. Alisha used
to babysit for Timmy on weekday
afternoons until a few months ago,
when she started working at a local
grocery store. From her car, Alisha
could see Timmy was flailing
around in the water next to the
dock of Maverick Pond, the
neighborhood's favorite fishing
spot. Alisha knew that the there
was a big drop off there where the
water was very deep, well over
Timmy's head. Because she was
late for work, she decided to keep
driving, thinking that Timmy's
parents must be close by. She feels
terrible later when she discovers
that Timmy is hospitalized in critical
condition after nearly drowning in
the pond. Did Alisha have a legal
obligation to come to Timmy's aid?
The United States Department of No, because the rule-making process was bypassed.
Agriculture (USDA) inspectors
received new manuals to use as
guides for their inspections of meat
packing facilities. Because they
were pressed by Congress to get
some new rules in place quickly
due to quality concerns at some
facilities, the USDA inserted the
new guidelines in the inspector
manuals through a commonly used
process called "bootlegging."
Based on a violation of one of
these rules, a large meat packing
facility has their packing line closed
down temporarily. Do you believe
these "bootlegged" rules are
allowable?

Erica owns a waterfront home in No. FOIA may not be used to obtain information from
South Carolina, and is interested to state governments.
know what information South
Carolina has about her property
and her tax obligations. May Erica
use the Freedom of Information
Act (FOIA) to obtain this
information?
In the 1950s, Illinois enacted No because it violated the Commerce Clause.
legislation requiring trucks to have
curved mud flaps. There was no
federal law about mud flaps at the
time, but the Illinois statute was
enacted because state legislators
asserted that the curved mud flaps
were more effective in preventing
accidents than the straight mud
flaps. Several trucking companies
brought suit, because the Illinois
statute conflicted with the laws of
other states that required straight
mud flaps. Was the Illinois statute
constitutional?

To protect Native Americans, the No, the statute violates the Supremacy Clause.
Federal government passes a law
prohibiting their taxation.
Oklahoma amends its own tax law,
adding a small tax on Native
Americans. Is the Oklahoma law
constitutional?

Marcel is picnicking in a crowded His nudity was not intended to convey a particularized
local park. He decides he would message.
be more comfortable naked, so he
takes off all his clothes. He can
enjoy only a few more bites of his
lunch before he is arrested for
violating city ordinances about
public nudity. Marcel sues. If the
court finds that Marcel's actions do
not warrant First Amendment
protection, it is probably because:
Eloise plans to build an addition on a violation of procedural due process requirements.
her house that she will operate as a
bed and breakfast. The town
rejects her plans on the grounds
that she must first purchase from
the town an expensive commercial
building permit. Eloise argues that
she is just modifying her own
residence and that she owned the
residence before the commercial
permit legislation was passed. She
argues she that, therefore, does
not need the expensive permit. At
the court hearing on her case, the
town mayor serves as judge. This is:

Andre is in 11th grade at his local The court will apply intermediate scrutiny, and the school
public high school. He wants to will have to demonstate that the rule is substantially
take the school's ballet class as one related to an important government interest.
of his required electives, but the
school allows only girls to enroll in
the class. Andre sues under the
14th Amendment. How will a court
determine the result?

This chapter is filled with examples Marbury v. Madison


of statutes that have been struck
down by the courts. A Texas law
banning flag burning was rejected
by the Supreme Court, as was a
Louisiana death penalty statute.
Where does the Supreme Court
get its power to strike down
congressional statutes as
unconstitutional?
You begin work at Everhappy No, because an employer has the right to regulate how
Corp. at the beginning of employees look at work.
November. On your second day at
work, you wear a political button
on your coat supporting your
choice for governor in the
upcoming election. Your boss
glances at it and says, "Get that
stupid thing out of this office, or
you're history." You protest that his
statement violates your
constitutional rights. Are you right?

Gilleo displayed a large sign on her Gilles, because the prohibition was overly broad.
front lawn that read, "Say No to
War in the Persian Gulf, Call
Congress Now." The city of Ladue
fined her for violating its
prohibition on signs on front lawns.
Gilleo sued. The city claimed that it
was regulating "time, place, and
manner." Who should win?
David paid $975,000 for two Yes, because the Beachfront Management Act renders
beachfront lots in coastal South David's property economically valueless.
Carolina, with the intention of
building residential homes on each.
Two years later, the South Carolina
legislature passed the Beachfront
Management Act, barring any
further development of the coast,
including David's lots. When David
files a complaint to seek
compensation for his property,
South Carolina refuses, pointing to
a passage in David's own complaint
that states "the Beachfront
Management Act [was] properly
and validly designed to preserve...
South Carolina's beaches..." Is
South Carolina required to
compensate David under the
Takings Clause?

Jason was a restaurant server with No, because the position of server has a bona fide
six years of experience who occupational qualification.
recently relocated to the
Northeast. The new town he lived
in had only a small number of
restaurants, the busiest one being
part of the chain "Hooters."
Although the Hooters restaurant is
known for only hiring young,
attractive females as servers, Jason
applies for a wait staff position, and
is turned down because he is a
male. Is Hooter's required to hire
Jason for a server position?
Claudia and Patrick would like to Mediation.
dissolve their business partnership.
They are confident that they can
work collaboratively on the terms
of the settlement, but they do not
have much money to spend on the
process and are nervous about the
prospect of the terms being
decided by a third party who is
unfamiliar with them and their
industry. Which of the following
legal processes is a good choice
for them?

Statutory rape is a crime in which a the jury in a trial court.


defendant has sexual intercourse
with a person who is too young to
give legal consent for the action. In
a criminal case involving an alleged
statutory rape, the age of the
victim at the time of the offense is
an issue to be determined by:
Eliza (female) has been working as File a class action lawsuit against Giant Coalmine.
an administrative assistant at Giant
Coalmine Corp. for two years.
Recently, several of her male
coworkers, including some of the
company's executives, have started
sexually harassing Eliza by
engaging in unwanted sexual
advances, abusive language, and
threatening behavior. Eliza has
discussed this situation with her
female coworkers, several of whom
have experienced a similar pattern
of inappropriate treatment. The
women feel intimidated and worry
about losing their jobs if they
complain. What can Eliza and her
attorney do to strengthen their
case against Giant Coalmine?

During discovery, lawyers for the the plaintiff's mental condition is relevant to the case.
defense find out that the plaintiff
was hospitalized in a psychiatric
facility last April. If the court grants
the defense access to the plaintiff's
full medical records from that
hospitalization, it is most likely
because:

Marshall claims that Victor both parties waived their right to a jury.
breached their contract and sues
him in district court, seeking
$80,000 in damages. If the trial is
being held without a jury, it is most
likely because:
Miles is floating happily on a Probably not. Intack does a significant amount of business
rubber raft in his swimming pool in in New Jersey and can reasonably expect to be sued
New Jersey when the raft busts and there.
cuts his eye. His doctor says he will
need corrective surgery and may
never fully regain his eyesight in
the damaged eye. Miles learns that
the raft was manufactured in
Louisiana by Intack Inc., a company
headquartered in New Mexico.
Miles bought the raft from a Kidz
Toyz store, a chain of stores in New
Jersey that sells many Intack
products. Miles sues Intack in a
New Jersey federal court for
$100,000. Intack files a motion to
have the case moved to federal
court in New Mexico, saying that
New Jersey lacks jurisdiction over
the case. Is Intack correct?

You and a partner in Turkey plan to Research Turkish law and hire a mediator familiar with
open a store in Chicago selling Turkish customs.
rugs imported from Turkey. You are
wise enough to insist on a contract
establishing the rights and
obligations of both parties and
would prefer a clause requiring
alternative dispute resolution
(ADR) of any conflicts that arise
under the contract. You want to be
sensitive to your partner's culture
and do not want to propose a
clause that will alienate him. What
is the best way you can accomplish
all of this?
In a civil case, a plaintiff wins a The burden is higher in a criminal case because the
lawsuit if he is 51 percent consequences that the defendant faces are more severe.
convincing and will collect 100
percent of his damages. In a
criminal case the prosecutor must
prove his case beyond a
reasonable doubt. Why is there
such a significant difference in the
burden of proof between civil and
criminal cases?

Prime Investments Corporation reverse the decision due to the trial judge's legal error.
sues one of its executives, Stewart,
for embezzling client funds. At
trial, Prime calls several of
Stewart's colleagues to the stand,
who testify that Stewart is a bad
person with no integrity and they
were not surprised to hear that he
was accused of stealing money.
Typically opinions of this type are
not admissible in court. The jury
finds Stewart liable to Prime for $1
million in stolen money. If Stewart
appeals, the appellate court will
probably:

Large numbers of employees have The employees will lose their rights to a class action.
signed mandatory arbitration
agreements in employment
contracts. Courts usually uphold
these clauses. Which of the
following is not an advantage of
using arbitration in place of
litigation?
Arthur met Rottweiler breeders No, because the breeders had insufficient minimal
Lynda and George at a dog show contacts with Rhode Island.
in Rhode Island. The couple, who
reside in New York State, later sold
several Rottweiler dogs to Arthur
who lived in Rhode Island. Arthur
later sued Lynda and George,
claiming that the dogs were sick
and infirm. He filed a complaint in
Rhode Island courts, claiming that
even though the dogs were sold in
New York, the breeders had
attended the dog show and
advertised in the Rottweiler
Newsletter which was distributed
at the event in Rhode Island. Based
on the facts, do you think Rhode
Island courts have personal
jurisdiction in this case?
An upscale restaurant in New York, No, because the requirements for diversity jurisdiction are
New York paid a considerable sum not met.
for the entire harvest of ice wine
from a small grower in Vermont.
After receiving and testing the
wine cases, the restaurant realized
the grapes used to make the wine
had spoiled, probably upon re-
thawing, when ice crystals can
disrupt the fruit. They requested to
return the wine, but the grower
refused to believe that there was a
problem. The restaurant files a
claim against the winery in federal
court, citing diversity as the basis
for subject matter jurisdiction. The
amount in controversy is over
$75,000 and both the restaurant
and the grower are incorporated in
Delaware. Can the federal court
hear this case?

Violent Violet, the main suspect in Because the police coerced Violet's confession.
a brutal double homicide, is
arrested and brought to the police
station. Once there, Violet refuses
to speak to the police. A team of
officers interrogates her, keeping
her awake for three straight days
and refusing to give her any food.
Eventually, she confesses to the
double homicide. At trial, the judge
rules that Violet's confession is
inadmissible. Why?
Harrison met Alejandro in an online Harrison and Alejandro will be found guilty of conspiracy.
chat room for people who like to
talk about violent fantasies.
Harrison and Alejandro spent
hours each week chatting online
with each other about elaborate
plans to kill Harrison's wife, Melody.
They chose a date to execute their
plan, and Harrison bought a gun,
some rope, and gloves for both of
them to wear. The police learned
of the plan and arrested Harrison
and Alejandro the day before they
planned to kill Melody. What will
happen at trial?

Marcus goes to class leaving five Yes, because Lina allowed the police in.
pounds of heroin on a table in his
bedroom. Shortly after, the police
knock on the door and ask Marcus'
roommate Lina if they may search
the apartment. Lina says yes, and
the police find Marcus' drugs. If the
police did not have a search
warrant for the apartment, was the
search permissible?

Jasper is indicted for larceny by the Because it is relatively easy for prosecutors to obtain an
grand jury but then found not guilty indictment.
at trial. Why is this possible?
Poppy is delighted when she meets wire fraud and romance fraud.
Sam on an online dating website.
He is charming and handsome and,
best of all, seems to be equally
enchanted with her. The two
exchange online messages and talk
on the phone. Sam asks Poppy to
transfer him some money so that he
can travel from his home in London
to visit her in Austin, Texas. Poppy
sends Sam money for a plane
ticket, plus additional funds to
cover medical expenses so he can
travel comfortably and feel his best
when he meets her family. As soon
as Sam receives the money, Poppy
never hears from him again. Sam
has committed:

AJ is studying in a busy student none of these crimes.


lounge when his friend arrives and
tells him that she just got great
tickets to a basketball game. AJ
grabs his books, backpack, and
cellphone and dashes to the game
with her. At the game, AJ realizes
that he mistakenly took someone
else's phone. At first, AJ plans to
return the phone, but then he
realizes that it is much nicer than
his so he keeps it. AJ is guilty of:
Arnie owns a house in a poor The prosecutor did not prove the criminal case beyond
section of the city. A fire breaks reasonable doubt, but Vicki proved her civil case by a
out, destroying the building, and preponderance of the evidence.
causing $150,000 of damage to an
adjacent store. The state charges
Arnie with arson. Simultaneously,
Vicki, the store owner, sues Arnie
for the damage to her property.
Identical evidence is presented to
juries in both cases. The criminal
jury acquits Arnie, while the civil
jury awards Vickie $150,000. Why is
this possible?

Kathy Hathcoat was a teller at a Embezzlement.


bank in Pendleton, Indiana. Her
branch manager, Mary Jane
Cooper, caught Hathcoat stealing
money from her cash drawer.
Rather than reporting Hathcoat,
Cooper joined in. The two helped
cover for each other by verifying
that their cash drawers were in
balance. They took nearly
$200,000 before bank officials
found out. What criminal charge
may the government bring against
Hathcoat?
A new British law requires British A police officer must tell the suspect that he has the right
police officers to say the following to a lawyer.
when placing a suspect under
arrest: "You do not have to say
anything. A record will be made of
anything you say, and it may be
presented as evidence if you are
brought to trial." What additional
information does a police officer in
the United States have to give
suspects at the time of an arrest?

Although video poker machines Money laundering.


are outlawed in Pennsylvania,
Conley placed them in bars and
clubs throughout the state. He
used profits from the machines to
buy more machines. What crime
has he committed, other than
violating the law prohibiting video
poker machines?

In 1995, after nearly ten months of A criminal jury's burden of proof is "beyond a reasonable
testimony, a criminal jury acquitted doubt" and a civil jury's burden of proof is
celebrity football player and actor "preponderance of the evidence."
O.J. Simpson of murdering his ex-
wife Nicole Brown Simpson and
her friend restaurant waiter Ron
Goldman. Then in 1997, a civil jury
found Simpson liable for the
murders, fining him $33.5 million in
damages. How can these juries
come to two such different verdicts
on the same issue?
Albert (Al) lived in a small town Yes, the crime of embezzlement.
and worked at the single bank
located there. Frustrated by the
knowledge that customers rarely
check their bank receipts or
reconcile their accounts, Al
decided to teach the folks in his
town a lesson. When a customer
came to the bank to withdraw
some money from the bank, he
would take extra funds out of their
account and pocket it. He
originally was planning on
restoring the money to the
accounts, but after gathering up
several hundred dollars, Al
decided to keep the money. He
then starts to do this regularly and
accumulates thousands of dollars
from customer accounts, and he
never returns any of the money.
Has Al committed a crime?

Dolly has a baseball bat and a It is possible for Dolly to batter Peter without assaulting
great deal of anger towards Peter. him, and it is possible for Dolly to assault Peter without
Which of the following is true: battering him.

D'Arcy, who is not a public figure, D'Arcy sued Mary because she wrote an article that said
brings a libel suit against Mary. At he had been convicted of tax fraud, when he had not.
trial, D'Arcy fails to prove injury or
actual malice, but he wins the suit
anyway. Which of the following
must be true?
One day, while drawing a patient's intentional infliction of emotional distress and slander.
blood, nurse Athena accidentally
pricks herself with the needle that
she had just used on the patient.
Rebecca, another nurse at the
hospital, warns Athena that the
patient has Hepatitis C, a virus that
can spread through a shared
needle. Athena becomes visibly
distraught, and Rebecca tells nurse
Edwin that Athena now has
Hepatitis C. In fact, Rebecca made
the whole thing up. She knew the
patient had no such illness and just
wanted to embarrass her co-
worker. Rebecca may be found
liable for:

After he is injured in an accident, No, under the single recovery principle.


Conrad successfully sues the driver
and the trucking company. The jury
awards him money for past lost
earning capacity, past and future
physical symptoms and discomfort,
and past emotional injury and
mental anguish. If Conrad's injuries
require him to be out of work for
much longer than anticipated, can
he later file another suit against the
driver and the trucking company
for his unforeseen lost wages?
Taterz Potato Chip Company Taterz is liable for commercial exploitation.
posted a billboard along the
highway with a photo of NBA
basketball superstar LeBron Smith
eating their chips. Smith was
furious. He did not endorse the
chips, and he did not want his fans
to think he ate unhealthy snacks.
Smith sues Taterz. What result?

Lucia is very interested in buying Charlotte loses because she did not have a contract with
Charlotte's apartment and has Lucia for the sale of the apartment.
toured it several times. She pays
Charlotte $5,000 to hold the offer
open (that is, not to sell the
apartment to anyone else) for
three days, so that Lucia can think it
over. The next day, Lucia's realtor,
Antonio, shows her an apartment
that she prefers. Lucia buys it on
the spot. Charlotte sues Antonio
for tortious interference with a
contract. What result?

You are a vice-president in charge No, posting the sign could give rise to a claim of
of personnel at a large defamation.
manufacturing company. In-house
detectives inform you that Gates,
an employee, was seen stealing
valuable computer equipment.
Gates denies the theft, but you
believe the detectives and fire him.
The detectives suggest that you
post notices around the company
informing all employees what
happened to Gates and why. This
will discourage others from
stealing. Should you post the
notices?
Caldwell was shopping at T-Mart False imprisonment, and she will win.
department store, carrying a large
purse. A security guard observed
her looking at various small items
for sale. At one point, Caldwell put
her reading glasses in her purse,
and the guard thought she might
have been shoplifting. The guard
approached her in the parking lot
and accused her of taking store
merchandise. The guard found no
stolen goods in her purse but had
her return to the store with him.
They walked around the store for
approximately 15 minutes, while the
guard said six or seven times that
he saw her steal something.
Another store employee indicated
she could go, and Caldwell later
sued. What kind of suit did she file,
and what should the outcome be?
Tata Consultancy of Mumbai, India, Tortious interference with a contract, and Tata will win.
is an international computer
consulting firm. It spends
considerable time and effort
recruiting the best personnel from
India's leading technical schools
and has its employees sign an initial
three-year employment
commitment. Desai worked for
Tata, but then quit and formed a
competing company, Syntel. His
new company contacted Tata
employees, offering a higher salary
and other perks. Several Tata
employees accepted Syntel's offer
and did not complete their three
year obligation. Tata sued Syntel.
What did it claim, and what should
be the result?

What general guidelines has the Punitive damages should not exceed compensatory
Supreme Court set forth for damages by more than a factor of nine.
awarding punitive damages?
Nine-year-old Simone was out Yes, because the detention was unreasonable.
shopping with her mother in the
local strip mall. While her mother
was looking in a furniture store,
Simone received permission to go
next door to a women's clothing
store. While in the store, she picked
up a package of socks and moved
to the front door to see if her
mother was outside. The manager,
thinking that Simone was
shoplifting, grabbed her by the
arm and placed her in an unlocked
storage room, where he asked her
questions for about 15 minutes.
After being satisfied that Simone
had not intended to steal
merchandise, he continued to keep
her in the unlit storeroom, alone,
for nearly two additional hours,
because he could not locate her
mother. Has the store manager
committed a tort by his actions?
Pam Perez hired Daniel Daniel Construction wins, because they didn't take any
Construction to complete a jewelry or valuables from Pam's home.
remodeling project in her
Washington, DC home. When the
work was finished, Pam asked the
construction foreman to do some
additional tasks before they left the
residence. When the foreman said
that work would need to be
rescheduled, Pam went on the
service review website "Angie's List"
and wrote the following entry: "Do
not hire Daniel Construction to do
any work for you. This company
didn't even finish the project, and
the work they did complete was
terrible. In my opinion, they are
also jewelry thieves, because
valuable items were missing from
my home, and Daniel Construction
was the only one with an extra key."
If Daniel Construction files a
defamation lawsuit against Pam,
what is the likely result?
Curious to see its worth, Irene Eleanor will be liable if Irene can prove that a reasonable
takes a beautiful oil painting that art appraiser would recognize the painting as being
she inherited from her valuable.
grandmother to Eleanor, a
respected art appraiser. Eleanor
tells Irene the artwork is a
worthless piece of junk, so Irene
sells the painting to Aziz at a yard
sale for $50. The following week,
Irene finds out that Aziz sold the
painting for $12 million dollars at
auction, after an appraiser at the
auction house determined it is the
work of a famous painter named
Thomas Gainsborough. Furious,
Irene sues Eleanor. What result?

Raquel is speeding in her car Adam will lose because Raquel's conduct was not the
through a busy town center when proximate cause of his injury.
she veers off the road to avoid a
cat and plows into a small
newspaper stand. The stand flies
into the air and smashes through
the glass windows of a nearby
yoga studio, where it startles
Adam, a yoga student, and sends
him flying into a set of lighted
candles. As a result, Adam suffers a
serious burn. Adam sues Raquel.
What result?
Lindsay decides to go skydiving for Yes, because Lindsay is an incite of the skydiving
the first time and chooses a company.
company with an excellent
reputation. On the day of the dive,
Lindsay slips on some oil on the
floor of the company's airplane
hangar and fractures her elbow.
Will Lindsay win a negligence suit
against the skydiving company?

Lucas is hosting Thorben and He did not know about the trap door.
several other dear friends at a
housewarming party at his new
home. During the tour of the new
place, Thorben plunges through a
trap door partially concealed by
the living room carpet and falls 20
feet into the concrete basement
below. Thorben breaks both legs
and sues Lucas. Which of the
following arguments will be Lucas'
best defense?
Aurelia has emergency major heart Yes, under the doctrine of res ipsa loquitur.
surgery at the best hospital in town.
The next week, she goes to her
doctor complaining of an acute
pain in her chest. The doctor
performs an x-ray and finds that
there is medical sponge inside
Aurelia's chest. Medical sponges
are tools used during surgery and
should never be left inside the
patient. Because Aurelia was
unconscious during the surgery,
she does not know who left the
sponge in her chest or how exactly
it came to be placed there. Can
Aurelia still win a negligence case
against the hospital?

Jasper buys a can of Zaps bug The shop that sold Jasper the bug spray had kept the bug
spray. The first time Jasper sprays spray in an extremely hot storage room, and the extreme
some on his arm, the can explodes, temperature is what made the can explode.
causing deep cuts on his face that
require surgery. Jasper sues Zaps.
Which of the following facts, if true,
would most help Zaps' defense?
Irving was a notary public who No, Irving has a duty of care because it was foreseeable
prepared income tax returns for that the failure to properly draft the will would cause
Mark. Irving agreed to draft a will injury to Sonja.
for Mark, leaving all of the
property to Mark's sister, Sonja.
When Mark died, the court refused
to uphold the will because it was
improperly drafted. As a result,
Sonja inherits only one eighth of
the estate, and Sonja sued Irving.
Irving defended on the grounds
that he had no duty of due care to
Sonja because he only had
dealings with Mark. Is Irving right?

A new truck, manufactured by No, because there was factual cause, and the injury was
General Motors Corp. (GMC), had foreseeable.
a defective alternator that caused it
to stall on a busy highway. The
driver set up emergency flares
around the stalled truck, but
congestion on the highway made
them difficult to see. Davis did not
see the flares, drove his car into the
back of the stalled truck, and was
killed by the impact. His widow
sues GMC. GMC moved for
summary judgment, alleging (1) no
duty to Davis, (2) no factual
causation, and (3) no foreseeable
harm. Should summary judgment
be granted?
Harris walked onto some train No, Harris' own actions were the factual cause of harm.
tracks, ignoring a yellow warning The train station provided safety measures that a
line painted on the station reasonable person would pay attention to that Harris
platform, shouts from concerned ignored.
onlookers, and the ringing bells
and flashing lights signaling the
approach of an express train. The
train killed Harris, and his widow
sued the railroad arguing that its
negligence caused her husband's
death. Will she win?

Powers drove a truck that his Big is liable because it knew about the problem with the
employer leased from Big Trucks. strap and did not fix it.
After Big repeatedly failed to
respond to Powers' requests to fix
the strap used to close the truck's
rear door, Powers replaced the
strap with a nylon rope. Later, this
nylon rope broke, causing Powers
to fall and break his back. When
Powers sued Big, what was the
result?
Bernard Hospital was in desperate Yes, because the hospital was negligent in omitting the
need of nurses during a shortage background check.
of qualified medical personnel.
Because the hospital was so short-
staffed, they bypassed the optional
background checks commonly
required of all incoming
employees, opting for only a check
of references provided by the
applicant. Melinda, a nurse with 20
years of experience, applied for a
position at Bernard, and provided
excellent references that all
checked out. After Melinda was
hired, it was discovered that she
was injecting herself with patients'
pain medications. An investigation
by the hospital, after a lawsuit was
filed by a patient against them,
discovered that Melinda had been
discharged after the same type of
incident at her previous employer.
Is the hospital liable for the
patient's failure to receive their
medication?
In 2016, two-year-old Lane Graves Failure to warn negligence.
was wading in the lagoon of an
Orlando Walt Disney World resort
and watching a family outdoor
movie, when an alligator snatched
the boy and disappeared. The
resort had signs which read "no
swimming, deep drop off" posted
along the beach area where Lane
was taken, and his parents were a
few feet away when the incident
occurred. Several hours later,
Lane's body was discovered about
15 yards from where he was taken
by the gator. A Disney employee
said this was not the first time an
alligator had been spotted in the
lagoon beach area. If the Graves
family files a lawsuit against the
resort, what theory would best
support their claim?

Mrs. Martin tells some Jake has made a unilateral contract with Mrs. Martin and a
neighborhood kids that she will pay bilateral contract with the hardware store.
$100 if any of them mows her lawn.
Jake goes to a hardware store,
purchases a lawnmower for $60,
and then mows Mrs. Martin's lawn.
Jake has entered into which types
of contract?
Olivia agrees that she will bring Olivia will win. The court will rule that they had an implied
Desiree a cherry pie every Monday contract.
for one month in exchange for $15
per week. Olivia delivers a pie to
Desiree for four weeks, and
Desiree pays her each time. Olivia
continues to deliver Desiree pies
every Monday, and Desiree
continues to pay for another five
weeks. On the tenth week, Olivia
brings a pie and Desiree refuses to
pay. Olivia sues for payment. What
will result?

Chef Jacquie is scheduled to teach Jacquie, because the class is primarily a service.
a cooking class to three students.
The class tuition is $1,100 per
student. In the class, each student
cooks a French meal under
Jacquie's expert supervision and
receives a cookbook (worth $30)
and a cooking pan (worth $150).
Tory, one of the students, tells
Jacquie the day before the class
that she will be unable to attend
and requests a refund. Jacquie
denies the refund and Tory sues.
Tory claims that the UCC should
govern the contract, and Jacquie
argues that it should be covered by
the common law. Who is right?
For the past seven years, The court will uphold the original contract, and
Sommerset Storage, Inc. has hired Mountbatten will lose.
Mountbatten Tax Associates to
prepare its annual tax return. This
year the parties agree to their usual
$1,000 fee, but Mountbatten finds a
loophole in the tax code and gets
Sommerset a refund four times the
usual amount. Mountbatten then
requests that Sommerset pay
$4,000 to reflect the increased tax
refund. There is nothing in their
contract about increased fees, but
Mountbatten argues it would be
unjust for Sommerset not to pay
extra. Sommerset refuses, and
Mountbatten sues. What will result?

While George travels for two The court will apply common law, and Mary will lose.
months, Mary agrees to housesit
and care for George's three horses
at her stables. The parties agree
that Mary will pick up the horses on
the first day of George's trip, and
George will pay Mary when he
returns. George returns home from
his travels and finds that Mary
never picked up the horses.
George sues. What will result?
Riley, age 16, and Samuel, age 36, Yes, the contract is voidable, and only Riley can cancel it.
enter into a contract in which Riley
will sell Samuel his car for $11,000.
The next day, Samuel decides he
no longer wants the car and tries
to get out of the contract. Samuel
argues that because Riley is a
minor, the contract is void. If Riley
wants to enforce the contract, will
he be able to?

Central Maine Power Co. (CMPC) Yes, CMPC is obligated to pay because this was a
made a promotional offer in which unilateral contract, and MSI performed.
it promised to pay a substantial
sum to any homeowner or builder
who constructed new housing with
electric heat. To qualify for the
offer, Motel Services, Inc. (MSI)
decided to install electrical heat in
a housing project it was
constructing in Waterville, Maine.
MSI built the units and requested
payment for the full amount of the
promotional offer. Is CMPC
obligated to pay? Why or why not?
Interactive Data Corp. hired Foley Interactive loses because it had an implied employment
as an assistant product manager, contract with Foley that incorporated the seven steps.
and over the next six years,
Interactive steadily promoted him.
Interactive officers repeatedly told
Foley that he would have his job as
long as his performance was
adequate. They also distributed an
employee handbook that specified
termination guidelines that
included a mandatory seven-step
pre-termination procedure. Foley
learned that his supervisor was
under investigation by the FBI, and
he told Interactive officers. Shortly
thereafter, Interactive fired Foley.
He sued, claiming that Interactive
could fire him only for good cause
after the seven-step procedure.
Who wins?
The Hoffmans owned and Yes. They can most likely recover damages based on
operated a successful small bakery. promissory estoppel.
Lukowitz, an agent of Red Owl
Stores, told them that for $18,000
Red Owl would build a store and
fully stock it for them to operate.
The Hoffmans sold their bakery
and purchased a lot on which Red
Owl was to build the store.
Lukowitz then told the Hoffmans
that the price had gone up to
$26,000. The Hoffmans borrowed
the extra money from relatives, but
then Lukowitz informed them that
the cost would be $34,000.
Negotiations broke off and the
Hoffmans sued. The court
determined that there was no
contract. Can the Hoffmans
recover any money?

While negotiating with Stewart to Voidable by Yasmine


purchase his house, Yasmine asks
him about the condition of the
roof. "Excellent," he replies. "It is
only 2 years old, and should last 25
more." In fact, Stewart knows that
the roof is 26 years old and has
had a series of leaks. The parties
sign a sales contract for $600,000.
A week before Yasmine is to pay
for the house and take possession,
she discovers the leaks and learns
that a new roof will cost $35,000.
What kind of contract exists
between Yasmine and Stewart?
Simon, aged 10, is invited to a No, because there was no intent to form a contract by
classmate's birthday party at an Simon's mother.
exclusive ski resort on March 15th.
The day will include 4 hours of
snowboarding, lunch and birthday
cake. Simon's mother checks a box
on the invitation that says "YES, we
will attend" and returns it to the
classmate's address. Unfortunately,
they later don't attend the party
when Simon comes down with the
flu. On March 17th, Simon's mother
receives an invoice in the mail from
Simon's classmate for $35 that says,
"Party No-Show Fee." Can Simon's
classmate collect the fee?

Xuechen graduated from culinary Yes, the employment contract has been breached, but the
school at the top of her class, and non-competition agreement has not been breached.
signed an employment contract to
work as a chef for one of Chicago's
best restaurants for a period of
three years. Additionally, Xuechen
signed a non-competition
agreement that stated she agreed
not work as a chef for any other
restaurant in the Chicago city area
for a period of five years. Shortly
after she began working, Xuechen
left her employer to work as a
manager at another Chicago
restaurant. Has Xuechen breached
either of the contracts she signed?
The town of Sanford, Maine, It will depend on whether the auction was with or without
decided to auction off a plot of a reserve, or minimum price.
land owned by the town. The town
advertised that it would accept
bids through the mail, up to a
specified date. Arthur and Arlene
mailed in a bid that turned out to
be the highest of all the bids the
town received. When the town
refused to sell them the lot, they
sued. Who will win?

The Tufte family leased a 260-acre Yes, because under the mailbox rule, an acceptance is
farm from the Travelers, Inc. generally effective when mailed.
Toward the end of the lease,
Travelers mailed the Tuftes an
option to renew the lease. The
option arrived at the Tuftes' house
on March 30 and stated that they
had until April 14 to accept. On
April 13, the Tuftes signed and
mailed their acceptance, which
Travelers received on April 19.
Travelers claimed there was no
lease and attempted to evict the
Tuftes from the farm. May the
Tuftes stay?

You are considering joining an It depends on what state you are in.
online club. Before continuing to
the membership page, the site
presents you with hundreds of lines
of dense legalistic text and asks
you to agree to them. You click the
"I agree" box. Will you be bound by
the terms?
Jackie offers to sell Mel a concert No. Mel made a counteroffer, which Jackie rejected.
ticket for $50, and Mel replies, "I'll
give you $40." Jackie refuses to sell
at the lower price, and Mel says,
"OK, OK, I'll pay you $50." Has a
contract been formed? At what
price?

While staying overnight at Lily's No, Lily's note was an invitation to bargain, not an offer.
house, Martin indicates that he
would like to buy her car. Lily says
she'll think it over. The next day
Martin finds a note on the kitchen
counter from Lily that reads, "I
might be able to sell the car for
about $22,000." Martin writes her a
check for $22,000, takes the car
keys off the hook by the front door
and drives off in the car. Have Lily
and Martin formed a binding
contract?
Mariposa Middle School is in need The offer will not be enforced because the offer is open
of more sports equipment, so a too long.
school administrator contacts Kiko's
Sporting Goods. The administrator
speaks to Kiko herself, and Kiko
offers Mariposa 15 percent off its
order. Kiko tells the administrator
she can order whenever she is
ready and still enjoy the discount.
Kiko faxes the administrator a
handwritten note that reads
"Pursuant to our conversation, I
hereby grant Mariposa Middle
School a 15% discount off the final
sales price of their purchase. Offer
good for one use only. Offer open
for the next 5 months. Signed, Kiko"
The administrator tries to use the
discount four months later but Kiko
says she has revoked the offer.
What will result?

Polyquan, Inc. emails a dozen Polyquan has a contract with Shirts-A-Lot but not with
companies, offering to sell them Bespoke Tourism
100 sweatshirts in either blue, red,
orange, black, or green. The
sweatshirts are $6 each. Shirts-A-
Lot Clothing emails back, "We
accept your offer of 100 sweatshirts
for $6 each. No orange
sweatshirts." Bespoke Tourism
emails back, "We accept your offer
of 100 sweatshirts for $6 each on
the condition that you do not
include any orange sweatshirts."
Does Polyquan have a contract
with either company?
On Monday, Billy receives an offer Billy and Andrew have a contract for $500,000.
from Andrew to buy Billy's house
for $500,000. On Tuesday, Billy
mails Andrew a reply, "I'll sell it to
you for $600,000, and not a penny
less." On Wednesday, Billy
reconsiders so he mails Andrew
the following note: "I accept your
offer for $500,000, the house is
yours." On Thursday, Andrew
receives Billy's first note. On Friday,
Andrew receives Billy's second
note. What result?

TuffGrip ran an advertisement for No, TuffGrip is liable unless the purchase orders also
its brake pads in several national included the limitation of liability.
automotive industry journals. The
ad listed a range of prices for the
brake pads and a few technical
specifications. At the bottom, the
ad stated that TuffGrip had no
liability in the event of a
malfunction. Three large mechanic
chains purchased the brake pads
after seeing the ad, and signed
standard TuffGrip purchase orders.
Many of the brake pads that these
mechanic chains installed in
customers' cars failed, causing
severe injuries to drivers who could
not stop their cars. TuffGrip
claimed it had no liability to the
mechanic chains. Is TuffGrip
correct?
Oliver found some cozy-looking Oliver will lose as long as the company can show it was
winter gloves for a great price in a acting in good faith.
catalog. He decided to purchase a
pair for himself and additional pairs
for every member of his extended
family. He mailed in the order form,
along with a check. Two weeks
later the catalog company
informed Oliver that it had run out
of gloves and would not charge
him. Oliver sues. What will result?

A national beverage company ran a John loses, because no reasonable person would believe
promotion where consumers could the advertisement was a serious offer.
collect "points" by purchasing the
company's products, then redeem
the accumulated points for items
such as baseball caps and t-shirts.
One television ad showed a
teenager landing a $33.8 million-
dollar aircraft in his schoolyard
while "Harrier Jet: 7,000,000 points"
flashed across the screen. After the
commercial aired, John gathered
the 7 million points and asked for a
Harrier jet, yet the company
refused to comply. Two days later
the points for the jet had changed
in the ad from 7,000,000 to
700,000,000. If John sues for the
airplane, what is the probable
outcome?
On August 15, Calvin offers to sell No, because Clarissa didn't meet all the requirements of
Clarissa his laptop for $500 cash. the offer.
He tells Clarissa she can only
accept the offer by mailing the
money to arrive by September 2 at
his university mailbox. Clarissa
mails a check for $500 to Calvin,
and the money arrives in the
mailbox by August 30th, although
Calvin doesn't retrieve his mail
(along with Clarissa's payment)
until September 4th. Did Clarissa
accept Calvin's offer?

Amy, a baker, has found her dream $0 because an agreement to accept different
home, but cannot afford the down performance in lieu of full payment of liquidated debt is
payment. Amy's brother agrees to binding.
loan her $30,000 for the down
payment, and Amy agrees to pay
him back in one year. Next year,
Amy offers to bake her brother's
wedding cake for his wedding next
month instead of paying back the
loan, so that she can buy new
equipment for her bakery. Amy's
brother agrees. How much money
does Amy owe her brother?
While visiting Sarah's art gallery, Yes, as there was no satisfaction of the debt because
Mark spots what he believes is an Mark's check bounced.
original painting by the artist
Vincent van Gogh and agrees to
buy the painting from Sarah for
$1,000,000. Upon returning home,
Mark has the painting appraised
and learns that it is not a Van Gogh
and is worth only $100,000. Mark
sends Sarah a letter saying that he
bought the painting under false
pretenses and will pay only the fair
market value of the painting,
enclosing a check for $100,000 with
"payment in full" written in the
memo line. Before depositing the
check, Sarah crosses out Mark's
note and writes "first partial
payment" over it. The next day
Sarah learns that Mark's check has
bounced. If Sarah sues Mark, will
the court find in her favor?
Florian, a busy surgeon, reads No. This is a requirements contract, and Florian acted in
about a new state-of-the-art good faith.
surgical mask developed by Myriad
Medical Devices (M.M.D.) and
thinks it would be useful for his
practice. Florian enters into a
contract with M.M.D. in which
M.M.D. agrees to sell the doctor all
the masks he needs in the next six
months. Shortly thereafter, a fire
burns down Florian's practice, and
he will be unable to see patients
for at least 9 months. Florian will
not need a single mask until the
practice is back up and running. If
M.M.D. sues Florian for breach of
contract when Florian does not
order any masks, will M.M.D. win?

Helena shares with her family her The element of the agreement about the bald eagle is not
two goals for her senior year in enforceable because shooting bald eagles is illegal.
college: join a synchronized swim
team and shoot a bald eagle.
Helena's cousin Anika is concerned
about the utility of both of these
goals and urges Helena to spend
the year looking for a job,
promising, "If, upon graduation, you
have secured a job, have refrained
from synchronized swimming, and
have not shot any bald eagles, I
will give you $10,000." Helena
immediately accepts the terms of
the agreement and at graduation
has met all three criteria, but Anika
refuses to pay. What argument will
be most helpful for Anika in court?
CeCe Hylton and Edward Meztista, Hilton's debt is discharged because there has been an
partners in a small advertising firm, accord and satisfaction.
agreed to terminate the business
and split its assets evenly. Meztista
gave Hylton a two-page document
showing assets, liabilities, and a
bottom line of $35,235.67, with half
due to each partner. Hylton
questioned the accounting and
asked to see the books. Meztista
did not permit Hylton to see any
records and refused to answer her
phone calls. Instead, he gave her a
check in the amount of $17,617.83,
on which he wrote "Final
payment/payment in full." Hylton
cashed the check, but she wrote
on it, "Under protest—cashing this
check does not constitute my
acceptance of this amount as
payment in full." Hylton then filed
suit, demanding additional monies.
Meztista defended on the grounds
that the parties had made an
accord and satisfaction. What is the
likely result?
American Bakeries had a fleet of American Bakeries wins because this was a requirements
over 3,000 delivery trucks. Because contract which does not require any purchase from
of the increasing cost of gasoline, Empire.
the company was interested in
converting the trucks to propane
fuel. It signed a requirements
contract with Empire Gas, in which
Empire would convert
"approximately 3,000" trucks to
propane fuel upon American
Bakeries' potential request, and
would then sell American Bakeries
all required propane fuel to run the
converted trucks. American
Bakeries never requested a single
conversion. Empire sued for lost
profits. Who wins?

Melnick built a house for Gintzler, Gintzler will win because he gave consideration.
but the foundation was defective.
Gintzler agreed to accept the
foundation if Melnick guaranteed
to repair any damage that was
caused by the defects in the future.
Melnick agreed, but when Gintzler
called Melnick two years later to
repair water damage resulting from
the foundation defects, Melnick
refused to make any repairs.
Gintzler sued, and Melnick argued
that his promise to make future
repairs was unsupported by
consideration. Who will win the
suit?
After going through her old No, because of Michelle's illusory promise, she and Farra
clothes, Farra asks her friend did not have an enforceable agreement.
Michelle if she would like to buy
any of them, stating she is hoping
to get at least $50. Michelle
responds that she will pay $5 for
each piece that she would like to
buy. The next day, Michelle tells
Farra that she does not want any of
the clothes. Has Michelle violated
the agreement with Farra?

Maybelline falls down a well and is Yes, because Ruben relied on the promise.
saved when her neighbor Ruben
makes a very daring rescue. Ruben
hoists her to safety, and Maybelline
hugs him and promises him a check
for $100,000 for his kindness.
Ruben is delighted and when
Maybelline visits him the next day
and again mentions the $100,000,
he tells her that he is going to quit
his job as a preschool music
teacher and use the money to start
a business. Two days later he
resigns from the nursery school
and calls Maybelline and asks
when he can expect his money. He
is startled when she replies that she
is not going to give him a cent. Will
a court enforce Maybelline's
promise?
For seven years, Stanford Owens Yes, because Stanford and DiggyWerx each receive and
has run a successful practice that incur a detriment.
helps small businesses file their
taxes, become incorporated, and
perform other legal tasks. Stanford
moves his practice to NYC, and is
happy to find his first local client,
DiggyWerx, almost immediately.
They negotiate a flat fee of $5,500
for six months of Stanford's
services. A month later, Stanford is
more familiar with the local
business landscape and realizes
that most people offering his
services in NYC charge around
$13,000 for six months of work
because of the higher costs to do
business in NYC. Is the deal
between Stanford and DiggyWerx
enforceable?

Tim's Lumber Co. agrees to provide Deck builders' demand for wood must be made in good
Deckbuilders, Inc. with all of the faith.
wood that Deckbuilders requires
for the next five years.
Deckbuilders agrees not to buy
wood from any other vendors.
What is the primary restriction on
how much wood Deckbuilders may
demand?
Lisa makes hand-crafted beads. No, Julia has made an illusory promise.
Lisa approaches Julia, a successful
jewelry maker, and offers to sell her
beads exclusively to Julia if Julia
agrees to buy all of the beads that
Lisa makes. Julia agrees to buy all
of the beads Lisa produces each
month if, after inspecting them,
Julia thinks they will sell well. Do
Lisa and Julia have an enforceable
contract?

Richard hires Paul to paint his Yes, Paul provided the additional consideration of working
kitchen in two eight-hour days of overtime and completing the work in a shorter time
work for $500. At the end of the period than originally agreed.
first day, the kitchen is 85 percent
complete. Richard asks Paul to stay
late and finish. Paul agrees to stay
and complete the painting if
Richard pays him a total of $750 for
the job. Richard agrees. Does
Richard owe Paul the additional
$250?

Rodolfo hires Tessa to paint the Rodolfo owes Tessa the additional money because Tessa
exterior of his house. Two days into promised to re-paint the stained areas of the house and
the project, smoke from a nearby needed more paint to do the job.
forest fire blows onto the property
and stains her work. Tessa will have
to start again. Tessa asks Rodolfo
to pay for the additional labor and
materials, and Rudolfo agrees.
Tessa completes the job, but
Rodolfo refuses to pay the
additional fee. Tessa sues. What
outcome?
Seth finds a lost dog and returns cannot collect the money because it was offered in
her to her owners. The overjoyed response to something Seth had already done.
owners promise Seth $500, but
never send him the money. Seth:

In March, Louisa's Hamburger The new terms may be binding because an oral
Stand contracts with modification may be binding under the UCC.
HydrationCorp to buy 100 bottles
of lemonade for $100 and an
additional 100 bottles of lemonade
for $115 on May 1. After the
purchase and delivery of the 100
lemonades in March, Louisa speaks
with a HydrationCorp
representative, and they agree that
on May 1 HydrationCorp will
instead sell Louisa 100 bottles of
iced tea for $115. What will result?

Mary pledges $30,000 to a school The school will be able to enforce the promise with
for the blind. In reliance on Mary's "moral consideration"
promise, the school hires an
architect to build a cafeteria. Two
months later Mary takes back her
promise. The school sues, and the
judge will likely rule that
__________________.
Alonzo graduates with a 3.8 GPA in Yes, on the theory of promissory estoppel.
Accounting from a New York
university and receives offers of
employment from three of the Big
Four accounting firms. While
considering his options, a mid-
sized firm in California calls and
offers him twice the starting salary
of his top offer. The only caveat is
he needs to be in Los Angeles
within the next week. Alonzo
excitedly decides to take the
California job. He refuses the other
employment offers, breaks his
lease in New York, sells or gives
away most of his possessions, buys
a new car, and heads for L.A. the
next day. When he arrives three
days later, the firm apologizes and
informs Alonzo that the position is
no longer available due to the loss
of a major client. Based on these
facts, does Alonzo have a potential
claim against the accounting firm?

Mallory hires Raghav to work for No, because both parties are not bound by the contract.
one year as an assistant manager at
$5,500 per month, reserving the
right to cancel the contract at any
time. After two months, Mallory
fires Raghav because the
company's order volume drops.
Can Raghav recover payment for
the remaining 10 months on the
contract?
Sophie is an accomplished plastic Sophie will lose because she does not have a medical
surgeon who has lost her medical license.
license due to her addiction to
illegal drugs. Vanessa hires Sophie
for a "filler party" in which Sophie,
using a simple medical procedure
designed to reduce wrinkles,
injects a filler into the foreheads of
Vanessa and ten of her friends.
Vanessa fails to pay Sophie, and
Sophie sues. What will result?

KwikKash is a payday loan Claudette will win because the loan violates usury laws.
company that gives cash to
borrowers for a large fee. KwikKash
recently loaned Claudette $1,000
and required that she pay back
$1,500 within 30 days. Claudette
fails to repay the loan, and
KwikKash sues. What is the most
likely result?

When Alistair drops off his tuxedo Alistar loses because this is a valid exculpatory clause.
at Whett Dry Cleaners for dry-
cleaning, he signs Whett's standard
exculpatory clause, which releases
Whett from all liability in the event
that any item of clothing is
damaged, lost, or stolen while in its
care. When Alistair returns to pick
up his tuxedo, he is informed that
the garment tore during the dry
cleaning process. Alistair sues.
Which of the following results is
most likely?
When Tremaine arrived in Tremaine will win because the contract was procedurally
Cincinnati, he rented an SUV from a and substantively unconscionable.
car rental company in the airport
terminal. A month after the trip, he
noticed that the charge on his
credit card for the SUV was much
higher than he anticipated because
the rental company had charged
him $30 per gallon to refill the gas
tank. He reviewed the eleven-page
rental contract and noticed on
page seven a provision about the
gas charge. He saw another
provision on page ten stating that
in order to file a dispute with the
company he would be charged a
$750 "Investigation Fee" for his
claim to be reviewed. Tremaine
sues the rental company. Which of
the following outcomes is most
likely?

Over her objections, Carlene's Evan, Persephone and Carlene.


husband Silvio borrows $10,000
from his friend Evan so that he can
achieve his dream of swimming
with great white sharks in South
Africa. Carlene knows Silvio has a
reputation for being reckless, but
she still finds it pretty alarming that
Evan and Silvio's friend Constantine
and Silvio's business partner
Persephone each take out a life
insurance policy on Silvio a month
before his trip. Who can legally
have an insurance policy on Silvio?
Rudolph hired Moe to walk his dog Rudolph owes Moe money because the licensing issue is
every weekday. Rudolph hands irrelevant to their agreement.
Moe a check for a month of
excellent work. "Thanks!" says Moe.
"This will help me pay for the silly
citation I just got from the city,
since I never licensed my dog
walking business. They make us get
licenses just to raise money for the
city! What a scam." When Rudolph
learns that Moe's business is
unlicensed, he refuses to pay. What
will result?

Zazzles Jewelry sells Chic Boutique The contract is governed by the UCC, so the court may
150 necklaces. Zazzles sues when choose to enforce the remainder of the contract.
Chic defaults on its payments, and
the court finds that some of the
repayment terms in the contract
are unconscionable because they
place undue burdens on Chic.
Zazzles is willing to make the
repayment terms more favorable to
Chic but wants the rest of the
contract enforced. What is likely to
occur?
For 20 years, Art's Flower Shop The clause is unenforceable because it is unconscionable.
relied almost exclusively on
advertising in the Yellow Pages to
bring business to its shop in a small
West Virginia town. One year, the
Yellow Pages printer accidentally
did not print Art's ad, and Art's
suffered an enormous drop in
business. Art's sued for negligence
and won a judgment of $50,000
from the jury. The printing
company appealed, claiming that
under an exculpatory clause in the
contract, the company could not
be liable to Art's for more than the
cost of the ad (about $910). Art's
claimed that the exculpatory
clause was unconscionable. Please
rule.

Guyan Machinery, a North Carolina No, because it is not essential to the employer.
manufacturing corporation, hired
Albert Voorhees as a salesman and
required him to sign a contract
stating that if he left Guyan he
would not work for a competing
corporation anywhere within 250
miles of North Carolina for a two-
year period. Later, Voorhees left
Guyan and began working at
Polydeck Corp., another North
Carolina manufacturer. The only
product Polydeck made was
urethane screens, which comprised
half of 1 percent of Guyan's
business. Is Guyan entitled to
enforce its noncompete clause?
Richard and Michelle Kommit live Richard and Michelle Kommit
in California, where gambling on
credit is illegal. They travel to New
Jersey, where they used their credit
card to withdraw cash from an ATM
conveniently located in the "pit"—
the gambling area of a casino. They
ran up debts of $5,500 and did not
pay. The California National Bank
(the issuer of the credit card) sued
for the money in a California court.
Who wins?
Professional Basketball teams have Yes, because the non-competition clause is oppressive.
been pursuing Cody, a recently
graduated high school athlete, for
nearly two years. Cody and his
father hired an experienced agent
to represent him in contract
negotiations with the various
teams. Unfortunately, just before
meeting with the agent to review
contracts, Cody's father was
hospitalized in critical condition
following a car accident. Despite
knowing that Cody would be
alone, the agent insisted that they
meet and Cody decide on a team
to work with. The agent convinces
Cody to sign a lucrative contract
with a top-rated team, but the
agreement contains a non-
competition clause that would
keep Cody bound to that team for
his entire career. The contract has a
10-day grace period for review and
cancellation with no penalty. After
Cody's dad recovers and returns
home from the hospital two weeks
later, they discuss the contract and
seek to cancel it. Is it likely that
Cody will be able to avoid the
contract?
Oceanview Enterprises offers No, because Malik did not have notice of the clause
scenic cruises to exotic before the ticket purchase.
destinations in the South Pacific,
such as Tahiti and the Marshall
Islands. In December of each year,
the cruise line provides customers
an opportunity to enter a lottery
for a limited number of exclusive
tickets sailing to a "hidden"
unnamed island. Malik is fortunate
enough to win the right to buy two
of the expensive cruise tickets for
herself and best friend Jecala.
When she receives the purchased
tickets through the mail, Malik
notices the back of each has the
following statement: "The
purchaser and/or passenger of this
ticket agrees that any claim against
Oceanview Enterprises or its
employees will only be resolved in
courts of Oceanview's home state
of Alaska. This ticket is non-
refundable." Is it likely the clause
on the ticket would be enforceable
against Malik?
For his 17th birthday, Asher bought No, because he ratified the contract.
himself a used car from a car
dealer for $10,000. The dealer
purchased it for $6,000, repaired it,
and then sold it to Asher. Eighteen
months later, Asher has almost
finished paying off the car when he
totals it by accidentally driving it
into his neighbor's swimming pool.
Asher's mother is a lawyer and tells
him to rescind the contract. Will
Asher be able to recover any
money from the dealer?

The McAllisters had several serious Yes, the Silvas are entitled to damages for both fraud and
problems with their house, nondisclosure.
including leaks in the ceiling, a
buckling wall, and dampness. They
repaired the buckling wall by
installing I-beams to support it, but
never resolved the leaks or the
dampness. When they decided to
sell the house, they said nothing to
prospective buyers about the
problems. If asked, they stated that
the I-beam had been added for
reinforcement. The Silvas bought
the house for $60,000 and
immediately began to have
problems with leaks, mildew, and
dampness. Are the Silvas entitled
to any money damages?
Susan drops by Dean's garage sale Yes, Dean is entitled to rescind. This is a mutual mistake
and buys a painting for $10 that because both parties made an important factual error.
both she and Dean think is a copy
of a piece by Matisse, a well-known
painter. Later, Susan is delighted to
discover that the painting is
actually an original Matisse and is
worth $50,000,000. Dean hears the
news and wants the painting back.
Will he get it? Why or why not?

Mina, a real estate developer, No, because she understood the risk she was assuming.
reads that the tiny town of
Lakeview is considering building a
large shopping center and adding
public transportation from the
nearby city to Lakeview. Mina
knows that these types of changes
would greatly increase the demand
for real estate in Lakeview. She
buys 10 acres of land in Lakeview
from Arlen, so that she can build
houses and take advantage of the
town's economic boom. There is no
reference in the purchase and sale
agreement or deed to the possible
development in Lakeview. A few
weeks later, the town declares that,
due to changes in zoning laws, it is
not building the shopping center
or adding the public
transportation. Can Mina rescind
on her contract with Arlen?
Sixteen-year-old Travis Mitchell Yes, Travis will get his money because a minor is permitted
brought his Pontiac GTO into M&M to disaffirm a contract and get a full refund of his money,
Precision Body and Paint for body even if he is unable to make restitution.
work and a paint job. M&M did the
work and charged $1,900, which
Travis paid. Travis later complained
about the quality of the work. M&M
did some additional work, but
Travis was still dissatisfied. He
demanded his $1,900 back, but
M&M refused to refund it because
all of the work was "in" the car and
Travis could not return it to the
shop. The state of Nebraska, where
this occurred, follows the majority
rule on this issue. Does Travis get
his money?
Roy Newburn borrowed money No, the release is not valid. This meets all of the criteria of
and bought a $49,000 truck from a clear case of economic duress. Treadwell made an
Treadwell Ford. A few months later, improper threat, forcing Newburn to enter into the
the truck began having contract.
transmission problems. Newburn
learned that the truck had 170,000
more miles on it than the odometer
indicated. Treadwell admitted the
mileage error and promised to
install a new transmission for free.
When Newburn came to pick up
the truck with the new transmission
installed by Treadwell, the
company refused to turn over the
truck until Newburn signed a
general release of the dealership's
liability for any claims based on the
inaccurate mileage, which
Newburn signed. One month later,
the truck broke down, and the
resulting delays cost Newburn so
much income that he fell behind on
his loan payments and lost the
truck. He sued Treadwell, which
defended based on the release. Is
the release valid?
Tom and Harriet, adult siblings, are The sale to Augusta is void; the sales to Luther and Tom
taking their parents on a weekend may be voidable.
trip. They pick up their father,
Luther, at his house where he is
singing a goodbye song to each of
his 20 plants, and eating pancakes
out of his coat pocket. Next, they
drive to an assisted living facility, to
collect their mother, Augusta, who
has lived there since a judge
declared her mentally incompetent
a few years ago. When they arrive
at their hotel, Luther writes
postcards to his plants, Augusta
knits a sweater, Harriet goes for a
walk, and Tom orders a glass of
scotch. When Harriet returns, she
learns that a hotel guest sold Tom
and her parents each a souvenir
snow globe for $1,000. The snow
globes are for sale in the hotel gift
shop for $5. Harriet is furious. Can
her family members get their
money back?
Jacobi is visiting Sheila at her If Sheila had a reasonable belief that she was telling the
apartment while she is truth, Jacobi can rescind the contract, but he cannot
redecorating, and Sheila asks him if collect damages.
he would like to buy her two sofas
that she is replacing. Sheila tells
Jacobi that the sofas are in great
condition and are the most
comfortable she has ever had. She
acknowledges that some of the
fabric on the sofas is slightly worn
but says that is their only issue.
Jacobi buys the sofas on the spot,
but once they are back at his house
he finds that one of them is
infested with bed bugs. Jacobi has
to throw out the infested sofa and
pay a special exterminator to come
and get rid of the insects. Jacobi
sues Sheila. Assume their
jurisdiction follows the majority
approach. How will the court rule?

Ellen and Allen are interested in Yes, because Lyle previously told them about the pool.
renting Lyle's country house for the
summer. Lyle tells them about the
house, volunteering the property's
square footage, the number of
bedrooms, and stating that the
house has a heated pool and a hot
tub. The next day, Lyle learns that
the pool is going to be drained for
the summer as it undergoes repair.
Must Lyle disclose this information
to the potential renters?
Jonah is 12 years old and absolutely That neither Ronaldo nor Jonah knew the real value of the
loves soccer trading cards. At the card.
local hobby shop, he finds a "Lionel
Messi" trading card in the case,
which has a handwritten tag by it
that reads "9/00-." The employee at
the front desk interprets this to
mean $9.00 and sells Jonah the
card for that amount. A few days
later, Ronaldo, the card shop
owner calls Jonah and asks for the
card back because it is worth
around $900. Jonah refuses, and
Ronaldo sues. Which of the
following is Jonah's strongest
argument?
Alex, a 25-year-old banker, joined Yes, because Janine can't void the contract.
his friends at a "pub-hopping" trip
that had participants visiting six
bars in a two hour span of time.
Between the fifth and final pub
stop, a highly intoxicated Alex
stopped to use the bathroom at a
local luxury car dealer, who was
holding a special evening sales
event. Seeing all the cars and
thinking he was signing up for
training as a race car driver, Alex
mistakenly signed a contract to
purchase the $188,425 emerald-
green Ferrari displayed in the
showroom. The next day, a sober
Alex found the contract paperwork
at the foot of his bed, and with no
memory of the previous evening,
quickly headed to the dealership.
As he began to explain to the sales
manager Janine about the drunken
purchase mistake, he changed his
mind, and decided to keep the car.
Manager Janine, however, took the
contract copy from Alex and
ripped it up, saying "You're not
responsible enough to own this
car." Does Alex keep the Ferrari?
Just before turning 18 years old, No, because the original contract was void.
Caitlynn goes to the local Driver
Examination Office to test for her
driving license. When she passes
with a perfect score, her parents
allow her to go alone to the local
car dealership and purchase her
first car. She signs a contract to buy
a used lemon-yellow Volkswagen
Beetle with payments of $225 per
month for 3 years. Unbeknownst to
Caitlynn and her parents, the car
was a trade-in, and the dealer was
still waiting to receive the title.
Caitlynn drives the car until six
months after she turns 18, making
the payments each month, but then
decides she hates the yellow color,
and returns it to the dealer. The
dealer refuses to take the car back.
Is Caitlynn obligated to the car
purchase contract?

Able orally agreed to pay Carr Enforceable because the work could be completed within
$800 to restore Able's antique car one year.
within 18 months, even though it
was possible for him to complete
the restoration in 10 months. The
agreement is:
Mast Industries orally offered to Batak.
sell certain textiles to Bazak
International for $103,000. After a
few days, Bazak sent a
memorandum, signed by a Bazak
officer, to Mast confirming the
agreement, describing the goods,
and specifying their quantity and
the price. Mast received the memo
but never responded. When Mast
failed to deliver the goods, Bazak
sued. Who will win?

When Griffiths sold his house to Griffiths wins. The purchase - or repurchase - of a house is
Hippen, Griffiths orally agreed to a classic interest in land, and any such promise must be
buy the house back at the selling written to be enforceable.
price if Hippen should happen to
move within the next three years.
Two years later, Hippen was
transferred to Miami and tried to
sell the house back to Griffiths, but
Griffiths refused. Hippen sued for
breach of contract, but Griffiths
claimed there was never an
enforceable contract. Who wins?

Landlord owned a clothing store The writing needs to state the effective dates of the lease.
and agreed in writing to lease the
store's basement to another
retailer. Both parties signed a
written lease that (1) described the
premises exactly, (2) identified the
parties, and (3) stated the monthly
rent clearly. However, a trial court
held that the lease did not satisfy
the Statute of Frauds. Why not?
Lorna and Igor make a verbal Lorna cannot enforce the deal because her actions were
agreement that Lorna will buy a not sufficient to be considered part performance.
house Igor owns. They agree on
the price and a closing date in sixty
days. Before closing, Lorna moves
into the house with her mother,
remodels one of the bathrooms,
and paints the kitchen. A week later
Igor tells Lorna he does not want
to go through with the deal. May
Lorna force Igor to sell her the
house?

Terrance, a recent college Gloria will not be responsible for Terrance's debt unless
graduate, moves to New York City the landlord can prove that Gloria made the promise out
to open the second branch of his of self-interest.
mother's business. He plans to
work from his apartment full-time.
The landlord of the apartment that
Terrance applies for is not
convinced that Terrance will be
able to afford the rent. So
Terrance's mother Gloria (who is
extremely rich) speaks with the
landlord and tells the landlord that
if Terrance fails to pay his rent,
Gloria will pay it for him. When
Terrance fails to pay his rent, the
landlord requests the money from
Gloria, who refuses. What result?
Sabrina tells her boyfriend Sabrina owes Alexander nothing because the agreement
Alexander that if they get married, was not in writing.
she will give him 50 percent
ownership of her tech startup
company. Alexander quits his job
as a teacher and begins working at
Sabrina's company. The two get
married but divorce ten months
later. Alexander claims that Sabrina
owes him 50 percent of her
company. Please rule.

Daquan designs and manufactures Magnus must pay for 8,000 ReaderWranglers.
ReaderWranglers, which are
beaded, nylon cords that attach to
reading glasses and sunglasses, to
help prevent people from losing
them. Daquan calls Magnus, the
CEO of several assisted living
facilities for senior citizens, and
they agree that Magnus will buy
10,000 ReaderWranglers for $3
each and that Daquan will deliver
them to Magnus at Mount Olympus
Seniors (MOS) in one month.
Daquan arrives at MOS on the
specified date with 8,000
ReaderWranglers and Magnus
refuses to accept the delivery.
Daquan sues. What result?
Piper is suing Dimitri for breach of Yes. The court will admit evidence that Dimitri signed
contract. Her lawyer admits into under duress.
evidence a fully integrated
contract signed by Piper and
Dimitri stating that Dimitri will sell
Piper his beach house for
$250,000. Will Dimitri be allowed
to testify that he only signed the
contract because Piper had a gun
to his head?

Lauren verbally agrees to sell Dante will win under promissory estoppel.
Dante 500 Bietigheimer apple
trees, which are currently too small
to be transported. Lauren and
Dante agree that Lauren will have
the trees delivered when they
reach maturity in 18 months, and
Dante will pay in full when he
receives them. Dante then buys a
10 acre parcel of land that has soil
suitable for Bietigheimer apple
trees, and he pays a farmer to clear
the land and begin preparing it for
the arrival of the apple trees next
year. Lauren then calls Dante and
tells him that she is selling the trees
to someone who offered her more
money. Dante sues. What result?
Norv and Wanda agree verbally Nor cannot enforce the deal because it was not in writing.
that Wanda will buy Norv's house,
and that Norv will also sell her his
living room furniture for $1,000.
They agree on a total price, and
that the deal will close in 30 days.
Norv prepares his house for
Wanda, taking out all his
possessions, and leaving the living
room furniture. Two weeks later,
Wanda tells Norv she does not
want to go through with the deal.
Can Norv enforce his agreement
with Wanda?

Arabella sells Ann all of her No, because the contract is fully executed.
bowling equipment for $1,100. The
next day Arabella regrets the deal
and tries to rescind the contract.
Arabella argues that the agreement
is invalid because it should have
been in writing, and therefore she
has to refund Ann the money, and
Ann has to return the bowling
equipment to Arabella. Is Arabella
correct?

Mariko wants to buy one of Clare's No, because it is too vague.


rowboats, so the two draft a
contract for its sale. The contract
reads: "Binding Agreement: Mariko
will buy one of Clare's small
rowboats in February. Mariko will
pay cash, and Clare will deliver the
rowboat to Mariko." If both parties
signed the agreement, will it satisfy
the Statute of Frauds?
Iggy verbally agreed to sell frozen Hachiro is bound by the memorandum because he never
hotdogs to Hachiro. He sent a objected to it in writing.
signed memorandum to Hachiro,
which read, "Pursuant to our
conversation, this is to confirm your
order of 100,000 frozen hotdogs
for $0.15 each, which you will pick
up at my manufacturing plant on
November 1 of this year." Upon
receiving the memorandum,
Hachiro called Iggy and told him
the deal was off. On November 1,
Hachiro did not collect any
hotdogs. If Iggy sues what will be
the result?

Doreen calls Peruvian Pastimes PP will win if it cannot sell the sweaters elsewhere.
(PP) and orders three sweaters
customized with the initials of her
grandchildren. A week after PP
starts knitting the sweaters, Doreen
sends a letter cancelling her order.
PP sues. What result?
The Uptown Mall has hired Bryce Bryce wins, because he fully performed his obligations
to be the mall's Santa Clause for under the contract.
the upcoming season. Bryce and
the mall manager discussed on
September 6, 2016 the terms for his
employment as "Jolly Ole' St. Nick"
for the December holiday seasons
of 2017 and 2018. After orally
agreeing to perform both holidays,
Bryce realized he had a conflict for
2017, so informed the manager on
September 10, 2016 that he could
only be available for the 2018
season. The manager told Bryce
"no problem" and found another
Santa for 2017. After performing for
the mall as Santa Claus in 2018, the
mall refuses to pay Bryce for his
services. If Bryce files a claim
against the mall for the wages he
earned, what is the likely outcome?
The Roberts family just received Yes, because Jo-Anne has completed part performance
news from Jo-Anne Roberts' on the contract.
employer that she was receiving a
prestigious promotion to
Corporate Vice President. The new
position was in New York City,
nearly 3,000 miles away from their
home in Los Angeles, California.
Because Jo-Anne needed to find a
place to live in the city quickly, she
agreed to purchase via phone a
2,000 sq. ft. apartment located a
few blocks from her new office. Jo-
Anne wired the owner a payment
of $500,000 against the $3.7 million
negotiated price of the apartment.
Without waiting for further word
from the owner, Jo-Anne and her
husband Albert loaded their 3
children and 2 cats into their SUV
and headed for the "Big Apple."
Once arriving New York, the family
moved into the apartment and
redecorated the entire interior, at a
cost of $350,000. After living in the
apartment for two months, Jo-Anne
wired another $500,000 payment
to the owner, who called
immediately after receiving the
payment and said the deal was off.
Can Jo-Anne keep the apartment?
Cozette hires Baldrick to cook The agreement will be discharged due to true
dinner for her family three nights impossibility.
per week for an entire year. Nine
months into the agreement,
Baldrick dies of food poisoning.
Baldrick's estate sues Cozette for
Baldrick's salary for the rest of the
year. What result?

Omkara, a furniture manufacturer, Omkara's agreement with Gnome is discharged due to


contracts with Foam Gnome for true impossibility. Omkara's agreement with Duke's is not
$50,000 worth of foam, which discharged and she must perform.
Omkara will use for making ten
sofas she has agreed to make for
Duke's Furniture. A day before
Gnome is going to ship the foam to
Omkara, a flood destroys its entire
inventory. Gnome tells Omkara it
cannot send her the foam in time,
but tells her that FirmFoam can
supply her with an identical
shipment for $65,000. This increase
in price will wipe out twenty
percent of Omkara's profit from her
contract with Duke's. Omkara wants
to get out of both contracts. Can
she?

Andy agrees to buy Charlotte's A condition precedent.


house. The purchase and sale
agreement states that if the house
passes an inspection, the parties
are obligated to go through with
the deal. The clause about the
inspection is:
Ted and Aisha enter into a contract No, because the contract contained a "time is of the
that states that Aisha will buy Ted's essence" clause.
vintage sports car after Ted
replaces the engine and fixes the
transmission. The contract contains
a "time is of the essence" clause,
requiring Ted to have the car ready
for the sale by February 13. On
February 12, Ted calls Aisha and
says he needs two more days to
complete the repairs. Aisha
declines and buys another car. Did
Aisha breach the contract?

Silas has agreed to dig five wells Silas has committed an anticipatory breach. Noreen is
on Noreen's property over the next discharged and may immediately hire someone else to
month, working each weekday. dig the wells. She may also sue Silas for breach of
One Friday, after Silas has contract.
completed three wells for Noreen,
he informs Noreen that on Monday
he is going to start a project
digging sixty wells for Romeo and
that job will take him about a year.
What does this mean for Noreen?

Alex contracts with Rashard to Rasher will lose. The parties did not agree on the exact
purchase thirty umbrellas. Rashard specifications of the umbrella, but the court will imply a
ships the umbrellas to Alex, and condition that the umbrellas be waterproof.
Alex mails Rashard payment. When
the umbrellas arrive, Alex is
shocked to see that the fabric
canopy at the top of each umbrella
is made out of paper towel and is
not waterproof. Alex sues, and
Rashard argues that he never
indicated that the umbrellas were
made out of waterproof material.
What result?
Evans built a house for Sandra, but Sandra wins because Evans did not substantially perform.
the house had some problems. The
garage ceiling was too low. Load-
bearing beams in the "great room"
cracked and appeared to be
steadily weakening. The patio did
not drain properly. Pipes froze.
Evans wanted the money Sandra
promised for the job, but Sandra
refused to pay. Who wins?

Franklin hires Angela to paint his Angela loses because this is a personal satisfaction
portrait. She is to be paid $50,000 contract.
if the painting is acceptable "in
Franklin's sole judgment." At the big
unveiling, 99 of 100 attendees think
that Angela has done a masterful
job. Franklin disagrees. He thinks
the painting makes him look like a
toad. Franklin refuses to pay, and
Angela sues Franklin. Who wins
and why?

Krug International contracted with Krug wins because of true impossibility.


Iraqi Airways to build equipment
for training pilots. Krug then
contracted for Power Engineering
to build the specialized gearbox to
be used in the training equipment
for $150,000. Power did not know
that Krug planned to resell the
gearbox to Iraqi Airways. When
Power had almost completed the
gearbox, the Gulf War broke out
and the United Nations declared
an embargo on all shipments to
Iraq. Krug notified Power that it no
longer wanted the gearbox. Power
sued. Please rule.
Rothman's clothing store had a 20- No, based upon the "good faith" requirement.
year lease in a shopping center in
Phoenix, Arizona owned by
Foundation Development. In
addition to monthly rent, Rothman's
was obligated to pay common-
area charges four times a year. The
lease stated that if Rothman's failed
to pay on time, Foundation could
send a notice of default. Then if the
store failed to pay all money due
within 10 days, Foundation could
evict. On February 23, Foundation
sent to Rothman's the common-
area charges for that quarter.
Rothman's believed the bill was in
error and sent an inquiry on March
18. On April 10, Foundation insisted
on payment of the full amount
within 10 days, but it sent the letter
to Rothman's Phoenix office, which
was not responsible for paying the
bill. The Phoenix office forwarded
the bill to the relevant executive in
New York, who received it on April
20. Rothman's issued a check for
the full amount on April 24 and
mailed it the following day. On
April 28, Foundation sued to evict.
On April 29, the company received
Rothman's check. Will Foundation
be able to evict Rothman's?

Thirty-five members of the Ortiz No, Mary has no theory that supports her claim.
extended family were spread
across three states—Illinois, New
York and Florida—and they rarely
were able to get everyone
together. For Thanksgiving 2018,
Mary Elizabeth Ortiz decided to
surprise her family by booking a
mountain retreat for the entire clan.
She especially wanted to celebrate
her uncle's return from overseas
military service. On September 15,
2018, Mary rented a spectacular
lodge in the Tennessee Smoky
Mountains with over 20 bedrooms
and an incredible view, paying in
advance the full $2,500 per night
fee for 3 evenings, but not
purchasing travel insurance. The
lodge had the following
cancellation policy: "A sixty (60)
day notice is required for
cancellation. Cancellations that are
made within 60 days of the arrival
date, for any reason, forfeit the full
advance payment and reservation
deposit. You are advised to
purchase travel insurance
separately if cancellation costs are
a concern." The rental contract also
stated that the only exception
would be for "acts of God, such as
fires or earthquakes." One week
before Thanksgiving, Mary's uncle
had a heart attack and was
hospitalized. The next day,
November 16, 2018, Mary cancelled
the reservation, and requested a
full refund, which the lodge
refused to provide. Does Mary
have a claim for the return of her
$7,500?
Alais is a clothing fashion designer Alais prevails because the breach was material.
known for her French style of
elaborate heirloom smocking
combined with industrial elements.
Planning for an upcoming show in
Paris on September 26, she orders
on September 10 300 yards of a
custom reflective material from
Rutherford Industries for her
seamstresses to hand smock. The
supplier promises to deliver the
material before September 20, so
Alais will have time for her staff to
complete the sewing. When the
cloth doesn't arrive until the
evening of September 21, Alais files
a claim for breach against
Rutherford. Who prevails?

Yvonne promises to sell 500 No, because it was not foreseeable to McKinley that the
pounds of rice to Zed at market shipping delay would cost Yvonne so much money.
price. To secure Zed's rice, Yvonne
enters into a contract with
McKinley Venturesto to ship the
cargo. The contract stipulates that
McKinley will deliver the rice to
Yvonne within 48 hours. However,
the rice does not arrive for 5 days,
during which time the market price
of rice decreases 11 percent.
Because of the delay, Yvonne lost
considerable profit. Can Yvonne
recover her damages because of
McKinley's breach?
John contracts with FashionWare John can recover from FashionWare the $500 he paid for
for the purchase of 1,000 zippers the zippers, any reliance interest, and the expectation
for $1 each. The agreement states interest from his contract with Campers' Crevasse.
that John will pay $500 when the
contract is signed and the
remaining $500 when FashionWare
delivers the zippers. In the
contract, John specifically states
that he is buying the zippers for the
manufacture of 1,000 windbreaker
jackets for Campers' Crevasse,
which he is contractually required
to deliver to Campers' Crevasse in
30 days. FashionWare breaches the
contract, causing John to miss his
delivery date with Camper's
Crevasse, which then cancels its
contract with John. What remedies
are available to John?

Desmond has been a fan of Crispin's estate will be able to recover $2,000 in
Crispin's paintings for years and is restitution.
thrilled when the artist promises to
give him painting lessons. The two
agree that Crispin will provide
Desmond with five weekly lessons
for $1,000 each, payable once the
lessons are complete. After
teaching Desmond two lessons,
Crispin dies of a heart attack.
Crispin's estate sues Desmond for
the $5,000 payment for the art
lessons. What result?
Harlowe purchases a watercolor Harlow will be able to get the painting but not the chair.
painting by a prominent local artist
and a RecumbentSloth brand
recliner from a yard sale. She pays
cash for both items, and the seller
promises to deliver them next
week. Two days later the seller
calls Harlowe and says he no
longer wants to sell the items and
sends Harlowe back her money. If
Harlowe sues, will she be able to
get the items she bought?

Over spring break, you go to The fee is too high, and the bank is simply imposing a
Florida and accidentally spend too penalty.
much on your checking account's
debit card. When you get back to
school, you are shocked to see
you owe your bank $220, which
you pay back immediately.
However, you have not even had
time to recover from your sunburn
before the bank informs you that,
pursuant to the liquidated damages
clause in the contract you signed
when you opened the account,
you are being charged a $55
"overdraft fee" for every transaction
you made on your debit card after
your balance was $0.
Unfortunately, you made nine such
transactions, which means you owe
the bank an additional $495 in fees
(9 x $55 = $495). What is your best
argument against the overdraft
fees?
Anna worries that her brother Nick The court will award Nick nominal damages.
does not have good health
insurance, so she writes a contract
stating that if he mows her lawn on
the last Saturday of every month
she will buy him an expensive
insurance policy that he could not
otherwise afford "for at least the
next six months, and longer if
necessary." The next week, Nick
gets a great job at an investment
bank, which provides him health
insurance coverage. Anna
congratulates Nick and revokes the
deal. The breach of contract
infuriates Nick, who sues his sister.
What will result?

Pamela was injured in an auto A claim for punitive damages.


accident by an uninsured driver.
Pamela filed a claim with her
insurer, American Mutual, for
$2,000 under her "uninsured
motorist" coverage. American
Mutual told her that if she sought
that money, her premiums would
go "sky high," so Pamela dropped
the claim. Later, after speaking with
an attorney, Pamela sued. What
claim was her attorney likely to
make?
Rick was in the process of buying Restitution.
320 acres of land when Rick signed
a contract to sell that same land to
Simon. Simon paid Rick $144,000,
the full price of the land. Before
Simon could complete the
purchase of the land, Rick went
bankrupt. Which of the following
remedies should Simon seek from
Rick?

Lewis signed a contract for the The market value evidence shows that Lewis suffered no
rights to all timber located on harm and, therefore, had no claim.
Nine-Mile Mine agreeing to pay
$70 per thousand board feet
($70/mbf). As he began work,
Nine-Mile became convinced that
Lewis lacked sufficient equipment
to do the job well and forbade him
from entering the land. Lewis sued.
Nine-Mile moved for summary
judgment. The mine offered proof
that the market value of the timber
was exactly $70/mbf for which
Lewis had no contradicting
evidence. The court granted
summary judgment. Why?

John sold a Vermont lakeshore lot Deborah will likely win restitution damages.
for $115,000 to Deborah who
intended to build a house on the
property. John indicated the land
was suitable for the project, but
Deborah soon learned that a
wetland protection law prevented
building near the lake. Deborah
sued, seeking rescission of the
contract. What is the likely
outcome?
Ike is building a water park in Include a liquidated damages clause int he contract.
Charleston, South Carolina, with
attractions that are larger and more
extravagant than anything in the
area. Ike cannot afford delays in
construction and is nervous about
his contractor's ability to finish the
project on time. What can Ike do
protect himself?

Hector is outraged by the Hector will win nominal damages.


Ellsworth Elementary School
charity auction when the school
invalidates his winning bid on a trip
to Paris, refunds his money, and
refuses to give him the plane
tickets he won. He suffers no real
harm because a group of
sympathetic parents pool their
money and buy him a comparable
trip, but the breach by the school
still stings, so he sues. What result
is most likely?
Romanita is offered $500 to climb No, but she has a potential claim for the cost of the
to the top of the academic center, climbing equipment.
the tallest building on the university
campus. Before climbing the 80
foot tower, Romanita purchased
ropes, boots and a safety harness,
as well as other equipment, at a
cost of close to $150. After two
weeks of preparation and
encouragement by the offeror,
Romanita arrives at the tower.
However, just before her feet leave
the ground on her ascent up the
tower, the offeror withdrew the
offer, citing safety concerns. Does
Romanita have a claim for the
$500?

Damon contracts with Steve No, because specific performance is not an appropriate.
Redmont of Redmont Roofers, Inc.
to replace the roof on his 1850
Greek Revival mansion in Ohio by
April 15. The project involves
cutting and shaping intricate trim
pieces all along the roofline, as
well as replacing the slate shingles
and underlying wood support
system. Steve begins to get
frustrated on April 2 because the
project is taking too long, so at
about the halfway point, he calls
Damon and says he won't finish the
project. Damon sues Steve in court
and asks specific performance. Will
the court likely grant this remedy to
Damon?
One provision in a contract states Ulysses has made a warranty and representation; Emma
"Ulysses vows he is a board- has made a covenant.
certified physician" and another
states "Emma promises not to eat
red meat for six months from the
date of this agreement." Which of
the following best describes these
contract terms?

Slimline and Distributor signed a Slimline, because Distributer's conduct is a material


contract that provided that breach.
Distributor would use reasonable
efforts to promote and sell Slimline
Diet Soda, which was already
being sold at Warehouse Club.
After the contract was signed,
Distributor stopped conducting the
in-store demos of Slimline it had
previously been doing at
Warehouse, and it did not
repackage the product as Slimline
and Warehouse requested. Sales of
Slimline continued to increase
during the term of the contract.
Slimline sued Distributor, alleging a
violation of the agreement. Who is
likely to win?

Malay, the CEO of Santi, Inc., Anthony, who tells Malay about the risks of that sort of
interviews four lawyers to help with deal and ways to accomplish Santi's goals safely and
a potentially lucrative business legally.
deal. He provides each candidate
the bare facts of the deal so they
can discuss it and present their
approaches. Which of the four
should he hire?
Lucian has every reason to believe Yes, because it is essential to the success of Lucian's
the new pizza shop he is opening business that construction goes smoothly.
will be a success, but he also has a
lot riding on it. He has borrowed a
significant amount and has
refinanced his house to pay for the
expenses of starting up. If the
business does not succeed, it will
be a disaster for his family.
Fortunately, the construction and
start up of the business will be
pretty simple, and he is hiring his
sister Allegra (an experienced
contractor) to do most of the work.
Should Lucian and Allegra put their
contract in writing?

Fred slips on some ice on Opal's The insurance contract was ambiguous, so the Community
front steps and shatters both his Rules will not prevent Opal from recovering.
kneecaps. Opal has a
comprehensive homeowner's
policy with Riskocity Insurance, but
Riskocity refuses to pay for Fred's
injuries as the policy does not
cover accidents caused by Opal's
"dangerous conduct." The
Community Rules of Opal's gated
community require residents to
keep their front steps free of ice.
Riskocity contends that because
Opal violated the Community
Rules, she is liable for Fred's injury.
Opal argues that the insurance
policy makes no mention of her
Community Rules and because she
could not foresee Riskocity relying
on the Rules, they should not
prevent her from recovering. What
result?
Louise Latour, a successful interior No. There is clear and convincing evidence that the
designer, won a contract to design mistake does not reflect the intent of the parties, and the
the interiors of Greentree court will reform the contract.
Broadcasting's corporate
headquarters. A few weeks after
signing the agreement, Greentree
receives a much cheaper bid from
another designer and decides to
get out of the contract. A
Greentree lawyer notices that the
top of the contract contains an
embarrassing typographical error:
It states that the agreement is
between "Louise Latour and
Grantree Broadcasting." Will
Greentree be able to use this
mistake to get out of the contract?

Stephan and his boss agree to Yes. This is a valid way to execute a written amendment.
extend his employment contract by
an additional six months. The
employment contract states that all
amendments must be in writing, so
Stephan and his boss cross out and
change the provision in his contract
that covers the duration of his
employment and rewrite it to
include the additional six months.
They each initial and date the
change, and they each keep a
copy of the agreement. Is the
amendment valid?
Rosa and Happy Movers sign a No, because Rosa did not sign the flyer.
contract that states Rosa will pay
Happy Movers $100 per hour to
move all of her furniture to her new
house on September 1. The
contract also states that any
modifications to the agreement
must be "made in writing signed by
the party to be charged with the
amendment." The day of the move,
a Happy Movers representative
shows Rosa a flyer for a promotion
offering to complete a move in
under two hours for an extra $300,
and Rosa agrees orally to the deal.
The Happy Movers representative
signs his name on the flyer, and
staples the flyer to the contract. If
Happy Movers finishes moving
Rosa's furniture in less than two
hours, is Rosa contractually
required to pay the extra $300?

Sam and Elias agree in writing that Elias will not be liable if the contract contains a clause
Sam will give Elias needlepoint discharging the obligation if there is a force majeure
lessons once a week for five weeks provision.
in exchange for $1,000 per lesson.
The contract stipulates that if either
party is going to cancel a lesson,
they must provide 48 hours' notice.
On the morning of the second
lesson, Elias calls Sam and says he
cannot make it because
earthquake damage has closed all
roads leading out of his town. Will
Elias owe Sam for the missed
lesson?
Fritz's Pet Shop has a contract with Yes, because the terms are reciprocal promises.
PlanetLazy Pet Products that states:
a) PlanetLazy will deliver to Fritz 25
pet beds on the second Saturday
of every month for one year; b)
Fritz will pay PlanetLazy $2,500 on
the first day of each month for one
year. If Fritz does not pay
PlanetLazy on a given month, is
PlanetLazy still obligated to
continue delivering pet beds?

PillPharm is negotiating a deal with Send HVH its own letter of intent stating that they do have
Happy Valley Health (HVH) for the a binding agreement.
sale of PillPharm's latest health
supplement. PillPharm would like
to package its supplements in
bottles produced by Frank, but
Frank says he will not enter into a
contract with PillPharm until the
HVH contract is finalized. HVH has
sent PillPharm a letter of intent, but
Frank thinks it is too vague and
refuses to enter into an agreement
with PillPharm. What should
PillPharm do?
Deborah, a lawyer for a minor- Add an integration clause to the contract.
league hockey team, is negotiating
a contract between the team and
its new practice stadium. The
contract, now in its fourth draft,
addresses all of Deborah's
concerns and incorporates all of
the changes she wanted. She is
ready to sign the agreement, but
she wants to be sure that any
future conversations she has with
the stadium lawyers are not
considered additional provisions to
the contract. What should Deborah
do?
Confluence Corporation, located The prices are in dollars.
in Champaign, Illinois, decided to
begin buying rare earth metal from
the Canadian Rare Earth Company,
with headquarters in Montreal,
Quebec Province. The following
clause was included in a signed
contract between the companies
to purchase quantities of the metal
Lanthanum for Confluence's
production of rechargeable
batteries: "During the Initial Term,
Seller shall sell the Products to
Buyer at the purchase prices set
forth on Exhibit A. All prices are in
dollars with no cents. Any
adjustment to prices will require
written modification signed by
both parties within ten (10) business
days." A few weeks after signing,
the Confluence Corporation
attorney calls the Canadian
company with a concern about the
clause. What do you believe this
concern may be?
The Onyx Company negotiated Severability provision.
intensely for months to hire Robert
Conyers as its new Chief Executive
Officer (CEO). Attorneys for both
the Company and Mr. Conyers
were now working on the final
employment contract, and it was
especially important to the Onyx
Company to keep their new CEO
for at least 10 years. Because Mr.
Conyers had exceptional skills and
considerable experience related
directly to the company's industry,
it was important for the contract to
include an atypical retention clause
in the event of a merger with
another entity. If Onyx's attorney
was concerned about
enforceability of the retention
segment, what could they add to
the contract to safeguard the rest
of the provisions?

Lionel is the personal assistant to Tell his boss about the tickets. His boss may choose to
the head coach of a professional keep the tickets for himself.
football team. At the end of the
football season, a football player's
wife gives Lionel front row tickets
to the opera. What must Lionel do?

Beatrix is a diamond dealer who Yes, but Beatrix must tell BMD that the salmon works for
also works as a jewelry designer at her.
Bunny Mayhew Designs (BMD).
Beatrix arranges to have one of her
own company's salesmen meet
with a BMD representative to try to
sell some of Beatrix's available
stock. May Beatrix do this?
Zephyr agrees to mow his brother Yes, because Zephyr's conduct was grossly negligent.
Zeke's lawn for free while Zeke
recovers from a broken leg. Zeke
tells Zephyr not to use the mower
if its warning light turns on, but
Zephyr ignores the warning. The
engine overheats, ruining the
lawnmower. Is Zephyr liable for the
damage?

Malik asks Petra, an expert in No. If Petra has no reason to believe that Malik would be
sports collectibles, to help him find interested, she does not have to pass the offer on to him.
a buyer for his collection of
autographed baseballs. Petra finds
a buyer who is willing to pay
$5,000 for the collection, but Malik
scoffs at the offer and insists it is
worth at least double that amount.
He says he would not even
consider selling for less than
$7,500. The next week, someone
offers to pay $1,000 for the
collection. Must Petra tell Malik
about the offer?
Heather and her employee Emilio Heather is liable for the indemnification of Emilio if Emilio
believe they see Ethel, a customer did not realize he was committing a tort.
of Heather's clothing boutique,
shoplift some mittens. Heather tells
Emilio, "Lock her in the back room,
and don't let her out until you
figure out what's going on. I have a
flight to catch." Heather leaves for
the airport. Emilio questions Ethel
in a back office for three hours, but
is unable to determine what
happened with the mittens. When
Ethel is finally released, she sues
for the tort of false imprisonment.
Assuming that Emilio's acts
constitute false imprisonment, is
Heather liable for the
indemnification of Emilio?

Liam tells Emily that he wants to The contract price of the car because Liam is an
buy her friend Tamara's car, but unidentified principal.
does not think Tamara will sell it to
him for personal reasons. Emily
tells Tamara that she knows
someone who would like to buy
her car, and the two draw up a
contract stating that Tamara will
sell the car to "an undisclosed
buyer" for $10,000. After Tamara
has signed the contract and given
Emily the keys, Emily has Liam sign
the contract, agreeing to pay the
$10,000 purchase price. Liam takes
the car but disappears without
paying. What, if anything, will
Tamara be able to recover from
Emily?
Alissa accepts a six-month unpaid Yes, because she is a gratuitous agent.
internship at Jonah's dental
practice. After one month, Alissa
wants to quit but knows that if she
does it will cost Jonah a lot of time
and money to find and train
someone new. Can she quit before
the end of the six months?

The German-American Vocational No. A formal contract is not necessary to establish an


League was formed in New York agency relationship.
during World War II to serve as a
propaganda agency for Nazi
Germany. Under U.S. law, all
foreign agents were required to
register, but neither the Vocational
League nor its officers did so.
When they were charged with
violating U.S. law, they argued that
they were not agents of the
German government because they
had no formal agency agreement.
Is this a strong argument?

An elementary school custodian Yes, the school district is liable if the custodian intended
struck a teenager who wrote to serve some purpose of the school when he struck the
graffiti on the school's wall. Is the student.
school district liable for this
employee's intentional tort?
The Fellowship is a not-for-profit No, because there was no control or consent between
corporation whose primary CCT and The Fellowship.
purpose is promoting goodwill
among churches and synagogues.
Its annual meeting featured various
vendors with display booths.
Keener, a church representative,
approached the booth of Chris's
Cars and Trucks (CCT) about
buying a bus for his church. While
Keener was test-driving one of
CCT's buses, the brakes
malfunctioned, and Keener was
killed in the resulting crash. If CCT
is liable for the accident, could The
Fellowship also be liable as CCT's
principal?

Jamal worked for a delivery Yes, because the stop was a mini detour from his work.
company, and he was driving
sometimes up to 10 hours a day.
One afternoon, on the way back
from making a delivery in his
neighborhood, Jamal stopped at
the uniform shop to pick up some
shirts he needed for work. Usually
he would run his errands in his
personal vehicle after the day
ended, but the uniform shop was
on the way to his next delivery
anyway. As he turned into the
shopping center, Jamal was not
paying attention and hit a bicyclist
crossing the road, seriously injuring
him. Is Jamal's delivery company
liable for the bicyclist's injuries?
Barbara had a dream of opening Yes, because she used The Bread Factory's resources and
her own bakery she planned to time to prepare for her own business.
name "Sinful Sweets." While she
saved money to start her business,
she worked for a big commercial
baker known as "The Bread
Factory." The Bread Factory made
many different types of bread, but
none of the sweet baked items
Barbara hoped to offer in her own
shop. So, while Barbara made
French baguettes and sourdough
loafs, she thought about cupcake
icing and baking the perfect
chocolate chip cookie. Soon, the
daydreaming led to snips of time
searching the internet on her
mobile phone for recipes; then she
progressed to starting a file on her
supervisor's computer desktop
filled with ideas and saved
searches. Barbara started testing
her recipes using the bakery's
materials, and she found that all
her efforts were making her a
better and more efficient baker.
Once she had tested all her recipes
at the bakery, Barbara's next step
was to take two accumulated sick
days to search for a storefront for
Sinful Sweets. She found the
perfect spot and provided two
weeks' notice to The Bread Factory,
including one week of vacation she
had saved. Has Barbara through
her actions breached a duty of
loyalty to her soon-to-be former
employer?
George and Susan open a dry- G & S Clean is liable, and both Susan and George are
cleaning business together, but do personally liable.
not execute any documents or
perform any formalities other than
obtaining a "d/b/a" certificate for
their business name: "G & S Clean."
One day, when George is out for
lunch, Susan burns a large hole in a
customer's fur coat while cleaning
it. Who is liable for the cost of the
coat?

Wanda, Derek, and Mitchell No, unless the partners have not complied with the LLP
formed B. Flowers, LLP three years statutory filing requirements.
ago. A year ago, Derek and
Mitchell decided that the company
was ready to open a new location
and had B. Flowers take out a loan
from a bank for the extra capital
needed to expand. The new
location has not performed well,
and B. Flowers is unable to pay the
remaining $50,000 owed to the
bank. Is Wanda liable to the bank
for the debt?

Mack is the local owner of a Yes, if the franchise agreement demands it.
restaurant franchise. Though the
national chain is known for its
hotdogs, Mack wants to sell
vegetarian burritos. Must Mack get
permission from the franchisor?
Alan and Ivan opened a kosher General Partnership
delicatessen, Main Court, which
failed after barely a year in
business. One supplier sued for
overdue bills. Alan and Ivan will be
liable to the supplier if Main Court
was which of the following types of
organizations?

In order to obtain limited liability, Yes, the court can pierce the veil of an Llc because Tom
Tom and Doris properly formed a and Doris commingled assets.
limited liability company (LLC) to
operate their catering business.
They sometimes deposited the
proceeds from catering jobs into
their personal checking accounts,
and if they needed to pay personal
bills and were short of funds, they
used the business account. If
creditors of the business cannot
get payment for their invoices, will
a court order Tom and Doris to pay
the creditors using their personal
assets?

Russell and Rachel have designed Corporation


a new type of cell phone that they
believe will revolutionize the
market. They would like to start a
company to produce, market, and
sell the phone, and they know that
they will need a considerable
amount of up-front capital
investment to develop a prototype
and later to create inventory to
sell. What is the best form of
business for Russell and Rachel?
Wilma is one of five equal Wilma (and each of the members) pays taxes on their $1
members of Polar, LLC. This year, million share ($5 million divided by 5) of profits.
Polar generates $5 million in
profits. The company reinvests $4
million into the company, leaving $1
million to be divided equally
among the members. How are
federal income taxes paid on
Polar's profits?

Fitness World is a corporation with Yes, if all of the shareholders are U.S. citizens or residents.
75 shareholders that are individuals
who all hold common stock. At an
annual meeting, the shareholders
unanimously voted that Fitness
World should register as an S
corporation with the IRS so that the
shareholders may enjoy the flow-
through tax benefits. Is Fitness
World eligible to register as an S
corp?
Clean Earth, Inc. is a Delaware Against the shareholders, if it finds that Clean Earth has
benefit corporation that produces acted in a "responsible and sustainable manner."
cleaning products using 100
percent recycled, organic, and
sustainably produced materials.
Several years ago, it became a
socially conscious organization and
has fulfilled all of the reporting
requirements since then. Clean
Earth executives have decided to
purchase its paper product
supplies (such as napkins and toilet
paper) from a new supplier, who
plants two trees for every tree it
uses. Since this supplier charges
five times more than Clean Earth's
previous supplier, profits from
Clean Earth's paper products are
expected to be cut in half. If a
Clean Earth shareholder
challenges the decision as being
unfair to shareholders, how will a
court likely rule?

Two general contractor firms, Atlantic Builders and North West Mechanical are both
Atlantic Builders and North West liable.
Mechanical, form a joint venture
for the purposes of completing a
large construction job. They are
collaborating on all phases of the
project. An Atlantic Builders'
employee operating a crane
accidentally drops a steal beam
onto a car parked near the
construction site. Who is liable for
the damage to the car?

Zack, Gavin and Breana were No, because the LLC was an entity independent from its
college friends who decided to go owners.
into business together as a party
equipment rental service. The three
budding entrepreneurs formed as
equal owners a Limited Liability
Company (LLC) named "Parties R
Us." The written operating
agreement gave only Zack the
authority to manage the business,
hire and fire employees, and
purchase equipment. Zach
contributed to the LLC his
knowledge in the party business
(his family had owned a similar
company), and Gavin and Breana
each contributed $100,000 capital.
Over the next year, Zack ran the
business, but he did not consult
with Gavin and Breana, and no
meetings were held. When Gavin
and Breana asked what was going
on with the company, Zach said he
was too busy to talk about it. At the
end of the first year, despite Zach's
best efforts, the LLC's cash was
depleted and its liabilities
significantly exceeded its assets. To
make matters worse, a defective
Parties R Us tent had collapsed
onto a wedding group, causing
serious injuries. One of the
wedding guests, Margaret, sued
the LLC and received a $500,000
judgement against them. Because
the LLC cannot pay Margaret, is it
likely that Zack, Gavin, and Breana
will be held personally liable for
the judgment?
Surgeons Dr. Martin and Dr. Only Dr. Kellogg would be liable for the $4 million.
Kellogg are partners who decided
to form a professional corporation
together. After the business was
established, the doctors went out
to celebrate the new enterprise
with some colleagues.
Unbeknownst to Dr. Martin, Dr.
Kellogg, who was on call that day,
had to leave the party to perform
emergency surgery on an accident
victim. Unfortunately, Dr. Kellogg
had too much alcohol at the dinner
and was drunk during the
procedure. As a result of his
intoxication, Dr. Kellogg accidently
cut a major artery in the patient's
abdomen, and the individual bled
to death on the operating table. A
jury in the malpractice suit that
followed awarded a $5 million
judgment against Dr. Kellogg and
the corporation. Insurance covered
$1 million of the damages. Who
would be responsible for the
remaining $4 million?

Xavier and William are both William and Xavier will each receive $25,000.
architects. They have formed a
partnership, but have not signed a
partnership agreement. Xavier
landed a big contract with a major
real estate developer and did all
the work for the project, which
generated $50,000 in profits.
Under the Uniform Partnership Act
(UPA), how will the profits from this
project be divided?
Percy and Yolanda are partners in a Yes, if the theft occurred through the ordinary course of
business that provides financial business.
advisory services. Percy steals
$50,000 from a client's account
and immediately disappears with
the money. Is Yolanda liable to the
client for the $50,000?

Andrew, Betty, and Charles form a Charles will be wrongfully dissociated, and Andrew and
partnership to produce five Betty must decide whether or not to continue the
independent films. After they partnership without Charles.
complete their third film, Charles
decides that he would like to go
back to school full-time, and tells
Andrew and Betty that he will not
produce the remaining two films
with them. What will happen to the
partnership?

FAB has three partners, Ramon, Jackie must pay a net amount of $5,000.
Janna, and Jackie, and $20,000 in
assets when it begins its winding-
up process. FAB owes $20,000 to a
supplier, $25,000 on a bank note,
and $5,000 to Jackie for a loan she
gave to the partnership. FAB does
not have a partnership agreement.
What financial rights and
obligations does Jackie have?
Joel owns a construction business. Yes, under a theory of partnership by estoppel.
Joel's brother-in-law, Barry, works
for him on several of his large
projects. At the grand opening of
one of Joel's buildings, a client
introduces Joel and Barry to Marie,
referring to the pair as partners.
Neither Joel nor Barry corrects the
client. Impressed with the building,
Marie asks Joel and Barry to
manage her real estate
development firm's next project.
Joel does not want to take the job,
but Barry wants to try to run his
own project, so he accepts. The
project is a disaster, and Marie
loses $2 million due to Barry's
mismanagement. May Marie hold
Joel liable?

Sarah and Mark formed MCS as Mark, Sarah, and MCS


partners years ago, which they
have now dissolved and are in the
process of winding up. Without
Mark's knowledge, Sarah cancels
MCS's insurance policy covering
the MCS van. Two months later,
while making a delivery for MCS,
Mark runs a stop light and hits a
pedestrian. Who will be liable for
the damages caused by the
accident?
Herman rightfully dissociated from Yes, but Herman can recover whatever he pays to Great
Oak Tavern, a partnership, 18 Grains from Oak Tavern.
months ago. Neither Herman nor
the remaining Oak Tavern partners
notified anyone with whom Oak
Tavern does business that Herman
had left the partnership. Oak
Tavern has not paid one of its
suppliers, Great Grains, for its
purchases over the past 10 months.
Can Great Grains recover the
money it is owed by Oak Tavern
from Herman?

Jasmine and Joanna run a bakery as No, but Joanna must turn the earnings over to the bakery.
partners. Joanna's sister asks her to
provide all of the desserts for her
company's annual New Year's
party, and Joanna agrees. Joanna's
desserts are such a hit with the
partygoers that the company sends
Joanna a check for $500. Joanna
would have provided the food for
free, but happily accepts the
check. When Joanna tells Jasmine
about the party and the check,
Jasmine insists that Joanna must
give her half of the money. Is
Jasmine correct?
Sports Destination is a sporting No, because a merger requires unanimous approval from
goods store owned by four the partners.
partners: Jerry, Kevin, Lisa, and
Marcy. They have a partnership
agreement, but it does not state
anything about voting rights. Gear
Head, a competing sporting goods
store, has made an offer to merge
with Sports Destination. Jerry, Lisa,
and Marcy vote in favor of
accepting Gear Head's offer, but
Kevin does not. Has the merger
been approved by Sports
Destination?

Ramon and Martin sign a written No, because they do not share profits.
agreement stating that they will co-
manage a hardware store together.
Martin agrees to provide 75
percent of the start-up capital for
the store and cover 75 percent of
any losses suffered during the first
year. Ramon is entitled to all of the
profits. Ramon manages and
operates the store while Martin
handles the book-keeping and
other administrative functions. Will
a court find that Ramon and Martin
formed a partnership?
Molly and Corbin met while both Only Green Fields LLP is liable.
studying horticulture in college,
then after graduating decided to
start a landscaping business
together. Their partnership—"Green
Fields"—started operating and
hired a landscape architect and an
employee to provide labor for
projects. Everything went well for
about four months, until the
employee negligently failed to
mark a deep hole in a client's yard.
The client's 4-year-old daughter
fell into the opening and was
severely injured. The client sued
Green Fields and was granted a
judgment of $250,000, which has
not been paid. After the judgment
was entered, Molly and Corbin
transitioned the partnership into a
Limited Liability Partnership (LLP).
They are operating the business
now as Green Fields LLP. To help
with continuing costs, Molly and
Corbin take on an investor-Partner,
who contributes $50,000 to Green
Fields LLP. Are Green Fields LLP
and the new partner liable for the
judgment against the original
Green Fields Partnership?
The Ford Motor Corporation and Joint Venture.
Toyota Motor Corporation planned
to join forces in 2011 to produce a
hybrid truck that would meet fleet
emission standards. Toyota had
considerable experience at that
point with hybrid vehicles, and
coupled with Ford's long history of
truck manufacturing, it would be a
potentially productive partnership.
What would be the best structure
for the collaboration between
these two automotive giants?

Wayne borrows money from No, because Retail is a corporation by estoppel.


Phyllis, who in good faith
represents herself as an agent for
Retail Lender, Inc. Wayne learns
that Retail was never incorporated
due to a significant error in Phyllis'
incorporation paperwork. If Phyllis
sues to recover the loan amount,
can Wayne avoid liability based on
the fact that Retail does not exist?

Marcus owns common stock in XO, No, unless there are assets remaining after all creditors
an oil and energy company that is and preferred stockholders are paid.
about to be liquidated. Is Marcus
guaranteed to be paid in the
process of dissolution?

Hannah owns non-voting stock Conversion Rights.


shares in Asset Management, Inc.,
but would like to vote at the next
shareholders meeting. What tool
could help Hannah gain the
authority to vote in shareholder
meetings?
George is president of Plumbers, George commingled personal assets with corporate
Inc. He signs a contract with Susan, assets.
which calls for Susan to perform
some services and receive $10,000.
Susan performs, but Plumbers does
not pay. Which of the following, if
true, will grant Susan the right to
recover from George personally?

Michi Corp.'s board of directors Yes, because a court may dissolve a corporation if it is
has determined that the insolvent insolvent or if its directors and shareholders cannot
company will not be able recover resolve a conflict over corporate management.
financially and it is best to
terminate the corporation. The
board unanimously votes to
dissolve and makes a
recommendation to dissolve to the
shareholders, but only 40 percent
of the shareholders vote to
approve dissolution. The
shareholders insist that Michi is
simply mismanaged and can be
saved if the board appoints a new
CEO. May the Michi board request
a court to order dissolution of the
company?
The Resolution Trust Corp. (RTC) No, because there is no indication that the board acted
sued the directors of the without authority or illegally.
Commonwealth Savings Corp.
(CSC) seeking to recover from
them personally $200 million that
the bank lost in bad real estate
loans. The directors approved the
loans after state and federal
regulatory agencies had issued
reports criticizing the bank's loan
practices, but the loans did not
violate any laws. CSC's charter
stated that the purpose of the
corporation "is to engage in any
lawful activity for which
corporations may be organized."
Under Texas law, RTC could
recover for CSC directors'
negligence only if their acts were
ultra vires. Will RTC be able to
recover from the directors?

MegaCorp has five directors and 176 shares if MegaCorp uses cumulative voting and 526
1,050 shares of voting stock. Jessica shares if MegaCorp uses regular voting.
would like to purchase enough
stock to elect herself to the board
of directors. How many shares of
MegaCorp stock will Jessica need
to own to secure a place on the
board of directors?
Denise was a promoter for a Yes, she remains liable as the promoter on a contract for
proposed corporation, EVR-Young which there has been no novation.
Corp. As promoter, she signed a
three-year lease to rent office
space from Landlord. She signed
her name and indicated below her
signature she is signing as
"promoter for EVR-Young Corp., a
company yet to be incorporated."
EVR-Young never files
incorporation documents with the
state. Is Denise personally liable for
the lease?

Isabel, Wesley, Gina, and Lucas Yes, they can sign a shareholder agreement.
worked together intensively to
form an environmentally conscious
corporation named Icon Corp.
They are the only shareholders. Is
there anything the group can do to
avoid having Icon shares sold to
someone who does not share their
vision for the environment?

Zoom-Bot has common stock and All cumulative preferred shareholders will be paid for all
cumulative preferred stock previously unpaid dividends along with this year's and any
shareholders. The company is remaining funds will go towards the payment of this year's
issuing dividends for the first time dividend to common shareholders.
in three years. How will the
dividend funds be divided by the
shareholders?
Kramer hired an attorney on March No, because Kramer and IBM treated WebVisions like a
1 to incorporate his start-up legitimate corporation.
website development company. He
began operating as president the
corporation "WebVisions", and
strictly adhered to all formalities
associated with the corporate
form. One of the actions Kramer
took was to sign a contract with
IBM for the purchase of a dozen
computers for his programmers to
use. For six months, WebVisions
carried on business of the
corporation as usual, and made
monthly payments to IBM on the
computer systems. Kramer then
discovered that the attorney he
hired had only just filed the
incorporation document on
September 15. When IBM
discovered the oversight, they filed
a claim against Kramer for the
$28,678 remaining on the computer
purchase contract. Is Kramer
personally liable for the balance?
An appraiser valued a subsidiary of Yes, because they acted in good faith.
Signal Co. at between $230 million
and $260 million. One month later,
Burmah Oil offered to buy the
subsidiary at $480 million, giving
Signal only three days to respond.
The board of directors accepted
the offer without obtaining an
updated valuation of the subsidiary
or determining if other companies
would offer a higher price.
Members of the board were
sophisticated, with a great deal of
experience in the oil industry. A
Signal Co. shareholder sued to
prevent the sale. Is the Signal
board protected by the business
judgment rule?

The board of directors of Athletic Probably yes, because courts generally will support a
Stride, a successful sneaker board decision if there is even a remotely rational
company, votes to have the business purpose and no other breaches of the
company purchase a professional managerial fiduciary duties.
basketball team for $500 million
dollars. The directors are all
knowledgeable about the
professional sports industry, and
none of them have a conflict of
interest in the deal. When the
shareholders oppose the purchase
as being unrelated to the
company's business, the board
argues that, as the team's owner,
Athletic Stride will have invaluable
marketing and sponsorship
opportunities. Would a court likely
uphold the board's decision to
purchase the basketball team?
Frank, the CFO of Springer, Inc., No, because the business judgement rule does not
concealed temporary financial protect illegal activities.
losses the company suffered early
in the fiscal year in Springer's
annual report submitted to the
Securities and Exchange
Commission (SEC). It is illegal to
submit fraudulent SEC filings, but
Springer's financial situation had
already improved greatly by the
time the annual report was filed
and Frank did not want to
unnecessarily alarm shareholders.
When news that Springer
submitted fraudulent financials to
the SEC broke, the company's
stock price dropped by 15 percent
and shareholders sued Frank under
applicable securities laws. Frank
defended on the basis that his
actions were protected by the
business judgment rule because he
was acting in the company's best
interests. Was Frank correct?

Sammy's, Inc. is a publicly traded Yes, the board has breached its duty of care.
company. Sammy's board of
directors appointed Jeb as CEO.
Jeb has made a number of
blunders, which drove Sammy's
deeply into debt. In its first meeting
in six years, the board of directors
votes for Sammy's to file for
bankruptcy. Has the board
committed any violations?
Data Management, Inc. is No, because the DocuWorld directors can issue the
interested in acquiring DocuWorld preferred stock as a poison pill.
Corp., but the DocuWorld board is
not interested in selling or merging.
DocuWorld's shareholders
authorized blank check preferred
stock in the corporate charter. Will
Data Management be able to
bypass the resistance of the
DocuWorld board by making a
tender offer to DocuWorld
shareholders?
Ocean View Inns recently Asset lockup
purchased 5 percent of Seaside
Resorts' stock. Seaside Resorts'
board of directors fears that Ocean
View Inn is going to attempt to
take control of the company in
order to acquire Seaside Resorts'
hotel in California; it is the only
Seaside Resorts location that
directly competes with Ocean
View Inns. Although it takes a
considerable amount of business
away from Ocean View Inns, it
accounts for only five percent of
Seaside Resorts' earnings. The
Seaside Resorts' board suspects
that Ocean View Inns would retain
the California location and sell off
the rest of the Seaside Resort
assets in pieces. Although the
Seaside Resorts' board has a good
faith belief that a takeover by
Ocean View Inns is not in the
shareholders' best interests, it does
not believe that a majority of the
shareholders will work with them to
resist a takeover. What antitakeover
device is the board's best option?
Power Wind is a corporation that Yes, if they are in a state that has codified Unocal.
specializes in designing and
manufacturing windmills. Power
Wind has a manufacturing plant in
Colorado and offices throughout
the southwestern United States.
NRGreen, a provider of a variety of
renewable energy design and
installation services, has offered to
buy Power Wind at a favorable
price. However, because
NRGreen's business model does
not involve any manufacturing, it
will close the Colorado plant
immediately after acquiring Power
Wind. May Power Wind's directors
decline NRGreen's offer in order to
keep the plant open?
Dominique is the CEO of a The court will rescind Dominque's decision and hold her
pharmaceutical company and must liable for any losses suffered by the company as a result
decide whether to continue the of the decision, unless Dominique can show the
company's research into a new skin transaction was entirely fair to the shareholders.
treatment. Originally, the company
had hoped the treatment would
help burn victims recover quickly,
but two years and $5 million in
testing has revealed that it is only
useful in relieving very dry skin if
applied hourly. In addition,
projections show that it will be so
expensive to produce the
treatment that the company will
have to charge $100 a day for it in
order to make a profit. However,
the researcher in charge of the
project is Dominique's best friend,
so Dominique approves another $5
million to be spent on continued
research of the treatment. If the
company's shareholders challenge
Dominique's decision, how will a
court rule?
Kurt owns 55 percent of the New Strike down the deal unless it is entirely fair to NELC.
England Lumber Company (NELC)
stock and is also the CEO. Kurt also
owns two percent of the Boston
Homes stock. Boston Homes is a
construction company that builds
houses throughout New England.
Kurt wants NELC to provide Boston
Homes with all of the lumber
Boston Homes needs and to give
Boston Homes a ten percent
discount on all of its large orders.
Four NELC board members who
have no interest in Boston Homes
form a special committee and
approve the deal. If a NELC
shareholder challenges the deal
between NELC and Boston Homes
in court, how should the court
rule?

Carly is a director at Dazzle, a No, because Dazzle turned down the opportunity to work
corporation that operates a chain with Zeb.
of stores that sell fancy shoes and
handbags. Zeb approaches Carly
about partnering with him in his
new business selling leather
handbags with large buckles and
other metal accents. Carly brings
Zeb's offer to the Dazzle board of
directors and shareholders, but
they turn it down because Zeb's
bags are too casual and do not fit
in with the rest of their stores'
merchandise. Carly loves Zeb's
bags, though, so she decides to
personally invest with him. Has
Carly violated her fiduciary duty to
Dazzle?
Mona was working as a medical Yes, because she has a responsibility to bring the
biller-coder supervisor for a opportunity to the Olaf Group first.
medical physician group in primary
care called The Olaf Group, and
she was also a shareholder and
director in the overall medical
company. After about two years of
supervising the Olaf Group, Mona
heard about an opportunity to do
billing for a dermatology practice
in town. Although she knew the
Olaf Group could do the work,
Mona thought that it would be
more effective for the
dermatologist if she did the work
independently. So, instead of
bringing the opportunity to the
Olaf Group, Mona decided to just
take on the dermatology work on
her own. Has Mona breached any
fiduciary duties to her group?

Unhappy with the management of Yes, if the shareholder has owned at least one percent or
Jackpot Enterprises, Inc., a $2,000 of Jackpot's stock continuously for a year.
shareholder asked the company to
include a proposal in the proxy
statement that would require the
board of directors to sell or merge
the company. Must Jackpot include
this proposal in its proxy
statement?
Pro Threads, Inc., an international Only D-Satisfaction shareholders need to approve the
designer clothing manufacturer, is merger.
acquiring by merger D-Satisfaction,
a small manufacturer that
specializes in fitted dresses.
Dresses of this type account for
one half of one percent of Pro
Threads' sales. Do Pro Threads and
D-Satisfaction shareholders need
to approve the merger?

Drug Co. paid $3 billion to settle a No, if the shareholders can show that demand was futile.
lawsuit and pay a criminal fine after
it illegally marketed 13 of its most
important drugs. Shareholders filed
a derivative suit against the Drug
Co. board and top executives.
Defendants responded with a
motion to dismiss on the grounds
that shareholders had not made
demand on the board. Should the
court grant the motion to dismiss?
After Classic Corp. went public at Yes, they have legal protection and are owed a fiduciary
$12 a share, it began suffering many duty from the majority shareholders.
years of losses. Isaac Fogel, who
owned 64 percent of the stock,
decided to take the company
private again by buying
shareholders' stock at a price of 20
cents a share. Classic hired two
financial advisers who opined that
the buyout price was fair. The
board of directors voted in favor of
the sale and then scheduled a
special shareholder meeting to
vote on the buyout. Do the
minority shareholders have any
rights?

George is a majority shareholder in No, because George has no right to control Soda Pop
Soda Pop, Inc., a large soft drink operations.
maker. Citizens in India are
threatening to sue Soda Pop for
stealing water in rural villages,
ruining crops, and depriving
livestock of water. If they are
successful in their suit, will George
be liable to the Indian plaintiffs?

Wanda, a realtor, owns stock in No. This is not a proper purpose to access company
Mackey Land, a publicly traded real information.
estate development company. To
market her own realty business,
Wanda demands that Mackey Land
provide her the list of Mackey Land
shareholders. Is Wanda entitled to
this information?
Gerald is the sole nominee to fill Yes. Gerald has received the necessary number of votes.
the open position on ABC Corp.'s
board of directors. Gerald has
been on the board for the past ten
years and his management style is
extremely unpopular. At the annual
meeting, Gerald receives two of
the 1,000 shareholder votes. May
Gerald remain on the ABC board?

Connor owns ten percent of the Yes, if the company has proxy access bylaws.
stock issued by Outdoorsmen, Inc.,
an outdoor sporting supply
company. The nominating
committee has nominated an
executive from a private equity firm
for the open seat on the
Outdoorsmen board. Connor
believes the nominee does not
have the industry knowledge to
properly manage Outdoorsmen
and that his good friend, Michael,
who has produced popular nature
documentaries and has a CPA, will
be much more effective. Connor
demands that Michael be included
in the proxy materials sent to
shareholders. Must the
Outdoorsmen board comply with
Michael's request?
The SEC discovered that Alliant's Yes. Allianz may claw back all of the profits the CFO
annual report contained misleading received from the sale of the stock six months ago.
financial statements, which
concealed the fact that Alliant
would not be able to collect $2
million worth of debt. When the
SEC releases a statement
concerning Alliant's fraudulent
filings, the company's stock drops
from $38 to $15 per share. Six
months prior to the SEC's
announcement, Alliant's CFO sold
half of his shares at the company's
all-time high stock price of $43.
May Alliant recover any money
from the CFO that he earned from
his sale of stock?

DCAM Corp. stock is held by Rita Rita may object to the merger and require DCAM to buy
(5 percent), Jack (50 percent), and back her stock at fair market value.
Taran (45 percent). DCAM's board
of directors has negotiated a
merger with EAE, Inc. Jack and
Taran support the deal, but Rita
thinks it will be disastrous for the
company. If DCAM is a privately
held corporation, what recourse
does Rita have?
Marley owns 10% of the common Yes, because Marley's request is related to his interest as a
stock in Hometown Industries, Inc., shareholder.
and Cecil and Jonathan each own
45%. Both Cecil and Jonathan sit on
Hometown's Board of Directors
(BOD) and are paid officers, while
Marley is only a shareholder with
no other role in the company.
Hometown has not paid dividends
to shareholders in a number of
years, despite the company's
financial success. When Marley
questions the lack of distributions,
Cecil points out that any profits
have been reinvested in the
company. Jonathan shows Marley
how the use of company profits
has improved and strengthened
the business, resulting in its current
success. Knowing Marley is
unhappy, Cecil and Jonathan offer
to purchase all of Marley's shares in
the company for $25 per share.
Marley believes his stock is worth
much more, and asks to inspect
Hometown's corporate books, but
Cecil and Jonathan refuse. Does
Marley have a right to inspect
Hometown's corporate books?
Marley owns 10% of the common No, because the directors are operating in good faith.
stock in Hometown Industries, Inc.,
and Cecil and Jonathan each own
45%. Both Cecil and Jonathan sit on
Hometown's Board of Directors
(BOD) and are paid officers, while
Marley is only a shareholder with
no other role in the company. Both
Cecil and Jonathan receive salaries
that are on par with the industry.
Hometown has not paid dividends
to shareholders in a number of
years, despite the company's
financial success. When Marley
questions the lack of distributions,
Cecil points out that any profits
have been reinvested in the
company. Jonathan shows Marley
how the use of company profits
has improved and strengthened
the business, resulting in its current
success. Both directors tell Marley
that if this trend continues, the
company should begin paying
dividends after a few additional
quarters. Marley is not satisfied
with Cecil's and Jonathan's
responses, and seeks to bring a suit
to force the payment of a dividend.
Is it likely that Marley will be
successful in his claim?

You might also like